Sie sind auf Seite 1von 324

0hemist

Patt I
Section I
Structure, Bonding, and Reactivity

Organ

Sections I-IV
Section II
Structure Elucidation

Section III
Stereochemistry

$ection IV
Hydrocarbon Reactions

'll'te

K.lE. V. I. E. W Speci alizing in II{CAT Preparation


@

Nomenclature

a) b)

IUPAC Nomenclature

General Nomenclature

Bonding and Molecular Orbitals

Section I
Structure, tsonding,
and

a) Lewis Dot Structures b) Bonding Model c) Covalent tsonds d) Molecular Orbitals and Bonds
e) g) a)
b)

i. ii. iii.

f)

Single Bonds Double Bonds Triple Bonds Molecular Structures Octet Rule (HONC Shortcut) Charged Structures Hybridization of Atomic Orbitals sp-Hybridzation sP2-HYbridzation sP5-nybridzation Common Shapes

Hybridization

Reactivity

"</ /V"
HH C-C. HH HH HH

by Todd Bennett

i. ii' iii'

Bond Energy

a) Bond Dissociation trnergy b) Ionic Bonds Intramolecular Features a) Resonance b) Inductive Effect

7ffiV"

c)

Steric Hindrance

d) Aromaticity

Fundamental Reactivity

a) Proton Transfer Reactions b) Lewis Acid-Base Keactions c) Acid and Base SLrength d)
Primary Effects Secondary Effects Values and Terminology Electrophiles and Nucleophiles

i. ii. iii.

Physical Properties

a) l-lydrogen-Bonding b) Polarity c) Van der Waals Forces d) Solubility and Miscibility

RNKIIELEY
l)n.n-v.r)E.w'
Speciahztng in MCAT Preparation

Organic Chemistry

Molecular Structure

Introduction

The perfect place to start any review of organic chemistry is the basics of molecular structure, which traditionally include bonding, hybridization, and electronic distribution. We shall consider a chemical bond to be the result of atomic orbitals overlapping to form molecular orbitais. We shall consider all bonds involving carbon to be covalent in nature. A covalent bond is thought to involve the sharing of electrons between two adjacent nuclei. According to the rules of electrostatics, the region between two nuclei offers a highly favorable environment for electrons, where they can be shared by neighboring atoms.
However, there are several other factors to consider in bonding. if bonding were purely a matter of electrostatics, then all of the eiectrons would be found between two neighboring nuclei, not just the bonding electrons. The sharing of electrons may be either symmetric (when the two atoms of the bond are of equal electronegativity) or asymmetric (when the two atoms of the bond are of unequal electronegativity). Sharing of electrons occurs when the atoms of a bond lack a compiete valence electron shell. By sharing electrons, each atom moves closer to completing its shell This is the driving force behind the formation of stable
covalent bonds.

Having looked briefiy at electron distribution, we can introduce the idea of electronic orbitals, which are three-dimensionai probability maps of the location of an eiectron. They represent the region in space where an electron is found 95% of the time. We shall consider the orbitals and the overlap of orbitals to describe the electronic distribution within a molecule. Once one has established a foundation in bonding, the classification of molecules can be made based on sinrilarities in their bonding of particular atoms, known as functional groups. Each functional group shall be considered in terms of its unique electron distribution, hvbridization, and nomenclature. Nomenclature, both that of the International Union of Pure and Applied Chemists (IUPAC) and more general methods describing the substitution of carbon within a functional group, shall be used to describe a particuiar organic molecule. The review of nomenclature is
continuous throtrghout all sections of this book.
Then, we shall consider the factors that affect the distribution of eiectron density within a molecule, including resonance, the inductive effect, steric hindrance, aromaticity, and hybridization. The distribution of electron density can be used

to explain and predict chemical behavior. The simplest rule of reactivity in organic chemistry is that regions of high electron density act as nucleophiles by sharing their electron cloud with regions of low electron density, which act as electrophiles. If you can correctly label a molecule in terms of the region that caries a partially negative charge (the electron-rich environment) and the region that carries a partially positive charge (the electron-poor environment), you can
understand chemical reactions better.

And so begins your review of organic chemistry. Fortunately, much of organic chemistry is taught from the perspective of logic, which makes preparing for organic chemistry on the MCAT easier. In organic chemistry courses you are required to process information and reach conclusions based on observations, which is also required on the MCAT. Reviewing and relearning this material will help you develop criticai thinking skills, which will carry over into your rerriew for other portions of the exam. Despite what you may have perceived was a girth of information when you initially studied organi.c chemistry, you don't need to review that much material to prepare successfully for the MCAT.
Copyright
@

by The Berkeley Review

Exclusive MCAT Preparation

Organic Chemistry

Molecular Structure

Nomenclature

IUPAC Nomenclature (Systematic Proper Naming) IUPAC Nomenclature is an internationally used system for naming molecules. Molecular names reflect the structural features (functional groups) and the number of carbons in a molecule. In IUPAC nomenclature, the name is based on the carbon chain length and the functional groups. The suffix indicates which primary functional group is attached to the carbon chain. Table 1-1 lists prefixes for carbon chains between one and twelve carbons in length. Table 1-2 lists the suffices for various functional groups. Be aware that "R" stands for any generic alkyl group. When R is used, it indicates that the carbon chain size is irrelevant to the reaction. Table 1-3 summarizes the nomenclature process by listing several four-carbon compounds.
Carbons
1

Prefix
metheth-

Carbons
5

Prefix penthex-

Carbons
9 10
11

Prefix
nondec-

2
J

propbut-

7
8

heptoct-

undecdodec-

12

Table 1-1

Functionality
R-CH3
R-O-R

Compound Name Alkane


Ether

Bonding

C-C

&

C-H

C-O-C o

R-CO-H R-CH2-OH R-CO-R

Aldehyde Alcohol
Ketone

C-Co il C_C-C o

il

C-O-H

R-CO-OH

Carboxylic acid Table 1-2

C_C_

il

OH

Formula
H3CCH2CH2CH3

IUPAC Name
Butane Butene Butanal Butanone Butanol

Structural Class Alkane Alkene Aldehyde


Ketone

H3CCH=CHCH3 H3CCH2CH2CHO
H3CCH2COCH3

H3CCH2CH2CH2OH H3CCH2CH(oH)CH3 H3CCH2CH2CH2NH2 H3CCH2CH2CO2H

Alcohol Alcohol
Amine Carboxylic acid

2-butanol
Butanamine Butanoic acid

Table 1-3

Copyright

by The Berkeley Review

The Berkeley Review

.ffi

:.,

::r'ii'*lllil+iji*i:il$i:

:::l':: .:.

::ri:::;ni:iiliiii:ri:*iiiliffii

Organic Chemistry

Molecular Structure

Nomenclature

Figure 1-1 shows examples of IUPAC nomenclature for four organic compounds with variable functional groups:

C1

3-methylpentanoic acid Longest chain: 5 carbons Carboxylic acid group

4-chloro-S-methyl-3-heptanol Longest chain: 7 carbons

Methyl substituent at C-3

Alcohol group Chloro substituent at C-4 Methyl substituent at C-5

3-ethylcyclopentanone Ring of 5 carbons Ketone group Ethyl substituent at C-3 Figure 1-1

3,3-dibromobutanal
Longest chain: 4 carbons

Aldehyde group
2 Bromo substituents at C-3

General Nomenclature (Common Naming Based on Substitution)

In addition to the IUPAC naming system, there is a less rigorous method of naming compounds by functional group and carbon type (based on carbon
substitution). Carbon type refers to the number of carbon atoms attached to the central carbon atom (carbon atom of interest). A carbon with one other carbon attached is referred to as a primary (1') carbon. A carbon with two other carbons attached is secondary (2"). A carbon with three other carbons attached is tertiary (3"). Figure 1-2 shows some sample structures.

H CH. HCH"HH carbon \r)yn"t .s r"'r,iu.y carbon \ l}}i""oary r_zt{ ./'\ oH ./-\ -/-\ Cl HgC CHg H3CH2C

carbon

H3CH2C

Isobutane

(2-Methylpropane)

Sec-butanol (2-Butanol)

n-Propyl chloride
(1-Chlropropane)

Figure 1-2 Nomenclature is an area of organic chemistry best learned through practice and experience. We will deal with nomenclature throughout the course, as we introduce each new functional group. Understanding nomenclature is especially important in MCAT passages where names rather than structures are given. Be sure to know the Greek prefixes for carbon chain lengths up to twelve carbons.

Copyright

by The Berkeley Review

Exclusive MCAT Preparation

Organic Chemistry

Molecular Structure

Bonding and Orbitals

Lewis Dot Structures (Two-Dimensional Depiction of Molecules) Lewis dot structures represent the electrons in the valence shell of an atom or bonding orbitals of a molecule. Typically, we consider the Lewis dot structures of elements in the s-block and p-block of the periodic table. For every valence electron, a dot is placed around the atom. Single bonds are represented by a pair of dots in a line between the atoms, or by a line itself. A double bond is represented by a double line (implying that four electrons are being shared.) Likewise, a triple bond is represented by a triple line (implying that six electrons are being shared.) Lewis dot structures are familiar to most chemistry students,
so recognize the exceptions to the rules, as they make good test questions.

Example 1.1 V\4rat is the Lewis dot structure for HcBF? A.


B.

:H-B-F:

u
C.

t"
D.

H- S-'F

t"

.ry

H-ri-r:
H

t"

H- B-F
H

t"

Solution
Boron has only three valence electrons, hence it can make only three bonds. There is no lone pair on the boron atom, eliminating choices A and C. Hydrogen has only one electron, which is in the bond to boron, so there is never a lone pair on a bonded hydrogen. This eliminates choices A (already eliminated) and B. Fluorine has a completed octet, so it makes one bond and has three lone pairs, as depicted in choice D, the best answer.

Bonding Model Bonding is defined as the sharing of electron pairs between two atoms in either an equal or unequal manner. As a general rule, a bond is the sharing of two electrons between two adjacent nuclei. The region between two nuclei is the most probable location for an electron. In most cases, with the exception ol ligand bonds (kaown also as Leruis ttcid-base bonds), one electron from each atom goes into forming the bond. When electrons are shared evenly between two atoms, the bond is said to be a coualent bond. Ylh,en electrons are transferred from one atom to another, the bond is said to be an ionic bond. The difference between a covalent and ionic bond is measured in the degree of sharing of the electrons, which can be determined from the dipole. The more evenly that the electrons are shared, the less the polarity of the bond. The relative electronegativity of two atoms can be determined by measuring the dipole of tl-re bond they form. When the difference in electronegativity between two atoms is less than 1.5, then the bond is said to be covalent. When the difference in electronegativity between two atoms is greater than 2.0, then the bond is said to be ionic. When the difference in electronegativity between two atoms is greater than 1.5 but less than 2.0, then the bond is said to be polar-covalent (or partially ionic).

E.

l"t

r
a

:-,'

l}.e Berkeley Review

The Berkeley Review

Organic Chemistry
Example L.2

Molecular Structure

Bonding and Orbitals

Which of the following bonds is MOST likely to be ionic?

A. C-O B. N-F C. Li-H D. Li-F


Solution A bond is ionic when the difference in electronegativity between the two atoms exceeds 2.0. This means that the bond that is most likely to be ionic is the one between the two atoms with the greatest difference in electronegativity. Lithium is a metal and fluorine is a halide, so they exhibit the greatest electronegativity difference of the choices listed. The best answer is therefore choice D.

Covalent Bonds Bonds can be classified in one of three ways: ionic, polar-covalent, and covalent. A covalent bond occurs when electrons are shared between two atoms having Iittle to no difference in electronegativity. As the difference in electronegativity decreases, the covalent nature of the bond increases. There are two types of :ovalent bonds: sigma bonds (o), defined as having electron density shared betvr'een the nuclei of the two atoms; and pi bonds (n), defined as having no electron density shared between the nuclei of the two atoms, but instead only above and below the internuclear region. Sigma bonds are made from many types of atomic orbitals (including hybrids), while pi bonds are made exclusively of parallel p-orbitals. In almost all cases, the sigma bond is stronger than the pi bond, with molecular fluorine (F2) being a notable exception. Figure 1-3 shows a generic sigma bond. You may notice that within a sigma bond, only about eighty to ninety percent of the electron density lies between the nuclei, not all of it.

o@o
Electron density

.Nuclei.

Figure L-3

Example 1.3

Which drawing depicts the electron density of

a carbon-carbon sigma bond?

Solution A sigma bond has its electron density between the two nuclei, which eliminates choice D. The two atoms in the bond are identical, so the electron density should be symmetrically displaced between the two nuclei. This eliminates choice B.
Most of electron density is between the nuclei, so choice A is a better answer than choice C. These drawings are ugly, so focus on the concept, not the pictures. Exclusive MCAT Preparation

Copyright

by The Berkeley Review

Organic Chemistry

Molecular Structure

Bonding and Orbitals

Figure 1-4 shows a generic n-bond. Within a n-bond, there is no electron density between the two nuclei. The electron density in a ru-bond results from electrons being shared between the adjacent lobes of parallel p-orbitals. Nuclei

Electron Densitv J

Figure 1-4

carbons have sp2-hybridization. The C=Cbond is made up of aoroz-rozbond and a n2r-2p bond. Bond length varies with the size of the orbitals ih ttre bond. For instance, a sigma bond composed of an sp2-hybridized carbon and an sp3hybridized carbon is shorter than a sigma bond comprised of two sp3-hyb ridized, carbons. Hydrogens use s-orbitals to form bonds. Figure 1-5 shows thiee sigma bonds with their relative bond lengths. The longer bond is associated wittrthe larger orbitals (bond radii: dz > dy > dil.

and pi bonds plesent. The n-bond Conslsts of p-orbitals side by side, und it,

account hybridization and overlap. In alkanes, carbons have sp3-hybridiiation and all of the bonds are sigma bonds. In alkanes, there are two typbs of bonds: c H lorps-r bonds) and c- c (o'sp3-sp3 bonds). In alkenes, t-h-"." uru sigma -

In organic chemistry, covalent bonds are viewed. in great detail, taking into

or-rp3

orp2-rp2

osp3_sp3

Figure 1-5

length of a bond is defined as the distance between the nuclei of the two atoms making the bond. Figure 1-6 shows an example of a n-bond between two Zprorbitals, which is typical for nearly all n-bonds encountered in organic chemistry, because carbon, nitrogen, and oxygen have 2p-orbitals in their valence shells.

in sigma bonds, no matter what the orbitals are from which the sigma bond originates. In pi bonds, electron density does not 1ie between the two nlclei. The

Figure 1-5 confirms that most of the electron density lies between the two nuclei

2p,-2p,

eo
The Berkeley Review-

ffi

Figure 1-6

Copyright

by The Berkeley Review

Organic Chemistry

Molecular Structure

Bonding and Orbitals

Pi bonds are found as the second bond present in double bonds and the second

and third bonds present in triple bonds. The first type of bond to form between atoms is usually the sigma bond. Once a sigma bond exists between two carbon atoms, then pi bonds can form between the atoms. Fluorine gas is an exception to the "sigma bond first" rule. Molecular fluorine (F2) has only one rc-bond, with no o-bond present. This is attributed to the small size of fluorine and the internuclear repulsion associated with a typical single bond. This is why the bond dissociation energy of F2 is less than the bond dissociation energy of Cl2, even

though chlorine is below fluorine in the periodic table.

Molecular Orbitals
Molecular orbital is a fancy way of describing a bond or anti-bond that exists between two atoms. An anti-bond is a molecular orbital that results in bondbreaking when coupled with a bonding orbital. It is important to recognize the shape and location of electron density in molecular orbitals. Figure 1-7 shows the common bonding and anti-bonding orbitals associated with organic chemistry.

.@.@o + cffi'c
sp
J

sp3

orp3-rp3

Sigma bonding molecular orbital

D*.c@ + D@c@ ,3 -3 ' sp"


, sp"
-

.-3 .-i oxsp--sp-

Sigma anti-bonding molecular orbital

I I
p

+
frzp-zp

Pi bonding molecular orbital

fB n tr= ffic r

5+6 P P

d6

lt*rr-r,

Pi anti-bonding molecular orbital

Figure 1-7
The shading of the lobes in each orbital represents the direction of spin for the electron. h:r order for electron density to overlap, the electrons must have the same spin. This is analogous to driving on the freeway. If you join a freeway in the same direction as traffic is flowing, you can easily blend into traffic. This is a favorable interaction. If you join a freeway in the opposite direction as traffic is flowing, you cannot easily blend into traffic. This is an unfavorable interaction.

Copyright @by The Berkeley Review

Exclusive MCAT Preparation

Organic Chemistry
l\lolecular Bonds

Molecular Structure

Bonding and Orbitals

greater interest than the sigma-bonds and pi-bonds are the single, double, and triple bonds present between atoms. single, double, and triple bond nature is discussed more so than the sigma and pi nature of bonds. In trganic molecules, there are only single, double, and triple bonds. Between like atoms, the descending order of relative strengths of boncls is triple bond > double bond > single bond. Another rule to consider is that for bonds between like atoms, the ionger the bond, the less the electron density overlaps between nuclei, and thus the weaker the bond. This is summarized as: longer ionds are rueaker bonds.

of

single Bonds: single bonds are composed of only one sigma bond between the two atoms. single bonds are longer than double and triple bonds between two atoms, even though fewe_r electrons are present. Ethane has sigma bonds only and is shown in Figure 1-8 in both stick figures and with the rele')ant orbitals.

HH

HH

/v"
C-C.

OC

ffi"p
orp3-5p3

/
H

\"

Figure 1-8 Double Bonds: Double bonds are composed of one sigma bond and one pi bond between two adjacent atoms. Ethene (czH+) has f6ur sigma bonds between carbon and hydrogen, a sigma bond between the two carf,ons, and a pi bond present between the two carbons to compiete the carbon-carbon double bond. Ethene is shown in Figure 1-9 in both stick figures and with the relevant orbitals.
ftp^-p^

Illtt,,,

H7:

\n

..rr\H
osp2-roz

%o="# @rffi%

Figure 1-9

Triple Bonds: Tripie bonds are composed of a sigma bond. and two pi bonds between two adjacent atoms. Triple bonds are shorter than either iingte or double bonds. Ethyne (CzHz) has two sigma bonds between carbon and hydrogen, a sigma bond between the two carbons, and two pi bonds between the two carbons to complete the carbon-carbon triple bond. Ethyne is shown in Figure 1-10 in both stick figures and with the relevant orbitals.
fipv-pv

@.- .P

Figure 1-10

Copyright O by The Berkeley Review

lo

The Berkeley Review

Organic Chemistry
Example 1.4

Molecular Structure

Bonding and Orbitals

\A/hat is the relative bond strength of carbon-carbon bonds in the molecule shown

below?

bondaS\
bond c

,rr..

ooloo

_-_*/

l, / C*C
\

,r,
-\ z .v ^CH^CH"r
C CHe

(H3.)2HC

""Y T
A. Bond a > Bondb > Bond c > Bond d B. Bondb > Bond a > Bond c > Bond d
C. D.
Bond d > Bond a > Bond c > Bondb Bondb > Bond c > Bond a > Bond d

bond d

Solution
There strongest C-C bond is a double bond, bond b, so choices A and C are -Bond eliminated. d is the weakest, because it is between two sp3 carbons. Bond c is stronger than bond a, despite both sharing an sp2 and an sp3 carbon, because bond c contains the more highly substituted carbon. Choice D is the best answer.

Molecular Structures We shall continue from the fundamental concept that a valence electron can be shared between two nuclei rather than being isolated to just one nucleus, because the attractive force of two positive sites is greater than the attractive force of one. This is the basic, perhaps oversimplified, perspective of a chemical bond. The sharing of electrons is what characterizes a covalent bond. One of the first rules of organic chemistry that you must understand is the octet rule. It is valid for carbon, nitrogen, and oxygen atoms. To understand organic chemistry, it is important that you recall VSEPR theory, which applies to bonding (in particular, to the subgroups of covalent bonding like single, double, and triple bonds and their component o-bonds and n-bonds). Table 1-4 shows the skeletal structures of molecules that contain carbon, nitrogen, oxygen, and hydrogen. Atom
Valence Electrons To Complete Shell
4 e- needed

Number of Bonds in Neutral Compounds

Carbon (C)

4. 5.
b

.c.
.N.

n
3

-tY'
_-6-_

-L_

Nitrogen (N)

3 e- needed

'"; - -''Y', \ -N'

i5:

Oxygen (O) Hydrogen (H)

- .ct.
..

2 e- needed
1 e- needed

:O.

1H.

1H-

Table 1-4

Copyright

by The Berkeley Review

ll

Exclusive MCAT Preparation

Organic Chemistry

Molecular Structure

Bonding and Orbitats

Octet Rule and the HONC Shortcut: Every molecular structure should have atoms that obey the octet rule (eight valence electrons for C, N, and o). The numbers of electrons needed to complete the shell in the Table 7-4 are derived from the electrons needed to obey the bctet rule. All neutral structures have atomic arrangements as described inTable 1.4. To complete the octet valance shell, carbon requires four electron pairs in the form of bonds, nitrogen requires one lone pair in addition to the thiee bonds it makes, and oxygen requires two lone pairs in addition to the two bonds it makes. You must be able to recognize valid structures by applying the bonding rules (HONC-1234). In a neutral compound, hydrogen makes onebond., oxygen makes truo bonds, nitrogen makes threebonds, and carbon makes four bonds. Neutral structures always obey this rule. Figure 1-11 shows examples of valid and invalid structures and a brief description of the bonding to the component atoms.

C:

-.
{j

::

:,:

::.

r)-l

-a

'-'

tt
/\ HH
L-L

/tnt

H \//
/\ HH
^-A L-L

,CHZ

All carbons have 4 bonds. All hydrogens have 1 bond.


Good Structure

Most carbons have 4 bonds, but one carbon has 5 bonds. All hydrogens have 1 bond.
Bad Structure

HsC- C- CAll

CH2CH2CH3

H"C-C:

/CH3

carbons have 4 bonds.

cH2cH3 AII carbons have 4 bonds. Ali hydrogens have 1 bond, but oxygen has 3 bonds.
Bad Strttctttre

A1l hydrogens have 1 bond.


Good Structure

Dn
:-..!i
tl__

_
_

:_

H '\
1\-L

/-"3

CH"

tra, N-C
H
A11

CHs

:_r

CHs

Ali carbons have 4 bonds.


A1l hydrogens have 1 bond. Nitrogen has 3 bonds.
Good Structure

All carbons have 4 bonds.


hydrogens have 1 bond, but nitrogen has 4 bonds.
Bad Structure

Figure 1-11

You can validate molecular structures by seeing whether they satisfy bonding rules (HoNC-7234) and conventions with regard to the number of bonds and lone pairs. If a structure does not satisfy the rules, then there must be a charge present. Generally, having too many bonds in a molecule results in a cation and too few bonds results in an anion, except with carbon. For instance, if oxygen makes three bonds and has one lone pair, it carries a positive charge. wnu.t nitrogen makes two bonds and has two lone pairs, it carries a negative charge. \Alhen carbon makes three bonds, the charge depends on the presence or absence of a lone pair (presence yields an anion, r,t'hile absence yields a cation). Copyright
@

by The Berkeley Review

t2

The Berkeley Review

-i!

Organic Chemistry

Molecular Structure

Bonding and Orbitals

Charged Structures Formal charges (charged sites on a molecule) occur when there is an excess, or shortage of electrons on an atom. For instance, an oxygen atom typically has six valence electrons and wishes to have eight. This means that oxygen makes two bonds to complete its valence shell (and thus satisfy the octet rule). However, if an oxygen atom had only five valence electrons, it would be short one electron from its original six and would consequently carry a positive charge. Having only five valence electrons, the positively charged oxygen would need to make three bonds (one more than its standard two) to complete its octet. We can conclude that oxygen with three bonds carries a positive charge. Table 1-5 shows somecommon organic ions to commit to memory: Number of Bonds
to Neutral Atom

Number of Bonds to Cationic Atom

Number of Bonds to

Anionic Atom

Carbon (C)

4
3

.c.
4
t'

3l
-/t\

? ..
-+ r+-

-'Y',-\
/-1--

Nitrogen (N)

'N'

z\':N\-5+=

-t-

Oxygen (O)

.o.

'-ij\,:t

:o*=

-p]'

Table 1-5

Drawing Molecular Structures Drawing molecular structures from a given formula requires following the octet rule for aII atoms except hydrogen. On occasion, thete will be charged atoms within the compound, but the number of charged atoms within the structure should be minimized. Figure 1-12 shows some samPle structures for a few
common molecules.

Molecular and Structural Formula

Lewis Structure

3D Structure

c2H60 cH3cH2oH

HH ll H_C_C- Otl HH
HHH

H
H
H

.C_C \t\ H.;


H

liu \it
.
.v II ^- rr

c2H7N cH3NHCH3

H_ C-N_ C_ H

rtl

HH
c2H50* cH3cHo+H

t..l

: C_ N' .( )Y HtH .C_H


H

ll H-C-C:
H

HH

"irH
.C-C \\
.g*-H

t..

O*- H

"'*
l5

Figure 1-12 Copyright


@

by The Berkeley Review

Exclusive MCAT Preparation

Organic Chemistry

Molecular Structure

Ilybridization

Hybridization
Hybridization of Atomic Orbitals Hybridization entaiis relocating electron density in atomic orbitals prior to bonding, in order to minimize the repulsion between electron pairs and thereby allow for bonding between atoms. There are three main types bi nyuria orbitals to consider: sp,sp2, and sp3 hybrids. Hybrid orbitals are atomic orbitals that are

represents general trends that are observed in neariy all molecules with hybridized orbitals involved in their molecular orbitals.

involved in making bonds between atoms. Listed in Table 1-6 are some pertinent facts and structural features for each of the three types of hybridization. lable 1-6

:r

Hybrid
Atomic Orbitals Angle
Shape

sp

sp2

sp3

s+p
180'

s+p+p
720"

s+P+p+p
109.5'

linear
2
2

trigonal pianar
3
1

tetrahedral
4 0

i!

o-bonds and e- pairs

r-bonds

Table 1-6
The number of n-bonds in Table 1-6 is for typicai molecules that obey the octet class of molecules that is an exception to the features in Table 1-6 is the boranes, such as BH3, BF3, and BR3. In boranes, the boron atom has only three valence electrons, so a neutral boron cannot satisfy the octet rule. The result is that boron has sp2-hybridization for its three sigma bonds, but no pi bond.

rule' A

Structures with Orbitals Lewis structures are used as shorthand representations of molecules. Flowever, in organic chemistry, molecules should be visualized in three dimensions, which hybridization helps to faciiitate. Determining the three-dimensional shape of a molecule requires first assuming a shape based on hybridization of the central atoms, then applying valence shell electron pair repulsion (vsEpR) theory. Figure 1-13 shows molecular structures witl-r orbitals and three-dimensional orientation. Structures should be drawn with and correct bond lengths and bond angles should be based on hybridization, steric hindrance, and vsEpR rules.
sp2

s,4

j-

S...'n, BH: H-p-1

-hybrid,ized

sp'-hybridized

,H

0-n

cH+

H-- \,,,*-,
H

sp3-hybridized

sp'-hybridized

,H

L
I

NHe

r-Ut'un
H
Figure 1-13

Hzo

ql

"t
The Berkeley Review

*i

Copyright O by The Berkeley Review

t4

Organic Chemistry

Molecular Structure

Hybridization

The orbital shown for BH3 in Figure 1-13 is actually an empty p-orbital, while the other orbitals depicted in Figure 1-13 are hybrid orbitals. The p2-orbital of BH3 is devoid of electrons, so the hybridization is sp2. \Mhile an empty p-orbital does

not actually exist, we consider the region where an electron pair could be
accepted. The three hybrid orbitals are detailed in Figures I-1.4,1.-75, and 1-16.

sp-Hybridization: sp-hybtidization is the result of the mixing of the s-orbital and the pporbital.

.&.o,
st'ayarialzation:

Two atomic orbitals

Hybridizes
to become

@&r.@
Two sphybrid orbitals

Figure 1-14 sp2-hybrtdtzation is the result of the mixing of the s-orbital, the pa-orbital, and the py orbital.

Py

.CJ@.O r)
Hybridizes
to become

o..

s^

three sp2-irybrid orbitals

Three atomic orbitals

Figure 1-15 sp3-hybtidi"ation is the result of the mixing of the s-orbital, the pa-orbital, the py-orbital, and the p2-orbital.

tt'-nyAnalzation:

i'rery.
Four atomic orbitals

Hybridizes

+@trb.e
sp3

,o3

ffi'I'^^,

tobecome ,or.m

Figure L-16

Example 1.5 What is the hybridization of each carbon in propene (H2C=CH-CH3)?

A. sp, sp, and sp3^ B. sp/,sp, and spr C. tp?,tp\, and sp3 D. sp', spr, and spJ
There are three carbons in propene. The first two carbons are involved in a rcbond, so they are each sp2-ltybridized. This makes the best answer choice C. The last carbon is not involved in any n-bonds, so it has sp3-hybridization.

Solution

Copyright

by The BerkeleY Review

Exclusive MCAT PreParation

Organic Chemistry

Molecular Structure

Hybridization

Common Three Dimensional Shapes Hybridization theory supports the notion that there are recurring molecular shapes (tetrahedral, trigonal planar, and linear) that can be seen within different molecules. This means that there is a electronic expianation for the structures that are observed within various molecules. Hybridization is a theoretical explanation to rationalize why electron pairs in the valence shells of bonding atoms assume orientations as far from one another as possible. Hybridization is
used to explain bond lengths and bond angles. Figures r-r7,1.-rg, and 1-19 show structures with their corresponding geometry and structural features.

Tetrahedral and

sp3

-nybridization

pairs) has tetrahedral orientation of the electron pairs about the central atom. This does not mean that the shape is tetrahedral, but that the orientation of electron pairs about the central atom (geometry) is tetrahedral. Figure 1-17 shows different structures with tetrahedral geometry about the central itom, but different molecular shapes.
Tetrahedral

A central atom with four electron pairs (any combination of bonds and lone

Structure
/

Trigonal pyramidal Structure

l-/-

t-t-'Y.tt -Hlu H
4

------^.. ------* ..

I
3

H/NY'H H/NY,H
atoms/1 lone pair
uecause of rone pair repulsion, bond angles decrease. N is r*ullu. than c, so Oond length N-H is less than C-H.

Because of

I I lengths are equal./ I C-H: 1.10A & < HCH: 109.5" /


-symmetry, all bond and bond angles

atoms/0lone

pairs

N-H:

1.00A & < HNH: 102.3"

l I Bent Structure
\\
\r'n'
rr

1o-ru
of lone-pair repulsion, bond

2 atoms/21one pairs
Because
angles decrease. O is smaller than N, so bond length O-H is less than N-H.

O-FI: 0.96A & < HoH:


Figure 1-17

104.5'

Trigonal Planar an d sp2 -Hybridization A central atom with three other atoms, two olher atoms and one lone pair, or one -of other atom and two lone pairs attached has trigonal p?ur,ut geometry th" th."" substituents (or electron pairs) about the central atom. This does not mean that the shape is trigonal planar, but that the orientation of electron pairs about the central atom is trigonal planar. Figure 1-18 shows the planar structure and spatial representation of the bonds in ethene. The stick and ball representation shows the three-dimensional perspective for ethene.
C-opyright

by The Berkeley Review

l6

The Berkeley Review

Organic Chemistry
Planar Structure

Molecular Structure
Spatial Representation

Ilybridization

H'/\' /\ HH
3

\i \ /
t,L

sn2-hvbridization

atoms/0 lone pairs


Each carbon in ethene has sp2-hybridization.

Figure L-18
Line ar and sp-Hybridiz

ation
one lone pair

A central atom with two other atoms or one other atom and

attached has linear geometry of the two substituents (or electron pairs) about the central atom, This does not mean that the shape is linear (although in most cases it is), but that the orientation about the central atom is linear. Figure 1-19 shows the linear structure and spatial representation of the bonds in ethyne. The stick and ball representation shows the three-dimensional perspective for ethyne'

Linear Structure

Spatial Representation

sr-hvbridization

C::

t\

' tt' /\

k\,
Figure L-19

180.0"

2 atoms/0lone pairs
Each carbon in ethyne has sp-hybridization.

Example 1.6

The hydrogen-carbon-hydrogen bond angle in formaldehyde (H2CO) is BEST approximated by which of the following values?

A. 108.3' B. 771.7" c. 118.5'


D.
121.5'

Solution
The first feature to look at is the hybridization of carbon. Carbon is involved in one n-bond, so the hybridization is sp2. The bond angle about an sp2-hybridized. carbon is predicted to be 120". The question here is whether the angle is slightly greater or slightly less than 120". Because there are two pairs of electrons on the

oxygen/ the electron density repels the electrons in the two carbon-hydrogen bonds. This forces the two bonds closer together, which compresses the hydrogen-carbon-hydrogen bond angle. According to valence shell electron pair repulsion (vsEPR) theory, the angle shouid be slightly less than 120". The best answer is thus choice C.

_:
Copyright
@

by The Berkeley Review

t7

Exclusive MCAT Preparation

Organic Chemistry

llolecular Structure

Bond Energies

Bond Energy
Bond Dissociation Energy chemistry, the energy required to cleave a bond in a homolytic fashion is commonly used to compare reiative bond strengths. Homolytic cleivage refers to the breaking of a chemicai bond into two fiee radical fiagments. This is typically viewed in the gas phase or an aprotic, nonpolar solvenl, where ions are too unstable to exist. It is important that you recall that energy is released when a bond is formed and that energy must be absorbed by the ilolecule to break a bond. By subtracting the energy released upon forming new bonds from the energy required to break bonds, the enthaipy of a reaction can be determined. This is shown in Equation 1.1.
L:r organic

AHReaction = lEnergy&onds broken) - Energy66nds formed)

(1.1)

the enthalpy of a reaction is known, then the bond dissociation energies for bonds that are formed and broken during the course of a reaction can be determined. It is this method that allows for the comparison of bonds between identical atoms within different morecules. For instance, the theory of aromaticity is supported by the excess energy that is reieased. upon the formation of a n-bond that completes the aromatic iing. The release of excess energy implies that the molecule is more stable than expected from the standard bond dissociation energies, so some other factor must be involved. Table I-T lists some bond dissociation energies for typicat bonds in some common organic molecules. Bond Dissociation Energies for

If

A-B

Bonds (Kcal/mole)
Ph

A
Methyl
Ethyl n-Propyl Isopropyl f-Butyl Phenyl Benzyl

B=H
105 101
101

Me
90
89 89

Et
89
B8

i-Pt
86

t-Bu
84
85 85

OH
93 95 95 96 96
111 81
7B

NHz
85

102
101

6/
86
B5

85 85
B5 B5

88

101

98 96
111

89 87

87
95 100 75 70 79 85 101

82 77 96
73

99
99 115

82

702
76
74

99

702
77 69

88 86 86

/+
70

90

Allyl
Acetyl Ethoxy

67
75

N/A
93,5
101 105 111

81

77

107 44

96

L04

83

N/A
100 98

N/A
95 96

Vinyl
H

1I2
704.2

102
105

N/A
IT9

N/A N/A
1.07

i01

Table 1-7

A greater value in Table 1-7 implies that the bond is stronger. you may note that the weakest bond listed in Tabie 1-7 is an o-o single bond within a peroxide molecule (Eto-oH). Because this bond is so r.r'eak, peroxides are highly reactive species, often used to oxidize other compounds. The data in Table 7-7 also reveal that the substitution of the carbon and the position of the bond within the molecule affect the bond energy. The effect of hybridization can also be extracted when comparing bond energies betl-een r-inr.l and methyl substituents.

Cop.,-r::: I :'. lie Berkeley Review

la

The Berkeley Review

Organic Chemistry
Example 1.7

Molecular Structure

Bond Energies

According to the data in TabIe


bonds is the MOST stable?

1.-7,

which of the following carbon-carbon single

B. An spt-carbon to a second.ary spr-carbon C. A secondary sp3-carbon to a primary sf -carbon D. A secon dary sp3-carbon to a iecondary sp3-carbon
Solution
The most stable bond is the strongest bond. The strongest bond has the greatest bond dissociation energy, so to solve this question, the bond energies from Table 1-7 must be referenced. An spt-hybridized carbon is found in the double bond of an alkene. This is described as a vinylic carbon, so the vinyl entry in Table 1-7 is necessary for choices A and B. Considering that we are looking at carbon-carbon bonds, a primary carbon (with only one bond to a carbon) would have to come from a methyl group. This value is necessary for choices A and C. Likewise, a

A. An sp2-carbon to a primary sf -carbon

secondary carbon would come from a group such as ethyl or n-propyl Considering only Et is listed as a substituent in Table 7-7, the value for Et is necessary in choices B, C, and D. Choice A is found by looking at the entry for Vinyl-Me, which is 102 kcal/mole. Choice B is found by looking at the entry for Vinyl-Et, which is 101 kcal/mole. Choice C is found by looking at the entry for Et-Me, which is 89 kcal/mole. Choice D is found by looking at the entry for Et-Et, which is 88 kcal/mole. The most stable bond is the one that requires the greatest energy to break. The greatest bond dissociation energy among these choices is 102 kcal/mole, so choice A is the best answer.

Ionic Bonds Ionic bonds are bonds formed between two oppositely charged ions. They are common between metals and nonmetals. The strength of an ionic bond can be determined using Coulomb's law, Equation 1.2. Coulomb's law states that the force between two charged species is equal to a constant k times the charge on each ion, divided by the square of distance between the two charges, which are
treated as point charges:

p - v t 1z t-^rt

7 q1q2 -4"% c
andto= "
8.85

0.2\

where F = force, q = charge, r = distance,

x rcjP -C*.
N.m2

The greater the charge on the ion, the stronger the bond; and the closer the ions are to one another, the stronger the bond. Ionic bonds are typically stronger than covalent bonds. However, because ions can be solvated in a polar, protic solvent,

ionic bonds are often cleaved more readily than covalent bonds in a protic environment. In other words, despite the strength of ionic bonds, they are easily broken by adding water to the ionic lattice. This implies that the Coulombic attraction of the ions to water is comparable to the attraction of the ions to one
another.

Copyright

by The Berkeley Review

l9

Exclusive MCAT Preparation

Organic Chemistry

Molecular Structure

Intramolecular Features

g
I

Intramolecular Features Intramolecular features encompass anything that affects the stability of a molecule and the sharing of electron density beyond the localized region between two neighboring, bonding atoms. There are various factors that dictate the chemical reactivity of a compound and explain the distribution of electron density within a molecule. I like to call them the "five excuses" to explain organic chemistry. They are reslnence, the inductiae effect, steric interactions, aromnticity, and hybridization. I/y'e have already examined hybridization and seen the effect it has on the structure of a molecule in terms of bond angles. Besides considering the three-dimensional position of the atoms within a molecule, we will consider electron density and thus establish reactive sites within a molecule. We shall
start by considering the ever-so-loved resonance theory. Resonance
Resonance is an intramolecular phenomenon whereby electron density is shifted

|r t a
{':

fi

,!
5

through regions of the molecule via ru-bonds. Resonance is defined as the delocalization of electrons through a continuous array of overlapping p-orbitals (n-bonds and adjacent lone pairs). Resonance theory can be used to determine the stability of a structure. There are three rules to follow to determine the stability of a resonance form prioritized according to importance from most to
least:
1.

#'

fr

The resonance structure should contain atoms with filled octets


(excluding hydrogen).
The best structure minimizes the number of formal charges throughout the molecule. the molecule contains a negative charge, it is best placed on the most electronegative atom. electronegative atom.

2.

3a. if

3b. If the molecule contains a positive charge, it is best placed on the least
Figure 1-20 shows two resonance forms for an amide compound that obey the octet rule, and a resonance hybrid that shows the composite effect. The resonance hybrid is an average of all the major resonance contributors.

ooMore stable

o6t:

.&-

,rr.A*rr.- ,r.4.**r, form


Less stable form

,rr.A'r'?",
Resonance

hybrid

.$r-

Figure 1-20
The resonance structure farthest to the left in Figure 1-20 is more stable than the middle structure, because there is no separation of charge. You must be able to

St

rank the stability of resonance structures and decide whether it is a major contributor. Typical questions based on resonance include determining where certain molecules are most reactive. You should be able to apply resonance theory to other features of chemical structure and reactivity. For instance, when viewing an amide, the electron-rich oxygen is the most nucleophilic site on the molecule. When protonating an amide, it is the oxygen that gets protonated. When amides form hydrogen bonds, the oxygen is the electron-donating site. This has a significant impact on molecular structure in protein folding. 20
The Berkeley Review

or * f, *
I

fl
q fi

Copyright

by The Berkeley Review

fti

Organic Chemistry

Molecular Structure

Intramolecular Features

Figure 1-21 shows four examples of resonance structures and the arrow pushing necessary to convert between resonance forms. To draw resonance structures that are stable, it is often helpful to start with a lone pair and push those electrons into a n-bond. The electrons from the adjacent n-bond turn into a new lone pair.

roi

H,C/t'lv'L'l

H ll l^<-+ \ll I tq.z ts -* g,Qfl*],. HHH


Resonance

(ot

nil,.
s-

:ij:o I to -

r-

\
I

r"

Hvbrid:

6-

/C1.6;2 HsC f
H

Ct

NH

ioi o
I

..

o6ti

6zt-'dl

aoy'c:.ooResonance Hybrid

-* ",P'q8
CH"

CH"

t' t'

CH"

t"

CH"

t"

3r.-t\.,,,
CH"

,.liz't'l3;,
Resonance Hybrid

Hrc?5l,',

{-*

l"

CH"

t"

*rr3-c\c*r,
Figure 1-21

Resonance Hybrid

"lll"'13.,

In Examples 1",2, and 3, shown in Figure 7-2I, the negative charge moves every other atom between resonance forms. The lone pair becomes a n-bond, and the n-bond becomes a lone pair two atoms away. This is true when the number of total charged sites remains constant. In Example 1, there is only one charged atom in each of the resonance forms. You must look for the all-octet resonance structures. A11 of the resonance forms except the carbocations in Example 4 are all-octet resonance forms. This satisfies Rule 1 on the list of resonance rules. Neither structure in Example 4 satisfies the octet rule. The resonance hybrid is a composite of the individual resonance contributors. The most stable resonance structures (major resonance contributors as they are called) have the greatest effect on reactivity and structure for a compound exhibiting resonance.
Copyright
@

by The Berkeley Review

2t

Exclusive MCAT Preparation

Organic Chemistry
Example L.8

Molecular Structure

Intramolecular Features

The C-O bond length is LONGEST in the compound on the:

C. D.

A. left, because nitrogen donates electrons through resonance. B. right, because nitrogen donates electrons through resonance,
left, because nitrogen withdraws electrons through resonance. right, because nitrogen withdraws electrons through resonance.

5-"

*
o

Solution
For this question, the resonance forms of the lactam should be drawn first:

$-"

Partially single bond

6-

The all-octet resonance form on the right has the carbonyl bond in single bond resonance is caused by the donation of a lone pair of electrons by nitrogen. This means that the C-O bond is longer in the compound on the left, because nitrogen donates electrons through resonance. The correct answer is thus choice A. The effect of resonance outweighs the inductive effect. The inductive effect predicts that the nitrogen would be electron-withdrawing,

form. The single-bond

because it is more electronegative than carbon.

Inductive Effect The inductive effect, as the name implies, induces charge separation in a molecule, just as induction in physics refers to the creation of charged sites through induction. From a chemist's perspective, the inductive effect is the delocalization of electrons induced by electronegative atoms. The inductive effect involves the transfer of electron density through the sigma bonds. A highly electronegative atom pulls electron density from its neighbor, which in turn pulls electron density from its neighbor. The effect dissipates over distance, but it can affect bonds between atoms up to three or four atoms away. We most
often consider the inductive effect when a molecule has a halogen. The inductive effect increases with the electronegativity of the atom. Fluorine is the most electronegative atom found in organic molecules, so it pulls electrons

from the carbon to which it is attached in a strong manner. This makes that it in turn pulls electrons from its neighbor. Ultimately, as we see with polarity, the electron density in the molecule is pulled towards the most electronegative atom in the compound. An electronegative atom therefore withdraws electron density and thus can increase a compound's acidity, increase its electrophilicity, decrease its basicity, or decrease its nucleophilicity.
carbon electron-poor, so

Copyright

by The Berkeley Review

22

The Berkeley Review

Organic Chemistry

Molecular Structure

Intramolecular Features

For the relative electronegativity of common atoms in organic chemistry, the followingrelationshipholds: F > O> N > Cl > Br > I > S > C > H. Justrecall "Fonclbrisch" and you'll be in good shape. It may seem strange, but alkyl groups are electron-donating by the inductive effect, because hydrogen is less electronegative than carbon. Figure 1-22 shows an example of the inductive effect as it applies to the nucleophilicity of amines reacting with an alkyl halide.

t:a"*-rt
HH

t.

t. t-.a*l*,
FF

Z-roftate=L44

I Z- -log rate = 4.31

Methylamine

Trifluoromethylamine

Rate for H3CNH2 > rate for F3CNH2 Less nucleophilic due to the electronegative fluorine atoms

Figurel-22 The withdrawal of electron density by the fluorine atoms decreases the nucleophilicity of the amine compound by pulling electrons away from the nitrogen atom. As electron density is removed, the compound becomes electronpoor and thus a worse electron donor. This can be verified by the reaction rate in a substitution reaction. As the negative log of the rate increases, the rate of the
reaction decreases.

Example L.9

\\hich of the following compounds undergoes a nucleophilic substitution


reaction with ethyl chloride at the GREATEST rate?

-{. H3CCHFNH2 B. FH2CCH2NH2 C. H3CCHCINHz


D.
CIHzCCH2NH2

Solution The greatest reaction rate (the fastest reaction) is observed with the best nucleophile. Each answer choice has one halogen, so all the choices have slower rates than ethyl amine. The question asks which experiences the least inductive "-cithdrawal. Chlorine is less electronegative than fluorine (Fonclbrisch), so choices A and B are eliminated. The inductive effect diminishes with distance, so tFre least electron withdrawal is observed with choice D. You must consider both proximity and electronegativity when looking at the inductive effect.

-\lthough not applicable in Example !.9, you must also consider whether the inductive effect involves electron donation or electron withdrawal. For instance, methyl amine is more nucleophilic than ammonia (NHg), because the methyl group is electron-donating. Varying the R-group changes the inductive effect. It
also changes the size of the molecule, so steric hindrance can affect the reaction. For instance, trimethyl amine ((H3C)3N) is less nucleophilic than dimethyl amine (H3C)2NH), because the electron donation by the additional methyl group does not compensate for the increase in molecular size.

Copyright

by The Berkeley Review

23

Exclusive MCAT Preparation

Organic Chemistry
Steric Hindrance

Molecular Structure

Intramolecular Features

Steric hindrance occurs any time two atoms attempt to be in the same place at the sarne time. It is repulsive in nature and increases as the atoms draw closer. No one is certain about the nature of the force, but it is believed to be electron cloud repulsion. The effects are similar to what is observed in general chemistry with VSEPR (valence shell electron pair repulsion) theory, except that it is considered

only when two separate atoms or functional groups interact. Electrons move to be as far apart as possible, so lone pairs and bonds spread out to accommodate the geometry that spaces the greatest distance between electrons. Figure 1-23 demonstrates the effects of steric hindrance on a couple of organic molecules. Because the alkene is planar, the substituents on the alkene carbons have a tendency to collide with one another. H.C
I

/\
'""/

L-

LJ

HsC

H.C / CHs / /\ t \,"/L-L_ \ HsC CHe


I ^-^

Larger C-C-C bond angle

Smaller C-C-C bond angle

HHH H"C CH. \.-./ \' { \/ \. H"C -\__/ CHN


/"t\

H_H H Reduced bond angle

in

dimethylbutene,

because the methyl group hydrogens repel.

Figure 1-23

Example 1.10

Which of the foilowing functional groups is MOST likely to be found in the equatorial position on cyclohexane?

A. B. -OCH3 C. -OCH2CH2CH2CH1 D. -OCH(CH3)CH2CH3


-oC(CH3)3
Solution On cyclohexane, substituents with axial orientation experience greater steric hindrance than substituents with equatorial orientation. Because of steric hindrance, the substituent most likely to assume the equatorial orientation is the bulkiest. The tert-butoxide substituent, choice D, has the most crowded alkyl groups/ resulting in the greatest steric hindrance. This makes the best answer
choice D.

Copyright

by The Berkeley Review

The Berkeley Review

Organic Chemistry

Molecular Structure

Intramolecular Features

Aromaticity Aromaticity is stability generated from having 4n + 2 ru electrons in a continuous, overlapping ring of p-orbitals, where n is any integer including 0. This is known as the Hilckel rule. The stability is rooted in the molecular orbital model, where an energy level is completely filled when there are 4n + 2 n-electrons in the cyclic n-network. Figure 1-24 lists experimental values for the enthalpy of hydrogenation of a series of alkenes. The large deviation associated with benzene is attributed to its aromatic stability.

o#o o#o o#o

ktul/.ol. AH = -2g

ktul/*ol" AH = -56

AH = -54

ktul/-n,"

o#* o
Figure 1-24

LH =

-49

ktul/r',or"

Despite the presence of three n-bonds, the 1,3,5-cyclohexatriene (benzene) yields far less heat from hydrogenation than expected due to its aromatic stability.

The first two entries show that the enthalpy of hydrogenation of an alkene is -28

kcal/mole per n-bond. The third entry shows that conjugation results in
stability, reducing the amount of heat released upon hydrogenation, but only by about 2 kcals/mole. Based on -28 kcal/mole for each n-bond, benzene should be expected to have a AH of approximately -84 kcal/mole. The difference of 35 kcal/mole (84 - 49) cannot be attributed to conjugation alone, hence it is said to be due to aromatic stability.

)'lot all cyclic, conjugated polyenes show such a large deviation from the expected value for the enthalpy of reaction. 1,3,5,7-Cyclooctatetraene (CgHg)
shows a deviation of only 8 kcal/mole from its expected value of -112 kcal/mole.

This implies that conjugation is useful for only a small fraction of the 35 kcal/mole difference observed with benzene between its expected and actual va-lues. Because benzene has 6 n-electrons in a continuous n-cycle, it obeys the
Huckel rule (it has 4n + 2 n-electrons where n = 1), whiIe1",3,5,7-cyclooctatetraene (CsHs) has 8 n-electrons in a continuous n-cycle and does not obey the Hrickel rule. This lack of aromaticity results in a less stable reactant, so more heat is generated in the hydrogenation reaction.

Copyright O by The Berkeley Review

Exclusive MCAT Preparation

Organic Chemistry
Example 1.1L

Molecular Structure

Intramolecular Features

The hydrogen-carbon-hydrogen bond angle about the terminal carbon following alkene is BEST approximated by which of the following values?

in the

H"CH"C /\/ HsC


A. 108.3" B. 711..7" c. 118.5" D. 121.5"
Solution
f-v-v

" '\

/a aH

First we must consider the hybridization of carbon. It is involved in one n-bond, so the hybridization is sp2. The bond angle about an sp2-hybtidized carbon is predicted to be 120". The question heri: is whether the angle is slightly greater or slightly less than 120". Because there are two alkyl groups on the other carbon of the alkene, the electron density repels the electrons in the two carbon-hydrogen bonds. This forces the two C-H bonds closer together, which compresses the hydrogen-carbon-hydrogen bond angle. According to steric repulsion theory, the angle should be slightly less than 120'. The best answer is thus choice C.
Example 1.L2

Which of the following explanations accounts for the pKu of L,3-cyclopentadiene being only 15, while the pKu for hydrogen on other spr-carbons is around 49? A. The strain of the five-membered ring forces the proton off. B. The proton is involved in resonance.

C. D.

The conjugate base is aromatic. The steric hindrance of the sp3-carbon weakens the C-H bond on that carbon.

The acidity of a compound capable of losing a proton ({+) can be determined by the stability of its conjugate base. When an ordinary spr-hybridized carbon (one that is not stabilized by resonance or the inductive effect) is deprotonated, the carbanion that is formed is unstable. Carbon is not electropositive, so it does not readily lose a proton. With 1,3-cyclopentadiene, however, the carbanion that is formed upon deprotonation has both resonance and aromatic stabilization once it loses the proton. The cyclopentadienyl anion that is formed is aromatic. This makes choice C the best choice. The reaction is drawn below:

Solution

HH

H'
Six conjugated n-electrons in a continuous planar arrangement of p-orbitals is aromatic.
Because it includes the word tesonance, choice B may at first seem apPealing. But a proton, having no electron pair, cannot be involved in resonance. Be careful of wording like this, because it is easy to pick resonance without thinking about it.

Copyright @by The Berkeley Review

The Berkeley Review

Organic Chemistry

Molecular Structure

Fundamental Reactivity

dffifitffi:xiRd#Ctffit$

Fundamental Reactions in Organic Chemistry In organic chemistry, perhaps the most common class of reaction is nucleophilic attack. In the simplest sense, a nucleophilic compound (one with an electron-rich site) attacks an electrophilic compound (one with an electron-poor site) to form a new bond. In some instances a leaving group is discarded, while in others a nbond is broken. No matter what the result, the reaction has the same tundamental drive and mechanics. The reactions can be viewed as Lewis acidbase reactions, so organic chemistry starts with a thorough look at Lewis acids and bases. Prior to that, we shall review Bransted-Lowry acid-base chemistry.
Proton Transfer Reactions (Brsnsted-Lowry Acid-Base Reactions) Brsnsted-Lowry acid-base reactions involve the transfer of a proton (H+) from
I,
is

)r

rf n
.e

the acid (defined as the proton-donor) to the base (defined as the protonacceptor). This means that to be a Brsnsted-Lowry acid, the compound must have a hydrogen that can be lost as H+. A hydrogen like this is often referred to as a protic hydrogen or proton. Throughout this section, we will be using the term grotonation to describe the gain of an H+. A hydrogen atom has one proton in its nucleus and one orbiting electron (in a 1s-orbital). when hydrogen loses an electron to become H+, all that remains is a proton. This is to say that H+ is a Flroton, and thus the gain of H+ can be referred to as protonation. Deprotonation is
the loss of H+.

To be a Brsnsted-Lowry base, the compound must have electrons available that can form a bond to H+. Because a lone pair of electrons is necessary to form a bond to the proton, all Bronsted-Lowry bases are also Lewis bases. Figure 1-25 shows a proton-transfer reaction, a one-step reaction.

trt,
..'

Hrc\\Y
H
Base

, a-\Hf-.Cl ' N:' + N:,


Bond forming

H"C

"\o
...

Bond breaking

son;/;iins
Acid
(Proton-donor)

=F-

Hrc'{,:*:, -I formed
H

N-

+ :cl: Bond
broken

..O

"

Conjugate

Conjugate
Base

v
te
re

(Proton-acceptor)

Acid

Figure 1-25

)t
is

it
is

ln the reaction in Figure 1-25, you should note that an arrow going from

a lone

lrt of

pair to an atom becomes a bond in the product, and an arrow going from a bond to an atom becomes a lone pair on that atom in the product. This is a standard convention in drawing mechanisms. The reaction shown in Figure 1-25 is very favorable, as indicated by the asymmetric equilibrium arrow. The favorability is attributed to the fact that HCI is a strong acid. Proton-transfer reactions proceed favorably (AG < 0) from the side with the stronger acid and stronger base to the side with the weaker acid and weaker base. This is to say that a favorable chemical reaction proceeds from the less stable species to the more stable species. There are five strong acids used in organic chemistry that you should recognize: H2SO4, HNO3, HCl, HBr, and HL An important fact to know is that as a Bronsted-Lowry acid gets stronger, it loses a proton more readily, so its conjugate base is less willing to gain a proton. The result is:
\Alhen comparing two conjugate pairs, the pair with the stronger acid has the weaker conjugate base.

Copyright O by The Berkeley Review

Exclusive MCAT Preparation

Organic Chemistry

Molecular Structure

Fundamental Reactivity

Lewis Acid-Base Reactions Lewis acid-base reactions involve the transfer of an electron pair from the base (defined as the electron-pair donor) to the acid (defined as the electron-pair acceptor). This means that for a compound to be a Lewis base, it must have electrons available that can form a bond to an electron deficient atom (such as, but not exclusively, H*). A Lewis acid can have a protic hydrogen, but a Lewis acid may have an empty valence shell capable of accepting electrons. Typical Lewis acids include BF3, AlCl3, FeBr3, and SOCI2. Figure 1-26 shows a Lewis acid-base reaction, where ammonia is the Lewis base and BF3 is the Lewis acid.

T I n fl il
,t d q
il

H.C

Bond forming

"'\o
H^C

ril

.N:

eti:'

E:

l[

tr!

H,CV
H

"r."'.?'13,"#;\.. H .F:
Acid
rrrt

Base

(Electron Pair Donor) (Electron Pair Acceptor)

Figure

1,-26

4
.&.
4&

The role of a base is essentially the same in both the Lewis and Bronsted-Lowry definitions. A base donates a lone pair of electrons to form a bond to an acid, whether the acid is a Brsnsted-Lowry acid or a Lewis acid. In organic chemistry, the terminology varies, and Lewis bases are frequently referred to as nucleophiles. Nucleophile means "nucleus loving", which implies that nucleophiles seek out positively charged sites (referred to as electrophiles). The simple guide to organic chemistry is that negative charges seek out and bond to positive charges.

&t.

Acidity Acidity is defined by three definitions: the Arrhenius definition, the BrsnstedLowry definition, and the Lewis definition. The Arrhenius definition is that an acid yields HgO* when added to water. The Bronsted-Lowry definition is that an acid is a proton (H+) donor. The Lewis definition is that an acid is an electron pair acceptor. The strength of an acid depends on the effects of intramolecular forces on the bond to the acidic proton. These are the electronic forces within a molecule. They are responsible for the distribution of valance electrons, which accounts for the chemical behavior (such as acidity) of the molecule. An acid is stronger when an electron-withdrawing group is attached to the backbone of the acid, because the molecule is electron-poor, and thus a better electron-pair acceptor. An acid is weaker when an electron-donating group is attached to its backbone, because the molecule is electron-rich, and thus a wolse electron-pair acceptor. The primary task associated with evaluating organic acid strength is to decide which groups are electron-donating and which are electron-withdrawing.
Figure 1-27 shows some common organic acids and their pKu values. OH

PKu

=10

.A';il"
Carboxylic acid
Phenol

o*'=e-10
H

PKa = 15

o-*Vt
Alkyl ammonium cation

R-

OH

|x:=17'20
Carbonyl cr-proton

Alkoxide

Figure L-27

Copyright Q by The Berkeley Review

The Berkeley Review

.ffii;'...:

3
lSE

Organic Chemistry

Molecular Structure

Fundamental Reactivity

air
rve
?S,

vis cal
r'\i

iS

acidity. Basicity is also defined by three definitions: the Arrhenius definition, th-e Bronsted-Lowry definition, and the Lewis definition. The Arrhenius definition is that a bas-e vields OH- when added to water. The Brsnsted-Lowry definition is that a base is a proton (H+) acceptor. The Lewis definition is that a base is an electron-pair donor. The rules that you use for acidity can be applied to basicity, but with the opposite effect. Electron-donating groups increase basicity (while they decrease acidity) and electron-withdrawing groups decrease the basicity. As a result, the strength of a base or acid can be determined from the stability of its conjugate. The more stable the conjugate, the weaker the conjugate and the strongei the respective compound (either acid or base). Figure 1-28 shows some common
organic bases.

Basicity Basicity is most easily thought of as the opposite of

-/\ ROIry

o Jl prn =e-71

Carboxylate

Phenoxide

=4
PKa= 4
PKu = -1

PKu=-6--a

o/*Y,'
H

R-O-:
Alkoxide

RACH,Anionic g-carbon

Alkyl amine

id,
ry,
Ies.

Figure 1-28 AJthough you are not required to memorize exact pK6 values, it is a good idea to know the "-5-10-75-20 general rule" for organic acids. The pKn for i carboxylic acid is about 5, for a phenol it's about 10, for an alcohol it's about 15, and for a proton alpha to a carbonyl it's about 20. These are close enough for good

)ut to
es.

guessing.
3d-

an hat :on

Example 1.13

ilar

na
ich

lis
the rair

tr\hat is the pK6 for p-nitrophenoi and the pK6 for its conjugate base? A. O2NC6H4OH has pKa = 7.2; O2NC6H4O- has pKU = t2.8 B. O2NC6HaOH has pKa = 11.6; O2NC6H4O- has pKU = 8.4 C. O2NC6HaOH has pKa = 7.2; O2NC6H4O- has pKb = 6.8 D. O2NC6H4OH has pKa = 11.6; O2NC6H4O- has pKb=2.4

Solution

its
rair

;to
ng.

-20

nitro group is electron-withdrawing, which makes nitrophenol a stronger acid than phenol. As an acid becomes stronger, its pKu value decreur"r. Thit means that the pKu for nitrophenol is likely tobe7.2 rather than 11.6. This eliminates choices B and D. The pKu and pK6 sum to L4, so choice C is the best answer. This question could also be answered from a base perspective. The nitro group is electron-withdrawing, which makes nitrophenoxide a weaker base than phenoxide. As a base becomes weaker, its pK5 value increases. This means that T" prr, for nitrophenoxide is greater than 4,0. This eliminates choice D. Again, the pKu and pK5 sum to 14, so choice C is the best answer. pick C andlain rncredible satisfaction doing what you should do.
The Pick the perspective (acid or base perspective) that works best for you, and use rvith these questions.

it

rew

Copyright O by The Berkeley Review

Exclusive MCAT Preparation

Organic Chemistry

Molecular Structure

Fundamental Reactivity

Choosing Where to Protonate and Deprotonate

A common question in organic acid-base chemistry is, "Which is more acidic?" This question can refer to two separate compounds, or two sites within the same
compound. There is also the complementary question, "Which is more basic?" In Figure I-29,we are dealing with two sites on the same molecule, so the question is, "Which lone pair on the molecule is most basic?" It shows the two possible products when acetic acid (CH3CO2H) is treated with a strong acid. Acetic acid (a carboxylic acid) has two sites with lone pairs (two oxygen atoms). Deciding which oxygen gets protonated (which forms a more stable protonated species) requires thai you consider many different factors, including the resonance stability associated with each protonated product. Resonance can help to stabilize the excess positive charge on the compound, and thus make the species
more stable.
protonation

o*ysJl';"' !/) <+ uuc-h


\i6-11
o:
<-'>

at

oxygen

^r,o} tt

f H3c-c
\

oxygen u :p-

oxygena

H.C"\

[. // i9-11
Protonation at Oxygen b

All-octet resonance form, #aking this the more stable product.

.g^-H

Oxygenb

H1C-u "

//)

\^ Oxygen t !g-- g /
H

H"C- CO " \^ Oxygen bi6;'/


H

:'6:o /

11

Non-octet resonance form with separation of charge, making this the less stable product. Figure 1-29
Because there is resonance stabilization when the compound is protonated at Oxygen a, and there is no resonance stabilization when the compound is protonated at Oxygen b, it can be concluded that carboxylic acids are protonated at the carboxyl oxygen (C=O), not the hydroxyl oxygen (OH). This means that the carbonyl oxygen is more basic than the alcohol oxygen. In reality, we rarely see carboxylic acids acting as bases, but amides are similar in structure, and it is critical in biochemistry that we know where they are protonated and thus where

they exhibit hydrogen-bonding. Example 1.14 Which of the following statements BEST explains why amides are protonated at oxygen rather than nitrogen? A. The oxygen is less electronegative than nitrogen, so it donates electrons more

B.
C. D.
Copyright
@

readily" The oxygen is larger than nitrogen, so its electron cloud attracts protons more

readily. Oxygen carries a partial positive charge due to resonance withdrawal from the nitrogen. Oxygen caries a partial negative charge due to resonance donation from the nitrogen.

by The Berkeley Review

30

The Berkeley Review

Organic Chemistry

Molecular Structure

Fundamental Reactivity

Solution As shown below, the nitrogen of the amide donates electron density to the carbonyl oxygen through resonance. This places a partial negative charge on oxygen (increasing its basicity), and a partial positive charge on nitrogen 'decreasing its basicity). Choice A is eliminated, because oxygen is more electronegative than nitrogen. Choice B is eliminated, because oxygen is smaller Jran nitrogen, not iarger. Because of resonance, oxygen carries a partial negative :harge, while nitrogen carries a partial positive charge. This means that choice C -: false and choice D is true. This explains the basicity of amides.
:

ll-l

' *ot"

basic due to negative charge'

^Ar,- NHz R
{cid Strength Factors

Aa -NHzlessbasicduetoabsenceofelec{rons. R-

',-cic proton. The weaker the bond to the acidic proton, the more readily it :::aks, and consequently more acidic the acid. With acids, the bond breaks in a --:rerolytic fashion, forming ions. Primary effects include size, electronegativity, .:.i hybridization of the atom directly attached to the acidic proton. Secondary ,.-.;ls depencl on the effect of the molecule on the atom bonded to the acidic :.,rtor-r. The more electron-rich that atom, the less acidic the proton. The more = .:tron-poor that atom, the more acidic the proton. secondary effects include -:Surflorc (the delocalization of electrons within a molecule through ru-bonds), -: - nductive effect (the delocalization of electrons within a molecule through o: -::.1s), and electron cloud repulsion (deformations of a molecule and elongation :: londs within the molecule). There is also aromaticity to consider, but that will := :-Cdressed as a special case. Secondary effects involve intramolecular forces,

affecting the strength of an acid or base can be broken down into primary =:iects and secondary effects. Primary effects depend on the bond directly to the
- ,'-ctors

it
;d

-.-:h dictate where the electron density within a molecule lies, and thus they ,:,::atethereactivityof acompound. weshalllookathowthesefeaturesaffect --- electron distribution within a molecule, starting with the primary effects. l,:r-e 1-B shows common acids and bases in organic chemistry, listed according - :elative strength (in both the acids and the bases).
Strong Acids
H2SOa >

rt
)'
rs :e

HI > HBr > HCI > HNO3

i\-eak Acids
: trong Bases

HF > HCO2H > H3CCO2H > H2CO3 > H3CSH > H3CNH3CI H5C6OH > H3COH CH3(CH2)3Li > NaNH2 > KH > NaOCH2CH3 > NaOH = KOH H3CNH2 > NaHCO3 > H3CCO2Na > HCO2Na > H3COH Table 1-8

l\'eak Bases
at

Fimary Acid Strength Factors : ,::-.arv factors directly affect the strength of the bond to the protic hydrogen.
--. ,'. i.'ond weakens (homolytic and heterolytic bond dissociation energy lessens), -. j strength increases and conjugate base strength decreases. The three primary :::ors to consider are atomic size (when comparing acids involving atoms -:rin the same column of the periodic table), electronegativity (when - --.raring acids involving atoms within the same row of the periodic table), and ' ::r;lization (when comparing acids where hydrogen is on the same atom).
,

:,, right

by The Berkeley Review

3l

Exclusive MCAT Preparation

Organic Chemistry

Molecular Structure

Fundamental Reactivity

Strcngth Dependence on Atomic Size When comparing acids within a column of the periodic table, the strength of the acid is dictated by the size of the atom to which hydrogen is bonded. Smaller atoms form shorter bonds to hydrogen than their larger counterparts. Given that longer bonds are weaker bonds (in both a homolytic and heterolytic fashion), the acidity of a compound directly depends on the length and strength of the bond' A significant difference in atomic size ultimately determines the relative strength of acids, because larger atoms make stronger acids. A prime example of this involves proton exchange (acid-base) chemistry between thiols and alcohols, where thiols are stronger acids than alcohols. This is because

sulfur (found in a thiol) is a larger atom than oxygen (found in an alcohol). in acid-base chemistry, you may also consider the reactivity from the base perspective. To apply this atomic size theory to bases, compare the distribution of electron density about a small atom velsus a larger atom. A conjugate base is more stable if electron density is spread out over more space. Because larger atoms stabilize negative charge more readily, they are not as reactive, and thus not as basic. This is often referred as polarizability. Il you keep in mind that the more the negative charge is spread out,.the harder it is for a proton to find the negative charge, then you see that the compound is not as basic. The thiolalcohol example is shown in Figure 1-30.

HH
PKa = 10'4

,AHS

\r
CH:
PKu = 15.7

HO

CHg

Stronger Acid

Weaker Acid

Figure 1-30 You should understand that atomic size (polarizability of the conjugate base) can be applied only when the protic hydrogen is directly bonded to the larger atom. The atomic size argument is used to explain relative acidity for haloacids. Relative haloacid strength for haloacids is: HI > HBr > HCI > HF, confirming that size is more important than electronegativity for elements within the same column of the periodic table. This contradicts what you would expect if you were to apply the rules of electronegativity. The relative strength in the case of haloacids is dictated by atomic size of the halogen. As mentioned before, it is because the bond is longest between hydrogen and the largest halogen, thus making it the weakest and most readily broken hydrogen-to-halogen bond. It can also be considered that the conjugate base (halide) is more stable as it gets larger, because of the greater space over which the negative charge is distributed.

In the halogen case, the less concentrated (more diffuse) negative charge on iodide (i-) is not as readily shared as the negative charge on bromide (Br-), chloride (Cl-), or fluoride (F-). The weaker the conjugate base, the stronger the conjugate acid, This is to say that because I- is the most stable anion of the halides, it is the weakest base of the halides. Therefore, HI (the conjugate acid of I-) is the strongest of the haloacids. This theory is also applied when looking at the relative reactivity of halogen containing organic compounds in reactions where the halide is a leaving gfouP. The relative acidity of haloacids should be familiar to you. The mathematics of acidity and basicity is important in general chemistry, but we shall consider only values such as pK6 to compare the relative strengths of acids. In organic chemistry, by quantifying acidity with pKa values,
we can support or disprove relationships.

Copyright

by The Berkeley Review

The Berkeley Review

Organic Chemistry
Str ength D ep en dence on El ectr oneg atia
e

Molecular Structure

Fundamental Reactivity

ity

r
rt

e
t.

I
R

Within a row of the periodic table, the strength of the acid is dictated by the electronegativity of the atom to which hydrogen is bonded. Given that atoms within the same row of the periodic table are approximately equal in size, the bond strength depends more on the distribution of electrons within the bond than it does on the length of the bonds. Because the electronegativity of the atom bonded to hydrogen affects the distribution of electron density within a bond, the electronegativity of the atom directly bonded to the acidic proton dictates the
acidity of the compound. The relationship is reasonable in that as an atom bonded to hydrogen has a greater electronegativity, the electrons are pul1ed more towards that atom, rather than toward hydrogen. This allows the bond to be cleaved in a heterolytic fashion rather easily. The result is that the more electronegative the atom, the
stronger the acid.

e n e n

r
s
e e
t-

Comparing electronegativity applies only for hydrogens bonded to atoms in the same row of the periodic table, not the same column of the periodic table.

The greater acidity associated with alcohols than amines of equal alkyl substitution can be explained by invoking the fact that the electronegativity of oxygen is greater than that of nitrogen and that oxygen and nitrogen have roughly equivalent atomic sizes. Figure L-31 shows the relative acidity when comparing a hydrogen bonded to fluorine, oxygen/ nitrogen, and carbon. Because fluorine is more electronegative than oxygen, which is in turn more electronegative than nitrogen, the following relationship holds true:

HH
H-F
n
L

Strongest Acid
PKu = 3'2

i.
rt

.4. CHs HO Amphoteric pKa = t57

\r

HH

\l

HH
CHg H

\r
CH:
PKo= 49

HzN
pKa = 33

Very Weak Acid

Weakest Acid

tl

Figure 1-31
The same rationale used to explain relative acidity in Figure 1-31 can be used to erplain relative basicity in the conjugate bases amines, alcohols, and hydrofluoric add. Because nitlogen is less electronegative than oxygen and fluorine, it more rcadiiy donates an electron pair to a proton, so the trend in basicity shown in Figure 1-32 holds true:

s
S

tt

s
t.

n
,),

ui.io
ir<o =

-rl

cH.,

iro

= -r.s

PKu

= 10'8

e
rf

Figure L-32 Amides (RZN-) are some of the strong bases that are used in organic chemistry. They are used to remove alpha protons (the protons bond to the carbon that is alpha to a carbonyl) and the terminal hydrogen of an alkyne, both with pKu
values above 17.

It

s
e

il
e
s/

Copyright

by The Berkeley Review

.'.'

Exclusive MCAT Preparation

Organic Chemistry
Strength
D

Molecular Structure
ependence on

Fundamental Reactivity

o
>e:

Hybridization

The hybridization of an atom affects the bond length and the distribution of electron density within a bond, so the hybridization of an atom directly bonded to the acidic proton affects the acidity of the compound. The relationship is not obvious in that it is not true that longer bonds lead to stronger acids, as is the case with most other acids. In fact, the relationship between length and acid strength is exactly the opposite. As the hybrid orbital gets smaller, the electrons are held closer to the nucleus of the atom bonded to hydrogen, so the bond can be cleaved in a heterolytic fashion more easily.
The result is that the more s-character in the hybrid orbital of the atom bonded to hydrogen, the stronger the acid,

ir":

::r
:

:r:-

:a
nt-:,:
r"ai:r:

:*ii l

*C*,

This results in the relative acidity being sp > sp2 > sp3. It is most commonly observed with carbon acidity, but can also be observed with nitrogen and oxygen. Figure 1-33 shows the comparison of acids where hybridization explains the difference in acid strength.

--. re5

-*,

:,*rl"

-HCH" sPL pKa=26 H- C= C- CH3


Stronger Acid

/ \ ^CrC: Ca ' CHs PKo=36 H


spr

Weaker Acid

L*

-{
oK"=26 H
I

: ::-

PKo=33 H
SP"

^"\r'cH3 SP.N
I

H
Stronger Acid

rr;N--.cu, lzHH
Weaker Acid

tir
il-ril:

Figure 1-33

l:.
i:.:. :-?;
STRONGEST
* i

Example 1.15

A compound with which of the following atoms would be the


base?

-!..-

A. B. C. D.

A weakly electronegative atom carrying a negative charge A highly electronegative atom carrying a negative charge A weakly electronegative atom carrying no charge A highly electronegative atom carrying no charge

A base is a compound that donates electrons. The best electron-donating group would be an atom that readily shares its electron density, so the atom should not be very electronegative. The stronger base carries a negative charge, rather than no charge, so it can more easily donate electrons to an H+. The best answer is
choice A,

Copvright O by The Berkeley Review

34

The Berkeley Review

Organic Chemistry

Molecular Structure

Flrndamental Keactivity

Secondary Acid Strength Factors Secondary factors indirectly affect the strength of the bond to the protic hydrogen. Secondary effects involve the electronic environment of the atom bonded to the protic hydrogen. Simply put: If the atom has electron density donated to it from the rest of the molecule, it will not pull away electrons,from the bond to hydrogen as readily, so the compound is less acidic. if the atom has electron density withdrawn from it by the rest of the molecule, it will pull away electrons from the bond to hydrogen more readily, so the compound is more acidic. Secondary factors to consider for now are resonance and the inductive effect. Resonance is a more significant factor than the inductive effect. Strength Dependence on Resonance Resonance, being ihe delocalization of electrons through an overlapping string of

;-bonds, can be either an electron donating effect or an electron-withdrawing

effect. It is important to distinguish between electron-donating groups and


electron-withdrawing groups, because they affect acidity in opposite ways. When looking at resonance, an adjacent atom making while an adjacent atom with a lone pair is electron-donating.
a n-bond is electron-withdrawing,

Trpical examples of organic acids that are affected by resonance include phenols and carboxylic acids. Figure 1-34 shows the resonance effect of an electrondonating group (OCH3) on a carboxylic acid. It decreases the acidity, raising the pKu only when it can delocalize n-electrons.
!\ ith no n-bond, there can be no donation
*uough resonance

oH,
Figure 1-34

uuco:

FoH

Methoxy lone pair can donate to acid group through the n-bond

Pf"=56

The methoxy substituent (OCH3) has a lone pair of electrons on the first atom adjacent to the carboxylic acid. The oxygen atom of the methoxy group donates eleckon density to the system, and thereby decreases the acidity of the carboxylic acid. An electron withdrawing substituent has the opposite effect, because it grakes hydrogen electron deficient, and thus more protic. Electron-withdrawing substituents increase the acidity of a compound, as shown in Figure 1-35.

".$.{'"o_Q
o*Jo o
Figure 1-35

pKa=10.0

It is important to know that electron-withdrawing groups make the compound a 5etter electron-pair acceptor (Lewis acid), so the acidity increases with electron-.'.ithdrawing substituents. Electron-donating groups make the compound a lt orse electron-pair acceptor, so the acidity decreases with electron-donating substituents. This holds true with all effects (not just resonance). It is easy to get caught up in the idea that any resonance makes a compound more stable, but this l-c not always true. Use the electron-donating or electron-withdrawing ;haracteristics to determine the effect on the acidity or basicity.
Copyright
@

by The Berkeley Review

t5

Exclusive MCAT Preparation

Organic Chemistry

Molecular Structure

Fundamental Reactivity

Strength Dependence on the lnductioe Effect The inductive effect is the delocalization of electrons through the sigma bonds of

molecules. It withdraws electron density from the carbon to which it is attached, which makes that carbon electron-poor. As a result, the electron-poor carbon pulls electron density from its neighbor. Ultimately the electron density in the molecule (as with polarity) is pulled towards the most electronegative atom and away from the other side of the molecule. Hydrogen is often at the other end of the molecule, so it becomes electron-deficient and becomes a better electron-pair acceptor (more acidic). An electronegative atom therefore withdraws electron denslty and thus increases an acidic compound's acidity or decreases a basic compound's basicity. For common atoms in organic chemistry, the order of electronegativityyou should recallis: F > O > N > Cl > Br > I > S > C > H' This helps you predict the relative acidity due to the inductive effect. The carboxylic acids in Figure 1-36 show the effect of electronegativity.

a molecule. Fluorine is the most electronegative atom found in organic

Noz

t
HH
PKo =2.59

cl>
HH pKo = 2.88

HO

oo ,"\"^o\,
HH

HH
PKa = 1'58

PKa=2.92

HH PKa = 3.17
1"-36

HH
PKa= 4.74

Figure

The inductive effect depends not only on the electronegativity of the withdrawing atoms, but also on their proximity. In Figure 1.-37, the proximity of
the chlorine atom to the acidic group affects the acidity of the carboxylic acid. The effect on the compound's acidity drops with distance and is negligible when they are over four atoms apart. The closer the electron-withdrawing substituent to the acid functionality, the greater the increase in the acidity.

CHg

HO

CHe

HO

cH2cl
HH
PKa = 4'51

Ho

Cl

HH
PKa=
4'07

HH
PKa = 4'82

PKa = 2'88

Figure 1-37

It may seem strange, but alkyl groups are electron-donating. This is attributed to the fact that hydrogen is less electronegative than carbon. When the substituent is electron-donating, the inductive effect decreases the acidity. As a result, alkyl groups reduce thJ acidity of a compound. In Figure L-38, the compounds become less acidic due to the electron-donating methyl group'

HoA.r '
PKa = 3'78

,to
H]
PKa = 434

'no\'"nu
HH
PKa = 4'82

Figure 1-38

Copyright

by The Berkeley Review

The Berkeley Review

itv
;of
ni.c

Organic Chemistry

Molecular Structure

Fundamental Reactivity

ed,
ron

Values, Terminology, and Applications Conceptuai questions in acid-base chemistry can often be reworded into: "\zVhich acid is stronger?" The answer can tell you the sign of values and the nature of a leaction. The following terms are ways in which an increase in a compound's acidity can be observed. As a compound becomes more acidic: acid dissociation increases :Ku of the acid decreases pH of a 1.0 M solution decreases for reaction with a base decreases conjugate base strength decreases -\G fKA of the conjugate base increases the stability of the conjugate base increases
Ku of the acid increases

the

rrd

lof
rair ron

sic of his rlic

.H

ou must also be able to use the qualitative concepts. A proton transfer reaction ;"rcs favorably from the side with the stronger acid to the side with the weaker :cid; a favorable reaction proceeds from stronger acid to weaker acid. To jetermine the K"O for a proton-transfer reaction, first decide which of the acids :l conjugate bases) is stronger. From there, it is a matter of determining the ;rection of the equilibrium. If the equilibrium lies in favor of the reactants, the K*" < 1. If the equilibrium lies in favor of the products, the K"q t 1.

Example 1.16 r\irich of the following compounds is the STRONGEST acid?

-4- H3CCH2OH
the

C D.

m.

H3CCO2H F3CCH2OH F3CCO2H

'of
id.
S'olution
len
ent

::n
H" J

-::boxylic acids are stronger acids than alcohols, due to the withdrawing of .!e;ton density by the carbonyl oxygen through resonance. This eliminates ::--..ices A and C. Fluorine is highly electronegative, so it withdraws electrons

the acidic hydrogen via the inductive effect, thus increasing the acidity. ::,e skongest acid is the carboxylic acid with fluorines attached, choice D.
Etample 1.17

i: --r,' can the difference in aciditv r-: be explained?

between trifluoroacetic acid and trichloroacetic

".4. Fluorine is larger than chlorine, so trifluoroacetic acid is a stronger acid. $" Chlorine is larger than fluorine, so trichloroacetic acid is a stronger acid.

Ito
ent

C lirichloroacetic acid is a stronger acid,


eleckone gative than fluorine.

because chlorine

is

more

kyl
rds

D. lirifluoroacetic acid is a stronger acid, because fluorine is more


e.iectronegative than chlorine.

iolution
F:-rcrine is smaller than chlorine, so choice A is eliminated. The acidic proton is :rar :,onded to the halogen, so atomic size does not dictate the acid strength. This

*:::nates choice B. Fluorine is more electronegative than chlorine, so F


,r-'.:-i.lrarvs electron density more than

Cl, making trifluoroacetic acid more *!"-:Ton poor and a stronger acid (better electron pair acceptor). The pKu of :'iuoroacetic acid (F3CCO2H) is 0.18, while the pK3 of trichloroacetic acid i-irCCQH) is 0.64. A lower pKu value confirms that trifluoroacetic acid F:CCO2H) is the stronger of the two acids. Choice D is the best answer.
rew

J:rrr, right O by The Berkeley Review

Exclusive MCAT Preparation

Organic Chemistry
Example 1.18

Molecular Structure

Flrndamental Reactivity

Which of the following statements is true

A.
B.
C. D.

it pertains to pKu values? A functional group that is electron-withdrawing by resonance lowers the pKu value, while a functional group that is electron-withdrawing by the inductive effect raises the pKu value. A functional group that is electron-withdrawing by resonance lowers the pKu value, while a functional group that is electron-donating by the inductive effect raises the pKu value. A functional group that is electron-donating by resonance lowers the pKu value, while a functional group that is electron-withdrawing by the inductive effect raises the pKu value. A functional group that is electron-donating by resonance lowers the pKu value, while a functional group that is electron-donating by the inductive effect raises the pKu value.
as

Solution
Regardless of the effect (whether it is resonance or the inductive effect), electronwithdrawing groups increase acidity and lower the pKu while electron-donating groups decrease acidity and raise the pKu. This eliminates choices A, C, and D and leaves choice B as the best answer. The effect of an electron-withdrawing group can be seen in the following trend: H3CCO2H (pKa = 4.74) is less acid than H2CICCO2H (pKa = 2.85), which is less acidic than HCI2CCO2H (pKa = 1.26), which in turn is less acidic than CI3CCO2H (pKu = 0.64). As the amount of electron-withdrawal increases, the acidity increases. Example 1.19

Which nitrogen atom in the following molecule is the MOST basic?

b d

NHz

NHz
A. B. C. D.
Nitrogen a Nitrogen b Nitrogen c Nitrogen d

Solution
The most basic nitrogen atom is the one most capable of sharing its lone pair with a proton or electrophile, which means that the nitrogen where the lone pair is

least shared within the molecule is the most basic. Nitrogens a and b have reduced basicity, because the electron pair on nitrogen is being donated to the aromatic ring through resonance. Nitrogen d has reduced basicity, because the electron pair is being donated to the carbonyl group through resonance. Only Nitrogen c is free to share its electrons (which are not being delocalized

""*^"r" -t,^t^ -" *

Copyright

by The Berkeley Review

5A

The Berkeley Review

ty

Organic Chemistry

Molecular Structure

F-undamental Reactivity

Example 1.20 l\hat is observed when histidine is protonated on its side chain?
le
te

A. B.

The imine nitrogen gets protonated, because the amine nitrogen.

it is more electronegative than

The imine nitrogen gets protonated, because it is less electronegative than the

le

le
\a
IE

C. D.

\a /e

amine nitrogen. The amine nitrogen gets protonated, because the imine nitrogen shares its lone pair of electrons with the n-bonds in the ring through resonance/ thus it cannot be protonated. The imine nitrogen gets protonated, because the amine nitrogen shares its lone pair of electrons with the n-bonds in the ring through resonance, so it cannot be protonated.

nrg

D
r8
an

6),

of

Solution tsoth nitrogen atoms are equally electronegative, because neither carries a charge. This eliminates choices A and B. One could argue that having different :"'bridization makes the electronegativity different, but the goal here is to find -ie best answer, and if it is necessary to stretch the definition of terms, you're :etter off finding a better answer choice. The histidine is protonated on the imine :rtuogen, because the lone pair on the amine nitrogen is being shared with the :;clic rc-system through resonance. This makes the ring aromatic, so the lone :air on the amine nitrogen is not available to be donated as a base. The best ins\{,er is choice D, as shown below.

o
HeN

oor

H.N

4;,
\:'
lmine nitrogen

CH,,

t-

H
H*

CH"
H

->
@

r\-j
the n-system remains aromatic.
@

Ax )^,J

R"ronur,ce is still possibie, so

t'*#,r'
Oil
th
is
ve
he he
Lly

HsN

Ir--CH?

7;;" \
NT.I\-

l-H

CH?

7 Amine nitrogen

1x,^
\Jz

ed

No resonance is possible, so the n-system is no longer aromatic.

:w

Copvright

by The Berkeley Review

Exclusive MCAT Preparation

Organic Chemistry

Molecular Structure

Fundamental Reactivity

Summary of Acids and Bases Equation

In acid-base chemistry, it is impossible to avoid equations, aithough general


chemistry typically involves more math than organic chemistry. Some equations define concepts, while others help in caiculations. Equations 1.3 through 1.10 are defining equations, and each should be tattooed in your memory bank:

hu6
-{llrt

hfr

tu;
miln$mfr

iMM
sMl

pH = -log[H3O+]
pOH = -log [OH-]
PKa = -1og Ku
PKb = -1og K6

(1.3) (1.5)
(1".7')

[HaO+] = 10-PH

(1.4) (1.6) (1.8) (1.10)

pH + pOH = 14 (at

(1.e)

25"C) Ku = 16-PKu pKa + pKb = 74 (at25"C)

,m$Mfl

,rm& dM d!

Given pKu values, the AG" and K"O for a proton transfer-reaction can be calculated. Listed below are four equations that you should be able to work with. Equation 1.11 is the Henderson-Hasselbalch equation. It is used to determine the pH of buffered solutions and is derived from the fundamental acid dissociation reaction. Equations 1.11 and 1.13 are common, and should be
committed to memory:

ry
ffi

mhq$h'

ffi ffi@
mmm

ilbttrrd

dQu

pH=pKu+logl{l 1o(Pu-pKu;- [A-]


IHA]

(1.11)

tHA]
(1.12\

ry'F ffi flmr


ffi

ForHA + H2O

-:i+

H3O+ +

A- ro=[H:-O*]-[A]
tHAI

(1.13)

Keq = 10Pkn(product acid) - PKa(reactant acid)

(1.1,4)

Equation 1.14 is derived from the fundamental reaction:

w r4 ry q0
ffi@

'm

HA + BBy definition: Ka(HA)

A- + HB, wirere

K". = [HB][A-]
lHAttB-t

= 10-PKa(HA) and Ku1Hs,=

t"i#it
'
x

= 1o-pKa(HB)

Using equilibrium:

,, =#F tHBltA-l
tHAltB-l

tA l tHAl

lHBl =[H3o+][A]

tB-l

irll
lH3o+llB-l
nffinW

tHAl

= Ka(HA) x
Thus:

1 - Ka(HA) Ka(HB) Ka(HB)

d
Ka(HB)
1g-PKa(HB)

Ko. =Ka(HA) = lO-pKa(HA) = 1g-pKa(HA) * 16+praluB) = 16(pKoen)-pKa111a))


HB is the product acid and HA is the reactant acid, so by strbstitution we derive Equation 1.13:
Keq = 1gpKa(product acid) - pKa(reactant acid)

rillld

rnffi M

ffimr

il@iifrfl'fr

Copyright O by The Berkeley Review


i#t,, i$ii:r

The Berkeley Review

1
'al
NS

Organic Chemistry

Molecular Structure

Fundamental Reactivity

re

,4) ,6) ,8)

Reaction Types Lr organic chemistry, there are a few fundamental reactions that describe the najority of reactivity in organic chemistry. In addition to acid-base chemistry, there are substitution and addition reactions. All of these reactions involve the aCdition of an electron pair to an electron-poor site. If you are able to identify the :lectron pair that can be shared and the electron-poor site within the reactants, '"-ou will be successful at predicting chemical reactivity. Addition and :ubstitution reactions are often further classified according to their nucleophile ,rl eiectrophile, but for now, we shall consider only a generic addition reaction to ::,iroduce reaction pathways, mechanistic logic, transition states, intermediates,

'nd energy diagrams.


Electrophilic Addition Reactions ;-ectrophilic addition reactions are common irr alkene chemistry. The alkene ::cleophile donates its most available electron pair (the electrons of the n-bond) :r an electrophile. The reaction involves a strong acid, often a haloacid, as the =-ectrophiie. If the alkene is asymmetric, then you need to consider orientation :.:tors in the reaction in terms of minimizing steric hindrance in the transition ':ate and stability of the intermediate. When looking at an alkene, the term .;:'-.:,tnrctric refers to a state where the two carbons have an unequal types of :-'-:bons bonded to each. When the reactant alkene is asymmetric, the reaction -:,-ion's Markovnikov addition rules. The electrophilic substituent adds to the .=ss hindered carbon of the n-bond, according to the rules associated with l, f:rkovnikov addition. \lechanisms (Reaction Pathways) l,le,chanisms are the step-by-step account of all species thought to exist as :=--tants proceed to form products. Mechanisms are commonly referred to as ':-;--;irrr pathzoays. Mechanisms focus on intermediates, propose transition states, ::rl break reactions into steps. Few reactions in organic chemistry occur in one : -:. so mechanisms often involve many steps before reaching a product. You :-.=.,' recall from general chemistry that the rate of a reaction depends on the rate-

0) be

rk
to

id
be

4)

:=:ermining step. Whenever there is more than one step in a reaction :re;hanism, there exists a rate-determining step. -l:: mechanism for a reaction is proposed based on the products that are formed, : :;-,' fast the reaction proceeds, and which changes in conditions alter the rate. It :::. be supported by monitoring isotopically labeled atoms on a molecule. A :.=:hanism can never be proven, only disproved. Because reaction data are ::=ed on the rate-determining steps, rate data are critical in supporting a :ri:-hanism. Mechanisms are the most probable pathway that accounts for the -::a presented. Most reactions in organic chemistry involve a nucleophile r-:-;krng an electrophile, which is often referred to as nucleophilic substitution. ie:-.re any mechanistic studies are encountered, be sure that you have a strong ;35p of the definitions of the terms. lL= reason for thoroughly addressing a reaction using arrow-pushing technique r. :o learn themes in reactions that are repeated in other reactions. In other ,,'.':ls, understanding mechanisms allows you to predict products of unknown :=::tlons. For instance, if a carbocation is formed during one reaction :,:iranism, an analogous reaction is likely to form a carbocation as well. From --:*. perspective, almost all organic chemistry reactions can be classified as Lewis l::i-base reactions. These include all reactions that involve a nucleophile :::"cking an electrophiie.

:w

- irlright O bv The Berkeiey Review

Exclusive MCAT Preparation

Organic Chemistry
Example L.21

Molecular Structure

Fundamental Reactivity

A carbocation can be classified A. an intermediate. B. a transition state. C. a product.

as:

D.

a catalyst.

Solution A carbocation is an intermediate, because it is not stable enough to be present at the completion of a reaction, eliminating choice C. It is not present at the start of a reaction, and it does not form an activation energy-lowering complex during the reaction, so it is not a catalyst. This eliminates choice D. It is present for a finite period of time, so it is not a transition state, meaning choice B is eliminated. It has a finite lifetime and is semi-stable, so the best answer is choice A. Arrow-Pushing in Mechanisms When drawing mechanisms, the atrow starts with the electron pair destined to form a bond. If a bond is broken, then the arrow starts at the bond being broken and finishes as either a new bond being formed, or as a lone pair on another atom. Figure !-39 explains the arrow-pushing associated with the electrophilic addition of hydrobromic acid, HBr, to a symmetric alkene, E-2-butene.
Step 1: There is no lone pair in the alkene, so the arrow starts from the most available electron pair (the rc-electrons) and goes to the partially positive hydrogen of HBr. The bond between H and Br is broken.

"").l.iYg.fil -f{ ,/n-Bond\


Bondbrbaking breaking CH3

*a^

a^

H"C Bond H

"Jj:"3>"
/\ H
CHs

r,t.

..o

Nucleophile (e- Pair Donor)

Electrophile (e- Pair Acceptor)

Planar carbocation Bromide anion (Electron poor) (Electron rich)

Step 2: A lone pair on bromine goes to the positive carbon of the carbocation intermediate to form a sigma bond between C and Br. The product is an alkane with bromine (alkyl bromide).

H.C "
.i_

\^ ;f

Bromide anion Planar carbocation (Carbon is electron-poor) (Bromine is electron-rich)

CHa

":',J-'t.",
Alkyl bromide product

Figure 1-39

Copyright

by The Berkeley Review

The Berkeley Review

Organic Chemistry

Molecular Structure

Fundamental Keactivity

at
Df

Competing Mechanisms lVhen an asymmetric alkene reacts with hydrobromic acid, there are multiple potential products. The reaction can produce either a secondary or tertiary alkyl bromide product. The pathway leading to the tertiary alkyl bromide has a lower activation energy, forms a more stable intermediate, and results in the more stable product. The pathway leading to the more stable product yields the lhermodynamic product. The pathway with the lower activation energy yields fie kinetic product. Some reactions may involve a competition between the pathway leading to the thermodynamic product versus the pathway leading to the kinetic product. Figure 1-40 shows the treatment of an asymmetric alkene rvith hydrobromic acid. Observed Reaction
(Leads to the More Stable Product by way of the More Stable Intermediate)

rg
a
al.

H"C

!o

HsC

CH:
Electrophile
(Strong haloacid)

H:C
(3" > 2" in

CHg

II
er

Nucleophile (Alkene ru-bond)

3'Carbocation Bromideanion stability) (Leaving group)

ic

H.CA 'J4.}H
+

H.C
A .I1f.
...

/\ HeC

H.c\7
:
S'

'\^ ;f
LL

CHe

Bi

\ CH:

3" Carbocation (Electron-poor)

Bromide anion(Electron-rich)

efiyf bromide product

(More stable than 2'product )

Minor Product Reaction


(Leads to the Less Stable Product by way of the Less Stable Intermediate)

H"C
H:C

A ').L.1 \*s-;ry;t/\
CHs
Electrophile
(Strong haloacid)

HeC
(2' < 3" in

CHs

(Alkene

Nucleophile n-bond)
H

2'Carbocation Bromideanion stability) (Leaving group)

H"C

Hsc.
...+

H$t"/
H:C

...c-

/, \

CHs
Bromide anion (Electron-rich)

\_ H\t"'/ HsC
-..
2"

\,

lP'':
\
CHs

\H ..

2'Carbocation
(Electron-poor)

Alkyl bromide product

(Less stable than 3" product )

Figure

1"-40

)w

Copyright

by The Berkeley Review

Exclusive MCAT Preparation

Organic Chemistry

Molecular Sllrrclrrre

Fundamental Reactivity

Energy Diagrams An energy diagram accounts for the energy of the system as a reaction proceeds. As energy is added to the system (to break bonds or reorient the molecule to a less stable structure), the energy goes up. As energy is released from the system (upon making bonds or reorienting to a more stable structure), the energy goes down. The overall difference from start to finish represents the energy change for the reaction (either AG or AH.) The diagram reaches an apex with a transition state, and a localized nadir in the middle with an intermediate. The start represents the energy of the reactants, and the end represents the energy of the products. Each apex represents a transition state in the reaction, which results in a step in the reaction, Figure 1-41 shows the two energy diagrams associated with the two possible reactions when adding HBr to an asymmetric alkene.

T
2(3')

Products
2": (H3C)2CHCHBTCH3
bo

>'

3': (H3C)2CBICH2CH3
Reactants (H3C)2C=CHCHa + HBr

rr

o
AJ O)

trr

li

/ aGr(3") = -8.77kcar /
AG"12.;

= -7.63kcar

mole mole

Reaction coordinate

Figure 1-41
Energetics and Kinetics

Will a reaction proceed or not? This question deals with thermodynamics (overall energy of the process) and kinetics (overall rate of the process).
Reactivity can be analyzed
as

follows:

1.
2. 3. 4.

Identify and label the reactants (in most cases as nucleophile and
electrophile).

Predict the products of the reaction. If the reaction occurs in multiple steps, predict products for each step. Show the flow of electrons in each of the steps. This is known as drawing the
mechanism. Evaluate whether the reaction is overall thermodynamically favorable. AG"

is the standard free energy. When negative (AG" < 0), the reaction is favorable in the forward direction. When AG' is negative, Ksq (the
equilibrium constant measuring the ratio of products to reactants) is greater than 1 (K"q > 1). Because AG" = AH'- TAS", and most organic reactions take place in sblution where entropy is relatively unchanging, AH" can be approximated from AG". In conclusion, to determine whether a reaction is thermodynamically feasible, look at O6', KeO, and AH".

Copyright

by The Berkeley Review

44

The Berkeley Review

ty_

Organic Chemistry
i.

Molecular Structure

Fundamental Reactivity

s.

Evaluate whether the reaction can proceed at the given temperature. There are favorable reactions that never take place, because the activation energy (E6s1) is too great. When looking at reactions, there is AGX to consider, the

n
}S

!e rn

rt
te

n
d

activation free energy for each step. There must be enough free energy present that a reasonable number of reactants can overcome the activation barrier. This factor is hidden in the rate constant (k.*), which takes into account the frequency of collision, the temperature, and the orientation of molecules during collision. In conclusion, look at: T (high temperature equals fast reaction), Eact (low Eagl equals fast reaction), and intermediates (stable intermediates equal fast reaction).
Effect of Temperature on Reaction Rate The rate of a reaction increases as temperatute increases, because there is more energy available in solution for reaction. Temperature is part the rate

=ee :onstant, kr*, mathematically expressed Equation 1.15.

k.* = A
-,q-here

"-Eut'/RT
E661

(1.1s)

A is the Arrhenius constant and

is the activation energy.

The energy diagrams in Figure 1-42 show the change in energy level as the :eaction proceeds and Figure 1-43 shows the molecular energy distribution iroughout the entire system at two different temperatures (T1 and T2).
state

(f)

bD
t+
q.)

>'

rl]

Reaction coordiante

Figure

1,-42

TztTr

Minium

Eo.1

for reaction

s
e

Kinetic energy Figure 1-43

tr

e
e

At T2, the average kinetic energy of the molecules is greater than it is at T1. Thus, at T2, more molecules have enough energy to overcome the activation energy 'rarrier
of the reaction.
@

Copyright

by The Berkeley Review

4s

Exclusive MCAT Preparation

Organic Chemistry

Molecular Structure

Physical Properties

Physical,, Propefties,
It is important to have an understanding of how molecules interact with one another. By understanding these interactions, it is easier to predict what will
Physical Properties and Intermolecular Forces

take place in a chemical reaction or physical change, CommonMCAT questions involving intermolecular forces include determining the boiling points of two related compounds, as well as a compound's solubility propeitils or melting point. The rule is simple: the greater the forces, the higher the boiling point. we will use relative boiling points to verify the relative intermoleCular forces between compounds. The following are intermolecular forces that affect the boiling points,listed in descending order of strength. Hydrogen bonding Hydrogen bonding is a weak bond (approximately 4 to g kcals/mole) that exists between a lone pair of electrons and a hydrogen that carries a substantial partial positive charge. A hydrogen has a substantial partial positive charge when it is bonded to a small electronegative atom such ai nitrogen, o*yg".r,-o, fluorine. There are no hydrogen bonds involving hydrogens thai are covalently bonded to carbon! You should be aware that not all hydrogen-bonds have the same strength. For instance, an amine lone pair binds the protic hydrogen of an alcohol more tightly than an alcohol lone pair binds the protic hydro-gen of an amine. The strength of a hydrogen bond can be estimated from thu bur" properties of the lone pair donor and the acid properties of the hydrogen donor. PoIar lnteractions Polar interactions are the Coulombic interaction between partially charged particles (approximately 1 to 3 kcals/mole). Negatively chirged sites attiact positively charged sites. The greater the partial charge on the site of the molecule, the stronger the force between opposite chargei. The strength of the force also increases as the distance between oppositely charged sites d"creases. A typical example of a polar interaction is the dissolving of ions and polar species into water. Van der Waals van der waals forces exist between all compounds. van der waals forces are considered only when no other forces exist to any extent. They are the result of

the attraction between temporary dipoles (a very weak force). They are the weakest of the three intermolecular forces between molecules that we shall consider. They are considered to be less than 1 kcal/mole.
The intermoiecular forces are the primary consideration when approximating physical properties, when forces are not enough to determine ihe physical properties such as boiling and melting point, then structural features such as molecular mass and molecular rigidity become the determining factors. The heavier the compound, the harder it is to remove it from a lower energy phase and place it into a higher energy phase (i.e., liquid to gas). what is meJnt by "molecular flexibility" is the ability to twist and conform to allow for more surface area, and thus more intermolecular interactions. Van der Waals forces are rarely used to explain anything except why there is not a complete absence of intermolecular force. Figure L-44 shows the different forces.

W ,*
O!

ft

Copyright

by The Berkeley Review

46

The Berkeley Review

Organic Chemistry
Hydrogen-bonding:

Molecular Structure

Physical Properties

gu
HeC

-?o \
CHs

-rr.
H

H
I

/
Dip ole - Dip ole I nte ractio
n

F_

kry
H3CV'066+l
HaC

T.)a%'"
H

Van der Waals interactions:

Tr--$T,{,

:-.$e?. H
F-xample 1.22

Figure L-44

'ir{-hv

is methanol (CH3OH) a liquid at room temperature, while

ethane

CII3CH3) is a gas?

3"

E-

D.

Ethane is more polar than methanol. Methanol is significantly heavier than ethane. Methanol has stronger van der Waals interactions than ethane. Methanol has hydrogen-bonding, while ethane does not.

Solution
!fiethanol (CH3OH) is a liquid at room temperature, while ethane (CH3CH3) is a Sas, so methanol has the greater boiling point. Ethane is a nonpolar molecule, so Cpice A is eliminated. Methanol has a molecular mass of 32 grams/mole, while e$lane has a molecular mass of 30 grams f mole, so methanol is only slightly (not significantly) heavier than ethane, so choice B is eliminated. The van der Waals rnEractions are roughly equal for all molecules, so choice C is eliminated. hfiethanol has hydrogen-bonding, while ethane has no protic hydrogen and Sr,erefore no hydrogen-bonding. The higher boiling point is due to the hr-drogen-bonding of CH3OH, so choice D is all yours!

Copvright O by The Berkeley Review

Exclusive MCAT Preparation

Organic Chemistry

Molecular Structure

Physical Properties

Hydrogen-Bonding Hydrogen-bonding is the strongest of the common intermolecular forces. It can be thought of as a weak covalent bond between a hydrogen that carries a partial positive charge and the lone pair on a nearby atom. The strength of a hydrogen bond varies between 4 and 8 kcals per mole. Hydrogen bonds are similar to the interaction of a base with an acidic proton in the transition state of a proton transfer reaction. This is where the term protic comes from, as a hydrogen capable of hydrogen-bonding is also slightly acidic. The partial positive on hydrogen is strong enough to form hydrogen bonds when the hydrogen is bonded to either fluorine, oxygerL or nitrogen (highly electronegative atoms). The strength of the hydrogen bond is best approximated by the acidity of the proton and the basicity of the lone pair donor. The strongest hydrogen bond exists between hydrogen on fluorine and a lone pair from nitrogen. Hydrogen bonding in alcohols is stronger than in amines, as supported by the greater boiling points of alcohols relative to amines with comparable mass. The best explanation for this is the greater acidity of the protic hydrogen of an alcohol than an amine. compounds that form hydrogen bonds are polar, so when a compound has hydrogen-bonding, it also has dipole-dipole interactions. This means that when you are asked to compare boiling points of compounds, you should first look for hydrogen-bonding. Figure 1-45 shows the structures and boiling points of butanol and butanal. Butanol is capable of hydrogen-bonding, while butanal is not. For this reason, the forces between butanol molecules are stronger than the forces between butanal molecules. The result is that butanol molecules bind one another more tightly, resulting in a higher boiling point. This comparison of the two is reasonable, because the two molecules are of roughly equal mass. OH

\//

,,^-'At
Butanal (b.p. = 76.1,'C)

Butanol (b.p. =

717.4' C)

Figure 1-45
Example 1.23

Which of the following compounds has the HIGHEST boiling point? A. (H3C)3N B. (H3C)2NH

C. (H3C)3CH D. H3COCH3
Solution

This question centers on intermolecular forces, particularly hydrogen-bonding. As a rule, the compound with the greatest intermolecular forces has the highest boiling point. Hydrogen-bonding is the strongest of the intermolecular forces, and if a compound has hydrogen-bonding, it also has dipole-dipole interactions, so the compound with hydrogen-bonding has the greatest intermolecular forces. To form a hydrogen bond, both a lone pair of electrons and a hydrogen on a highly electronegative atom (N, O, or F) are required. Choice C does not contain a lone pair, so it is eliminated. Choices A and D have all of their hydrogens on carbon, so they are both eliminated. This leaves choice B as the only molecule that forms hydrogen bonds.

Copyright

by The Berkeley Review

The Berkeley Review

gs

Organic Chemistry

Molecular Structure

Physical Properties

ln
ai

ln
1e

)n

tn
)n is
I

Folarity ?olarity is defined as an asymmetric distribution of electron density within a :roiecule. Perhaps an easier way to think of this is as a tug-o-war for the =-ectrons between atoms. The more electronegative atoms pull the electrons ::ore tightly. If the molecule has more of one atom on one side of the molecule -:ra-n another--that is, if it is asymmetric about a ceniral point-- then it is polar. :-:ure 1-46 shows a series of chlorinated methane derivatives that demonstrate ::-arity (or lack of polarity) based on structure.
H
I

CI
I

H
I

H
I

CI
I

:IE

rd

ln
er

st ol
a ris

l'--.npoiar

Polar

Polar

'/Y,.'ct/V.
Polar

Nonpolar

Figure 1-46 -. : --iar compound has a dipole, which for a1l intents and purposes is a line :::,1 r'r form the positive side of the molecule to the negative side of the molecule ri- : i".-av that sums up the polarity vectors of each bond in the molecule. The line rL'-:ates the direction in which the electron density shifts. Figure 7-47 shows the m: from Figure 7-46, wlth dipoles now drawn in. 'e;ules
F{
I

)u

il.
n/

ln
re is

C1

,r'"{'Jn H .ztyrn
'',,:

f'

CI/IV:, ct o./Y.,
Polar

l:O1af

Polar

Nonpolar

Figure

1,-47

-r,e :,:elaction of the dipoles between two nearby molecules accounts for a weak r'ur,:: l-- shown in Figure 1-44. The alignment of dipoles is best when the partial J,"$itr-- e of one molecule aligns near the partial negative of another molecule.

{x.mrnie 1.24

:i the following molecules has a dipole moment of zero? -::ion monoxide 4.lr* il" -l::*oromethane : -:-Dchloropropane I l:':r.-1,1-dichlorocyclohexane
rL,r,'tu::
$uluruc.ou-r

1r :a ^: a dipole moment of zero, the molecule must be symmetric. Carbon :ru n r,:, :ie i C=O), dichloromethane (see Fi gure 1,- 47 ), and 2,2- dichlor oprop ane are r:r-;-=h'x; and thus polar. The trvo carbon-chlorine bonds of trans-1,4lLr:-:r!:::.:i'clohexane are on opposite sides of the molecule, symmetrically lrsrJ:,i-n,i. about a central point in the molecule, so their dipoles cancel out.
.sil$r

-]ilu "L':t

rs the best answer.

:w

l*um-nr,,::.=--

C by The Berkeley Review

49

Exclusive MCAT Preparation

Organic Chemistry

Molecular Structure

Physical Properties

trg
Solub
Sneubd
brmscift

Van der Waals Interactions

Van der Waals forces are weak intermolecular forces that exist between all molecules' These weak attractive forces account for the minimal attraction between hydrocarbons. In biochemical discussions of hydrophobic interactions, it is in fact van der Waals forces that are being considered. Figure 1-48 shows why lard (a saturated fat) has a higher melting point than vegetable oil (an unsaturated fat). Because the molecules in laid are more flexible, they are better able to interact with another molecule (tie knots around another molecule, if you will) than is vegetable oil. Perhaps it is easier to picture lard as a pile of strings that can tie knots around themselves, while vegetible oil is like a pile of straws. If you were to build two separate piles, one a pile of strings and the other a pile of straws, then intertwine each within itself, from which would it be easier to remove a piece? It would be far easier to remove a straw from the straw pile, because the straws are not tangled up. Their rigidity prevents interactions between the straws. This is why saturited fats 1*hose molecules are flexible like strings) are solid at room temperature, while unsaturated fats (whose molecules are rigid like straws) are llquid at room temperature. The greater the number of rc-bonds in a compound, the lower its melting point, and the greater the odds it is a liquid at room temperature. The nbonds in the fatty acids in a phospholipid bilayer affect the fiuidity of a cell
membrane.

ln*e s[
hu'}"e0

maFrfff,

*oto,.{ Src

mrMl
lmaru
he$p
rwmnmrhi

lx.{ryd dmpdl
ffir.mmm

tr
il

f,'*rmt

ill*

t"
Vegetable OiI (More rigid structure)

ffiLard (More flexible structure) Figure 1-48

ffig"-

,IMffi

@fu s

Example 1.25

Cell membranes are composed of many molecules, including phospholipids. A phospholipid is a molecule with a glycerol backbone (HocH2-cH(oH)CH2oH), plus two fatty acids, and a phosphate attached to the oxygen atoms of glyceroi. A cell membrane would be most rigid if both of its fatty acids were: A. completely saturated and short molecules. B. completely saturated and long molecules. C. uqsaturated and short molecules. D. unsaturated and long molecules.

{a

*"n

il

mhi mm

Solution
For the membrane to be rigid, there must be many interactions between the chains. The maximum degree of interaction occurs with long, saturated acids. Pick B and feel good yet again.

lipid

fitty

Copyright O by The Berkeley Review

50

The Berkeley Review

Organic Chemistry

Molecular Structure

Physical Properties

Solubility and Miscibility Solubility is defined as the ability of a solute (solid) to dissolve into solution. Miscibility describes the ability of a liquid to mix (dissolve) into another liquid. We shall look at both of these abilities in terms of physical properties and intermolecular forces. The intermolecular forces of greatest concern here are hydrogen-bonding and polarity. The basic rule of polarity governing miscibility and solubility is that like dissolaes like. This means that a polar molecule dissolves most readily in a polar solvent, and a nonpolar molecule dissolves most readily in a nonpolar solvent. Miscibility and solubility can be used as diagnostic tests to help determine the identity of an unknown substance. There are three combinations of properties that a solvent may have. It may be polar and protic (capable of hydrogen-bonding), polar and aprotic (no hydrogen-bonding, but has dipole-dipole interactions), and nonpolar and aprotic (weak intermolecular forces). Types of solvents and their properties are described in the Table 1-9.
Type
Polar, Protic
Polar, Aprotic

Intermolecular Forces

Examples

H-bonding, dipole-dipole, Water. Alcohols and van der Waals Dipole-dipole and van
der Waals
Ketones, Ethers,

Alkyl halides

Nonpolar, Aprotic van der Waals Table 1-9

Oils, Petroleum

The followrng three solubility observations can be explained by the solubility rules in Table 1-9. Salts dissociate into water, because ions are stabilized by the protic nature of water. Sugar dissolves into alcohol, because of the large amount of hydrogen bonding. Wax dissolves into oil, because it is entropically favorable to randomize.

Individual van der Waals interactions are weak; but over a long
molecule, they quickly become significant.

It is important that you have

a good understanding of which common solvents have which properties. The solvent properties come into play when dealing with chromatography and extraction. In chromato graphy, the greater the solubility of a solute in the solvent, the greater the affinity of the particle for the mobile phase, meaning it can travel farther and faster. In extraction, solutes are separated from one another by their relative solubility (or miscibility) in two solvents. The difference in solubility is attributed to functional groups on both the solvent and soiute. Micelles can be employed to increase the apparent solubility of a solute in a solvent in which it is said to be insoluble.

How can a nonpolar particle dissolve into water? Soaps help to make a nonpolar, aprotic species dissolve into water. For a soap (surfactant) to do this, it must be both hydrophilic (water-soluble) and hydrophobic (water-insoluble) simultaneously. Such molecules contain a polar (or charged) end (referred to as the head) and a alkyl chain (referred to as the tnil). Originally, soaps were made by treating animal fats with strong base to convert the ester to a carboxylic acid, and then further convert the carboxyiic acid to carboxylate (its conjugate base), a
Copyright
@

by The Berkeley Review

5l

Exclusive MCAT Preparation

Organic Chemistry

Molecular Structure

Physical Properties

o
IUo
m_

species with an organic tail and a charged head. soap molecules form micelles when placed into water. Micelles are little pockets (spherical in shape) with an organic core and polar heads sticking out from the core to interact with the water. They are shuttle pods for nonpolar, aprotic species through water.

How do candles burn? This question requires an understanding of how phase changes with temperature. Is it the solid form or the liquid form of the wax that burns? what is the melting point of the wax in a candle? Is heat distributed evenly in a candle (i.e., is it a thermal conductor or a thermal insulator?) These

melts to form an oil. The melting point for wax i.s greater than room temperature, but less than the temperature of the flame. The oil and vapor that form burn when exposed to heat from the flame and to oxygen. only the oil near

questions all ask about the physical properties of wax, a conglomerate of carbon chains usually containing between 31 and 50 carbons. As heat is applied, wax

f,o
1*

the flame burns, so heat is not evenly distributed in the system. The workings of a candle can be explained using fundamental principles in science. euite often, the simplest things in life are organic chemistry in action,

Example 1..26 \Alhich of the foiiowing compounds would make the BEST micelle?

A. H3C(CHz)gCOzH B. H3C(CH2)3CO2C. H3C(CH2)1aCO2H D. H3C(CHz)tqCOzSolution


The best micelle has an ionic (charged) head and a long carbon chain for the organic tail. Choices A and c are eliminated, because they have uncharged heads. Although a carboxylic acid group is polar and protic, a charged site is better, because it is more hydrophilic when charged. Choice D is better than choice B, because it has a longer organic tail. Pick D to score big on this question.

At L

E{
tn"-

Copyright O by The Berkeley Review

52

The Berkeley Review

C{

Organic Chemistry

Molecular Structure

Section Summary

Key Points for Molecular Structure (Section 1)

-\omenclature 1. IUPAC Nomenclature (Names are assigned systematically based on functional groups and carbon chain length) a) Name Root (Assigned according to the longest chain) b) Suffices (Assigned according to functional group--generally, the most oxidized functional group gets top priority) c) Prefixes (Assigned to note stereochemistry; i.e., R or S, E or Z, and a or B) d) Common Nomenclature (Prefixes based on substitution and relative positions of functional groups; i.e., geminal diol and secondary alcohol)
Bonding and Orbitals 1. Bonding (An attractive interaction between neighboring atoms) a) Covalent Bond (The sharing of electrons between atoms; carbon makes
covalent bonds)

Single bonds are made of a sigma-bond; they are weaker than both double and triple bonds. ii. Double bonds are made of one sigma-bond and one n-bond; they are weaker than triple bonds, but stringer than single bonds. iii. Triple bonds are made of one sigma-bond and two n-bonds; they are stronger than both single and double bonds. iv. Sigma-bonds share electron density between nuclei while n-bonds share electron density in the plane above and below the nuclei. b) Molecular Orbitals i. Like atoms, bonds have electrons in regions of high probability, so there are sigma and pi orbitals to describe molecular bonds. ii. Sigma bonding orbitals are more stable than pi bonding orbitals, while

i.

iii. c) i,

sigma anti-bonding orbitals are less stable than pi anti-bonding orbitals. They fill 62n4v*46*2 Anti-bonding orbitals have no overlap between atoms

Structural Rules (Atoms obey predictable behavior when making bonds) Octet Rule (Atoms in the second row of the periodic table seek to complete their valence shell by obtaining eight electrons.) ii. HONC Rule (In neutral molecules, H makes one bond, O makes two bonds, N makes three bonds, and C makes four bonds. If an atom deviates from these values, it carries a charge.)

Hybridization 1. The mixing of atomic orbitals (s and p in organic chemistry) to form hybrid
orbitals capable of combining to make molecular orbitals. a) sp-hybridrzation results in linear compounds, often with two ru-bonds, a 180" bond angle, and the shortest of all hybrid orbitals. b) sp2-hybridi"ation results in trigonal planar compounds, often with one nbond, a 120" bond angle, and a medium sized hybrid orbital. c) sp3-hybridlzation results in tetrahedral compounds, with no n-bond.s, a 109.5'bond angle, and the longest of all hybrid orbitals. Bond Dissociation Energy 1. The energy required to break a bond in a homolytic fashion (into free radicals) is the bond dissociation energy. Higher BE refers to a stronger bond. a) BDE depends on the atoms, the substitution, and electron distribution b) Ionic bonds, rare in organic chemistry, break in a heterolytic fashion. Copyright O by The Berkeley Review

5t

Exclusive MCAT Preparation

Organic Chemistry
a)

Molecular Structure

Section Summary

Intramolecular Features

1. Forces Affecting Electron Distribution within a Molecule


Resonance (Sharing of n-electrons through an array of p-orbitals)

i, Most stable resonance structure has an octet for all atoms but
hydrogen, minimal charged sites, and if there must be a charge, it sits on an atom of appropriate electronegativity. ii. Also know as conjugation when dealing with just n-bonds iii. Atoms with lone pairs are generally electron-donating while atoms with a n-bond and no lone pair are generally electron-withdrawing. b) Inductive Effect (The pull of electron density through sigma bonds) i. Depends on electronegativity of atoms ii. Diminishes over distance, becoming negligible after four carbons. c) Steric Hindrance (Repulsion of two atoms at the same location) d) Aromaticity (Stability for cyclic systems with a set number of n-electrons) i. Must contain a continuous cycle of overlapping p-orbitals ii. Must have 4n + 2 n-electrons, where n is 0, 1, 2, etc... (Hrickel's Rule) Fundamental Reactivity

1. Organic Chemistry at it foundation is Lewis Acid-Base Chemistry

a)

Acid-Base Chemistry

Bronsted-Lowry deals with proton transfer while Lewis deals with the accepting and donating of electron pairs. ii. Electron pair donors are Lewis bases and nucleophiles, while electron pair acceptors are Lewis acids and electrophiles. b) Determining Acid Strength (stronger acids have lower pKu values) i. Primary factors affecting acid strength are the size, hybridization and electronegativity of the atom to which H is bonded. ii. Secondary factors affecting acid strength are resonance and induction. iii. Base strength goes in the opposite fashion as an acid. c) Acid and Base Terminology and Facts t. Acids: pH = -log[H3O+], [H3O+] = 10-PH, pKa = -log Ka, K6 = lQ-pK2 t1, As acid strength increases: 1) acid dissociation increases, 2) Ku increases, 3) pKu decreases, 4) pH in an aqueous solution decreases, and 5) conjugate base strength decreases and stability increases 111. Bases: pOH = -log [OH-], [OH-] = 10-POH, pKb = -Iog K6, K6 = 1g-pK6
iv.

i.

As base strength increases: 1) base hydrolysis increases, 2)

KU

increases, 3) pKU decreases, ) pH in aqueous solution increases, and 5) conjugate acid strength decreases and stability increases

d)

Mechanisms involve tracking the pathway of electron transfer

Physical Properties

1. The physical properties of a compound are affected by intermolecular forces a) Hydrogen Bonding (Occurs between H on N, O, or F and a lone pair) i. Increases boiling point and melting point by increasing attraction.

b)

Polarity (Interaction between dipoles of adjacent compounds Polar interactions are weaker than a hydrogen bonds c) Van der Waals Forces (Weak interaction between temporary dipoles) i. Small impact on physical properties d) Solubility and Miscibility (Ability to dissolve into a solvent) i. Based on the idea that "Like dissolves like." ii. Solid into solvent is solubility while liquid into solvent is miscibility

i.

Copyright

by The Berkeley Review

54

The Berkeley Review

Stmcture, Bonding, and Keactivity


Passages
15 Passages
I OO

Questions

Suggested Structure, Bonding, and Keactivity Passage Schedule: I: After reading this section and attending lecture: Passages I - III & VI - VIII Grade passages immediately after completion and log your mistakes. II: Following Task I: Passages IV V & IX, (20 questions in 26 minutes) Time yourself accurately, grade your answers, and review mistakes.

III:

Review: Passages X - XIII & Questions 92 - IOO Focus on reviewing the concepts. Do not worry about timing.

,1fu

REKI{ELEY
Speciahzrng in MCAT Preparation

L)R

E.v.r.n.w'

betn engt and

is to

I. II. III. IV. V. VI. VII. VIII. IX. X. XI. XII. XIII.

Bond Dissociation Energies Structure of Caffeine Solubility of Dyes and Soap

(l -8)
(e

carb inao

ofb

- 14)

(rs - 2L)
(22 - 28) (2e - 34) (55 - 42) (43 - 4e)

Amide Protonation
Physical Properties and Intermolecular Forces Molecular Structure and Polarity
Micelles IR Determination

It'c
lnrc fut

l"i

bood

of 1", 2', and 5"Alcohols

(5o - 56) (57 - 63) (64 - 70)

PetroleumDistillation Acidity and Hybridization


Ksq and Acidity

I,nrc

l'"r.

lEtr

(7r - 77)
(78 - 84) (85 - el) (92 - lOO)

@
@
ThErc
wrqE TanilE

t**. l'"r.

Alkoxides and Alkyl Sulfides Nucleophilicity and Basicity


Questions not Based on a Descriptive Passage

T guury

Thcrt

Structure, Bonding, and Reactivity Scoring Scale Kaw Score 84 - 100


MCAT Score

0fbil
rm a&

66-43 47 -6s
34 46

l5 l5 10 l2 7 -9

lb'@ ilm ilE

&s,

4-6

t-53

t-3

Fassage

(Questions 1 - 8)

I
=

,\s an approximation when determining the enthalpy of **n;tion fiom bond energies, it is assumed that a bond
rri'aeen two atoms has a fixed value for its bond dissociation sme'rsv, regardless of the substituents on the molecule. This
$ :,J say that one assumes a covalent bond between carbon nr,i iodine always has the same bond energy, whether the ;*rron is a teftiary or primary carbon. A more critical view

The presence of an alkyl group on an alkene strengthens its rc-bond. Alkyl groups on vinylic carbons are considered to be electron donating, so fi-bonds must be electron
acceptors.

I.

What bond dissociation energy would you expect for the

bond between carbon-l and hydrogen and the only carbon-carbon single bond in H-C:C-CH:?

rr :ond

energies, however, shows this assumption to be l:rlru;urate. Table I lists a series of energies for common

rmml: in a wide range of organic molecules. BE 1-keal-; mole Bond

. B. C. D.
A

C1-H 92 kcal/mole; CZ-CZ 86 kcal/mole C1-H 1 16 kcal/mole; C2-C3 86 kcal/mole C1-H 92 kcallmole; C2-C3 110 kcal/mole C1-H I 16 kcal/mole; C2-C3 1 10 kcal/mole

BElleel;
mole
88 85
84
81

H3C-CH3 H5C2-CH3
(H3C)2CH-CH3

2.

Bromine would make the STRONGEST bond with


which type of carbon?

(H3C)3C-CH3 H2C=CH-CH3

91

HrC-H
H5C2-H (H:C)zCH-H (H:C)rC-H H2C=CH-H H3CO-H H5C2O-H

t04
98 95
91 108

A. Methyl B. Primary C. Secondary D. Tertiary

ffilJ:C-I

3.

t02
103

What can be concluded about the relationship between atomic size and bonding?

A.

Smaller atoms form longer, stronger bonds than


larger atoms. than than

Table
T-l: r'alues in Table

B. Smaller atoms form longer, weaker bonds


larger atoms.

,1111p

demonstrate the effect of alkyl g{lrum$ -rn neighboring atoms and the bonds that they fbrm. ^i'brmry s a correlation between carbon substitution and bond 1mruilu:- ls rvell as between atomic size and bond strength. lllhmm ,s also a correlation between hybridization and bond itimmrsfir- L'ut it is not substantiated by the limited data in Tliutulre :" nhich presents too few examples of varying degrees mi m.uqrrir;lzation to reach a solid conclusion about the effect of fllilril$mflnliilriiion on bond strength. The effect of substitution on rtiil[,,rrrc on bond strength can also be evaluated using bond

C. Smaller atoms form shorter, stronger bonds


larger atoms. larger atoms.

D. Smaller atoms form shorter, weaker bonds than

4.

rflrilllfiHrHmr-i.

Table 2 lists the enthalpy of reaction for the reactions of various alkenes. The dilferences irilttil t'r,fri r!611-ts5 tbr the variOus hydrogenation reactions are due rui' lm* {re.Jt of alkyl groups on the strength of a n-bond.
iltltlltrNflrurgfl:r.tion

The GREATEST amount of energy is released by the oxidative cleavage of an alkene that is:

A. unsubstituted. B. monosubstituted. C. disubstituted. D. trisubstituted.

Alkene

AH

lteal;

H2C=CH2 RHC=CHz
cis-RHC=CHR R2C=CH2 trans-RHC=CHR R7C=CHR R2C=CR2

-32.6
-30.2

-28.s
-28.3 -27.4 -26.1

5.

The difference in enthalpy ofhydrogenation between the cis and trans alkenes can be attributed to a difference in:

-26.4

A. B. C. D.

resonance.

hybridization. the electronegativity of carbon.


steric hindrance.

Table

J;nrlght

by The Berkeley Review@

GO ON TO THE NEXT PAGE

6.

The hybridization of the carbons in H2C=CH-CH:


can best be described as:

Passage

ll

(Questions

I - 14)

. B. C. D,
A

sp, sp, and sp2. sp, sp,andsp3.

tp3, sp3, and sp2. tp2, sp2, and sp3.

Caffeine, a drug extracted from tea leaves, coffee beans, and cacao plants, exists in two forms, depending on the pH at which it is processed: the neutral (freebase) form or the protonated (acid salt) form. Caffeine has medicinal uses as a

stimulant and as a coagulant. For years, beverages


containing this drug have been popular for their stimulant
effects and in some cases flavor. Caffeine can be extracted from tea leaves by employing acid-base extraction using standard extraction techniques on a pulverized conglomerate of the leaves. Caffeine isolated from the leaves in this way is relatively pure. Once extracted, either the acidic form or basic form of caffeine can be isolated. Both forms are air-stable in their crystalline solid state and in solution, but they exhibit different physical properties due to the ionic nature ofthe acid salt form. The acid salt form is more water-soluble and has a higher melting point than the free-base form. The acid salt form of caffeine can be converted into the free-base form by treating the acid salt with a strong base. Equally, the freebase form can be converted into the acid salt form by treating the free base with a strong acid. Drawn in Figure 1 are the
free-base and hydrochloric acid salt forms of caffeine.
Freebase

Which of the following single bonds is the strongest?

A. H3C-I B. H3C-CI C. (H3C)2CH-I D. (H3C)2CH-C1

The GREATEST amount of energy is required to break which of the following carbon-carbon bonds?

A. B. C. D.

H3C-CH3 (H3C)3C-C(CH3)3
H2C=CH2 (HgC)zC=C(CHl)z

Acid salt

HrC .\ N

HrC -\

clcH:

'\*--CHr

\*z

(
N

*Ao
I

(
+N

*Ao
I

CH: Figure L

cHr

Freebase and acid salt forms of caffeine

Because the reactivity of caffeine varies with its form, predictions about its reactivity must be based on the pH of its environment. The conjugate acid form of caffeine has a pK6 value that is slightly higher than 6, making caffeine a weak acid. It exists primarily in its deprotonated form at a pH of 7 in an aqueous solution. The extraction of caffeine from tea leaves can be carried out using vinegar to lower the pH of aqueous layer and using an ether solvent (organic layer) as the second layer of the biphasic system. The caffeine cation dissolves into the aqueous layer, while the other orgi components of the leaves dissolve preferentially into organic layer. 9

The acid salt form of caffeine can be converted to free-base form most readily by adding which of

following reagents?

A. HCI B. NaCl C. CaCO3 D. NaOH

Copyright O by The Berkeley Review@

GO ON TO THE NEXT PAGE

I
beans,

0.

What is the hybridization of the imine nitrogen of the purine ring that is protonated in the acid salt form of
caffeine?

14.

rpHat

or the
9SASA

)rages

nulant racted using


rrate

A. B. c. D.

The compound formed by replacing the N-CH3 group between the two carbonyl carbons in caffeine with an O atom is classified as an:

rp tp2 tp3 Nitrogen atoms do not exhibit hybridization.

A. acidanhydride. B. acid ester. C. ester. D. lactone.

of

way is

rrm of n their :xhibit


re acid

i 1. Which of the following strucrural descriptions


caffeine?

BEST

describes the relationship between the four nitrogens in

Jhasa
rid salt

rrm by
e freereating

A . Perpendicular planar B. Coplanar C. Tetrahedral D . Inverted planar

ue the

clCH3

The length of the carbonyl bonds (C=O) in the caffeine molecule are BEST described by which of the following
sLatements?

'o
e

B.
C D

form, { of its
a PKa

. .

Both C=O bonds in caffeine are longer than C=O bond in cyclohexanone. Both C=O bonds in caffeine are shorter than C=O bond in formaldehyde. Both C=O bonds in caffeine are shorter than C=O bond in carbon dioxide. All C=O bonds are of equal length, regardless of
compound.

the the the the

ll :'

r weak

rHofT
om tea tof the ) as the

cation
rrganic rto the

Compared to the acid salt form of caffeine, the melting point of the free-base form is:

l to the of the

. B. C. D.
A

higher, because it is the more polar form. lower, because it is the more polar form. higher, because it is the less ionic form.

lower, because it is the less ionic form.

IAGE

O by The Berkeley Review@

GO ON TO THE NEXT PAGE

Passage

lll

(Questions 15 - 21)

6.

Which of the following would be the BEST solvent to remove an ink dye that has hydroxyl (OH) groups
attached to a carbon backbone?

Many common household items are the products of basic organic chemistry. Dissolving one or more colored dyes into a volatile organic solvent, such as isopropanol, for instance, makes ink. Paint, like ink, is the combination of a dye and a solvent. When ink is applied to a porous surface such as paper, the pores of the material absorb the solution. Then as the volatile organic solvent evaporates away, the solid dye is left bound to the pores of the material. This is why ink can smear when initially applied, but once it has dried (once the solvent has evaporated away), the ink does not smear.
is possible to remove dried ink from paper by treating it with organic solvent. A problem with this method is that the solvent diffuses radially across the paper, taking the dissolved dye with it as it travels. This is commonly referred to as running and is the basis of paper chromatography. Inks that run when water is spilled onto the paper to which they are bound are made out of water-soluble dyes. The eraser capable of removing erasable ink has a surface to which the dye in the ink adheres more tightly than it adheres to the
paper.

A. Propanone B. Propanol C. Propanal D. Propanoic acid

17.

What is the IUPAC name for the following compound?


CH
3

It

CH

H 2CH 2CH 2CH2CH2CH2CH 2C O 2H

A. Nonionic acid B. Decanoic acid C. Undecanoic acid D. Dodecanoic acid 8.


Some kinds if ink run when water is spilled on paper to which they adhere. This can best be explai by which of the following reasons?

Another common household product derived from organic compounds is soap. Each soap molecule has a hydrophilic (waterJoving) end and a hydrophobic (water-fearing) end. In water, the hydrophobic portions of several soap molecules form an aggregate pore in which nonpolar, hydrophobic species (dirt and oil) can gather. This pore or nticelle (the spherical cell formed by several aligned and coagulated soap molecules) is water-soluble, because the hydrophilic end of each molecule composing it solvates in the water. A micelle is lemoved by continuous exposure to running water, into which it dissolves and migrates. soaps is sodium dodecyl sulfate (SDS), found in many commercial shampoos and hand soaps. Soaps can be made by treating animal lard (fatty-acid triglycerides) with a strong base (such as NaOH).

. B.
C D

The organic solvent of the ink is miscible in water.

The organic solvent of the ink is immiscible i


water. The dye of the ink is soluble in water. The dye of the ink is insoluble in water.

. .

19.

Which of the following would be the BEST soap?

One

of the most common industrial

A. B. C. D.

CH3CH2CO2H CH3CH2CO2Na
CH3 CH2CH2CH2CH 2CH2CH2CO 2H

CH3CH2CH2CH2CH2CH2CH2CO2Na

This forms glycerol (HOCH2CH(OH)CHZOH)

and

carboxylate anions (fatty-acid carboxylates) by a reaction

referred to as s(tponification. Carboxylic acids once deprotonated form carboxylates (the conjugate base of the

20.

Which of the following compounds is MOST sol


in water?

acid). The organic chain of a soap molecule is most useful when it contains at least eight carbons. Longer carbon chains are common in soaps that are used to remove oils having
longer carbon chains.

15.

A11

of the following would be ideal properties for

A. CHqCHzCO2H B. CH3CH2CO2K C. CH3CH2CH2CH2CH2CO2H D . CH3CH 2CH2CH2CH2CO 2K

solvent used to dissolve a dye within an ink EXCEPT:

. exerting a high vapor pressure at room temperature. B. containing functional groups similar to the dye. C. being highly reactive with cellulose. D. having a boiling point slightly above room
A
femperature.

21.

Which of the following reactions forms CH3CO2Na?

A. CH3CO2H + CH3MgCl B. HCO2H + CH3MgCl C. Ethanoic acid + NaOH D . Propanoic acid + NaOH

Copyright

by The Berkeley Review@

GO ON TO THE NEXT PA

Passage

lV

(Questions 22 - 28)

23.

For years, chemists pondered whether amides were ;rotonated on the nitrogen or oxygen atom. The amide is an
.nalog to peptide linkages, so the root of this question is - runded in the chemistry of proteins. By determining the site -: protonation, conclusions about hydrogen-bonding in the ';;ondary structure of proteins can be made. Before the .jvent and advancement of x-ray crystallography, protein !-iucture could only be hypothesized. Due to the importance . - nydrogen-bonding in protein structure, detemination of the ::rtonation site was critical. Figure 1 shows the structural
:-:3cts of protonation on the oxygen atom of the amide.

We know that amides are protonated at oxygen rather than nitrogen, because oxygen:

A. B. C. D.

is less electronegative than nitrogen, so it donates


electrons more readily.

is larger than nitrogen, so it's electron cloud attracts protons more readily. carries a partial positive charge due to resonance withdrawal of n-electrons by the nitrogen. carries a partial negative charge due to resonance donation of n-electrons fr:om the nitrogen.

ound?
,2H

ao.n o o.H H-- ii i Ll <+ --c\N-R R,,\N-R *-c\p.,R' ttl


HHH
Figure 1 Protonation of amide on oxygen
on the
rlained
water.

4.

Which of the following statements correctly describes


the geometry of the molecule shown below?

HT
A. B.

lt

-C-. /H
H

=rre 2 shows the structural effects of protonation on the r.:osen atom of the amide.

The nitrogen has trigonal pyramidal geometry, so


the two hydrogens are outside of the plane created by the other four atoms.

o
il

ible in

-c\ "N
I

/R'
H

H+ _>

*- \g,,R'
/\ HH

lt

No resonance

The nitrogen has tetrahedral geometry, so the two hydrogens are outside of the plane created by the
other four atoms.
has tetrahedral geometry, so the carbon hydrogen is outside ofthe plane created by the other five atoms.

C. The carbon D.

Figure

Protonation of amide on nitrogen

The six atoms are coplanar.

Frotonation at the oxygen site is favored because of the 'rri.-:rlnce stabilization of the protonated product, similar to ,;":;- is observed when protonating esters. Despite the greater
'r
'f

':.:-t)' of nitrogen relative to oxygen (oxygen is less basic, ir;:-se it is rnore electronegative), the resonance stability is

25. What is the MOST basic site on the following


molecule?

-r::i:: -nough to favor O-protonation. This manifests itself in i':.:.n structure through the formation of hydrogen-bonds !1,- :hs carbonyl oxygen (lone-pair donor) to the nitrogen

r-::-r (partially positive proton).


soluble

B-pleated sheets and helices rr,::. ed in the secondary structure of proteins.

'r: :sion is found in the planar

Support

for

this

l:;ause of the resonanca structures with O-protonation, :ir :.{ atoms of the amide are all coplanar. This is due to :-h1'bridization of carbon, oxygen, and nitrogen. :: r :L

A. B. C. D.

nbUr$sn'
Site a

Site b
Site c Site d

l . Alich

of the following statements CANNOT be true?

I
2Na?

The C=O bond of an amide is shorter than the C=O bond ol a ketone.

The C-N bond of an amide is shorter than the C-N


bond ol a plimary amine.

26. The MOST

stable hydrogen-bond between amides

fi. Amides are more basic that aldehvdes. \. I only B. III only C . I and II only D. I and III only
PAGE

extends from the:

A. B. C. D.

carbonyl oxygen to the H on nitrogen.


the carbonyl oxygen to the H on carbon. amide nitrogen to the H on another nitrogen. amide nitrogen to the H on carbon.

-.,:'"-risht

by The Berkeley Review@

GO ON TO THE NEXT PAGE

7.

Which of the following is NOT a resonance structure of


an amide?

Passage

(Questions 29 - 34)

A.

o
il

B.
*,
I

o-

The boiling point


R'

of a compound is defined

as the

*,.c-*.,R'

C--**.
I

temperature at which the vapor pressure of the compound equals the atmospheric pressure. It is also thought of as the

c.

oI

D.
**..

o-

highest temperature at which a compound may still be observed in a liquid state. Boiling points vary with atmospheric pressure, so when comparing the boiling points of different compounds, a standard pressure is referenced.

R,,clN,,R
I

C-*,,R'
I

Under standard pressure, a compound's boiling point


corelates with the heat energy required to vaporize
a

molecule

of it from solution. As the heat energy of vaporization (AHuaporirulion) increases, the boiling point for the
compound increases.

Two chemists speculate about the reasons for the differences they observe in the boiling points of various

28.

organic compounds.

Which arrangement accurately relates the boiling points


of acetamide (H3CCONH2), acetone (H3CCOCH3), and propane to each other in descending order? Chemist

A. B. C. D.

BPnsslsmi6s ) BPgsslene ) BPpropane BPsssl6ng ) BPssslam;6. > BPpropane BPpropans ) BPas6l6mide ) BPu."ion" BPpropans ) BP6ss16ng ) BPsssl2mi6s

Chemist I proposes that the difference in boiling poin for two similar organic compounds is related to the differences in their molecular masses. The heavier molecule, the more energy that must be required to liberate the compound into the gas phase from the liquid phase. liberate the molecules into the gas phase, heat energy m be added to the solution, which increases the temperature the solution. Chemist 1 concludes that heavier molecul have higher boiling points than lighter molecules. Chemist 2

Chemist

2 proposes that the boiling point of

compound depends primarily on the strength of the attracti intermolecular forces between molecules in solution. stronger the attractive intermolecular forces bet molecules, the harder it must be to remove a molecule f the solution to the gas phase. As it becomes more diffic to liberate a molecule from its liquid phase into its gas more heat energy is required to carry the process out. result is that the boiling point of a compound increases as molecules bind to each other more tightly in solution. W the intermolecular forces are greater, fewer molecu vaporize, so the boiling point of the compound increases, the vapor pressure of the compound decreases.

The hierarchy in attractive intermolecular forces hydrogen-bonding first, polarity is second, and van der W forces rank third in strengths. Hydrogen bonds contain greatest amount of energy of these three forces, but not hydrogen-bonds are equal in strength. For instance, alco have stronger hydrogen bonds than amines, because t hydrogen of the alcohol is more acidic than the hydrogen the amine. The greater acidity allows the hydrogen to
electron density more readily. Any compound capable forming hydrogen bonds is also polar. Polar attractions stronger than van der Waals forces, the weakest of
intermolecul ar forces.

Copyright

by The Berkeley Review@

GO ON TO THE NEXT PA

19. All of the following


theory EXCEPT:
the

observations support Chemist l's

3.

gases

rund
s the

A. B.
C

As you climb higher in the mountains, the amount of in the atmosphere decreases. This affects the
decreases, because the amount ofhydrogen-bonding
decreases.

(H3C)2CHOH has a higher boiling poinr rhan


H3CCH2OH.

boiling point of propanol such that it:

lbe
with
oints
rced.

H3CCH2OH has a higher vapor pressure than


H3CCO2H.

A. B.

. .

H3CCH2CH2OH has a higher boiling point than


H3CCH2OCH2CH3.
H
3

decreases, because the amount ofhydrogen-bonding


rncreases.

roint

COH has a higher vapor pressure

C. decreases,
than
increases.

because

the atmospheric the atmospheric

pressure pressure

xule ltion
the

H3C(CH2)6CH3.

D.

decreases, because
decreases.

- the
rious

3tt.

11sw would Chemist 2 rank the following compounds according to their boiling points?

L H3CCH2OH tr. H3COCH3


Itr. H3CCH2NH2 A. H3CCH2OH> H3CCH2NH2 > H3COCH3 B. H3CCH2OH> H3COCH3 > H3CCH2NH2 C. H3CCH2NHz > H3CCH2OH> H3COCH3 D. H3CCHzNHz > H3COCH3 > H3CCH2OH

34. How do the boiling points of the following


chlorohydrocarbons compare with each other?
HsQ

three

cH3

'ornts

r r

the the

cl II.
ct

terate

r. To
must
rre

of

cules

ll.

ct
The hydrogenation of an eight-carbon diene has which of the following effects on the physical properties of the
compound?

m.

ofa
rctive
The ween

ct-t

A. B.

Both the molecular mass and the melting point


increase.

from Ticult
rhase,

The molecular mass increases, while the melting


point decreases.
decreases, while the melting

CI

GI

C. The molecular mass


point increases.

The as the When :cules


s, and

D. Both the molecular


decrease.

mass and the melting point

A. B. C. D.

Compound I > Compound fII > Compound II Compound I > Compound II > Compound III Compound II > Compound I > Compound III Compound III > Compound I > Compound II

:es is
Waals .in the

According to Chemist 2, as intermolecular hydrogenbonding increases, which of the following trends should
be observed?

rot all
:ohols
se the gen of
accept

A.

Both the boiling point and the vapor pressure


increase.

B. The boiling point


pressure decreases.

increases, while the vapor

ble of
Ins arg

The boiling point decreases, while the vapor


pressure increases.

D. Both

of

the boiling point and the vapor pressure

the

decrease.

,AGE

63

GO ON TO THE NEXT PAGE

Passage

Vl

(Questions 35 - 42)

8.

The dipole of a bond can be fbund by considering the distribution of charge within the bond and the length of the bond. The larger the difference in electronegativity between the two atoms forming the bond, the greater the magnitude of the partial charges on each atom, resulting in a larger overall dipole. When considering the dipole associated with molecules, the electron density of the entire structure is determined by the symmetry of the structure. Each bond is treated individually, and the sum of the component vectors is
the approximate dipole. The estimated dipole is good enough to predict chemical behavior. Figure 1 shows examples of a polar and a nonpolar cyclohexane derivative.

Infrared spectroscopy involves radiating a compound with electromagnetic radiation of a known wavelength and observing any changes in the lengths of bonds within that molecule as they stretch. When the bonds are stretched, the dipole moment changes and thus can be detected. Which of the following would show the LEAST change in dipole moment?

A. B. C.
D

Stretching a carbonyl bond


molecule

in an asymmetric

Bending a carbonyl bond


molecule

in

an asymmetilc
a

Stretching
molecule

a carbonyl bond in

symmetric

Bending a carbonyl bond in a symmeric molecule

9.

Which is the BEST description of the nonpolar structu

of Fe(NH3)aC12?
Nonpolar
Polar

Figure 1 Polar and nonpolar disubstituted cyclohexanes


The magnitude of a dipole is measured by placing the compound between the two charged plates and observing the drop in voltage. A large voltage drop in the capacitor implies that the dielectric constant for the compound is large, so the molecule has a large dipoie. This technique works because of the ability of a neutral polar compound when added to an electric field to align with the fleld. If there is a net charge on the molecule, it migrates toward the capacitor plate with
the opposite charge.

A. Octahedral shape with the two Cl ligands cis B. Octahedral shape with the two Cl ligands trans C. Tetrahedral shape with the two Cl ligands cis D. Tetrahedral shape with the two Cl ligands trans
0.

Which of the following compounds shows a dielect


constant of zero when placed in a capacitor?

A. 1,1-dichloroethane B. cis-1,2-dichloroethene C. trans-1,2-dichloroethene D. E-1-chloro-2-fluoroethene


41.

35.

Which of the ibllowing compounds, when added to the gap between the two plates of a capacitor, produces the GREATEST reduction in voltage?

A. B.

ArHF
C6H6
CO,I

Which compound has the LARGEST dipole moment?

D.

c. Nz

B . 1,1,2,2-tetrafluoropropane C . 1,1-difluoro-2,2-dichloroethane

I,1,2,z-tetrafluoroethane

I ,I

-difluoro-2,2-dichloropropane

6. In which of the following reactions is it possible to


form a nonpolar organic product?

42. Which of the following

statements CANNOT be true?

A. B. C. D.
7.

Hydrolysis ofan alkene Halogenation of an alkane Hydrogenation of an alkene Reduction of an amide

I. II. il. A. B. C. D.

For a tetrahedral structure, if any of the four li are not equivalent to the others, the molecule
polar. A11 1,4-disubstituted cyclohexane molecules polar.
A11

optically active molecules are polar.

Which change does NOT result in an increased dipole


moment?

A. B. C. D.

Replacing iodine with brornine on an alkyl halide Oxidizing a primary alcohol into a carboxylic acid Replacing iluorine with chlorine on an alkyl halide Adding HBr anti-Markovnikov to an alkene

I only II only I and II only II and III only

Copyright O by The Berkeley Review@

GO ON

TO THE NEXT PA

pound
rlength

Passage

Vll

(Questions 43

49)

3.

bonds bonds
ius can rw the

metric metric

A common problem facing pharmacists is developing -rugs in a form that can be easily ingested by human beings, :.rticularly the problem of getting organic compounds to :-ssolve in water. As a general rule, organic compounds are rt water-soluble, so it is difficult for them to migrate ,.rough the bloodstream. Because most organic compounds :..aibit little to no hydrogen-bonding, they are referred to as '.^Trophobic (Greek for "water fearing"). To overcome the
- . Jrophobic nature of organic compounds, one .;hniques can be employed.

Which of the following compounds should be used in order to make a dication more soluble in an organic
solvent?

A. B. C. D. 4.

H3C(CH)nCO2H H3C(CH)nCOz'
H3C(CHz)nNHz

H3C(CHz)nNH:+

of

two
4

How would a micelle appear in an organic solvent?

.,lnique
iecule

I
1 involves the use of micelles,
three-

Technique

mlcture

NS
5

bulbs composed of compounds that are partly .:rc and partly hydrophobic (organic). A prime example of - r.mpound that forms a micelle in water is the conjugate ::.: of a fatty acid (H3C(CH2)nCO2- Na+). The micelle is a . *erical membrane that forms when the organic tails ,-ire-qate as shown in Figure l.
Pol

: rensional

Hydrophobic tails

lns

lil:l:l:
4

5.

Which of the following compounds could MOST likely


be taken into the body through respiration?

Fizure

Aqueous arrangement of molecules in a micelle


as an antibiotic) prefers the

A. B. C. D.
6.

(H3C)2CHOCH3 (H3C)2CHCH2OH

(H:C)zCHNHCH3 (HrC)zCHCH2NH2

, : of the micelle over the aqueous solution. Overall, the -:;l1e is water-soluble due to the polar heads of the
rfatty-acid carboxylate anions. After migrating - :"idual an aqueous environment to a hydrophobic environment
rrl
,lecule i

.\n organic compound (such

Which of the following compounds would be MOST


soluble in water?

..1), a micelle turns itself inside out and releases the ::'iric compound in its core. This mechanism is what :-::les water-insoluble drugs to be transported through the - , jstream (an aqueous environment) to hydrophobic target
j rxs of the body (such as lipid membranes). -. ::'.r:ique 2

B.

"\y::
D.

" ,lves converting neutral organic compounds into ions by : .:.lr protolotion or deprotonation or altering a functional
""-.

Technique 2 involves converting the compound into a ,'..:r-soluble derivative that decomposes into its active, "..:r-insoluble form once inside the body. This often

;.:h an acyl group. The drug returns to its original active , -:r it is exposed to physiological conditions.
-

-:.

such as converting an alcohol into an ester by reacting

-:r right

by The Berkeley Review@

b5

GO ON TO THE NEXT PAGE

7.

Which of the following compounds would make the


BEST micelle?

Passage

Vlll

(Questions 50 - 56)

A. B. C. D.

H3C(CH2\CO2H

hydrogen bond is formed between an atom able to

H3C(CH)zCoz' H3C(CH)13CO2H H3C(CH)nCOz-

donate a lone pair of electrons and an electropositive hydrogen (an H covalently bonded to nitrogen, oxygen, or fluorine). A hydrogen capable of forming a hydrogen bond is said to be protic. A protic hydrogen can form one covalent bond and one hydrogen bond. As the hydrogen bond becomes stronger, the covalent bond becomes weaker. This is to say that as a

lone pair is donated to a protic hydrogen, the original


covalent bond to hydrogen weakens.
4

8.

Which of the following compounds would require micelle to make it water-soluble?

Covalent bonds can be studied using infrared


spectroscopy. Different bonds have different characteristic
absorbances based on their bond strength and atomic masses. Because the degree of hydrogen-bonding affects the strength of the covalent bond, a hydrogen bond can be seen indirectly in

A. B. C. D.

An alcohol (RCH2OH) A carboxylic acid (RCOZH) An amine (RCHZNHZ) An alkene (R2C=CR2)

the IR stretch of the hydroxyl peak. Figure

shows the IR

absorbances associated with four different hydroxyl groups.

3'Alcohol

9.

What force holds the organic tails of a micelle together?

A.

'Van

der Waals forces

B. C. D.

Polar attractions
Hydrogen-bonding

VV
2'Alcohol
1" Alcohol

,01",,

,oL.*,
Carboxylic acid

3396crnl

Covalent bonding

Figure

1 IR signals for hydroxyl

functional groups

53.
Because the covalent bond is weakened by hydrogenbonding, the IR signal of a covalent bond between atoms involved in hydrogen-bonding broadens as the degree of hydrogen-bonding increases. Not all hydrogen bonds are equivalent, so the signal becomes a range of absorbances that

appear as one broad

band. The wave number of

the

absorbance lowers, because the energy decreases. The absorbances in Figure 1 show that as hydrogen-bonding

increases, the

IR signal broadens and the maximum

absorbance occurs at a lower wave number.


50

According to the IR absorbances in Figure 1, which of the following compounds exhibits the GREATEST
amount of hydrogen-bondin
g?

. B. C. D.
A

The tertiary alcohol The secondary alcohol The primary alcohol The carboxylic acid

Copyright O by The Berkeley Review@

66

GO ON TO THE NEXT PAGE

Which of the following amine compounds should show the BROADEST signal above 3000 cm-l?

5.

As dimethyl sulfide is mixed into a pure sample of an


alcohol, the O-H absorbance:

-{. Ammonia B. Propylamine C. Dipropylamine D. Tripropylamine

A. B. C. D.

broadens and shifts to a lower value on the wave


number scale. broadens and shifts to a higher value on the wave number scale. sharpens and shifts to a lower value on the wave number scale. sharpens and shifts to a higher value on the wave number scale.

How is the absorbance value in the IR for a covalent


bond between oxygen and hydrogen affected by the bond trength and hydrogen-bonding to other atoms?

.\

As the bond length increases, the wave number (cm-l) ofthe absorbance decreases; so as the degree of hydrogen-bonding increases, the bond length increases and the wave number (cm-l) of the
absorbance decreases.

6.

Which of the following statements CANNOT be true?

I. II. m.

B.

The IR absorbance of a covalent bond involving an atom engaged in hydrogen-bonding is not affected by the hydrogen-bonding.

As the bond length increases, the wave number


(cm-l; of the absorbance increases, so as the degree of hydrogen-bonding increases, the bond length increases and the wave number (cm-1) of the
absorbance increases.

The bond length of the covalent bond to the protic


hydrogen increases with hydrogen-bonding.

The acidity of a proton is increased by hydrogenbonding.

C. As the bond length increases, the wave number (cm-l) ofthe absorbance decreases; so as the degree of hydrogen-bonding increases, the bond length decreases and the wave number (cm-l) of the
absorbance increases.

A. I only B. II only C. I and II only D. II and III only

D. As the bond length increases, the wave number (cm-l; of the absorbance increases; so as the degree of hydrogen-bonding increases, the bond length decreases and the wave number (cm-1) of the
absorbance decreases.

Hydrogen-bonding occurs within which of the following


compounds?

.{. Aldehydes B. Esters C. Ketones D. Primary amines

The STRONGEST hydrogen bond is formed between:

. B. C. D.
A

the lone pair of O and a hydrogen bonded to O. the lone pair of N and a hydrogen bonded to O. the lone pair of O and a hydrogen bonded to N. the lone pair ofN and a hydrogen bonded to N.

by The Berkeiey Review@

ol

GO ON TO THE NEXT PAGE

Passage

lX

(Questions 57 - 63)

Hydrocarbon

Octane Rating
93

Boilins Point
28'C

6'1"

The petrolcurn industry provides roughly forty percent of the annual cnergy needs of the United States. Crude oil is a mixture of hydrocarbons that is refined to produce tuels,

2-Methvlbutane
Benzene

r06
25

80'c
69"C

including heating

and petroleum. Many lightweight, alkene by-products fron.r the refinement of crude oil are used as raw materials in making polylners. The industrial process fbr refining crude oil into useful components is relerred to as cracking and is sirnilar to fractional distillation. Figure shows a schematic lepresentation of the cracking column used to refine crudc oil and the fi'agrnents collected at different
1

oil

n-Hexane

Toluene
n-Heptane

120

104'c 98'C 88'C 82'C


104"C

0 12

2-Methvlhexane

2.2.3-Trimethvlbutane
2,2, 4 -T r imethy I pen tane

t25
100

levels of refinement.
5

Table

7.

Which of the following eight-carbon hydrocarbons


the GREATEST octane rating?

has

Vapor

Petroleum

25"C-t]5'C

A. B. C. D.

2-Methylheptane
n-Octane

2,2-Dimethylhexane 2,2,4-Trimethylpentane

----+-

8.

Which of the fbllowing components is MOST likely


component of kerosene?

fr}..

Kerosene

175'C-280'C

___>

Heating oil 250'C-350"C

A. n-Octane B. n-Decane C. 2,2-Dimethyloctane D, 2,2,4,4-Tetramethyldecane

_+

Lubricating oil 300'c-375"C


5

9.

Which is NOT an effect of branching in a hydrocarbor


chai
n?

-------->Tar
Figure

Cracking column used to refine crude oil

. An increase in octane rating B. A decrease in boiling point C. A increase in density D. An increase in hydrogen-bonding
A

Petroleum distillaie is sold as gasoline, the luel most commonly used in internal combustion engines. 'Ihe best air-petroleum mixture for such engines is the one that produces the most unilbrm distribution o1' heat over the pcriod of timc that the piston is doing work. This allows for an even expansion of the gas in the piston, which results in more useful wolk. The result is a smooth lifting of the piston, rather than an cxplosive jerk. Engine efficiency depends on tl-rc unilbrmity ol heat distribution within it, so the choice of fLrel influenccs engine etTiciency. Gasoline is given an octane rating that is based on its combr-rstion rate. An octiine rating is e measule of a fuel's tendcncy to caLtse knocking (non-unifbnrr combustion.) The scale is set using 2,2,4-tlimethylpentane, wl-rich is assigned an octane rating of 100, and n-heptanc, which is assigned an
octane rating of zcro. A higher octane rating implies a better fuel. Table 1 lists the octane ratings and boiling points lbr some compollents of pctroleurn distillate.

0.

The cracking (refining) column operates according to the principle that:

A. B.
C D

more dense hydrocarbons rise higher than less dense hydrocarbons. hydrocarbons with lower boiling points rise highe: than hydrocarbons with higher boiling points.

. .

hydrocarbons with higher boiling points rise higher than hydrocarbons with lower boiling points.

aromatic hydrocarbons rise higher than nonaromatic hydrocarbons.

Copyright O by Tire Berkeley Review@

GO ON TO THE NEXT PAGE

1.

The efficiency (octane rating) ofa fuel depends on the:

Passage

(Questions 64 - 70)

A. B. C. D.

enthalpy of combustion. entropy of combustion. ratio of CO2 to water in the exhaust.


r'ate o1'combustion.

The acidity, bond strengths, and bond lengths of


hydrocarbons depend on the hybridization of the carbons within the compound. Hybridization is defined as the mixing

of atomic orbitals to fbrm new hybrid orbitals that are


correctly aligned to make up the covalent bonds. Hybrid orbitals are oriented to align the atoms within the molecule into the least sterically hindered position fbr bonding. The

2.

Which of the following statements must be true?

I. II. il.

Aromaticity incleases octane rating.


Ethylbenzene has an octane rating of less than 100.

orientation of electrons allows the molecule to form structures with a central carbon that has either tetrahedral 1sp3-hybridized), tri gonal planar 1sp2 -hybridized), or linear
(sp-hybridized) geometry. Although the geometry dictates the hybridization, the hybridization oI a carbon within a molecule can be used to predict the structure of the molecule. The more p-character thele is in the hybrid, the longer the hybrid orbital is, and thus the further away the electrons ale f}om the nucleus. This variation in length can be used to

2,2,3-Trintethylbutane is a good fuel additive to


incrcase fuel eff iciency.

A. I only B. III only C. I and II only D. I and III only

oly

63

The hybridization of carbon in the aerobic cornbustion of 2,2,4-trimethylpentane changes from :

explain differences in chemical reactivity and physical properties. When estimating properties of a bond, one must consider that acidity results frorn heterolytic cleavage, while bond energies are determined from homolytic cleavage, Figure 1 shows both heterolytic and homoiytic cleavagc Ibr the C-H bond of a telminal alkyne.

A. B. C. D.

rp3 to sp2. rp2 to sp3. sp to sp3.


sp3 to sp.

Hetreolytic Cleavage (into ions)

R-C:n'
-_^E^:Lr
Figure

__ R-C:C: .o +

..@
H

Homolytic Cleavage (into free radicals)


=: o-^E^. a u.

Heterolytic and homolytic cleavage of a C-H bond

to the
dense

righer righer non-

Acidity can be explained in tenns of heterolytic cleavage. The closer the electrons of a carbon-hydrogen bond are to the carbon nucleus, the more acidic the hydrogen on that carbon. This is to say that as the electrons in the bond get closer to the carbon nucleus, the bond is easier to break heterolytically, and thus the acidity of the hydrogen increases. Electrons get closer to the nucleus of carbon when the bond is shorterHowever, as the bond gets shorter, it becomes more difficult to break the bond in a homolytic fashion. It is more difficult fbr hydrogen to remove one bonding electron from the bond to carbon. This means that as the hydrogen becomes more acidic, the homolytic bond energy increases.
The less s-character in the carbon hybrid, the longer the length of the bond between carbon and the atom to which it is bonded. As the bond becomes longer, it becomes weaker

in a homolytic

sense.

AGE

Copyright

by The Berkeley Review@

GO ON TO THE NEXT PAGE

64.

The MOST acidic hydrogen on 3-methyl-1-penryne is on which carbon?

9.

Which of the following compounds has the WEAKEST


carbon-carbon single bond?

fuc
il
otrErt
qiffiEr

A. B. C. D.

Carbon-l
Carbon-2 Carbon-3 Carbon-4

A' H-c:c-H
g. H:..

B. Hsc-cEc-cr-rs

H/u
6

D' ^.^_ t- Hs

'

K^('v-

\^-^.zH t- tt^/ ,

ThEB
fli:t$ fi
H

isau
gnhflGf,

from

5. The LARGEST Ku is associated with which of the


following compounds?

wmsm

"Go.

ur\o,

70.

The LOWEST pK6 is associated with which of the


following nitrogen containing compounds?

mufu tr lwt
rerirj i uquilil

oG*,,

Tr

B.

.\*.\
C.

-NH (,\*,
D.

&][!DeEl|

*sasti

ilfum

G,"
66. Which of the following organic
STRONGEST base?

Eq lu0msh

compounds

is

the

A. B. C. C.

Kq"

CH3CH2CH2CH2Na CH3CH2CH=CHNa CH3CH2CNa=CH2

CH3CH2C=CNa

,l\ r

cl'ick$

7.

NaNH2 is a base strong enough to deprotonate the first hydrogen on a terminal alkyne. Which of the following hydrogens could it also deprotonate?

A. . C. D.
B

H on carbon-l of 2-methyl-1-butene H on carbon-2 of 2-methyl-1-butene H on carbon-l of 2-methyl-1-butanol H on oxygen of 2-methyl-l-butanol

8.

The LONGEST carbon-carbon bond can be found in which of the following compounds?

Th

A. H- CE C"'*..-r n-n./u3

Ct-ls

B. H-C= cD' H:^^//H t\

i.r dEE WurftlE

,/"

'-

o,

,/u:

mesrmt fuErudl md rce diiffieru ryid cm cqldliihi


mfirm*

ffiuc rtpr

Copyright O by The Berkeiey Review@

GO ON TO THE NEXT PAGE

Passage

Xl

(Questions 71 - 77)

Table 1 lists the theoretical equilibrium constants for the six acid-base reactions patterned after Reaction 2.
Organic Acid (HA)

Most reactions using organic reagents require a solvent :.her than water, so acid-base chemistry must be viewed frour ::irer the Brpnsted-Lowry definition or the Lewis definition. lie Brgnsted-Lowry definition of an acid is a compound that .!;:s as a proton donor, while the Lewis definition of an acid i a compound that accepts electron pairs. In an organic :'::t'ent, acid-base reactions involve the transfer of a proton

Equilibrium Constant (Ksq)

H3CCOCH3 H3CCOCH2COCH3 H3COH H3CCH25H CI3CH H3CNO2

2.Ox1O4
1.2

106

3.9 5.2 8.0 8.2

x 10-i

::rn one reactant to another. There is an equilibrium


;,nstanl associated with this process that is predictable based - r the pKz values of the two acids in the reaction (one acid is

x
x

104

10-9

I04

i :eactant, and the other

acid is a product).

To determine the Ku value for organic acids, an organic

Table
constant greater than 1?

:;r,l is

l:rilibrium constant (K"q) is

added quantitatively

to an organic base. The determined from the

71. Which of the following reactions has


A

an equilibrium

--ncentration of each species, once equilibrium is reached. -r-;;ction 1 is a generic reaction between an organic acid and .:,e conjugate base of a second organic acid

HA + B-

==::\

A- +
1

HB

. B. C.

CI3CH + H3CCH2S-

-+ H3COH + H2CNO2- + H3CCOCH3 + H3CO- *

C13C'+ H3CCH2SH

H3CO- + H3CNO2 H3CCOCH2-+H3COH

Reaction

D.

H3CCOCH2COCH3 + C13C-

Equation 1 can be used to determine the equilibrium : , rstant for Reaction I .

H3CCOCHCOCH3- + CI3CH

H* = tA-llFIBl '

Ka (acid HA) Ka (acid HB)

tHAltBl

= tg(pKa

(HB) - pKa (HA))

72. Which of the following compounds can deprotonate


c5H6?

Equation I
A series of six different organic acids are treated with -r;lopentadienyl anion, as shown in Reaction 2.
1,3-

A. H3CCOCH3 B. H3CCH2SH
C. H3COD. HzCNOz-

HA+CsHs-5+A-+C5H6
Reaction
The structur"e
2

of 1,3,-cyclopentadienyl anion is shown in


7

L:ure

1,

3.

For the following reaction: H3CCH2SH +

H3Co- --

H3CCH2S- + H3COH

what is true about the relative concentrations of each species at equilibrium, if the reactants are mixed in
equal molar portions?

Figure

1,3-Cyclopentadienyl anion

A. B. C. D.

[H3CO-] > [H3CCHzS-]; [H:Co-] > [H:CoH]; [H3CCH2SH] > [H:CO-]


[H3CCH2S-] > [H:CO-]; [H:COH] > [H:CO-];
[H3CCH2SH] > [H:CO-]

. determined using UV-visible spectroscopy whenever :,:ssible. In cases where no n-bond is present in both the
::actant acid and the product acid, the concentrations are ::termined using gas chromatography. The conjugate base trd reactant base concentlations are determined by the :rference between initial acid concentration and equilibrium

The concentration of each organic species at equilibrium

[H3Co-] > [H:CCHzS-]; [H:CO-] > [H:COH]; [H3CCH2SH] = [H3CO-1 [H3CCH2S-] > [H3CO-]; [H:COH] > [H:CO-]; [H3CCH2SH] = [H3CO-J

::id concentration. The concentrations are used to determine


rquilibrium constants. The calculated values are compared to ,:lues found using pKu numbers in Equation 1. It is found

iat

the error is greatest when

Kst is greater than 104.

Jopyright O by The Berkeley Review@

7l

GO ON TO THE NEXT PAGE

74, The acidity of the C5H6 is


hydrocarbons, because:

abnormally high for

Passage

Xll

(Questions 78 - 84)

79.

A. B. C.

it is aromatic.
it has an aromatic conjugate
base.

its hydrogens withdraw electron density through the inductive effect.

D. in the conjugate

base, the hydrogens withdraw

electron density through the inductive effect.

The effect of atomic size on reactivity is perhaps most pronounced when comparing the reactivity of thiols and alcohols. In protic solvents, such as water, alkoxides (RO-) are less nucleophilic than alkyl sulfides (RS-), because alkoxides are capable of forming hydrogen bonds. In aprotic solvents, alkyl sulfides are less nucleophilic than alkoxides,
because they are less basic and thus less able to donate their lone pair of electrons to an electrophile. To compare the nucleophilicity of alkoxides and alkyl sulfides, Reaction 1 is carried out with a range of combinations of one solvent and one nucleophile.

^l

-9"

5.

Which of the following relationships accurately shows


the relative pKa values for the given acids?

tlrf
H:C

&

.l

A. pKalct3cH) > pKa(cH3oH) > pKa(HqcNoz) B. pKagr3cNoz) > pKalgu3oH) > pKalgr3cH) C. pKalcn3oH) >pKalgl3CH) > pKalu3cNo2; D. pKalcu3oH) > pKalg3cNo2) >pKalcr3cH)

Tffi n.c4ff.
Reaction

Nuc-

+r

I
it

6.

Table I lists the negative logs of the reaction rates for Reaction 1 observed in a series of solvents reacting with a series of nucleophiles. In each case, the reaction is carried out at 30'C, and with an initial concentrarion of 0.10 M for the nucleophile and of 0.1 1 M for the electrophile. Which of the following compounds is NOT an example
of
a

fr. tf
A

:F

ni{

Lewis acid?

Nucleophilic H3COH3CS-

Solvent
Ether Ether Ether Ether Ethanol Ethanol Ethanol Ethanol
Water Water

-Log rate

a
C

A. CI3CH B. H3CNO2 C. NaCH3 D. BFr

t.44
1
9',7

HO'
HS-

1.03

1.16 3.19

H3COH3CS-

1.

-\,r

2.12
3.35

fol

HO-

77.

Which of the following acids has a pKa value close 10.0, given that the pKa for C5H6 is 15.0?

HSH3CO

r.96 4.22
.t ^'7

A. B. C. D.

H3CCOCH3

H:CS
HOHS

A. B. C. D.

CI3CH
H3COH H3CNO2

'Water
Water

5.62 2.14

Table I
Because the value in Table 1 is the negative log of the rate, the magnitude of the effect of nucleophile and solvent on

ft'c
Se

-{.
B.

the reaction rate is not immediately apparent. The smaller

the negative 1og of the rate, the greater the rate. The difference between the rate of SH- and the rate of RS- is attributable to differences in their molecular size. The difference in reaction rates btween the various solvents is attributable to a change in the mechanism from Sy2-like to Syl-like (as the solvent changes from ether to water). The
more the solvent binds the nucleophile, the less rapidly the nucleophile can attack the electrophile and thus the slower the rate of the nucieophilic substitution reaction. This affects the reaction rate of nucleophilic substitution in a protic solvent.

C. D.

Copyright O by The Berkeley Review@

GO ON TO THE NEXT PAGE

)st nd

8.

Which of the following sets of conditions results in the


FASTEST reaction rate?

3.

Which of the following changes results in an increase in


the reaction rate?

I)
ise

tic
3S,

A. B. C. D.

An An An An

alkoxide in an aprotic solvent alkoxide in a protic solvent alkyi sulfide in an aprotic solvent alkyl sulfide in a protic solvent

A. B. C. D.

Changing the nucleophile from HS- to H3CSChanging the solvent from ether to water Changing the reaction temperature from 30'C to
25"C

Changing the solvent from water to ethanol

eir
he

is
nd

9.

What value should be expected for the negative log of the reaction rate, if ethyl sulfide (CHjCH2S-) were
added to 2-iodopropane in ether solvent?

4. In comparing
A

the properties

of alkoxides and alkyl

sulfides dissolved in ether, one notes that alkoxides have

A. B.

1.05

c. 3.09

2.04 4.52

D.
for

. B. C. D.

higher pK6 and react faster than alkyl sulfides. higher pK6 and react slower than alkyl sulfides.

lower pK6 and react faster than alkyl sulfides. lower pK6 and react slower than alkyl sulfides.

na
ied

\ill

If iodine was replaced with bromine in Reaction 1, one should expect that lhe negative log of the reaction rate
would:

for

. B. C. D.
A

increase, and the reaction rate would increase. decrease, while the reaction rate would increase. increase, while the reaction rate would decrease.
decrease, and thc reaction rate would decrease.

1. According to the data in Table 1, which of


lbllowing bonds is the WEAKEST?

the

.{. c-H B. C-I


C. C-O D. C-S

From the data in Table 1, what can be concluded about the effect of hydrogen-bonding on reaction rate?
the

ton
rller
The
)lS

A. B. C. D.

The rs is

eto
The the
r the

Hydrogen bonding hindels nucleophilic attack and thus lowers the reaction rate. Hydrogen bonding enhances nucleophilic attack and thus lowers the reaction rate. Hydrogen bonding hinders nucleophilic attack and thus increases the reaction rate. Hydrogen bonding enhances nucleophilic attack and thus increases the reaction rate.

; the
:nt.

GE

-..pvright O by The Berkeley Review@

GO ON TO THE NEXT PAGE

Passage

Xlll

(Questions 85 - 91)

86. The nucleophilicity ofeach reagent in

aqueous solution:

Esters are semi-reactive carbonyl compounds that undergo substitution chenristry at the carbonyl carbon.
Several biological reactions (including transesterification and transamination) proceed through standard carbonyl chemistry. The reactivity of the carbonyl depends on both the nucleophile and the leaving group. A researcher set out to

. . C, D.
A
B

decreases as the pH is increased.

is best when the species is a cation. is best when the species is neutral.
depends only on the size of the nucleophile.

illi

determine the reactivity of three dil'f'erent nucleophiles in a standard substitution reaction using an ester electrophilc. For the reaction in Figure 1, three diff'erent compounds (Compound A, Compound B, and Compound C) were used.

87. Which of the following compounds would be the


MOST reactive nucleophile at pH = 9.0? A. H3CCH2CH2OCH3

ol,cur
o
.-------->

B. H3CCH2CCO2CH3 C. H3CCH2CH2CONH2 D. H3CCH2CH2CH2NH2


88.

NI

If

the pKu fbr H3CNH3+ is 10.3, which of the following is the BEST approximation for the pK6 for
17.3 12.3

CH 1

Cl3CNH3+?

Figure

Deacylation of an ester

A. B.

Contpound A has the forniula C13H24S, Compound B has the lormula C9H1gO, ancl Cornpound C has the folrnula C7H9N. Figure 2 shows three graphs ciepicting the change in reaction rate of the deacyltrtion reaction as a function of the solution pH fbr each of the thlee separate compounds.
Compound Cornpound Cornpound C

D. 1.0
89. If the pKu for NH4+ is 9.25, then pK6 for NH3 must
be equal to which of the fbllowing?

c.

8.3

A. B.

9.25

c. 5.75
4.15

7.00

D.
6.0

pH

1.0

8.0 6

pH
2

7.0

8.0 6

7.0

8.0

pH

90. How does the hybridization of the carbonyl carbon


change during the reaction'J

Figure

Reaction rate as a lunction ol'solution pH

Each reaction obeys standard mechanistic behavior fbr carbonyl substitution. They arc believed to proceed throLrgh a

mechanism where the nucleopl-rile attacks the calbonyl carbon, breaking the C=O n-boncl and forming a tetrahedral

A, B. C. D.
9

It changes fiom tp2 to tp3 and back to sp2. It changes from tp3 to t1t2 and back to spj. It rernains sp2 throughout the reaction. It remains spJ throughout the reaction.

intermediate. A lone pair ol clectrons on oxygen then reforms the n-bond, ejecting lhe leaving group. Although the nucleophilicity of the dil'fcrent compounds is not equal, the similar mechanisms make the reactions comparable. At low pH, the carbonyl compound can be pl'otonated, making it a better electrophile. This ncgates the effbct of decreasing nucleophilicity of alcohols and thiols, beceruse they remain
uncharged at low pH values.
85

1.

Aniline and benzylamine, drawn below, are both:


NHz

Aniline

Benzylamine

Which of the tbllowing types \lOST basic'l

ol conipounds is tl.re

.{, B. C. D.

Primary alcohols
Esters Secondary amines

A. B. C. D.

primary amines.
aromatic amines. conjugated amines.

nonalkyl amines.

Teltiary thiols

Coprright O by The Berkeley Reviewo

GO ON TO THE NEXT PAGE

@irqplifi

Questions

92

through

97.

Which of the following isomers has the HIGHEST


boiling point?

descriptive passage.

2.

Which of the following compounds has the HIGHEST boiling point?

A. H3CCH2OCH2CH3 B. H3CCH2CH2OCH3 C. (H3C)2CHoCH3 D. H3CCH2CH2CH2OH

A. 2-pentene B. Diethyl ether C. Heptanal D. Cyclohexanol .

8.

Which of the following molecules is NOT polar?

,1

The correct IUPAC name for the following molecule is:

. . C. D.
A
B

cis-1,3-dichlorocyclopentane
trans1 ,3-dichlorocyclopentane E-1,4-dichloro-2-butene 1,2,2,3-tetrabromopropane

9. Which of the following

compounds releases the

GREATEST amount of heat upon combustion?

A.\
B

-\

. . C. D.
A

2,4-diethyl-3,5-dimethylheptane 4-ethyl-3,5,6-trimethyloctane 3-ethyl-5,6-dimethylnonane


3,5,6-trimethyldecane

4. Which of the following compounds


conjugation?

exhibits

C.

I. 1,4-cyclohexadiene II. 3-ethylcyclohexene m. 2-methyl-1,3-cyclopentadiene A. I only B. II only C . III only D . I and III only
5.

o
CH(oCH:

D.

100. Which of the following functional


C2H5
)C (o) CH( CH:) z?

groups is found in

'r

Which of the following compounds is MOST stable?

A. Aldehyde B. Ester C. Ketone D. Oxirane

. B. C. D.
A

2-methyl-I,4-pentadiene
3-methyl-1,4-pentadiene

2-methyl-l,3-pentadiene
1.S-hexadiene

i 6. Which of the following molecules would have a dipole


moment NOT equal to zero?

L Z-1,4-dichloro-2-butene II. E-1,4-dichloro-2-butene m. cis-1,2-dichlorocyclopentane A. Compound I only B. Compound II only C. Compound III only D . Compounds I and III only
Copyright
@

l.D 2.A 3.C 4.A 5.D 6.D '7.8 8D 9.D 10.8 11.8 12.A 13. D 14. A 15. C 16. B r1. B 18. C 19. D 20. B 2t. C 22. A 23. D 24. D 25. C 26. A 21. D 28. A 29. C 30. A 31. A 32. B 33. D 34. A 35. A 36. C 31. C 38. C 39. B 40. C 41. D 42. B 43. B 44. A 45. A 46. B 47. D 48. D 49. A 50. D 51. A 52. A 53. D 54. B 55, D 56. A 51. D 58. D s9. D 60. B 61. D 62. D 63. D 64. A 65. D 66. A 61. D 68. C 69. C 70. A 11. D ',72. C 13. D 14. B 15. A 16. C 17. D 78. A 79. B 80. C 81. B 82. A 83. D 84. C 85. C 86. C 87. D 88. C 89. D 90. A 91. A 92. D 93" B 94. C 95. C 96. D 91. D 98. C 99. B 100. C
/5

by The Berkeley Review@

iCHEMICA ES FIN!

Structure, Bonding, and Reactivity Passage Answers


Choice D is correct. Table 1 does not list any aikynes, so the bond energies must be estimated from trends in the data. According to data in Table 1, a bond between two spJ-hybidized carbons has a bond dissociation energy, BDE, between 81 and 88 kcals/mole. A bond between an sp2-hybridized carbon and an sp3-hybtidi"ed carbon has a BDE ol 97 kcals/mole. This means that the bond between C2 and C3, a bond between an sp-hybridized carbon and an sp3-hybridired carbon, should have a BDE greater than97 kcals/mole. This eliminates choices A and B. A bond between a hydrogen and an sp3-hybtidi"ed carbon has a BDE between 91 and 104 kcals/mole. A bond between a hydrogen and an sp2-hybtidited carbon has a BDE of 108 kcals,imole. This means that the bond between hydrogeir and an sp-hybridized carbon should have a BDE greater than 108 kcals/moie. This eliminates choice C and makes choice D the best answer.

Choice A is correct. According to the bond dissociation energies listed in Table 1, iodine and chlorine both make their strongest bonds to methyi carbons. It thus can be assumed that bromine would exhibit the same behavior as these other halides, and that the strongest bond to bromine is formed by a methyl carbon. Choice A is best.
Choice C is correct. Table 1 shows an increase in bond strength for bonds formed between equivalent carbons and atoms of decreasing size (I, C1, ar-rd O). From this, it can be concluded that shorter bonds are generally stronger than longer bonds, and that as atomic size decreases, the bond length to a neighboring atom decreases. No bondlength data are provided in the table, but this can be inferred from the passage. Choice C is best.

J.

4.

Choice A is correct. The hydrogenation of an unsubstituted alkene yields the greatest energy (according to the heats of reaction listed in Table 2). A less stable reactant yields a greater amount of heat upon reaction, so the n-bond must be weakest in an unsubstituted alkene. The energy generated from oxidative cleavage, or any reaction that breaks the n-bond, is greatest when the alkene is unsubstituted. The best answer is choice A. Choice D is correct. The difference in reactivity between the cis and trans geometrical isomers of an alkene is attributecl to i.ntramolecular steric hindrance in the cis compound, because the substituents are on the same side of the molecule. The resonance, hybridization, and electronegativity of carbon are the same in both geometrical isomers of the alkene. This eliminates choices A, B, and C and makes the best answer choice D.

5.

6.

Choice D is correct. This question should be a lvelcome freebie, relative to other questions in the passage. Alkene carbons have sp2-hybridization, and alkane carbons have sp3-hybridization. Two of the three carbons in the compound are alkene carbons, while tire other carbon is an alkane carbon. The best answer is choice D, two sp2-hybriclized carbons and one sp3-hybtidized carbon. Pick choice D to get your point for correctness.
Choice B is correct. Table 1 lists the bond dissociation energy for various bonds, which is the energy required to break the bond in a homoiytic fashion. This in essence means that Table 1 lists the bond strength. The strongest bond, according to that data listed in Table 1, is the bond with the highest bond energy. The highest value among the answer choices is the bond between the methyl carbon and chlorine. The best answer is choice B. Choice D is correct. Double bonds are stronger than single bonds, meaning that more energy is required to break a double bond than a single bond. Choices A and B are thus eliminated. The lower heat of hydrogenation in

7.

the second chart implies that the reactant alkene molecuie is more stable. The more stable the alkene compound, the stronger its n-bond. This means that the double bond in the tetrasubstituted aikene is stronger
than the double bond in the unsubstituted alkene. The best answer is choice D.

Choice D is correct. To convert the acid form of a nitrogen-containilg compound (in this case, a protonated imine species) into a neutral species, you must add a strong base (strong enough to deprotonate the iminium cation). Choice A is eliminated, because it is a strong acid, which protonates rather than deprotonates the compound. Choice B is eliminated, because it is inert and has no effect on caffeile. Choice C and choice D are both bases, but the stronger base is NaOH, so choice D is the better choice. The carbonate base is not strong enough to fu111' deprotonate ihu i*ir-ri.rrrr cation. For best results in a case like this, pick choice D.

Copyright C bv The Berkcley Reviewo

76

Section I Detailed Dxplanations

E
r

Choice B is correct. We know that the nitrogen in questions has sp2-hybridization for three reasons: It is involved in one rc-bond, it has three substituents attached and no lone pairs, and it is pianar with respect to the adjacent atoms bonded to it, with bond angles of approximately 120'. For so many good chemistry reasons, the nitrogen in question is sp2-hybridized,, so why choose anything but choice B?
Choice B is correct. Because of the delocalization of electron density throughout the n-network of the caffeine molecule (achieved by the resonance between all adjacent non-hydrogen atoms), the compound must be planar to allow the p-orbitals to overlap correctly. This means that all of the atoms in a caffeine molecule, and thus all of the nitrogens, must be coplanar with respect to one another. Pick choice B for a whopping good answer. Choice A is correct. Because of the resonance donation of the lone pair of electrons on nitrogen to the carbonyl carbon, the C=O bond takes on some single-bond properti"es, which makes it longer than a standard sprhybridized C=O bond that lacks any resonance effect. The best choice is therefore choice A. |ust as a side note, the C=O bond in formaldehyde is longer than the C=O bond in carbon dioxide because of the varying hybridization of carbon in the two compounds. In formaldehyde the hybridization of carbon is sp2,while in carbon dioxide it is sp. The more s-character in the hybrid orbital, the shorter the orbital and thus the shorter the bond. The stable resonance structures for an amide are shown below:

the

tW'
'bon ized
rices

role.

:the Ihis

rake

vior
-t ,1,

and llger

lt

:nd-

\r.-/t\ N-

<----->

:9: lF \gZL\ N-

..o

longer than a stanc{ard C=Obond

tl

the r the any


r

Choice D is correct. Ionic forces are stronger than polar forces, so the ionic compound (acid salt form) should have both a higher melting and a higher boiling point than the polar uncharged compound (free-base form). This makes choice D correct, and you want that which is conect. Follow society's infiuence and pick D.

Choice A is correct. If the N-CH3 group is replaced by an oxygen atom, the compound that remains has an oxygen between two carbonyls. This is referred to as an acid nnhydride (from the fact that the compounds forms when two acids combine in a dehydration reaction). The best answer is choice A. There is no such term as an "acid ester", so choice B is elin-rinated. An ester inl,olves just one carbonyl, so choice C is eliminated. A lactone is a cyclic ester, and given that the compound is not an ester at all, it can't be a lactone. Choice D is eliminated.

-5. Choice C is correct. A

good soivent for dissolving a dye to form an ink is one that is a liquid at roorn temperature, evaporates quickly, and exhibits a high degree of dye solubility. Having a high vapor pressure impiies that it evaporates readily, so choice A is eliminated. If it has functional groups that are similar to the dye, then the dye is like1y to be highly soluble in the solvent, so choice B is eiiminated. If the boiling point is slightly above room temperature, then it is a volatile liquid, so choice D is eliminated. The solvent should not react with paper (celluiose), so the correct answer is choice C. Choice B is correct. Becar-rse like dissolves like, the best solvent for dissolving the dye should also have hydroxyl groups attached to it, just as the dye does. The best choice is therefore the alcohol, The carboxylic acid is not a good choice, because carboxylic acids are not as volatile as alcohols. If you have melting points memorized, then yor-l may be aware that carboxylic acids that are three carbons or greater in length are solids at room temperature. If yor-r don't have them memori.zed, like 99.999"/0 of us, that's okay too. Aldehydes and ketones may work, but not as well as the aicohol. The best choice is B. Choice B is correct. Determining the IUPAC name for the compound requires that you count the longest continuous chain of carbon atoms in this straight chain compound, which yields a total of ten. Next, you must identify all functional groups on the molecu1e, including alkyl groups that are not a part of the carbon backbone. The only functional group on this compound is a carboxylic acid. If there were multiple functional groups, the more oxidized functior-ral gror,rp gets higher priority in the name of the compound. For instance, if there were also an OH group, then it would be namecl a hydroxy substituent, rather than an alcohol. Having ten carbons and a carboxylic acid group makes the compound decanoic acid. Choice B is the best answer.
@

reak
rn

-:"

in

(ene nger

q
nrne ion). und.
ases/

:-.

rully

-:ryright

by The Berkeley Review@

77

Section I Detaited Explanations

18.

Choice C is correct. When water is spilled on paper, it diffuses across the surface of the paper. If the ink bound to the paper is soluble in water, it dissolves into the water and spreads out, or runs. So if an ink runs, it must be soluble in water. The colored portion of the ink is the dye, not the solvent. Running ink means ihat the ink dye is water soluble. Pick choice C for best results.

19.

Choice D is correct. As stated in the last paragraph of the passage, a compound must contain at least eight carbons in its chain to be a good soap. The best soap has a polar and nonpolar end associated with the molecule. The negatively charged carboxylate is at one end and an organic tail is at the other end. Molecules with charged and organic ends are optimal for making a soap. Choice D has both eight carbons and a charged end.
Choice B is correct. To be soluble in water, a compound must be either charged or polar. Because choices B and D are ionic, they are better in this regard than choices A or C. The organic tail is smaller in choice B, so it dissolves into water more readily than choice D. Pick choice B, and feel the sensation of correctness.

20.

21.

Choice C is correct. Carboxylates are formed when a carboxylic acid is deprotonated. The Grignard reagent in A deprotonates the carboxylic acid to form the carboxylate, but the cation is not sodium, so choice A is eliminated. Choice B is invalid, because the Grignard reagent deprotonates the carboxylic acid (formic acid), and the carboxylic acid does not have enough carbons to make sodium acetate. Choice D is invalid, because it has too many carbons (propanoic acid has three carbons). The H3CCO2Na molecule results from the deprotonation of acetic acid by a base with a counterion of Na+. The best choice is therefore choice C.
choice

Choice A is correct. Because of the resonance donation from nitrogen, the carbonyl bond (C=O) of an amide has some single-bond character. Since a single bond is longer than a double bond, the single-bond character of the amide carbonyi bond results in a longer carbonyl bond than the unconjr-rgated carbonyl (as observed with the

ketone). This makes statement I a false (not true) statement. Because of the previously mentioned resonance, the carbon-nitrogen bond has some double-bond character, making it shorter than a standard carbon-nitrogen single bond (as seen with a primary amine). This makes statement II a true statement. Because of the resonance, the carbonyl oxygen carries a partial negative charge. This makes the oxygen more basic than typical carbonyl oxygens (such as the one in an aldehyde). Statement III is also a true statement. Only statement I is ttot true, so choice A is the best answer.
23.

Choice D is correct. As emphasized in the passage, nitrogen donates electron density to oxygen through resonance. This places a partial negative charge on oxygen (increasing its basicity) and a partial positive charge on nitrogen (decreasing its basicity). Choice C is thus eliminated, and choice D is correct. Choice A should be eliminated, because oxygen is more electronegative than nitrogen. Choice B should be eliminated, because oxygen is smaller than nitrogen. If you want to do what you should do, pick D and gain incredible satisfaction doing what you should do. Choice D is correct. Because of the resonance donation from nitrogen, the nitrogen has sp2-hybridization. Having sp2-hybridization results in trigonal planar geometry. The carbonyl carbon aiso has trigonal planar
geometry, so the central two atoms force the three hydrogens and one oxygen to assume a coplanar orientation. Choices A, B, and C therefore ail must be eliminated as incorrect geometric descriptions, making choice D the best answer. The two resonance forms and the resonance hvbrid are drawn below:

fll =

o
il

oI

o6

sp2-Hybridizati,n
Trigonal planar

I'

,.'C\

N
I

/H

n-----------*

H/ \N*-H
I

-7 Cf. ./H H a]T---._


H

li

sp2-Hybridization Trigonal planar

25.

Choice C is correct. From the passage, we know that amides are protonated at the carbonyl oxygen, so choice D is eliminated. Because nitrogen is less electronegative than oxygen, it donates more electron density to the carbonyl oxygen (through resonance) than the ester oxygen donates to the ester carbonyl oxygen. This places a larger partial negative charge on the amide carbonyl oxygen than or-r the ester carbonyl oxygen. The larger negative charge makes Site c the most basic site. Choice C is therefore the best answer.

Copyright O by The BerkeJey Review@

7a

Section I Detailed Explanations

Choice A is correct. The most stable hydrogen bond forms between the best lone-pair donor (most basic site) and the hydrogen with the greatest partial positive charge. Because amides are protonated at the carbonyl oxygen (as stated and drawn in the passage), the carbonyl oxygen is most basic and thus donates one lone pair of its electrons. This eliminates both choice C and choice D, Of the answer choices remaining, the hydrogen on nitrogen carries the partial positive charge, not the hydrogen on carbon. This means that the hydrogen bond forms between the carbonyl oxygen and the hydrogen bonded to nitrogen, thus the best answer is choice A. If you are unsure, think of the hydrogen-bonding in B-pleated sheets.

Choice D is correct. Choice A is the most stable resonance structure of the amide (all octet and no formal charges are present). When nitrogen donates electron density to oxygen, choice B becomes the resonance structure. This is a minor contributor due to the formation of charges on the molecule. The fact that it is an alloctet resonance structure is favorable. If the nitrogen were to pull its n-electrons back from the carbon in answer choice B, the resonance structure represented by answer choice C would be formed. Because carbon does not have a complete octet in this resonance structure, it is a very minor contributor, but it is none-the-less a resonance structure of the amide. It is not possible to form a double bond to the R-group, because that would require five bonds to carbon. In order for carbon to donate in that manner (and have only four bonds), it must have had a lone pair (and thus a negative charge) in the original structure. R was not drawn as having a lone pair, so it is assumed that the R represents a standard alkyl group. The best answer is therefore choice D.

r\

,. c5-R' ..1\
I

/o \il

oI

oI

o.r\*,R'

H
Choice A (major)

HH
Choice B (minor) Choice C (very minor)

lt

-* o-tlN.R

Choice A is correct. Given that the molecular masses of the three compounds are roughly equal (59 g/mol, 58 g/llrol, and 44 g/mol), the top consideration for determining their boiling points is the intermolecular forces. An amide has hydrogen-bonding, while a ketone and a hydrocarbon do not. At room temperature, most amides are solids, acetone is a volatile liquid, and propane is a gas. Based strictly on the phases, the best answer (and the only answer that lists acetamide as the highest) is choice A, BP'sslnmide > BPacetor.,u > BPpropane. Acetone has a higher boiiing point than propane, because it is polar and more massive.

Choice C is correct. Chemist 1 considers molecular mass to determine the relative boiling points of compounds. We are looking for the exception, so the correct answer is the choice where the lighter compound has the higher boiling point (or lower vapor pressure). A higher vapor pressure at room temperature corresponds with a lower boiling point. In choice A, the heavier compound of the two has the higher boiling point, so choice A is not an exception to Chemist 1's general rule. In choice B, the heavier compound of the two has the lower vapor pressure (and thus higher boiling point), so choice B also follows the rule. In choice D, the heavier compound of the two has the lower vapor pressure (and thus higher boiling point), so choice D follows the rule too. In choice C, the heavier compound of the two has the lower boiling point, so choice C contradicts Chemist 1's theory. Choice A is correct. Chemist 2 considers intermolecular forces to determine the boiling point of a compound. The strongest intermolecular forces correspond to the highest boiling point. The passage states that alcohols have stronger hydrogen-bonding than amines. This means that the alcohol (I) has the highest boiling point (and thus is listed first), because ethanol has the strongest H-bonds of the three compounds. The ether (II) has the lowest boiling point, because it cannot form hydrogen bonds with itself (due to its lack of an electropositive hydrogen). The order of the boiling points is therefore: I > III > II, making choice A the best answer. Choice A is correct. Hydrogenation is the addition of H2 gas to an alkene to break the n-bond and reduce the compound to an alkane. For every n-bond that is lost by the alkene molecule, two hydrogens are gained. This increases both the molecular weight of the compound and the molecular flexibility of the compound (the product is both more massive and more flexible than the reactant). Both of these effects increase the melting point of the compound, making the melting point of the product greater than the melting point of the reactant. This makes choice A the correct answer. Pick A and you'il be an MCAT supernova.

- :vright O by The Berkeley Review@

79

Section I Detailed Explanations

32.

Choice B is correct. According to Chemist 2 (and as a general rule), the stronger the intermolecular forces, the greater the boiling point for a compound. The greater the boiling point for a compound, the less of it there is that evaporates, thus the lower its vapor pressure. Pick choice B, to correctly interpret the logic of Chemist 2.

33.

Choice D is correct. Higher elevation means fewer molecules of gas per volume of air, and thus a lower atmospheric pressure. The elevation and atmospheric pressure have no effect on the intermolecular forces between molecules. Flowever, the lower atmospheric pressure means that less energy (heat) is required to rgach a temperature at which the vapor pressure (Prrupo.) is equal to the atmospheric p."rs.tte (Patmosph"ric), the definition of the boiling point. The boiling point is therefore lowered as elevation increases. This'makes choice D correct. Choice A is correct. The boiling points of Compounds I and II are directly comparable, because they are geometrical isomers. Compound I (the cis isomer) is poiar, while Compound II (the trans isomer) is nonpolar. This means that the boiling point of Compound I is greater than the boiling point of Compound II, which eliminates choice C. Because Compound III is an alkane, it is flexible (whereas Compounds i and II are rigid, due to the n-bond), so Compound III is able to rotate between conformers. The most stable conformation of Compound III is nonpolar, but because it can assume polar conformations on occasion, the compound is slightly polar. The boiling point of Compound III is less than the boiling point of Compound L Compound III shbuld have the second highest boiling point, because it is slightly polar, while Compound II is non-polar. Thus, the correct order is I > IiI > II, making choice A the best answer.

34.

J5.

Choice A is correct. The greatest reduction in voltage is caused by the compound with the greatest dielectric constant. The greatest dielectric constant is associated with the most polar compound. Choices B, C, and D are all symmetric, so they are a1l nonpolar. This eliminates choices B, C, and D. Only ArHF (choice A) is polar, meaning that ArHF has the greatest dielectric constant. Choice A is a fine choice in a situation like this.

36.

Choice C is correct. The hydrolysis of an alkene forms an alcohol. An alcohol is polar, so choice A is eliminated. The halogenation of an alkane forms an alkyl halide. An alkyl halide is polar, so choice B is eliminated. The hydrogenation of an alkene forms an alkane. An alkane is most often nonpolar, so the best answer is choice C. Reduction of an amide forms a primary amine. An amine is polar, so choice D is eliminated.
Choice C is correct. Assuming that an alkyl iodide is polar to begin with, then replacing iodine with bromine results in a more polar compound, because bromine is more electronegative than iodine, so that the difference in

J/.

electronegativity between the halogen and carbon has increased. A carboxylic acid is more polar than a primary alcohol (or any alcohol, for that matter), so choice B results in a more polar compound. Alkenes are typically nonpolar, so the addition of HBr forms an alkyl bromide, which increases the polarity, so choice D is eliminated. Because fluorine is more electronegative than chlorine, replacing a fh,rorine substituent with a chiorine substituent results in a compound that is less polar, making choice C the choice that does nof result in increased poiarity. Pick choice C to be a star of chemistry.
38.

Choice C is correct. The dipole moment changes only when a compound's bonds are either stretched or bent, if the compound is asymmetric. This makes choices A and B less likely to exhibit the least change in dipole moment. The dipole moment does not change drastically (if at al1), when the chemical bonds of a symmetric compound are either bent or stretched. Therefore, the least change in dipole is observed in a symmetric molecule. Stretching a symmetric molecule often balances out, meaning that the electron density is shifted uniformly in opposing directions, The result is that the dipole of the molecule does not change. The best answer is choice C. Bending a symmetric molecule can make it asymmetric, so choice D is not as good as choice C.

39.

Choice B is correct. To be nonpolar, all of the ligands must pu1l in such a way that the vectors of each individual bond cancel out. Tetrahedral structures are not possible with six ligands, so choices C and D are eliminated. It is only when the two chlorine ligands are trans to one another that they cancel out one another in terms of polarity. The best answer is therefore choice B.
Choice C is correct. A dielectric constant of zero results from a nonpolar molecule. The only nonpolar molecule among the answer choices is trans dichloroethene. Cis alkenes are polar, so choice B is eliminated. Choice D is trans, but there are different substituents on each carbon, so it is polar. The best answer is choice C.
@

40.

Copyright

by The Berkeley Review@

80

Section I Detailed Dxplanations

1e
15

:1,

CI
ES

Choice D is correct. Choice A is nonpolar, because the vectors expressing the electron withdrawal of the fiuorines cancel out, so choice A is eliminated. Fluorine is more electronegative than chlorine, so the asymrnetric electron distribution is found n'ith choices C and D, which rules out choice B. In choices C and D, the fluorine atoms withdraw the electron der-rsity, making the molecule asymmetric. In the propane molecule (choice D), the methyl substituent donates eiectron density to the electron-poor central carbon, placing a partial positive charge on the methyl groupi therefore, it increases the dipole moment. This makes the
propane molecule more polar than the ethane molecule. The structures are shown below:

to
:),
ES

re
rr.

,h
.1,

All vectors cancel

.'. nonpolar

C-F vectors cancel, br-rt methyl donates

of

.'. slightly polar

lr.
td
1e

I
]C

Vectors almost cancel .'.

slightly polar

C-F vectors almost cancel C-Cl vectors,

but methyl donates .'. polar Choice B is correct. For a tetrahedral structure, if the four ligands are not all equivalent, then the structure is asymrnetric. If the compound is asymmetric, then it must be polar (have an asymmetric distribution of electron der-rsity). This makes statement I true. Figure 1 shows an example of a 1,4-disubstituted cyclohexane molecuie

.t,

Lc

is

St

that is tlot polat, which means that statement Ii is rzol true. A11 optically active compounds must be asymmetric in order to be optically active, so they must be at least slightly polar, This makes statement III trtre. Only statement lI is not true, so choice B is the best answer.

'.e

tn
a

:e is

Choice B is correct. A dication carries a +2 charge, so it must be coupled with an anion that is organically soluble. The only organic anion among the answer choices is choice B.
Choice A is correct. A micelle turns inside out from its aqueous structure rt'hen it is added to an organic solvent (as stated in the passage). Figure 1 shows a micelle as it appears in water, where the polar heads are exposed to the liquid, and the organic tails are protected in the core. In a hydrophobic (organic) solvent, the organic tails are exposecl, and the polar heads form a protective core. This is best illustrated in choice A. Choice C may look familiar, in tl-Lat cell membranes arrange themselves in such a manner. Choice D is a "throw-away" answer, because the tail and the head of the compound exhibit no attractive forces.
Choice A is correct. To be absorbed through respiration, a cornpouncl must be a gas or a vapor, because only gases are absorbeci through respiration. This means that any compound intended to be taken into the lung must be either a gas or a liquid r,r,ith a low boiiing (one with a high vapor pressure). There is hydrogen-bonding in choices B, C, ancl D, but not in choice A. Hydrogen-bonding increases the boiling point and thus lowers the vapor pressure. All of the compounds have roughly comparable masses (either 73 or 74 grams per mole). The only factor to consider in approximating the relative boiting points is hydroger-r-bonding. The best answer is the ether, choice A. As a point of trivia, it is estimated ihat the average human adult takes in approximately
3500 gallons of air a

_:1

day. Just thought you might like to know.

1S

Choice B is correct. To be water-soluble, the compound should be able to form hydrogen bonds. Choices C and D are eliminated immediately, because they are hydrophobic. Although choice A has an alcohol group, it is primaril;z organic. Choice B has two hydroxyl groups and an amide group. All of these functional groups form hvdrogen bonds, so choice B exhibits the greatest amount of hydrogen-bonding. The best answer is choice B.

'-'vright O by The Berkeley Revien'@

8l

Section I Detailed Explanations

47.

Choice D is correct. The best micelle has an ionic (charged) head and a long carbon chain for its organic tail. Choices A and C are eliminated, because they have uncharged heads. Choice D is a better answer than choice B, because it has a longer organic tail.
Choice D is correct. A micelle enhances the water solubility of a compound that is normally insoluble in water. This question therefore is asking for the least watersoluble compound. An alcohol, a carboxylic acid, and an amine all exhibit hydrogen-bonding (although within a tertiary amine, there is no protic hydrogen for hydrogen-bonding), so they should all be water-soluble to some degree. Because an alkene has no hydrogenbonding (it has neither a lone pair nor an electropositive hydrogen), it is unlikely that it would be watersoluble at all. The best answer of the given choices is therefore choice D. Choice A is correct. The organic tails of micelles are held together by the weak attraction associated with van der Waals forces (choice A). The organic tails are alkyl-based, so they are nonpolar, and they contain neither nitrogen, oxygen/ nor fluorine. This means that choices B and C are both eliminated, because to form hydrogen bonds, a compound must have an electropositive hydrogen bonded to either nitrogen, oxygen, or fluorine. Choice D is eliminated, because covalent bonds are formed in chemicai reactions, and the organic tails in micelles exhibit only attractive forces, nothing as strong as covalent bonding. The best answer, and thus choice to make, is choice A.

48.

49.

50.

Choice D is correct. Hydrogen-bonding weakens the covalent bond to hydrogen and thus makes the bond easier to vibrate. This means that as the degree of hydrogen-bonding to a protic hydrogen increases, the IR absorbance for the bond decreases in energy (in terms of wave numbers) and the peak broadens (showing a variety of strengths associated with the hydrogen-oxygen covalent bond). The greatest amount of hydrogenbonding is found with the carboxylic acid, as shown by the smallest wave number and broadest absorbance in the IR. The best answer is choice D.

51.

Choice A is correct. The broadest peak is associated with the compound having the greatest amount of hydrogen-bonding. As is observed in alcohols, the amine with the least steric hindrance exhibits the greatest amount of hydrogen-bonding. The least steric hindrance is found in ammonia. The best answer is choice A. As a point of interest, the tertiary amine has no N-H covalent bonds, so it has no hydrogen-bonding.
Choice A is correct. The relationship between bond. strength and IR absorbance is that the lower the absorbance value in the IR (as measured in cm-1), the lower the energy associated with the stretching vibration of the bond. The lower the energy necessary to stretch a bond, the lower the energy necessary to break the bond, and thus the weaker the bond. Longer bonds are usually weaker bonds. Thus, as bond length increases, the wave number of IR absorbance decreases. This eliminates choices B and D. An increase in the degree of hydrogenbonding weakens and thus lengthens the bond. This eliminates choice C and makes choice A the best answer.

\)

53.

Choice D is correct. Of the choices, only primary amines have a protic hydrogen, which means that only primary amines exhibit hydrogen-bonding. The best answer is choice D. Choice B is correct. The strongest hydrogen bond comes from the more basic lone-pair donor (found on the nitrogen atom, which is less electronegative than oxygen) being donated to the most protic hydrogen (found covalently bonded to the oxygen). This makes choice B the best choice. Choice D is correct. The addition of dimethyl sulfide to solution reduces the degree of hydrogen bonding exhibited by the alcohol, because less alcohols will be adjacent to one another to form hydrogen bonds. The absorbance associated with a hydroxyl peak sharpens with the reduced hydrogen bonding. Associated with reduced hydrogen bonding is a stronger covalent bond and thus an IR absorbance with a higher wave number. Pick choice D for optimum correctness satisfaction. Choice A is correct. The IR absorbance of a covalent bond is affected by hydrogen-bonding as stated in the true. As hydrogen-bonding increases, the covalent bond lengthens, so Statement II is true. The acidity of a proton increases with hydrogen-bonding, because the covalent bond to hydrogen is weakened. This is why acidity is higher in water than in other solvents. This makes Statement III true. The only not true statement is Statement I. The best answer is thus choice A.
@

54.

55.

56.

passage/ so Statementl is not

Copyright

by The Berkeley Review@

a2

Section I Detailed Explanations

57.

Choice D is correct. From the data in Table 1, it can be seen that branching in a hydrocarbon increases its octane rating. The greatest amount of branching is observed with 2,2,4-trtnethylpentane, choice D, which can be inferred from reading that 2,2,4-trimethylpentane has an octane rating of 100, higher than the straight-chain hydrocarbons. Don't be a dodo, pick D.

58.

r
,r e
S

t,

T
tr
R
a

Choice D is correct. According to Figure 1, kerosene has a boiling-point range of 175"C to 280"C, so the component most likely to be found in kerosene should have a boiling point in that range. The four answer choices are saturated hydrocarbons of eight, ten, ten, and fourteen carbons. The eight-carbon compound should be found in the petroleum range, and the ten-carbon compounds are probably found in the petroleum-to-naphtha range. You should use test-taking iogic to eliminate choices B and C, because their boiling points are simiiar, given that their molecular masses are identical and their structures are similar. The best answer is choice D, with the highest molecular mass (and thus the higher boiling point) of the choices. To make an estimate of the boiling points for both n-octane and n-decane, you can use the trend in other straight-chain hydrocarbons, where n-hexane has a boiling point of 69"C and n-heptane has a boiling point of 9B'C. Following this trend predicts that n-octane has an approximate boiling point of I25"C - 130"C and n-decane an approximate boiling point of 775'C - 180"C. The branching of Z,2-dimethyloctane reduces the boiling point from that of n-decane (the straight-chain, ten-carbon alkane) to somewhere around 165"C to I70'C. The2,2,4,4-tetramethyldecane is most likely to have a boiling point in the 175"C to 280"C range. Your job, should you accept it, is to pick D, Choice D is correct. From the data in Table 1, it can be seen that branching increases the octane rating of a hydrocarbon. For example, as branching increases, so does octane rating for the seven-carbon aliphatic hydrocarbons 2,2,3-trtmethylbutane > 2-methylhexane > n-heptane. This makes choice A a valid statement, thus eliminating choice A. From the data in Table 1, it can be seen that as branching increases, the boiling point decreases (for hydrocarbons of comparable mass). This can also be seen with the boiling points of the seven carbon aliphatic hydrocarbons, which have relative boiling points of n-heptane > 2-methyihexane > 2,2,3-trrrnethyibutane. Choice B is a valid statement, so it is also eliminated. Due to branching, the hydrocarbon with the greatest number of alkyl substituents has the greatest mass of compound occupying the smallest volurne. This results in an increase in density with branching. Choice C is a valid staternent, so it is
also eliminated. Hydrocarbons have no hydrogen-bonding, so regardless of the arnount of branching, hydrogenbonding neither increases nor decreases from hydrocarbon to hydrocarbon, This makes choice D an invalid statement as to the effect of branching. You should smile brightly when you pick choice D.

59.

rf
)L

:e
IC
Ld ,-e

60.

n-

Choice B is correct. As density increases for a hydrocarbon (or any gas), it does not rise as easily. This means that as density decreases, the ability of the vapor to rise (ascend the cracking column) increases. This eliminates choice A. Choice D is eliminated, because as shown in the apparatus in Figure 1, the aromatic hydrocarbons are not collected in the highest chamber of the cracking column. You could have immediately deduced that the correct answer is either choice B or C, because they are opposites and the boiling point is listed in the diagram. As indicated by the picture in Figure 1, the hydrocarbons with the lower boiling points are collected towards the top of the cracking column, which makes choice B the best choice. You'd be sad if you were to choose anything except choice B.

61.
rd

irg he
rth er.

Choice D is correct. Octane rating is based on the ability of a compound to distribute heat uniformly as it combusts. This ability is found in compounds that are capable of reieasing their heat energy steadily over an extentied period of tirne. The best answer is therefore choice D. Do what is best, and pick choice D. The octane rating does not depend on the enthalpy or entropy of combustion, although the favorability of the combustion reaction does. The ratio of carbon dioxide to water depends only on the number of carbons and hydrogens in the fuel" Choice D is correct. Because toluene and benzene have octane ratings higher than the other six- and sevencarbon saturated hydrocarbons, it can be inferred that aromaticity increases octane rating. Statement I is therefore a true statement. Because toluene (methylbenzene) has an octane rating of 120 and benzene has an octane rating of 106, it is assumed that ethylbenzene should also have an octane rating in excess of 100. Statement II is therefore a false statement. Because of the brar-rching associated wrth 2,2,3-trimethylbutane, it has a high octane rating. A high octane rating is a quality associated with a good fuel additive, so a branched hydrocarbon such as 2,2,3-trirnethylbutane is a good fuel additive. Statement IIi is therefore a true statement. Because statements I and III are both true statements, the best answer is choice D.

62.

he

III
.is
he

Copyright O by The Berkeley Review@

83

Section I Detaited Explanations

63.

Choice D is correct. Because 2,2,4-ttimelhylpentane is a saturated hydrocarbon, all of its carbons have a hybridization of sp3. In the final product, the carbons are all pr"r"r-,i in the form of carbon dioxide. The hybridization of^ carbon in carbon dioxide (CO2) is sp. This means that in this reaction, the hybridization changes from sp3 to sp. The best answer, and on- we highly recommend to all parties interested in success, is
choice D.

Tt-

64.

Choice A is correct. The start of the third paragraph states that the closer the electrons within a carbonhydrogen bond are to the carbon nucleus, the more acidic the compound is. To determine the relative acidity, you must make a decision about how close the electrons are to the nucleus. The passage also states that the more p-character there is in the hybrid, the longer the bond is. Connecting the two concepts, you should reach the conclusion that the shorter the bond, the closer the electrons are to the nucleus. This means that the less pcharacter there is in the hybrid, the more acidic the hydrogen. The most acidic hydrogen is thus found on an sp-carbon. In 3-methyl-1-pentyne, carbons 1 and 2 are sp-hybridized, but only carbon t has a hydrogen attached. Pick choice A for optimal results. Make a note from the conclusions that sp > sp2 > sp3 for uiiaity. sphybridized

H- C: C- Ct I

i\

CH

2-

CH3

cll3
55.

Choice D is correct. The largest Ku is associated with the strongest acid. All of the choices are hydrocarbons, so the most acidic proton is the one on an sp-hybridized carbon, as opposed to either an sp2-I.rybridized o, sp3hybridized carbon. Of the four answer choices, only choice D has a hydrogen bonded to an sp-hybridized carbon, so choice D is the best answer. Choice A is correct. Al1 of the compounds are depro^tonated hydrocarbons (with a lone pair on carbon), so the strongest base is the one with the lone pair on an spJ-hybridized carbon. The only choice with a lone pair of electrons ot-t an sp3-hybridized carbon is choice A. The iation is irrelevant to the pioblem, because it is sodium in each answer choice. Choice D is correct. NaNH2 is a base strong enough to deprotonate a hydrogen on an sp-hybridized carbon. Although this is true, it is not critical information in solving this question. Only one answer choice can be correct, so the correct choice must be the compound with the most acidic hydrogen. This means that this question is reduced to asking "Which compound, of the choices listed, has the most acidic proton?" The most acidic hydrogen is attached to the oxygen, so you had better pick D.

66.

67.

68.

Choice C is correct. The longest carbon-carbol bond is a single bond between the two largest orbitals. The largest of the three possible hybrid orbitals is sp3 , so the longest carbon-carbon bond is formed between an sp3hybridized carbon and an spJ-hybridized carbon. Choices B and D are eliminated immed-iately, because they contain no C-C single bond. Choice A is a bond between an sp-hybridized carbon and an sp3-ltybridized carbon, while choice C is between an sp2-hybtidized carbon and an sp3-hybridlzed carbon. An sp2-6t6rid orbital is longer than an sp-hybrid orbitai, so choice C is the best answer.
Choice C is correct. The weakest carbon-carbon bond is associated with the longest carbon-carbon bond. Choice A is eliminated, because the C-C bond is a triple bond, and triple bonds are the shortest of carbon-carbon bonds. Choice B is between an sp-hybridized carbon and an sp3-hybridized carbon, choice C is between an sp2hybridized carbon and an sp3-hybtidi"ed carbon, and choice D is between an sp2-hybridized carbon and an sp2hybridized carbon. Choice C is the longest, so it would terrific if you would pick choice C. Choice A is correct. The iowest pK6 is associated with the strongest base. Because the most acidic proton is found on an sp-hybridized atom, the strongest base must be a lone pair on un sp3-hybtidrzed atom. Choices C and D are spz-hybridized nitrogens, so they are both eliminated. Choice A is better than choice B, because the lone pair of electrons on nitrogen in choice B is tied into resonance r,r'ith the adjacent alkene n-bond. Electronwithdrawing resonance reduces a compound's basicity.

69.

70"

Copyright O by The Berkeley Reviewo

a4

Section I Detailed Explanations

Choice D is correct. For a reaction to have an equiiibrium constant greater than 1.0, the reaction must be favorable in the forward direction as written. A favorable acid-base reaction proceeds from stronger acid to weaker acid in the forward direction as written. The larger the equilibrium constant, the more favorable the reaction, so the strength of each acid can be inferred from the K"O values in the table. In choice A, the reaction proceeds from the weaker acid (C13CH with KuO of 8.0 x 10-9) to the stronger acid (H3CCH2SH with K"O of 5.2 x t0a). This means that this reaction is unfavorable and thus has u Keg < 1. Choice A is therefore eliminated' In choice B, the reaction proceeds from the weaker acid (H3COH witfi K"O of g.g x 10-1) to the stronger acid (H3CNO2 with K"O of 8.2 x 104). This means that this reaction is unfavorable and th.y_s \a1 l Sgq < 1. Choice B is therefore eliminated. In choice C, the reaction proceeds Jrom the weaker acid (H3CCOCH3 with Kg* of 2.0 x 10-4) to the stronger acid (H3COH with K"O of e.g x 10-1). This means that this reaction is unfavorable and thus has a K"O < 1. Choice C is therefore elimlnated, It is only in choice D that the reaction proceeds from a stronger acia 6IiCCOCH2COCH3 with Keq = 1.2 x 106) to a weaker acid (CI3CH with Kuo = 8.0 x tg-v;. The
correct answer is choice D.

Choice C is correct. Reaction 2, the experimental reaction from the passage, involves the protonation of CSHSto form CSHO. This question asks for the reverse reaction. This means that any acid that shows an equilibrium constant less than 1.0 has a conjugate base that is strong enough to deprotonate C5H6. Choices A and B are eliminated, because they are acids, not bases. Because only methanol (CH3OH) shows an equilibrium constant less than 1.0, only methoxide anion (CH3O-) is strong enough to deprotonate C5H6. The best answer is choice C.

Choice D is correct. The reaction as drawn proceeds from the stronger acid to the weaker acid, therefore the equilibrium constant is greater than 1.0. When the equilibrium constant is greater than 1.0, the products are in higher concentration at equilibrium than the reactants. This means that H3CCH2S- is in higher concentration than H3CO-. This eliminates choices A and C. To distinguish choice B from choice D, the initial concentrations must be known. Because H3CCH2SH and H3CO- are mixed equally initially, they must be equally concentrated at equilibrium, The best answer therefore is choice D'
Choice B is correct. The conjugate base of the 1,3-cyclopentadiene species has six n-electrons in a continuous cyclic planat affay of p-orbitals. These conditions result in aromatic stability. The best explanation for the relative ease with which the 1,3-cyclopentadiene loses its proton is the aromaticity associated with the conjugate base ( 1,3-cyclopentadienyl anion). The more stable that the conjugate base is, the stronger the acid is. Pick choice B and be satisfied. Choice A is correct. The weaker of the two acids has the larger of the two pKn values. This question is asking for the weakest acid relative to the strongest acid. As the acid gets weaker, the reaction with CSHS- becomes less favorable, so the equilibrium constant for the reaction gets smaller. C13CH shows the Iowest equilibrium constant of the answer choices, so it is the weakest acid and thus has the highest pKo value. It is only in choice A that CI3CH is listed as having the highest pK3 value, which makes choice A correct. Choice C is correct. A Lewis acid is an electron-pair acceptor. The classic example of a Lewis acid is choice D, BF3, with highly electronegative fluorine atoms and an empty p-orbital that can readily accept electrons. This makes the boron severely eiectron-deficient. Both CH3CI and CH3N 02 are listed as acids in the table, so choices A and B are not good choices. NaCH3 cannot accept a lone pair, but instead readily donates a lone pair. This means that choice C is not a Lewis acid, and in fact is a Lewis base. Pick C to be terrific. Choice D is correct. Using Equation 1, the K"q for a reaction is found by taking L0 to the power of the product acid pKu minus the reactant acid pKu. In the itandard reaction, C5H6 is the product acid andits^pKuvalue is 10 0) = 195' given as fS.0" If the pKu of the reactant acid is 10.0, then the equilibri.tm cor-rsiar-,t wouid be 10(i5'0-ihe question is thereiore: "Which acid in Table t has an equilibrium constant of roughly 105?" The best answer is choice D, CH3NO2, with an equilibrium constant of 8.2. x 104 when it reacts with C5H5-.

L-

e
.S

'i\

.ls

iC
ire rn-

-i.

Choice A is correct. Reading from Table 1, the fastest reaction rate corresponds to the lowest negative log value. Of the answer choices, the slowest reaction is observed with an alkoxide in an aprotic solvent. The 1.44 makes it the fastest reaction rate of the choices offered to you. The best answer is therefore choice A.
@

-pyright

by The Berkeley Review@

a5

Section I Detailed Explanations

79.

Choice B is correct. Ethyl sulfide would react with 2-iodopropane just slightly more slowly than methyl sulfide, so the negative log of the reaction rate would be slightly higher tnai lsir for the ethyl sulfide. The best answer is choice B, 2.04. Following the trend for ether s6lvent silows a negative 1og value for HSof I.76 and for H3CS- of 7.97. It can be conciuded from this that the negative log of-rate for "H3CCH2S- should be greater than 7.97, but no iarger than about 2.2. The only choice in this .utlgJi, 2.04. The tact of linearity in the trend is due to the fact that the change in steric hindrance from H3CS- to u3CCu2S- is less drastic than the change from HS- to H3CS-. The difference il the change in steric hindrance can be seen in the smaller change i1 negative log value for the reaction rate.
Choice C is correct. Because bromine is smaller than iodine, bromine forms a stronger bond with carbon than iodine, so bromine is a worse ieaving group than iodine. Tl-re rate of the reaction dJpends on the electrophile. The worse the leaving group, the worse the electrophile, and the slower the nucleophilic substitution reaction. The bromine leaving group wotild yield a slower reaction (a decrease in the ,"u.tior-, rate) than the iodine leaving group, and consequer-rtly a larger negative log of the reaction rate. The best answer is thus choice C. Choice B is correct. The weakest bond is the one that would be broken in a nucleophilic substitution reaction, such as Reaction 1. In Reaction 1, the bond that is broken is the one between carbon und iodi.,", thus the C-I bond must be the weakest of the choices. Pick choice B. Choice A should be eliminated, because the H is not the leaving group in Reaction 1. Because a C-O bond and C-S bond are formed when a C-I bond is broken in separate reactions listed in Table 1, it can be inferred that both the C-O and C-S bonds are stronger than the C-I bond. This eliminates choices C and D and further supports choice B as the best answer. Choice A is correct- As the solvent is changed from ether to ethanol and finally water, the degree of hydrogen bonding in solution increases. It can be observed from the data in Table 1 that the reaction rate decreases. The conclusion must be that hydrogen bonding decreases the reaction rate. Choices C and D are eliminated. The rate must decrease due to hindrance of the nucleophile, not enhancement. Your answer is choice A" Choice D is correct. According to the data in Table 1, regardless of the solvent, the negative 1og of the reaction rate is greater with H3CS- than HS-. This means that changing the nucleophile from HS- to H3CS- decreases the reaction rate. Choice A is therefore eliminated. the negitive log values of the rate are lower with the ether solvent than the water solvent, therefore water solvent must decrease the reaction rate. This eliminates choice B. Decreasing the temperature always produces a clecrease in the reaction rate, so choice C is eliminated

80.

81.

82.

83.

iE
r!9lti

too.

ethanol solvent must increase the reaction rate. That makes choice D the best answer. This question requires determining the relationship between the reaction rate ancl negative 1og of the rate.
84.

The negative log values of the rate are greater with the water solvent than the ethanol solvent,

so

Choice C is correct. In ether, aikoxides have lower negative 1og values for their reaction rates than alkyl sulfides, so the alkoxides must react faster. This eliminates choices B and D. The passage states that the difference in reactivity can be attributed to alkoxides being better bases than alkyl sulfides. The stronger the base, the lower the pK6 value. Pick C and feel jovial for just a moment, at least until the next question stirts.

85.

and D. The degree of substitution is irrelevant. Esters have no lone pair of electrons that can be readihdonated to a proton, so choice B is not correct. The best answer is an amine, independent of whether it is primary, secondary, or tertiary. This means that you really should pick C for the sensation of correctness.

Choice C is correct. The most basic species is the compound containi-ng nitrogen. In general, nitrogen compounds are more basic than oxygell- and sulfur-containing co-mpounds of eqtr"at hybiidization. This eliminates choices A

86'

choice D is eliminated. Afier eliminating the wrong choices, vou shor,rld settle for choice C as the best answer.

Choice C is correct. The rate referred to in Figure 2 is for a nucleop}rilic substitution reaction at a carbonyl site. There is a direct corelation between nucleophilicity and the rate of reaction. The graphs show that above a pH of 7, as the pH increases, so does the reaction rate. This mea11s that ihe nucleoptritcity increases. As pH incteases, compounds are no longer cationic. This eliminates choices A ancl B. Size ls noi applicable here, so

dt

87.

Choice D is correct. At pH = 9.0, all of the compounds should be neutral (although the amine in choice D marhave a small fraciion that rernains protonated). Tl-re most reactive compound is the best nucleophile. For nucleophilicity, an arnine is better than an ether, an ester, or an arnide. Foi this reason, pick D.
@

.il1

Copyright

by The Berkeley Review@

Section I Detailed Dxplanations

'-l
IE
-bt

te
1e

',e

Choice C is correct. The chlorine atoms are electron-withdrawing by the inductive effect (chlorine is more electronegative than carbon). Electron-withdrawing groups make the compound more acidic and thus lower its pKu value. Both choice C and choice D are lower than 10.3. Choice C is the better choice, because the inductive effect is not so substantial that it will make the ammonium cation that acidic. For the pKu to drop down to 1.0 would mean that the three chlorine atoms on the methyl group increased the acidity by a factor of 109'3 = 2 x 709 = 2,000,000,000 times. That is too much. Be conservative and pick C.
Choice D is correct. The sum of pKu + pKU for a conjugate pair in water is equai to 14.0 al25"C. This means that pK6 for NH3 (the conjugate base of NH+*) is equal to 14.0 - 9.25 = 4.75. Pick D and score with the best of them (whoever they are). Choice A is correct. The hybridization of carbon in a carbonyl compound, such as an ester (which contains one 7[bond), must be sp2 1th" n-bond requires one p-orbital, so only two p-orbitals remain for hybridization). This can also be deduced from the trigonal planar structure of the carbonyl compound. The hybridization of carbon in the tetrahedral intermediate (which contains no lr-bonds) is spJ. The final product again has the carbonyl

in

functionaiity, only now with the nucleophile attached. Tl." carbonyl product still has trigonal planar geometry. The hybridization therefore changes from spt to spJ and back to spt in the overall reaction, making choice A your choice. Make that choice today!
Choice A is correct. In each case, there is a nitrogen with one alkyl group and two hydrogens. This defines a primary amine , so both compounds are primary amines. It just so happens that the R-groups are aromatic rings/ but they are not aromatic amines per se, because the nitrogen atom is not a part of the aromatic system. The best answer is choice A. Of the two amines, only one is conjugated (aniline), so choice C does not describe both
structures.

Choice D is correct. Alcohois exhibit hydrogen-bonding, which increases their intermolecular forces. The it harder to move a molecule from the liquid phase into the gas phase. This raises the boiling point of an alcohol compared to a molecule of comparable size, so choice D has the highest boiling point. Molecular mass is of concern as well, but choice D is also the heaviest of the choices.
stronger forces make Choice B is correct. The longest chain is eight carbons, so based on that alone, you know that the best answer is All you need to do is find the longest chain to decipher the correct answer choice.

choice B (octane).

L,.

.i._
:he

Choice C is correct. Conjugation is defined as consecutive, alternating n-bonds. The structures are drawn below. OnIy Compound III has conjugation, so choice C is correct.

il
^ i-

1,,4-cyclohexadiene 3-methylcyclohexene 2-methyl-1,3-cyclopentadiene


Choice C is correct. Choice C is the most stable compound, because it is the only diene that has conjugation. Note that the structures are straight chains and not rings. It is easy to insert the word "cyclo" inadvertently into the name. Avoid careless mistakes and choose C. Drawn below are the structures of all four choices: 2-methyl-1,4-

rl.)' _:- 1:

3-methyl-1,4-pentadiene 2-methyl-1,3-pentadiene 1,5-hexadiene

'ite,

\4,/
I

\,.

,.^

';H
Choice D is correct. Saying that a compound has a dipole that is not equal to zero is equivalent to saying that the compound is polar. Cis compounds (both alkenes and cyclic structures) are aln'ays polar. This makes both Compound I and Compound III polar. You need not even examine Compound II, because no answer choice includes all three compounds. Pick choice D to score more MCAT points. F..l
LOns

- :vright

by The Berkeley Review@

Section I Detailed Explanations

97.

Choice D is correct. All alcohols have hydrogen-bonding, which increases their intermolecular forces and thus increases their boiling points, so choice D has the highest boiling point. Note that all of the compounds have exactly the same formula (and thus the same molecular mass). This eliminates the need to accbunt for any differences in molecular mass (which would also affect the boiling point). Because of the linear nature of choice B and its asymmetry (which makes it more polar than the two remaining choices), it should have the second-highest boiling point.
Choice C is correct. For a compound not to be polar, it must be s).'rnmetric. Cis compounds are asynunetric about a

98.

point (although they may have mirror-plane symmetry, rather than point symmetry), and thus are always polar, so choice A is eliminated. This leaves choices B, C and D as possible answers. An odd-numbered ring must be polar when it has two substituents, so choice B is polar and thus ruled out. In choice D, the middle carbon has varying substituents attached (not all four groups are identical), so it cannot be symmetric, thus it is polar, too. By eliminating three choices, choice C must be the correct answer. Drawing choice C out shows that the individual dipoles for the bonds cancel each other out, making the compound nonpolar.
E-1,4-dichloro-2-butene
The individual vectors cancel out, so there is no net vector. The compound is nonpolar.

99.

Choice B is correct. The most heat is generated by the least stable compound; thus finding the least stable compound is the task at hand. All of the choices have the same formula (C6Hg) so it comes down to structural features. The four-membered ring is unstable, so choices A and B are good. Choice B has no conjugation, while choice A does (conjugation is a stabilizing feature), so this makes B the least stable compound a*ot g the answer choices. The bond angles are not the optimal 109.5", whether the n-bond is in the ring or not.
Choice C is correct. Translating from the chemical formula into the structure yields the compound below:

100.

H3CH2C

H3CO H H

CFI3

There is no aldehyde group (which would have been represented as CHO), so choice A is eliminated. There is no ester group (which would have been represented as CO2R), so choice B is eliminated. There is a carbonyl adjacent to two alkyl groups, so the compound has a ketone functionality. This eliminates choice D and makes choice C the best answer.

Copyright

by The Berkeley Review@

Section I Detailed Bxplanations

lsomerism

Section II
Structure Elucidation
by Todd Bennelt

a)

b)

Isomers i. Constitutional Isomers ii. Stereoisomers iii. Configurational lsomers iv. Optical and Geometrical Isomers Conformational Isomers i. Eclipsed vs. Staggered Conformation ii. Oauche vs. Anti position !ii. Newmanprojections iv. Cycloalkanes v. Cyclohexane vi. Chair and Boat Conformations vii. trquatorial vs. Axial Orientation

H:C

Structural trnsights a) Structural Symmetry b) Units of Unsaturation


Spectroscopy and Analysis

a) b)

c)
d)

Qeneral Spectroscopy Infrared Spectroscopy i. Theclry and Key Signals ii. CommonAppplications iil Hydrogen-Bonding Ultraviolet Spectroscopy l. I heory ii" Common Appplications NMR Spectroscopy i. Theory ii. Structural Symmetry iii. Proton NIVIR iv. Shift Values v. Splitting Pattern oi: Signal Integration vii. Structural Features

L)K.E'v.r.-b.wt
Speci ahzing

RERI{EIEY

in MCAT Preparation

Structure Elucidation Section Goals


Be able to identifv isomers from both their structure and their name. There are several types of isomers. Be familiar with structural isomers (identified by different connectivity), stereoisomers (identified bv asymmetry), optical isome-{s (and their ability to rotate plane-polaijzed lieht), geometrical isom'ers (found #ith rings and alkenes), and conformational isomers (identified"by ro-tation about bonds or ring-flips). You should know how the different types
of isomers are related to one another.

Be able to identify the more stable chair conformation for six-membered rings. Cvclohexane and pyranose sugars involve three-dimensional ring structures. The most stable conformation result3 in the leaEt steric hindrance. As a general rule, axial orientation results in greater steric hindrance than equatorial orientation. Know the difference in stability between axial Srientation and equatorial oriehtation. Recognize the steric repulsion associated *ith 7,2-diaxial, 1,3-diaxial, and 1,4-diaxial orientation.

@3 @3

Be able to identify the more stable Newmann proiection for a compound. Structures can orient themselves in a staggered conformation, an eclipsed conformation, or some conformation between eclipsed and staggered. You must be able to identify the most stable orientation for a structure and distinguish between gauche and anti positions.

*t
,-11-

Be able to use the molecular formula to determine the units of unsaturation.


Some auestions reouire vou to determine the potential functional groups of a molecule. The presence

*I-

of eithbr a n-bond'and 1 rine within a strucfure results in a uniFof dnsaturation, which manifests itself as two fewer hvdroeefrs in the formula. A fullv saturated hvdrocarbon or carbohydrate has a total of 2n + 2 hydrdgen itoms in the compouLnd, where n is the nurhber of carbons in the fompouLnd.

-:ii
rM':lT

:tr:I,
Jtll

Be able to translate structures from two dimensions into three dimensions. Know what the terms staggered, eclipsed, gauche, and anti mean, and be able to draw structures in the Newmann projectidi to show ihe orlentation of substituents in these structures. Be able to
rotate about o-bonds.

I'L

[ht*
arLi:r:

ll l:-i,

t?

f,

JIl

Be able to deduce structural features using IK spectroscopy. You should have a basic understanding of the operations of an infrared spectrophotometer. You must know the IR stretches for a carboiyl and a hydroxyl bond. You shou-ld be able to determine which structural features correspond to #hich IR absorbairces. You must be able to eliminate and/or confirm possible structures, using IR data. You must be able to decipher IR spectroscopy graphs and identify the key peaks. Be able to deduce structural features using NMR sPectroscopy. You must know the NMR shifts for carbonyl compor-rnds, alkene compounds, and aromaLic compounds. You should be able to determine the struch-re of in unknown compor.rnd using the spechal information from the NMR. Most structures you will encounter on the MeAT are sm"all and symmetrical, so thev are easilv solved. You mustbe able to eliminate incorrect structures based onNMR data. Be s'.rri: to underitand. what the shift value (measured in ppm) tells you, what the integration tells you, and what the peak shaoe and couplins constants tell vou. Each piece of information can be used to help deterniine the s'tructure of in u"nknowt-, compound. Use these data in coniunction with the units of unsaturation.

Jllli.ru

l8,itr

lll

[.i

ri

slru ItI l[

ffir*
;r Ii|]U

ryr* rux
rllietu
1ll[ir'u

]L u LJ

4$r
W,ot!,r $mfigll

Organic Chemistry

Structure Elucidation

Introduction

dimensional folding of the structure. Structure elucidation involves determining the number of isomers that fits a molecular formula and then systematically eliminati.g isomers that do not fit the data untit, only one structure remains.

structure elucidation involves applying ail available information, from spectroscopic data to chemical reactivity, to ascertain the three-dimensional shape of a molecule. It entails determining the atoms within the molecule, the functional groups present on the molecule, and in advancecl cases, the three-

In this section, lve shall address the concept of isomerism and the many classes of isomers. Isomers have the same atoms within the molecule, but they differ in some manner, so that the molecules are not superimposable on one another. The difference could result from different bonding *il-,ir-, the molecules, similar bonding but different three-dimensional distribution about a stereogenic center, or the same bonds and stereogenic syrnmetry with different conformational orientation. A significant part of structure elucidation is determining the exact isomers that are formed in a chemical reaction.

Other structure elucidation tools shall be discussed. Questions that involve structure elucidation are often made easier by first determining the units of unsaturation from the molecular formula of a compound. Thii information provides hints as to the presence of n-bonds and./or iings within the structure. Chemical tests can be carried out to determine the nuriber of rc-bonds, which h'hen combined with the units of unsaturation, can specify the exact number of rings and ru-bonds within a molecule.

about the

In this section we shall also address spectroscopy a.d the information about structure it can provide. I'frared (IR) spectros.opy ir typically used to determine the functional groups within u .o*por'rd. It iin atso give some information
_symmetry

reatures. In its simplest application, NMR can show the carbon skeleton of a molecule. In its more sophisticated application, NMR can show the presence of stereoisomers and the exact positions of functional groups. we shall address :roth carbon-13 and proton NMR. combining xvn data with uv-vis spectroscopy and IR spectroscopy data allows for precise determination of threelimensional molecular structure.
' ou distinguish compounds by NMR?" .ur-r b" reduced to ,,How many different :r'pes of hydroge^s are there in each compound?" Multiple-choice NMR questions can be answered easilv by predicting the spectra from possible :tructures. For instance, if you can narrow down the potential struciures to (etones, then it's just a matter of systematically eliminating ketones that do not fit :he spectral data. This is the perspective from which w.-e will approach NMR. i he abiiity to predict spectra from structures is best attained thiough practice. -\s you do the multiple-choice qtiestions il the spectroscopy sections,-pr"di.t th" spectra for the structures in this same manner. the cliiference belween the :pectra in each answer choice (A,8, c, or D) is what often answers the question.

spectroscopy describes the connectivity of a molecule and its specific structural

presence of groups capable of forming hydrogen bonds. Ultraviolet-visible (uvYis) sPectroscopy tells us information about the n-bonds and conjugation within a molecule' Although all molectiles absorb ultraviolet radiation, for practical irurposes/ we use it only to detect n-bonds. Nuclear magnetic ,"ror-rur.,.u (NMR)

of the molecule, the hybridizatiJn of carbon, and the

-t is best to review NMR with symmetry as your focus. The question ',How can

Copyright O by The Berkeley Review

Exclusive MCAT Preparation

Organic Chemistry
Isomerism
Isomers

Structure Elucidation

Isomerism

Isomers are structures with the same formula, meaning they are made of the exact same atoms, but they differ in the location of each atom. The difference in

position can be the result of different connectivity (bonds), different spatiai arrangement because of asymmetry in the structure, or different orientation about a bond. The result is that there are several different types of isomers. Figure 2-1 shows a flow chart for determining the type of isomers.
ISOMERS
Structures made of the same atoms

Constitutional (Structural) Differ in connectivity (bonds)


Conformers
Di f fer by orieniaion in space

Stereoisomers

Differ in spatial arrangement of atoms Configurational Differ by orientation in space Can't rotate to become identicai
Geometrical Differ by orientation in space Can't rotate to become identical due to the presence of a ring or n-bond Enantiomers Nonsuperimposable mirror images

Identical after rotation about o-bond

Optical Differ by orientation in space Can't rotate to become identical due to asymmetry in the structure
Diastereomers Nonsuperimposable and not

( (

qn

rt
mirror images
Figure
2-1

&

ffi

Constitutional isonrcrs, which have different bonding, ate more commonly referred to as structtLrsl isomers. Structural isomers are most easily recognized by their difference in IUPAC name. The difference may arise from the functional groups (like an alcohol versus an ether, or a ketone versus an aldehyde) or it may arise from the connectivity of the carbon backbone (like 2-methyihexane versus 3-methylhexane). Structural isomers can be further divided tnto functional group
islnrcrs, positionnl isonrcrs, and skeletal isonters.
Stereoisorners have exactly the same bonds (and therefore the same connectivity), but they differ in the spatial arlangement of their atoms. On a more general note, stereoisomers can be categorized as either configurationnl isomers (which differ in spatial arrangement and cannot be converted into the other isomer without breaking a bond) or conformational isomers (which differ in spatial arrangement but can i:e converted into the other isomer by rotation without breaking a bond.) Within configurational isomers, there are optical isomers (isomers that rotate plane-poiarized light differently) , geometrical isomers (isomers that vary in orientalion about a n-bond), ensntiorners (nonsuperimposable mirror images), and

ffi m

diastereomers (nonsuperimposable and not mirror images). Configurational isomers are most easily distinguished by their IUPAC prefix. The IUPAC prefix contains either R or S, if the isomers differ in chirality at a stereocenter, or E and Z, if the isomers differ in their arrangement about a n-bond. We shall address stereoisomers in detaii in later sectrons'

Copyright

by The Berkeley Review

92

The Berkeley Review

Organic Chemistry

Structure Elucidation

Isomerism

B. C. D.

\vhat types of isomers are 2-methyl-3-pentanol and 3-methyl-2-pentanol? A. Conformational isomers


Geometrical isomers

Structural isomers
Stereoisomers

:hey are also positional isomers. The question was not that specific, so the best .rnswer is choice C. The two structures are drawn below:

S ol uti on The two compounds have different IUPAC names, so they are structural isomers. The two structures vary in the position of their alcohol and side chain methyl, so

3-methyl-2-pentanol

2-methyl-3-pentanol

Constitutional Isomers -

tlstance, 3-metllylhexanal and 3-methyl-2-hexanone are constitutional isomers. They each have the formr-rla c7H140, but they have a different sequence of oonds. They can also be referred to as positional isomers. Using nomer-rclature l'relps to determine whether two structures are consiitutional isomers, because :onstitutional isomers must have different IUPAC names. Figure 2-2 shows three pairs of structural isomers, one set of functional group isomers, one set of positional isomers, and one set of skeletal isomers. stntcturnl: Different arrangement of atoms (i.e. different bonds)

constitutional isomers (also referred to as structural isomers) are unique molecules that have the same formula, but different connectivity. In other rgords, they have the same atoms, but the atoms have different bonding. For

HoM
LI

Ho- cH2cH2cH2cH3

&

H3cH2c-

o-

cH2cH3

1-butanol
I{1C-

diethyl ether
HICCH2

'l

CH-

CH2CH2CH3

- CH CH2CHT
I

=/\
CHq

cl
2-chloropentane
H3C-

&

LI

m
C1

3-chloropentane

CH-

CH2CH2CH3

H:C- CH- CH&


CHs

-l-.-^-.

CHa
2-methyipentane

lt

CHI

CHr

I-

CHa CHs

2,3-dimethylbutane

Structural isomers have different IUPAC names. Figure 2-2

Copyright

by The Berkeley Review

Exclusive MCAT Preparation

Organic Chemistry
Example 2.2

Structure Elucidation

Isomerism

How many possible constitutional isomers exist for a molecule with the
molecular formula CaHlg?

A. 1 8.2
D.4

c.3
Solution

The maximum number of hydrogen atoms possible on a four-carbon alkane is ten, so there are no units of unsaturation in C4H1g. This means that there are no

n-bonds or rings in the molecule. To solve this question, chains of varying carbon connectivity (skeletons) must be considered. There is always one longest chain structure (C-C-C-C). There is also the possibility of a three-carbon chain with a methyl group ofi of the second carbon (if the methyl were on the first carbon, it is still butane). This means that there are two constitutional isomers for C4H1g, butane and 2-methyipropane. Pick choice B for the smile that
a correct answer brings.

Example 2.3 Which of the following pairs of molecules is NOT A. 2-Methylpentane and 3-methyipentane B. Cyclobutanol and tetrahydrofuran

a set

of constitutional isomers?

C. 1-Chlorobutane and 2-chiorobutane D. 4-Ethylchlorocyclohexane and 3-methylchlorocyclopentane


Solution
COH1+ and

In choice A, both compounds have the formula

different IUPAC

names, so they are constitutional isomers. In choice B, both compounds have the formula CaHgO and different IUPAC names, so they are constifutional isomers.

In choice C, both compounds have the formula CaHgCl and different IUPAC
names, so they are constitutional isomers. In choice D, the first compound has the formula C3H15C1, while the second compound has the formula C6H11Cl, so they are not even isomers, let alone constitutional isomers. This makes choice D the correct answer.
Shfls
0m!um

Ttu$

dnr:mrln

lrMffi

For a given formula, tirere is a finite number of possible structural isomers. The

htw

un

number of possible structural isomers depends on the molecular formula. Saturated aliphatic compounds (linear alkanes) are the simplest case. For each
extra carbon, the number of structural isomers increases. For instance, C3Hg has only one structural isomer, while C6H14 has five different structural isomers. It is important to realize that both formulae (CgHs and C6H1a) are for structures that are fully saturated (have no units of unsaturation). There is no easy formula for determining the number of structural isomers possibie for a given formula, but there is a systematic way to determine the number. Figure 2-3 shows all of the structural isomers for C3Hg, C+HfO, CSHfZ, and C6H14, and lists them in terms of charn length and substituent location.

WillfieffiD
mfrems

mr{
dtfimd

ru[

@!-'0J;ut.E

-;.*-

illifirrfnt

nery:l

pirffiii
* au]l

lllcmr

$riffwe
nUemr:

Copyright

by The Berkeley Review

The Berkeley Review

i[r_,Mplilu

Organic Chemistry
C3H8 (1 total): CaHro (2 total):

Structure Dlucidation
HsC
I

Isomerism

./ H:rC'

CH.

'\
CHg

Hr./cHt\ ,r{"'
4-Carbon chain

"ra/tt-a", 3-Carbon chain


HqC "\/
-CH3 CHr

CsHtz (3 total):

HrC/

CHr -\.f

-CHz '-.ur,

"rf

Hrc/ -."i
4-Carbon chain

_CH

HSC

-zc\

CH:

S-Carbon chain

3-Carbon chain

CeHr+ (5 total):

ur./cH'\.t'ttt'' .r{"t
5-Carbon chain

HrC/

"l

CHr --

an-

-CHz

cH,

HsC S-Carbon chain

"rf -zcH\ .zCH' Hzc' -cH, -c",


S-Carbon chain

"rl
t"\ ,t"'
chain

*C/t-.t'tt' 4-Carbon

"..-t"Figure 2-3

"l

,r('^u
CHs

4-Carbon chain

C
IS

;o
D

\e
a.

*r
AS

It
ES

1a ,4,

of

in

This procedure of determining the number of structural isomers is systematic. First, start with the longest continuous chain of carbons (equal to the total number of carbons in the formula). In the case of CoHt+, the longest possible chain is six carbons. After drawing the longest chain, draw a carbon chain of one Iess carbon (five carbons) and systematically deduce all of the possible isomers by moving the methyl group across the chain one carbon at a time. In the case of C6HI4, the next chain down from six carbons is five carbons and the extra (sixth) carbon is attached to one of the interior carbons in the chain. If the extra carbon were attached to a terminal carbon, then the longest chain would be six carbons, not five. In the case of C6H14, it is not possible to have 1-methylpentane, because that is really n-hexane. A guideline to follow as you deduce isomers is that structural isomers must have different IUPAC names. If you are ever in doubt about whether or not two compounds are structural isomers of one another, name them using IUPAC conventions. To complete the process of determining the isomers, systematically count isomers for each possible chain length, reducing the length by one carbon each time. When you are finished with eich possible chain length, sum all of the structures and that's your answer. For alkanes with functional groups attached, the procedure is the same except once all of the skeletal structures are determined, there is an additional itep of systematically placing the functional group at all unique carbons. Example 2.4 demonstrates this procedure. Copyright
@

:w

by The Berkeley Review

95

Exclusive MCAT Preparation

Organic Chemistry
Example 2.4

Structure Elucidation

Isomerism

How many structural isomers are possible for the formula CaH9C1?

A.3 8.4
D.6

c.5
Solution For a problem of this type, the possibilities for the carbon skeleton must be determined first. The four carbons can either be aligned four in a row or three in a row with the fourth carbon coming off of the second carbon of the three-carbon
chain.
STEP 1: 2 possible carbon skeletons

u ft

C-C-C-C
4-Carbon chain

C-C-C
I

fii

ni

C
3-Carbon chain

o!

ffi

The second step is to determine how many unique carbons each chain contains. STEP

2: Each skeleton has two unique carbons

/i

Co- Cu- Cu-

Co

Co2

Cu
I

Cu

ca
2 Unique carbons

Unique carbons
a a

The last step requires placing a chlorine on each unique carbon one structure at time and verifying your answer by checking to see if each structure has

different IUPAC name.


STEP 3: 4 structural isomers total

C_C-C-C
I

C-C-C-C
I

C1

C1

1-chlorobutane

2-chlorobutane

C_C-C tl CIC

C-C-C /\ CCl

1-chloro-2-methylpropane 2-chloro-2-methylpropane
The best choice is answer B, because there are four possible structural isomers. This systematic procedure works every time. It is assumed that isomer problems much beyond this example in terms of difficulty will be avoided on the MCAT because of time constraints. The skills employed when deducing the number of structural isomers can also be used when deducing structure from spectral data, such as IR and NMR information.

Copyright

by The Berkeley Review

The Berkeley Review

Organic Chemistry
Example 2.5

Structure Dlucidation

Isomerism

How many possible constitutional isomers have the molecular formula CaHg?

A.4 8.5

c.6

D.7
Solution
These questions are time-consuming, but unfortunately, there is not a convenient

rvay around

it.

To start, you must determine the units of unsaturation (also

l<rrown as degrees of unsafuration).

Degreesof unsaturation

=z(+)+?-(8) =8+2-8=10-8

2222

-2 -

\Vith one unit of unsaturation, the structure must contain either a n-bond or a ring. To get the correct answer, you must systematically consider each linear connectivity and each cyclic connectivity. Be sure not to count stereoisomers i'including geometrical isomers). The alkenes are listed first, followed by the
cvclic structures.
Possible alkene structures:

4-carbon chain (2 total)

3-carbon chain (1 total)

HzC: CHCH2CH3
1-Butene

H3CHC:

CHCH3

H. H
,C=

,CH'
C,,

2-Butene Note that there are two possible


geometrical isomers for 2-butene.

CHs

Methylpropene

Possible cyclic alkane structures:

4-carbon ring (1 total)

3-carbon ring (1 total)

H?C- CH, H2CCH2

-tt

CH"

/\' H?C-\

CH
CHg

Cyclobutane

Methylcyclopropane
Because there are five total constitutional isomers in all, the best answer is choice
B.

Copyright O by The Berkeley Review

97

Exclusive MCAT Preparation

Organic Chemistry
Stereoisomers

Structure Elucidation

Isomerism

Stereoisomers are molecules of the same formula that have the same bonds (connectivity), but a different spatial arrangement of the atoms. included in stereoisomers are configurational isomers (molecules that cannot be converted into one another through rotation about a o'-bond) and conformational isomers (caused by rotation and ring-flipping). Configurational isomers can be broken down further into either geometrical isomers (associated with nonrotating structures, such as rings and alkenes) and optical isomers (isomers that rotate piane-polarized light differently). Not all configurational isomers rotate planepolarized light, as you have seen with meso compounds, but opticai isomers differ in the magnitude and possible direction in which they rotate incident plane-polarized light. Stereoisomers of all types are not superimposable (they

I (

fi

ru

&

d[
m[

,0u

cannot be superposed onto one another.) Figure 2-4 shows the types of
stereoisomers. Rotamers are conformational isomers that vary in orientation in space because of rotation about a sigma bond.
Stereoisomers; Same bonds,

but a different spatial arrangement of the atoms

HOH

H^ccHz/

\s -(-

HOH

&

at,

.z H:C

\s C:.
CH2CHl @

(S)-2-butanol

(R)-2-butanol

Configurational isomers (Optical isomers)

,Q

,{F
.'rtl CH3

CH:
(cis)-4-methylcyclohexanol

&

(trans)-4-methylcyclohexanol

Configurational isomers
(Geometrical isomers)

Note: Configurational isomers have different prefixes in their IUPAC names

HH

C:. -z H3CCH2- CH2OH


1-butanol

\r

&

H"CCH' H-z

is C:.

CH2OH

1-butanol Conformational isomers


(Rotamers)

Figure 2-4

Configurational Isomers Configurational isomers are a subgroup of stereoisomers that have the same bonds, but a different arrangement of their atoms in space, no matter how the structures are twisted and rotated. Common examples with which you are familiar include optical isomers and geometrical isomers.

Copyright

by The Berkeley Review

9a

The Berkeley Review

Organic Chemistry
Optical Isomers

Structure Elucidation

Isomerism

optical isomers are molecules of the same formula and the exact same bonds 'connectivity), but a different spatial arrangement of the atoms due to asymmetry -'r'ithin the structure. An optical isomer cannot be rotated or manipulated tb assume the structure of another isomer. They cannot be converted into another cptical isomer without breaking a bond. Optical isomers rotate plane-polarized light differently from one another. Figure 2-5 shows an example of a pair of
..ptical isomers.

optical Isomers: Identical bonds with a different spatial arrangement about an asymmetric carbon that rotate plane-polarized light differently.
H

",?l7t-

cH2cH2cH3

&

\ CtrrrttHaC-

CH2CH2CHI

(R)-2-chloropentane

(S)-2-chloropentane

Optical Isomers Figure 2-5

Jptical isomers are a class of configuratisnal isomers. Configurational isomers :an also be classified as enantiomers and diastereomers, so some optical isomers ::l also be referred to as enantiomers or diastereomers.
E.rample 2.6

:{orv can the relationship between the following two molecular structures BEST

::

described?

\ CH2C\IZ .4,. Identicalmolecules B. Optical isomers Ll. Skeletal isomers f. Structural isomers

CIH,CH?CH?C.

.\

CH2CH2CH2CI &

.rut7tH

CH2CI

Solution
',-e

. iive-carbon chain. The chlorine

can start by naming each of the structures. Both have three chlorine atoms on atoms are on carbons 7, 2, and 5, so each rlecule has the IUPAC name 1,2,5-trichloropentane. To be skeletal or structural

':ereochemistry at the second carbon, while the right structure has R:::reochemistry at the second carbon. This confirms that the two structures are .:rical isomers, so the best answer is choice B.

-:-rmers requires that the two compounds have different IUPAC names, so -:-oices C and D are eliminated. No matter how you rotate the first structure, it :::rnot be superposed onto the second structure. This implies that they are not :entical molecules, so choice A is eliminated. The left structure has s-

-".pyright

by The Berkeley Review

99

Exclusive MCAT Preparation

Organic Chemistry

Structure Elucidation

Isomerism

Geometrical Isomers Geometrical isomers, simply put, are the cis and trans forms of a rigid compound (where rigid implies that it is not free to rotate between conformations). They are sometimes referred to as cis/trans isomers, which applies to rings and alkenes. They are nonsuperimposable, because they are locked into an orientation by the cyclic structure or n-bond. If two substituents on a cyclic compound are on the same side of the plane or if two substituents on alkene are on the same side of the carbon-carbon n-bond, then they are said to be cls to one another. If two substituents on a cyclic compound are on opposite sides of the plane or if two substituents on alkene are on opposite sides of the carbon-carbon n-bond, then they are said to be trans to one another. IUPAC convention does not use cis or trans in the naming of alkenes; instead, the letters E and Z are employed. In general nomenclature, the terms cis and trans are common. For geometrical isomers, which have different spatial arrangement about a n-bond, the prefix of E is given for trans orientation of the two highest priority groups, while Z is given for cis orientation of the two highest priority groups. Figure 2-6 shows several examples of pairs of geometrical isomers, both alkenes and cyclic structures.
Geometrical Isomers: Identical bonds with a different spatial arrangement (found with double bonds and rings).

Alkenes

H
\/LL

CH,CHTCH3

/\ HsC

c:c
H
Trans (E)

HH \/
&

/\ HsC

L-L
CH2CH2CH3

Cis (Z)

CHg

H?CO

&
HsC

CHe

Z ("Zis")
Cyclic Systems

cH"

(uP)

(up) HrC-H-)
(up)

&

(up) /YcHs HzC-_/--/


Trans

(down)

Cis
(t.tp)

(up)

(down)

Hrc\rA.rCH"

\J&
Cis

H3C\.n,.,rt\CH3

U
Trans

Figure 2-6

Copyright

by The Berkeley Review

loo

The Berkeley Kevieu

Organic Chemistry

Structure Elucidation

Isomerism

respective alkene carbon are trans to one another. The term Z is derived from the cerman word zusnmmen meanine "together" and refers to a compound where the two highest priority groups on each respective alkene carbon are cis to one another. Think of cis as "Zis", and you will always remember which is which.

The term E is derived from the German word entgegen meaning "across from" and refers to a compound where the two highest priority groups on each

Example 2.7

All of the following are true about geometrical isomers EXCEpT that: A. the E designation for an alkene refers to the highest priority groups on each

B. in both the E and Z isomers of an alkene, c. D.

alkene carbon in trans orientation.

the atoms directly bonded to the alkene carbons are all coplanar. molecuies capable of forming geometrical isomers have greater entropy than linear alkanes of equal carbon chain length geometrical isomers have relatively static structural features, such as polarity and solubility.

,qtatement, so

two alkene carbons, using the Cahn-Ingold-Prelog rules for assigning priorities to substituents by sequentialiy looking at the atoms attached to the site of interest. If the two highest priority groups are across from one another with respect to the louble bond, then the compound is trans and thus is assigned the letter E. If the :rvo highest priority groups are both on the same side of the double bond, then '&e compound is cis and thus is assigned the letter Z. This makes choice A a true

Solution To determine the geometry of an alkene, first locate the two carbons that constitute the alkene. Determine the highest priority substituent on each of the

it is eliminated.

coplanar with an orientation perpendicular to the substituents on the alkene carbons, the four atoms bonded to the two carbons of the alkene must be .oplanar. It is not possible to rotate around the doubie bond, because the p.rrbitals would no longer be coplanar, breaking the n-bond. This makes choice B a true statement, which eliminates it. There are only two geometrical isomers possible for an alkene. Because it is not possible to rotate about a double bond the ru-bond would have to be broken), it is not possible to convert between the cis and trans geometrical isomers without adding a great deal of energy. The .onsequence is that alkenes are rigid and thus have less entropy than alkanes of a lomparable carbon chain length. This makes choice C a false statement, and thus

Because the

two p-orbitals of the rc-bond

are

lhe best answer. The structures of both an aikene and cyclic molecule are

relatively static and do not change drastically. The ring may flip-flop a littie, but rotation is observed only for the substituents on the ring, and not observed for Jee bonds in the ring. Because the structure is static, the molecular features are constant. This makes choice D a true statement, eliminating it.
lVe shall discuss stereoisomers oniy at this superficial 1evel for the time being. In the stereochemistry and carbohydrate sections, stereoisomers will be discussed in greater detail.

Copyright O by The Berkeley Review

lol

Exclusive MCAT Preparation

Organic Chemistry

Structure Elucidation

lsomerism

Conf ormational Isomers

Conformational isomer s, ot conformers,

with identical connectivity because of rotation about a bond or (bonds) that are nonsuperimposable The most iontortion (often referr"d to u, iing-flipping) of the molecular structure' of tne motlculai structure is predictable based on
are molecules

stable conformation

hybridization,stericrepulsion,andVSEPR(ValenceShellElectronPair because nlpurcion) theory. Understanding conformational isomerism is vital, anl stability of a compound. Figure 2-7 shows strlcture ii.tut", the reactivity
two pairs of conformational isomers'
Conformationnl Isomers: Identical bonds with different of ring structures' carrs"d by either rotation about sigma bonds or contortion OH

spatial orientations

Hzc-=L--,/
OH

H-4

| |

CH"

.._ -

t\
t"
I

oH

OH

H:C

"--/--" H\/t/cwzcH3
cH2cH3
CHa

Figure 2-7

molecule' Figure Conformational isomers are different orientations of the same orientations) for butane' 2-8 shor"s six possible conformations (three-dimensional temperature, Conformational isomers are in dynamic equilibrium at room with the energy available' because rotation about sigma bonds is possible not remain in jusi one of Because molecules are consiantly rotating, butane does at one time or another' the conformations, but rather urrn*", all confirmations frequently' This means that there It assumes the most stable conformation most is a most favored and least favored conformation'

nT1.,,
nrr/ Y;"
Structure #1
HeC

CH:

CHs

Structure #2
HsC CHa

H Structure #3 H

V^

CHe

H\ul
H

v^
Structure

H\9)
HsC

#4 Structure #4

#5

btructure ffo Structure #6

Figure 2-8

from one conformer into Rotation about a carbon-carbon bond converts butane

another.Forinstance,al80"rotationabouttheCz-Czbondofbutaneconverts60' six structures represent Structure #1 in Figure 2-8 into Structure #4. The incrementalrotationsabouttheC2_.Cgbond,acrossafull360"rotation.
Copyright
@

bY The BerkeleY Review

t02

The BerkeleY Keview

Organic Chemistry

Structure Elucidation

Isomerism

Of the conformers in Figure 2-8, Structure #4 is the least stable, due to steric hindrance between the two methyl substituents (the largest groups). Structure #4 has hydrogens on carbon 1 and carbon 4 colliding with one another. Structure #1 is the most stable, because repulsion is minimized. To maximize stability, the largest groups on each carbon (CH3, in this case) need to be as far apart from one another as possible (tn anti orientation), if they repel. This minimizes the steric repulsion between the two groups. The least stable conformation (eclipsed, as it is cailed) has the two largest groups colliding into one another. The different orientations are referred to as confonnational isomers. Figure 2-9 shows a sider.iew of steric repulsion for the fully eclipsed butane molecule in Structure #4.
'.i.

HzC.. \-/

/\'H

H.

Steric repulsion between

,CHz hydrogens makes this


structure less stable.

H\YZ VN HH

Figure 2-9

A major part of learning about conformational isomers is learning


isomers based on the structures shown in Figure 2-8.

the

terminology. Listed below is a giossary of terms that apply to conformational Eclipsed: Eclipsed refers to the orientation where from a front view of the
molecule, you are prevented from seeing all of the substituents on the rear tarbon, because the substituents on the front carbon block the view of the rear. Structures #2, #4, and #6 are exampies of the eclipsed orientation of butane, Because of steric hindrance, the eclipsed conformations are less stable than the staggered conformations. Figure 2-10 shows the eclipsed conformation of ethane. Eclipsed Drawings of Ethane

HH

"Jy'-'Y';,.
Figure 2-10 Staggered: Staggered refers to the orientation where from a front view of the rrolecule, you can see all of the substituents on the front and rear carbons. There ,s a 60" dihedral angle between neighboring substituents. Structures #1, #3, and =5 are examples of the staggered orientation of butane. Staggered is the most .:able conformation. Figure 2-11 shows the staggered conformation of ethane.
Staggered Drawings of Ethane

n
H

c-c

v"
H

Figure 2-11

-opyright

by The Berkeiey Review

l03

Dxclusive MCAT Preparation

Organic Chemistry

Structure Elucidation

Isomerism

Within staggered conformation, there are two terms used to describe the relative position of substituents on adjacent atoms. These terms arc gauche and anti, and they refer to the position of a substituent on one atom relative to the position of a substituent on an adjacent atom. In the case of ethane, where each carbon has three hydrogen atoms attached, each hydrogen has two hydrogens gauche to it and one hvdrogen anti to it. Butane, having a methyl group on both carbon-2 and carbon-3, is a simplistic molecule to consider gauche and anti orientation. Rotation about the bond between carbon-2 and carbon-3 of butane generates conformational isomers where there is a methyl group on each carbon. We shall demonstrate gauche and anti orientation by referencing the methyl substituent on carbon-2 of butane relative to the methyl substituent on carbon-3 of butane.
The terms are defined belorv.

m
L0

jj

il\

:[
5

Gsttclrc: Gauche refers to a staggered conformation of the molecule, where the two groups of interest (often the iargest groups) have a dihedral angle of 60". Structures #3 and #5 have the CH3 groups gauche to one another. Three different perspectives of the staggered conformation of butane, with carbons 1 and 4 gauche to one another, are shown in Figure 2-12.
Staggered Conformation of Butane with CH, groups Gauche

Hfi!-cHr

CH,

H.C

C-

C..

'>("
H

n'/

v"
H

,/tn.

l :

Figure 2-12

:.
t!,

Anti: Anti

refers to a staggered conformation of the molecule, where the two groups of interest have a dihedral angle of 180". Structure #1 has the CH3 groups

anti to one another. This is the most stable structurel Three different
perspectives of the staggered conformation of butane, with carbons 1 and 4 anti to one another, are shown in Figure 2-13. Staggered Conformation of Butane
^1

Er

with CH, groups Anti

ar

IC

+
CH: Copyright O by The Berkeley Review

-{
H

c-c. ,/'n'
H:C

B. C.

\,"

Sc B

L<
B"

Figure 2-13

br,

ar

st:

The Berkeley Review

Cr

Organic Chemistry

Structure Elucidation

Isomerism

Because atoms within a molecule are in constant rotational motion, a structure loes not exist in one fixed conformation. Figure 2-14 shows the energy diagram that corresponds to a complete 360' rotation about the CZ-CS carbon-carbon bond rf butane, starting and finishing with Structure #L from Figure 2-8. The energy :iagram starts with the lowest energy structure at 0' (which is the conformation carbons 1 and 4 anti to one another) and rotates 360" about the C2-C3 bond ".-ith :o return to the same orientation. Note that the highest energy structure is eractly 180" apart from the lowest energy structure, and that a 60" rotation takes -"-ou from an apex (a localized energy maximum) to a nadir (a localized energy ninimum) on the energy diagram.

io
:!
0" 60"
1.20'

180'

240'

300'

360'

Degrees of rotation

Figure 2-14

ihe energy diagram for butane is symmetrical, because butane is a symmetrical inolecule. When the compound is chiral (contains stereogenic centers), the energy diagram is not symmetric. The energy axis of the diagram is not ;uantified in Figure 2-74, so it demonstrates only a conceptual relationship. If any calculations are required on the MCAT, values will be provided for the
energy of the eclipsed and staggered conformations, and for the gauche and anti
::'rteractions of various substituents.

Example 2.8

Ilow does the energy diagram for the complete rotation about a central bond of an asymmetric (chiral) compound compare to the energy diagram for its
complete rotation about the CZ-CS bond of butane?

A. The energy diagram is symmetric like butane, but with higher energy values. B. The energy diagram is symmetric like butane, but with lower energy values. C. The energy diagram is asymmetric like butane, but wiih greater differences
D.
in energy values. The energy diagram is asymmetric, with different energy values than butane.

Solution
Because the structure is asymmetric, the energy diagram depicting the rotation associated with the molecule must also be asymmetric, eliminating choices A and

B. The energy diagram for the complete rotation about the C2--4.g bond of butane is symmetric, so choice C is eliminated. Choice D must be the correct
answer. The three staggered conformations are of unequal energy.
staggered conformational isomers of l1-2,3-dimethylpentane are: The

Copyright

by The Berkeley Review

l05

Exclusive MCAT Preparation

Organic Chemistry
H3c<.^
L

Structure Elucidation

Isomerism

H,.

^/

.cH2cH3

-(. HsC

V"
CH:

.cH.cH, H"C. r&.- _/tn"n' r-r.'c'32.^ ^,/tn,t',

Hz9

n,c/'-

\:,f

n)'- \:,t

I
i

HsC

HsC

2 methyl/methyl and 1 ethyl/methyl interactions

$
CHs

CH2CH?
CHa

HgC\.

CH" J

HgC

H
CHs

H:C
H
1 2

H
methyl,/methyl and

methyl/methyl and I ethyl/methyl interactions Lowest Energy Conformation


1

ethyl/methyl interactions

first 120" of rotation of ethane. The graph shows that at 20" and 100" the
conformers at 40" and 80".

Figure 2-15 shows the exact orientation of the atoms at various times during the

structures are symmetric and thus of equal energy. This is also observed for the

Rotation about the

C-C

bond of ethane

20

40

60
Figure 2-15

80

100

720

Degree of rotational displacement from staggered conformation

one another, then the anti orientation is lowest on the energy diagram. However, when the force between neighboring substituents is attractive in nature, tl-re gauche orientation is most favorable. A good example is a vicinal diol, where the two hydroxyls on adjacent carbons exhibit hydrogenbonding with one another.

If the substituents repel

Copyright

by The Berkeley Review

The Berkeley Keview

Organic Chemistry

Structure Elucidation

Isomerism

Erample 2.9 .he best explanation for why the most stable orientation about the C2-C3 bond of

:i.e amino group and the carboxylic acid group in 3-aminopropanoic acid is :auche is which of the following? \. Gauche is the most stable, because it minimizes the steric hindrance between the amino group and the carboxylic acid group. B. The gauche conformation is always more stable than the eclipsed conformation. C. The carboxylic acid and amino groups form a hydrogen bond best from gauche orientation. D. The carboxylic acid and amino groups form a hydrogen bond best from anti orientation. the substituents attract one another, then the most stable conformation is s:aggered with the two attracting groups gauche with respect to one another. I-re best choice is C, because gauche has the amino and hydroxyl groups close .nough to form a hydrogen bond. Hydrogen-bonding is a stabilizing force. The :lrongest hydrogen bond is formed by the lone pair of nitrogen with the acidic :,r'drogen of the carboxylic acid group.

il

Solution

n&J*n,
H

,o,-./
o

ll"
(.

\:r:"
o?c
zHvdrogen bond-------

Anti orientation

- ,,H-

U-

"&_J

o-

ta ^.'.4

u^xr.

\Hz

,'/

Y3"

Gauche orientation

Hydrogen-bonding is possible when the two groups are only 60' apart, but not when they are 180" apart.

Copyright

by The Berkeley Review

r07

Exclusive MCAT Preparation

Organic Chemistry

Structure Elucidation

Isomerism

Newman Projections Understanding the nuances of conformational isomers requires good threedimensional viewing skills, so you may wish to dig out your molecular models if you still have them. Be able to recognize structures from the stick figure view (with dashed and bold wedges), as well as from Newman projections. Newman projections are front views of a molecuie. In drafting, three views are given to see the whole. It is no different in organic chemistry. The side view is a dashedand-bold wedge representation, the front view is a Newman projection, and a top view is a Fischer projection. Figure 2-16 shows a pictorial explanation of the conversion from a dashed-and-bold wedge drawing to a Newman projection,

r,n

str

ery drit
trmr
I[tuP

wT

MW

od

while Figure 2-77 shows a pictorial explanation of the conversion from


Newman projection to a dashed-and-bold wedge drawing.

When viewed from the right, substituents pointing out of the plane in a dashed-

and-bold wedge drawing are on the left side in a Newman projection.

mft

Substituents behind the plane in a dashed-and-bold wedge drawing are on the right side in a Newman projection. Substituents on the left side in the Newman projection end up pointing out of the plane in the dashed-and-bold wedge drawing. Substituents on the right side in the Newman projection end up behind the plane in the dashed-and-bold wedge drawrng.
Conversion from dashed-and-bold wedge drawing to Newman projection:

CJ q,)

,/
HgC

Y;,.",
Figure 2-16

o. HsC o o L

H
CHs

9.

-o

Conversion from Newman projection to dashed-and-bold wedge drawing:

behind

H
CHs

+)

in front
of

plane ,/ L-\, ,/
H

:f&1"'

OH
behind

Y,::*,"
ln frontH of piane

Figure 2-17 Cycloalkanes

Cyclic alkanes which contain only one ring have the chemical formula C.,H2., and contain no n-bonds. The stability of a given cycloalkane is rooted in its ability to form bond angies of approximately 109.5', the norm for sp3-hybridized carbons. The farther from 109.5" the angle is, the greater the reactivity of the cycloalkane. For this reason, three- and four-membered rings are reactive, while five- and six-membered rings are stable. When treated with hydrogen gas (H2), cyclopropane and cyclobutane readily form straight chain alkanes (propane and butane). Cyclopentane and cyclohexane do not undergo hydrogenation. Copyright
@

by The Berkeley Review

loa

The Berkeley Keview

Organic Chemistry

Structure Elucidation

Isomerism

The reactivity of three- and four-membered rings is attributed to ring strain. Rtng
T

:I o
t-

a e

strain is defined as the energy difference between the linear and cyclic alkanes of equal carbon iength. Because the bond angle in cyclopropane is 60", vastly different than 109.5' associated with a normal spr-hybridrzed carbon, there is a great deal of ring strain. To relieve this angle problem, cyclopropane forms what are referred to as bent bonds (sigma bonds in which the electron density does not lie between the two nuclei). Because the electron density is not between the two nuclei, the bond is much weaker and thus easier to break. Figure 2-18 shows the orbital and bonding pictures for various cyclic alkanes. Cyclopropane

I, a

tI.
e

Ring strain 27.4 kcal / mole

n
e

The carbon-carbon bonds are bent (not coilinear), making them weaker than standard carbon-carbon singie bonds.

Cyclobutane

Ring strain 26.2 kcal /

nole

Cyclopentane

uri-."r-)rr,
Rrng strain 6.8 kcai/mole

H2C-

cH2

Cyclohexane

HH

H
Figure 2-18

ln
its
ed

:le

t!.
-J

lecause of their stability, most cyclic organic and bio-organic molecules are =rther five-membered or six-membered rings. Six-membered rings are slightly rore stable than five-membered rings. Given their frequent presence in :iological molecules, five-membered and six-membered rings have a high :robability of appearing on the MCAT.

3I'{

-opyright

by The Berkeley Review

l09

Dxclusive MCAT Preparation

Organic Chemistry

Structure Elucidation

Isomerism

Cyclopentane Cyclopentane does not require much^distortion of its bonds and shape to accommodate the 109.5" angle for the sf -hybrid. A perfect pentagon has angles of 108', so there is only a small discrepancy from 109.5". This angle difference does not account for the small ring strain of 6.8 kcals per mo1e. To achieve the correct angle and alleviate this torsional strain, cyclopentane forms what is referred to as an enaelope shape, where one of the carbons is not coplanar with the other four. The major problem with cyclopentane is not the ring bond angles, but the substituents on the ring that are in an eclipsed conformation as a result of the near-planar ring structure. This eclipsing of hydrogen atoms causes further contortion of the structure, which accounts for the ring strain energy. Still, cyclopentane structures are relatively stable, Their stability makes them common in such biological structures as ribose, deoxyribose and the purine ring of the DNA bases adenine and guanine. Figure 2-19 shows a few other common fivemembered rings frequently encountered in the biological sciences.
COr-

ui"

lh

LU

lllln

m
il
rilm

*H"N-

"l

t-

OHH
B-D-Fructofuranose

HN\---zN
L-Histidine

CH"

t'

OH

HO

OHH
2-Deoxy-B-D-ribofuranose Figure 2-19

B-D-Ribofuranose

Copyright O by The Berkeley Review

lIo

The Berkeley Review

L-l

Organic Chemistry
Cyclohexane

Structure Elucidation

Isomerism

Cyclohexane has the most stable ring structure of all of the cycloalkanes. This is evident in biological molecules, which have multiple rings of six atoms. The

most stable form of cyclohexane is the chsir conformation. There are two different chair conformations for cyclohexane. The two conformational isomers
can interconvert through a process referred to as ring-flipping. In interconverting,

the structure passes through the boat conformation. Because the two chair conformations are equally stable, AGr* - 0. The interconversion between the two chair conformations of cyclohexane requires 10.8 kcals/mole in activation

energy. The chair conformation offers two substituent positions: equatorial (named for its orientation around the equatorial plane of the ring) and axial (named for its vertical alignment like an axis). Equatorial is more stable than
axial, so the most stable conformation of a cyclohexane compound has the largest substituents in the equatorial positions. Figure 2-20 shows chair conformations of cyclohexane with detailed positions.

H
Cyclohexane showing equatorial Hs Cyclohexane showing axial Hs axiai (up)

axial (up) equatorial (up)


(up)
eq (down)

eq (down) eq (up)

eq (up)

axial (down)

equatorial (down)
axial (down)

axial (down)
Cyclohexane showing all substituents

Figure 2-20 Copyright


@

by The Berkeley Review

lll

Exclusive MCAT Preparation

Organic Chemistry

Structure Elucidation

Isomerism

Interconversion between Cyclohexane Conformers

The interconversion between the two chair conformations of cyclohexane requires activation energy, but the free energy (AGr*) is zero, because there are no substituents on the cyclohexane ring. As cyclohexane undergoes ring-flip between chair conformations, it passes through intermediate structures. The two most significant intermediate structures are tLle twist form and the boat form. The twist form is of slightly lower energy than the boat form, because the hydrogens are not eclipsed in the twist form. The ring-flip conversion of cyclohexane from
chair to boat and on to the other chair conformation is shown in Figure 2-21.

il
H
+

ry

dffi

H
p1 H

tu

ro

Twist

ffi

mh

tu

@ry

ffi
il[m

ffim

HH
Figure 2-21
Because molecules can readiiy rotate and contort at room temperature, an equilibrium exists between the two chair conformations of the cyclohexane molecule. Either at low temperatures or'in cases where a substituent is too bulky to orient itself in an axiai manner, one conformer is present exclusiveiy. The proton NMR can be used to determine the more stable structure by focusing on the different signals for the two orientations of hydrogen. You should be able to determine the most stable conformation. The boat conformation is not as stable as the chair conformation, because its substituents encounter high energy eclipsed interactions its substituents encounter, while the substituents in the chair conformation are staggered with lower energy gauche and anti interactions.

@M

Monosubstituted Cyclohexane The ring-flip process is the same when there is a substituent on the ring as it is for cyclohexane, but the energetics are different. When a substituent is present, the activation energy for interconversion increases, and the two chair conformations differ in stability. Because the two chair conforrnations are no longer of equal energy, the two twist forrns are also no longer of equal energy. The most stable chair conformation is the structure r,r'ith the least steric repulsion. Substituents with axial orientation on the same side of the ring are close enough to repel (known as 1,3-diaxinl interactions), so axial orientation is less favorable than equatorial orientation. This is indicatecl in Figure 2-22, where the two chair conformations of both methylcyclohexane ar-rd cvclohexanol are shown. The two chair conformations of methylcyclohexane differ in stabiliiy by 7.69 kcals,/mole. Because a hydroxyl grorlp is smaller than a methyl group, the difference in energy for the two chair conformations of cr-clohexanol is only 1.04 kcals/mole. Copyright O by T'he Berkeley Review

tt2

The Berkeley Review

Organic Chemistry

Structure Elucidation

Isomerism

H
Methylcyclohexane Equatorial > Axial by 7.69 kcals/mole
Figwre 2-22

oH
I

Cyclohexanol Equatorial > Axial by 7.04 kcals/mole

Disubstituted Cyclohexane Disubstituted cyclohexane exhibits different dynamics than monosubstituted cyclohexane. The greatest energy difference is observed when comparingT,3diaxial to 1,3-diequatorial, because the steric repulsion of 1,3-diaxial substituents is the strongest repulsion encountered in cyclohexane. This is shown in Figure 222 for dimethylcyclohexane. When comparing 7,2-diaxial to 1,2-diequatorial, there are no diaxial interactions of the bulkiest substituents, because the two bulky substituents are trans to one another. This makes the energy difference between chair conformations less than what would be observed with the 1,3cyclohexane. The energy difference between the 1,2-diaxial and 1,2-diequatorial orientations is also less than the energy difference between 1,4-diaxial and 1.,4diequatorial (which is another compound having the two bulkiest substituents
trans to one another), because the 1,2-diequatorialcyclohexane species has gauche

interactions between the bulkiest substituents. The energetics of the

conformational isomers of dimethylcyclohexane are shown in Figure 2-23.

CH"

L;-/ :
/-=--/4
I

l"

H.c/114 "'[.\1-\

CH. "

Trans-7,2-dimethylcyclohexane

1,z-Diequatorial > 1,2-Dtaxialby 2.8 kcals/mole

N
CHs

.r,
9H:

:- "''H*"
H:c-\/-\

Cis-1,3-dimethylcyclohexane

1,3-Diequatorial > 1,3-Diaxial by 8.4 kcals/mo1e

F/Fcrr.

CH, "

Traus-7,4-dimethylcyclohexane

t,+-Diequatorial > 1,4-Diaxialby 3.4 kcals/mo1e Figure 2-23

Copyright O by The Berkeley Review

ll3

Exclusive MCAT Preparation

Organic Chemistry

Structure Elucidation

Isomerism

either up/up or down/down. Cis and trans do not refer to equatoriil or axial orientations, but rather to whether the substituents are above or below the ring.
Possible Orientations for Disubstituted Cyclohexanes
7,2-trans (upldown) or (down/up) 1,2-cis (uplup) or (down/down) 7,3-cis (uplup) or (down/down) 7,3-trans (upldown) or (down/up)

Table 2-1 lists the possible orientations for disubstituted cyclohexane. Trans refers to substituents on opposite sides of the ring, equating to either up/down or down/up. Cis refers to substituents on the same side of the ring, equating to

:
J:,

si

t.

<+ e; e> 7, -trans (upldown) or (down/up) e \, -cis (up/up) or (down/down) ++

C-1 C-1 C-1 C-1 C-L C-1

axial axial axial axial axial axial

C-2 C-2 C-3 C-3 C-4 C-4

axial or C-1 equatorial C-2 equatorial equatoriai or C-1 equatorial C-2 axial axial or C,1 equatorial C-3 equatorial equatorial or C-1 equatorial C-3 axial axial or C-1 equatorial C-4 equatorial equatorial or C-1 equatorial C-4 axial

Table 2-1

It is essential that you be able to translate from nomenclature to the most stable conformation. For instance, trnns-3-methylethylcyclohexane is a 1,,3-trans compound. A compound with 7,3-trans orientation has both an axial and an
equatorial substituent. The ethyl group is larger than the methyl group, so the ethyl group occupies the equatorial orientation in the most stable conformation of trans-3-methylethylcyclohexane. This is shown in Figwe 2-24.

-:
rr.x

llhr

tl!
I"}ff

:lhu
ilIJI,

b--_LcH:
I

f__4

IIIIIII

:ilh,r
dlilrru

[]rgft

I[!r!

cH2cH3
Figure 2-24

You should take note that when you convert from one chair conformer to the other, the axial substituents become equatorial (as seen with the ethyl group), and the equatoriai substituents become axial (as seen with the methyl group). The most stable conformation has the least steric repulsion.

Example 2.10 The most stable conformation of cis-L,2,4-trimethylcyclohexane has which of the following orientations for the three methyl groups?
CHg

A. The chair conformation B. The chair conformation C. The chair conformation D. The chair conformation

witl-r 3 equatorial methyls and 0 axial methyls with 2 equatorial methyls and 1 axial methyl with 1 equatorial methyl and 2 axiai methyls with 0 equatorial methyls and 3 axial methyls

Copyright

by The Berkeley Review

ll4

The Berkeley Review

Organic Chemistry
Solution

Structure Elucidation

Isomerism

The most stable conformation has as many methyls equatorial as possible. 7,2-cis

has one axial and one equatorial substituent, so there must be at least one axial substituent. Answer B is best. Drawn below are the two chair conformations of

cis-7,2,4-trimethylcyclohexane (or (1R,2S,4R)-1,2,4-trimethylcyclohexane


correctly name using IUPAC nomenclature rules.)

as

H:.cd

/
1

r-)

CH.

l"

CH,

l'

CH.

1"".

\-\4
WcH.,

equatorial;2axial

-\

2 equatorial; 1 axial

This covers the topics associated with isomerism. These topics are applicable in both the physical and biological sciences areas, so know them well. To implant this section as a working knowledge base, there are many passages with which to work. From the very beginning, you want to emphasize the logic behind your answers. The MCAT may not have passages that are verbatim duplicates of what you see in here, but if you answer these questions using sound logic and fundamental concepts, then you will slowly get acclimated to the MCAT way of thinking. At this point, passages may seem like an absurd form of asking questions, but hopefully you wiil take a liking to the style. Passages present information that you must incorporate into your background knowledge, and then using all the information you have, you must reach a conclusion concerning their questions. Multiple-choice tests require that you find the best, most reasonable answer. You are not required to solve detailed questions or derive fundamental concepts. Just find the best answer, as fast as you can.

Copyright

by The Berkeley Review

I l5

Exclusive MCAT Preparation

Organic Chemistry

Structure Elucidation

Structural Insights

ffi#ffiilffit

Structural Symmetry
\A/hen deducing the molecular structure for an organic molecule, it helps to know

imaginary mirror in the middle of the molecule. In a structure with point symmetry, there is an inversion point at the center of the molecule such that if two lines are drawn in opposite directions along the same axis, then both line segments intercept identical atoms at the same diJtance from the inversion point. This may not seem clear in words, but looking at a structure helps illustrate the concept. Figure 2-25 shows one compound with mirror plane symmetry and another with inversion symmetry. Molecules with inversion points are nonpolar, because all of the individual bond dipole vectors cancel each other out.

something about the symmetry of the compound and its units of unsaturation. symmetry can be broken into plane symmetry and point symmetry. In plane symmetry, the compound has two halves that are evenly displaced about an

cH20H

HOH"C

H '\r.-o1rY*'

HgC

*--*oof''
-,H
Figure 2-25

"\..
CH2Q.H

Molecule with mirror symmetry

Molecule with an inversion point

Symmetry within a molecule affects its NMR and IR spectra. As symmetry increases, the number of signals in a spectroscopic study decreases. coupling symmetry information with units of unsaturation helps to deduce the structural feafures and connectivity of a molecule. Units of Unsaturation Units of unssturation are calculated from the molecular formula. The units of unsaturation give us information about the number of rings and/or n-bonds present within a molecule. There is some minimum number of bonds needed to hold the atoms in a molecule together, and any additional bonds beyond the minimum are the units of unsaturation. To hold two atoms together, it takes one bond (Atom1-Atom2). To hold three atoms together, it takes two bonds (Atom1-Atom2-Atom3). The minimum number of bonds required to hold a molecule together is always one less than the number of atoms. The minimum number of bonding electrons is two times the minimum number of bonds. Any electrons beyond the bare minimum needed to hold the molecule together can be used to form additional bonds. For every extra pair of electrons, there is a unit of unsaturation. To determine the units of unsaturation, the strategy is to determine the number of excess bonding electrons. There are a few different methods for doing this.

1) C3H6 contains eleven atoms, which requires

at minimum ten bonds (and

thus twenty bonding electrons). There are three carbons with four bonding electrons each. There are eight hydrogens with one bonding electron each. This means that propane (C3Hg) has exactly the twenty bonding electrons needed. There are no extra bonding electrons, so propane has a linear unsaturation.

structure with no n-bonds. This is to say that propane has no units of

Copyright O by The Berkeley Review

rr6

The Berkeley Review

Organic Chemistry

Structure Elucidation

Structural Insights

'1.

ie

n rt
iJ
re
L-

The units of unsaturation for hydrocarbons and carbohydrates can be derived from the formula for aliphatic alkanes, Cr,H2', 1 2. "Aliphatic" refers to a structure that has no rings or n-bonds. An aliphatic alkane has the bare minimum number of bonds, so there are no units of unsaturation. For every ur-rit of unsaturation, there are two fewer hydrogen atoms than the maximum. Thus, the units of unsaturation can be obtained by comparing the actual formula to the fully saturated formula. For instance, C5H6 has four hydrogens less than the fully saturated formula for five carbons, CSH1Z. Because it has four fewer hydrogens, it has two units of unsaturation.
The units of unsaturation depend on the surplus of bonding electrons. To keep any chain propa gated, every member of it must make two connections. ln a molecule, each atom must make two bonds to keep it intact. This means

re

rl

that every atom needs a minimum of two bonding electrons. Using this perspective, we can determine the number of excess electrons per atom. Hydrogen makes just one bond, so you subtract one for each hydrogen in the molecule. Oxygen atoms are ignored, because they make the minimum two bonds that are needed. Carbons are multiplied by two, because carbons make four bonds, two beyond the minimum to propagate the chain. There are two ends to every chain, so two is added to the total. The units of unsaturation refer to bonds, rather than bonding electrons, so the sum of excess electrons must be divided by two. This is summarized in Equation
2.7.

Units of unsaturation =
:\
)g 'D

2(#C)+2-(#H)
2

(2.1)

a1

l"lethod 3 works with other atoms, too. Nitrogen makes three bonds, which is ::-e more than the minimum needed, so you add 1 per nitrogen atom. Halogens :-':ke one bond, which is one less than the minimum, so you subtract 1 per . -:-,de. Equation 2.2 includes nitrogen and halogens.
Units of unsaturation = 2(#c) + (#N) - (#H) - (#x) + 2
2

(2.2)

0t

ls
to
:IC

Erample 2.11 :::",r' many units of unsaturation are present in a compound with the molecular

1e

::::nula C7H9N3O2Cl2?

ls ta
II1 -t\' be

{. 8,2 :.3 1.4


1

of
to

t rlution -:,s question is solved by applying Equation 2.2.

nt
.,J ng

-. )+ (3)-(9)-(2)+2 =r4+3-9-2+2

2222

=72

-9 - B -

4Unitsof unsaturation

: -rr units of unsatuiation, the compound

ln.
'l:s

lar

oi

:::ause there are four units of unsaturation, choice D is the best answer. With could contain three n-bonds and one ::ng, meaning it is potentially a benzene derivative. When there are four units of ,:.saturation, you should immediately consider the possibility that the . --mpound contains an aromatic ring. While there are other combinations of four -::ts of unsaturation, there is a high probability of having an aromatic ring.
-:p.r'1i*1't,
@

:*'

by The Berkeley Review

tt7

Exclusive MCAT Preparation

Organic Chemistry
Spectroscopy

Structure Elucidation

Structural Insights

l
il

Spectro3bop$

an Andysis

,il

unknown, you may also have to assign signals and peaks to an existing compound. The spectra should be interpreted using the typical features you have learned. To date, the test has emphasized only a few features that have been stressed in course work. We shall start by reviewing the basic operations of the IR and its applications to structure elucidation. From there we sha1l consider ultraviolet,/visible spectroscopy, proton NMR, and carbon-13 NMR.
Infrared Spectroscopy
Er,'ery molecule produces a unique IR spectrum. Infrared spectroscopy starts by adding a monochromatic beam of IR photons to either a thin oil suipension (if the compound is a solid) or a neat solution (if the compound is a liquid) between

The MCAT togics include infrared absorption spectroscopy, uitravioiet-visible spectroscopy, 1H (proton) nuclear rnugtl"ti. ,"ronun." spectroscopy, and 13C (carbon) nuclear magnetic resonance spectroscopy. There is nothing for you to fear, however. At just over one minute per problem, if you have to deduce a structure from spectroscopic information, then it will likely be easy or symmetric. For example, on a previous exam, there was a proton NMR of ethanol that many students said was very easy. Besides just having to determine the structure of an

d d

Jffi

tffi

mh

l@G

h ffi

salt plates. The molecule absorbs electromagnetic radiation that causes transitions between vibrational energy levels within it, meaning that the molecule vibrates more or less frequently as energy is absorbed or emitted. when a molecule soaks up the EM radiation and vibrates at a higher energy. This change in stretching (vibrating) between atoms within the molecul" .u.6ei'a change in the dipole moment, which can then be monitored. An infrared spectrometer uses light of wavelength 2,500 nanometers to 17,000 nanometers
(recorded as 4000 cm-1 to 600 cm-1 on the graph). \A/hat we record is the change in the intensity of the EM radiation from when it enters the molecule to when it
exits the molecule. This is compared to a reference beam that traverses a path of identicai length but does not pass through the compound itself. If the compound. absorbs a given wavelength of light (corresponding to some transition), then we observe an absence of light exiting the sample tube. This is known as absorption spectroscopy. The graph records transmittance as a function of wave number (cmr), so absorbances are represented by drops rn intensity.

The frequency at which light is absorbed is specific for each type of bond. As you may have learned in physics, the frequency of light is directly proportional to the masses of the two atoms in the bond and the bond strength. This is to say that the potential energy in a resonating system (such as a spring that obeys Hooke's law) is described by Equation 2.3 P.E. = 1 kx2
2

(2.3)

The k-term is the spring constant, which we can say describes the bond strength. The x-term describes the distance from equillbrium that the bond has stretched. The absorbance can be thought of as increasing the potential energy of the bond, so the absorbance is proportional to the energy of the bond. As a resuit, the bond

dissociation is directly proportional to the energy that is absorbed. This is not exactly true, but close enough to help approximate spectra. Because a wave number is measured in cm-l, it is an inverse of the wavelength. The inverse of the wavelength is directly proportional to the energy of the photon. This means that the higher the wave number is, the greater its energy.

Copyright

bv The Berkeley Review

rr8

The Berkeley Review

Organic Chemistry

Structure Dlucidation

Spectroscopy and Analysis

For instance, a C=O boncl absorbs around 1700 cm -1, while a C-O bond absorbs around 1300 cm -1. this is because a C=O bond is stronger than a C-O bond. Carbonyl functional groups a-re common, so you should know the absorbance value for a C=O bond. An sp3-C-to-H bond absorbs just below 3000 cm-1, while an sp2-C-to-H bond absorbs just above 3000 cm-l, because an sf -C-to-H bond is stronger than an sp3-C-to-H bond. This is because the sp2-hybrid, having more scharacter, is smaller than the sp3-hybtid. The result is that an sp2-C-to-H bond is shorter and thus stronger than an sp3-C-to-H bond. The stronger bond, having a higher bond dissociation energy, has a higher energy absorbance.

Although the molecule as a whole absorbs the EM radiation, we can use the absorbances we measure to fingerprint particular functional groups and bonds rvithin the molecule. The skill needed to make IR useful is an active process. Scientists use IR not only to confirm the presence of certain functional groups, but to also to help decide which functional groups are not there. IR is most useful as a supplement to the moiecular formula and the NMR spectra for molecules. Table 2-2lists several useful IR absorbances. The values are listed in terms of wave numbers. Note that the absorbance of a given bond varies with ihe compound in which the bond exists.
Bond type Stretching (cm-1)
3640 - 3580 v) 3600 - 3200 (s, broad) 3500 - 3350 m) 3450 - 3200 m) 3300 - 3220 s) 3100 - 3000 v) 3060 - 3020 m) 2980 - 2860 s) 880 660 (v) 1620 - 1590 (v)

Bending (cm-1)

O-H alcohol (no H-bonding) O-H aicohol (H-bonding) N-H amides N-H amines C-H alkvnes C-H aromatic C-H alkenes C-H alkanes C-H aldehyde O-H acids (H-bonding)
C=C alkvnes C=N nitrile C=O ester
C=O aldehyde C=O ketone C=O acid C=O aryl ketone C=O amide C=O cr,B-unsaturated ketone C=C alkene C=C aromatic

1000 - 700 (s) 1470 - 7320 (s)

2900+,2700+ (m, 2 bands)


3000

2500 (s, broad)

1655 - 1510 (s)

2260 -2720 (v)


2260 - 2220 (v) 7750 - 1735 (s) 7740 - 7720 (s) 1725 - 1705 (s) 1725 - 7700 (s) 1700 - 1680 (s) 1690 - 1550 (s) 1685 - 1665 (s) 1680 - 1620 (v) 1600 - 1450 (v) 1300 - 1000 (s) 1220 - 1020 (w)

C-O C-N

alcohols, ethers, esters amines, alkyl


s = strong absorption

m = medium absorption Table 2-2 Copyright


@

w = weak absorption v = variable absorption

by The Berkeley Review

ll9

Exclusive MCAT Preparation

Organic Chemistry

Structure Elucidation

Spectroscopy and Analysis

using Table 2-2, you can evaluate IR data given in spectrum form to identify structures. As for memorizing peaks, according to the MCAT student Maru,Lal you are required to "know the important ones," which is open to interpretation. You don't necessarily have to memorize all of the values, but if you do enough
problems, the values you repeatedly see should become second nature.

fr

As a diagnostic tool, IR is used to detect certain functional groups. You have reached the pinnacle of utility when you use it to determine which functionalities are not present as well as which functional groups are present. Just as peaks confirm the presence of a certain bond, the absence of a peak supports the absence of that bond. Here is an example of how IR spectroscopy is used:

I (

An unknown compound with formula C_aHgO is analyzed by IR spectroscopy. An intense band is detected at 1770 cm-1 (IR absorbances are listeh by energy according to the wave number as measured in cm-1). By comparing the value to a chart of IR absorbances, this peak can be attributed to a C=O. The compound has one degree of unsaturation attributable to a C=O, which makes it possible to narrow it down to a small number of isomers. The structure cannot be cyclic and has a carbonyl. Given that the longest chain is four carbons, the carbonyl can be only on carbon 1 or carbon 2. This narrows it down to only two butane derivatives. The iongest chain could be only three carbons, with a methyl substituent on carbon 2. In that particular structure, the carbonyl group has to be
on carbon one. This leaves only three possibilities, and they are:

$ T
oru

u
lflr,

,&

efl

mm @u

1. H3CCOCH2CH3 (butanone) 2. H3CCH2CH2CHO (butanal) 3. H3CCH(CH3)CHO (2-methylpropanal)


Thus, we can reduce the choices from many types of compounds having one unit of unsaturation and one oxygen (a cyclic ether, for example) to a few. Ketones

and aldehydes have different chemical reactivity and physical properties, so when we combine IR information with chemical tests and the melting point of the compound, we can eliminate two of the three structures. This is a structure elucidation technique you have done many times in the past. Through examples ar-rd practice, you can familiarize yourself with the peaks and become talented at solving the problems using deductive reasoning.

Example 2.12 Which of the following compounds with the formula C5H19O cannot have an IR absorbance peak between 1700 cm-1 and 1750 cm-1?

h tu

w ffi &

TM

A. B. C. D.

An aldehyde A ketone A cyclic ether


A11

of the above have an IR absorbance between 1700 cm-1 and 1750 cm-1.

Solution An IR absorbance between 1700 cm-1 and 1750 cm-1 implies that the compound has a C=O in its structure. Because it has no absorbance between 1700 cm-1 and 1750 cm-1, it does not have a C=O bond. Choices A and B have a C=O in their structure, so they can be eliminated. The best answer is choice C. The one degree of unsaturation associated wlth the formula is used in the ring. The one oxygen in the formula is in the ether, which contains carbon-oxygen single bonds. Choice D is also eliminated, because choices A and B are eliminated.

Copyright

by The Berkeley Review

The Berkeley Review

Organic Chemistry
Example 2.13

Structure Elucidation

Spectroscopy and Analysis

How many structural isomers of C4H3O2 are possible that have an IR absorbance peak between 7735 cm-1 and 1750 cm-1, u p"ik around 1200 cm-1 and no broad
peaks above 2500 cm-1
?

4.2 B.3
D.5

c.4
Solution
The absence of a broad peak above 2500 cm-1 indicates that the compound does not have an o-H bond, which eliminates the possibility of it being an alcohol or carboxylic acid. It is most likely an ester, although it could have both a carbonyl and ether functionality. According to Table 2-2, the peak betweenIT35 cm-1 and 1750 cm-1 indicates ihat there ;s a c=o bond of an ester and not a ketone or aldehyde. The compound must be an ester, so the question now becomes, "How many esters are there that contain only four carbons in their structure?" There are only four four-carbon esters, as drawn below, so the best answer is choice C.

1)o2)
H,/-\
C

o
lt
OCH"CH2CH3

il

H3CH2C/C-

Oarr,

3)o
il

4)o
OCH(CH3)2

H,ZC\

nrC/

toarrra*ru

il

The two aldehyde structures (1 and 3) could be confirmed or eliminated by the presence or absence of two peaks at 2900 cm-1 and2700 cm-1. If this were a real iaboratory scenario, it would be far easier at this point to use proton NMR to deduce the structure of the unknown compound. Structure t has four types of hvdrogenin a 1:2:2:3 ratio. Structure2hasthree types of hydrogenina3:2: i ratio. Structure 3 has three types of hydrogen in a 1 : 1 : 6 ratio. Structure 4 has three types of hydrogen in a 3 : 2 : 3 ratio. The integration would be enough to distinguish anything except Structure 2 from Structure 4. To distinguish these trvo structures requires identifying the ppm shift value of each type of hydrogen.

iVe shall address NMR spectroscopy later in this chapter. IR spectroscopy r""ta r"

"tttt"a

;
f

1r

te te

ie

Copyright O by The Berkeley Review

t2l

Exclusive MCAT Preparation

Organic Chemistry

Structure Elucidation

Spectroscopy and Analysis

Hydrogen-Bonding in Infrared Spectroscopy Because the formation of hydrogen bonds affects the covalent bond between an atom and a partially positive hydrogen involved in hydrogen-bonding, any spectroscopy techniques focusing on the covalent bond to hydrogen, or the hydrogen itself, is affected by hydrogen-bonding. The effect is a broadened peak (observed in both the IR and NMR techniques). In the case of IR, the broadening of the hydroxyl absorbance associated with hydrogen-bonding is caused by the weakening of the covalent bond between the hydrogen and the atom (nitrogen, oxygen, or fluorine) to which it is bonded. This lowers the energy of the covalent bond and thus lowers the energy of absorption for the bond. As the hydrogen bond increases in strength, the covalent bond weakens. Because not all of the hydrogens have the same degree of hydrogen-bonding, their covalent bonds exhibit many different absorptions, ranging from unaffected and therefore standard covalent bonds to covalent bonds that are highly weakened by the hydrogen bond. This range of covalent bonds gets grouped together into the one broad peak. The same alcohol exhibiting two different degrees of hydrogenbonding is shown inFigure2-26.

l""o"Tro"*

\ ts

\""0

I j"''-.'" .$
I

Stronger hydrogen-bot-td

HH
*t

o/t\ o/\n
st.ong", cluut"r,t

bond

o/c\
Figure 2-26

\,.s
o<n

\I

i ' '\

s c<H
I

Weaker cJalent uona

Given that molecules are in continuous random motion within a liquid, some alcohols have strong hydrogen bonds, while others have no hydrogen-bondingThis means that the solution has a random distribution of hydrogen bonds and therefore a random distribution of covalent bonds. The result is a distribution of signals in infrared spectroscopy. To see each individual peak for each different covalent bond requires a high resolution IR spectrophotometer. It is unlikely you used such an instrument, so the signal with which your are familiar is a broad composite signal covering the range of the individual signals. Figute 2-27 s a high-resolution IR signal and the standard-resolution equivalent.

3440

3420 3400 3380 3360 3340 Wave number (cm-1) High-resolution spectrum

3320

3440

3420 3400 3380 3360 3340 Wave number (cm-1) Low-resolution spectrum

3320

Figure 2-27 Copyright


@

by The Berkeley Review

t22

The Berkeley

l5

0rganic Chemistry

Structure Dlucidation

Spectroscopy and Analysis

te

Example 2.14 ',"hat is true for the compound associated with the following IR spectra?

1457

cm-l ll

3317 cm-l 2903 cm1 2987

r:ii .,n ' & 736'7 cm-l

cn-l

-{. It exhibits no hydrogen-bonding. B. It has a carbonyl group. C, Ithas ahydroxyl group.


D.
It has a molecular mass that is less than 30 grams per mole. Solution - he compound represe-nted by the lR spectrum above in the question has a broad :-eak around 3300 cm-l and no peak near 1700 cm-]. These are the first areas to :onsider when looking at IR spectra. The compound has a hydroxyl group, but ro carbonyl group. This makes choice B incorrect and choice C correct. Because :t has the hydroxyl group, it can exhibit hydrogen-bonding. This eliminates ;hoice A. Because of the peaks in the 1300-1400 cm-1 range, we know the :ompound has a carbon-carbon bond, so it must have at least two carbons. The Lightest compound with two carbons is ethyne (HC=CH), which has a molecular nass of 26 grams per mole. However, because there is an oxygen present, the compound must have a molecular mass greater than 30 grams per mole. Choice D is eliminated.

spectroscopy. \A4rile infrared photons (in the 3-to-10 kcals/mole region) affect the vibrational energies of a molecule, ultraviolet (in the 70-to-300 kcals/mo1e region) and visible (in the 40-to-70 kcals/mole region) photons affect the electronic energy levels. when uv or visible photons are absorbed by a molecule, an electron is said to be excited from the ground state to an excited state. Because o-bonds are so much stronger than n-bonds, the lowest energy absorbance for alkanes is significantly higher than the lowest energy absorbance for alkenes. To excite an electron from the o-level (sigma bonding orbital) to the o*-level (sigma anti bonding orbital), photons of approximately 140 nm to 120 nm are necessary. However, because molecules in the air readily absorb energy in this region, the spectra must be obtained in a vacuum, Because this constraint is rather impractical, uv-visible spectra typically range from 200 to 800 nm, where air does not interfere. As a result, we typically use only UV-visible spectroscopy to analyze molecules with n-bonds, especially conjugated systems. uv-visible spectroscopy in organic chemistry focuses on transitions between the n and n* energy levels. For systems with conjugation, there are several n-levels, but we care about only the lowest energy transition.
3W

In addition to infrared spectroscop/, there is also ultraviolet/visible

Ultraviolet/Visible Spectroscopy

Copyright

by The Berkeley Review

t23

Dxclusive MCAT Preparation

Organic Chemistry

Structure Elucidation

Spectroscopy and Analysis

I
J]iL

The transition of interest is from fi to

known aslambda max of L,,'.u* depends on the amount of conjugation. As the conjugation increases, so does the tnaaelength of L*o*. Figure 2-28 shows the l"*u* values associated with the lowest energy n-to-n* transition of various conjugated hydrocarbons.

n*. The wavelength of highest absorbance, (),^u*), changes with the amount of conjugation. The value

F[
dffi.

r@

Il*

_Tn
217nm

mm

fuil

t/\Arl

r75nm

V\nrl
TE

iI

'r3

-rH"C:

l*ln3 n2
ir1

t/V\-l

z58nm

Tn4 -I.
I

ryr

lrillt[

{m M
@M

,tE.
rc1

Md

'tlllllillF

ffie
ffiar

H2C:

CH

H,C: -CH:
CHZ

CH

-cn
ll

CF{2

@
@m
,ilmhm

H2C:
Figure 2-28

/C'
CH

mry

muil

,m

mum

{ffrol
Because the energy gap between n and n* decreases as the conjugation increases,

the wavelength of maximum absorbance increases. 1,3,5,7-octatetraene has a I*u* of 304 nm and 1,3,5,7,9-decaquintene has d Imax of 353 nm. When more conjugation is added, the absorbance shifts into the visible range. Color resulF from excessive conjugation within a molecule. For instance, B-carotene (with 1L n-bonds) has absorbances at 483 nm and 453 nm. Substituted benzenes have a number of peaks. Conjugated aldehydes and ketones have about the same n-:i* absorbances as conjugated alkenes of the same number of n-bonds. However' conjugated aldehydes and ketones have other, more intense absorbances (e > 10,000) that are of longer wavelength than their hydrocarbon counterparts. ThL" is attributed to the n-to-fi* transition associated with aldehydes and ketones, possible because of the lone pair of electrons on the carbonyl oxygen. Figwe2-29 shows the L,^u* values associated with the lowest energy n-to-fi* and n-to-n* transitions of various ketones.

mru rym

o
il

o
CHs
n n

-\ HsC
n -+ 7tn
TE

I-u, = 270 nm
I-u,
= 187 nm

H^cD ^r.W
-t fi*
I,,'u* = 324nm
n
-->

7t*

I*u*

= 368 nm

-)

1l*

-t

Tl*

I-u*

= 219 nm

Tl

-+ 1l* l*u* = 267 nm

Figure 2-29

Unlike infrared spectroscopy, ultraviolet/visible spectroscopy can also


applied in a quantitative fashion. Ultraviolet/visible spectroscopy can be used determine the yield of a reaction, if it involves a UV-visible active compound. organic chemistry, a compound must have a n-bond to be UV-visible active'

Copyright

by The Berkeley Review

124

The Berkeley

Organic Chemistry
Nuclear Magnetic Resonance
as

Structure Elucidation

Spectroscopy and Analysis

The fundamental principle behind nuclear magnetic resonance,

:he external magnetic

terms of what is absorbed. The energy levels that are affeited are for the spin of a nucleus in the presence of an external magnetic field. Normally all of the nuclei have spins of the same energy. However, when an external magnetic field is applied, spins can either align with the field or align against the fie[, so multiple energy states are possible. In the case of 1H, theie or"u t*o energy levels: o line one aligned with the external magnetic field), and G (the one aligned against the external magnetic field.) The B energy level is defined as higher tlian the cx energy level. The energetics of the two levels depends on the strength of the erternal magnetic field and the magnetogyric ratio of a particular nucieus. This :neans that the energy gap between the two levels also clepends on the strength of

NM& is the same for other forms of spectroscopy. Energy, in the form of electromagnetic radiation in the radio frequency band, is added to the system and analyzed in

::equency of the EM radiation needed to flip the spin in"r"as"s proportionally.

field. As the external tnug.r"ii. field increarui th"

\ny nucleus with an odd number of protons (Z number) or an odd number of :-ucleons (A number) has a net spin. what is meant by "spin" is that as the :-ucleus precesses, it generates a weak magnetic field (as etections do). Just as a :rarged particle in linear motion generates a radial magnetic field, a charged :article in rotational motion generates a linear magnetic field. when the atomic :,ucleus has an odd number of protons (or nucleoni), the spins cannot pair up to :ancel one another out. The result is that the nucleus has a net spin. In the cises -t 1H, 13c, and 19F, it happens that there are only two energy ievels associated , ',1th the spins, so they can be analyzed without complicationl-A nucleus such as -)-i has spin, but there are more than two energy states, so its NMR spectrum is -.o complicated to analyze conveniently.
-:. the absence of any surrounding electrons, all identical nuclei exhibit the same -:in and therefore require the same energy for excitation in an external magnetic

:::ld. within a molecule, two identical nuclei may be in different eleclronic . r'ironments. As a result of the difference in their local magnetic fields, caused
::
-

:-'' the moving electrons, they do not require exactly the same amount of energy excite the nucleus to a higher-energy spin state. This can also be viewed is

- :eded to get excitation

:-achines can be designed to vary the frequency of the radiation or vary the .::ength of the magnetic external field. The NMR graph we observe typically :=:ords changes in the magnetic field strength along the x-axis, so we tnint or l,\lR in terms of varying external magnetic field strength.

cal magnetic fields altering the strength of the apptied external magnetic field (spin-flip) at a set frequency for the EM radiation. NMR

l:oton NMR (which uses the 1H nucleus) is the most common form of NMR and rkes advantage of the magnetic spin associated with the hydrogen nucleus. The l.:cAT test-writers focus on analyztng the graphs produced uy lumran. There is :,lrimal 13cxuR on the MCAT. rn"e scaie rot tuNvn is set from 0 to 10 parts : =r million (pp*) of the total magnetic field of the machine. Just as an inchls an ..;h because someone made it a standard unit of measurement, NMR is :easured in ppp of the external magnetic field, because that is the arbitrary .::.ndard. All lHNMR shift values are relative to a standard compounj, =:ramethylsilane ((H3c)4si). All twelve of the hydrogens on tetramethylsilane .:e equivalent, so they absorb at the same value. This value is arbitrarily ';signed to be 0 ppm, and all shift values are referenced against it. Rather than
.-

- Lnto other intricacies of NMR, we shall concentrate on how to read the graphs.

-,rpvright O by The Berkeley Review

125

Exclusive MCAT Preparation

Organic Chemistry

Structure Elucidation

Spectroscopy and Analysis

Symmetry and NMR Signals The best place to begin NMR for the MCAT is with molecular symmetry. Based on the symmetry of a molecule, you can determine the number of equivalent hydrogens that it contains. We will consider symmetry within different groups of molecules, starting with the four six-carbon esters shown in Figure 2-30'

&

tu

[M

l!..

ffi

o
oarr"cHrcH)cH1 "
h' c'de

MM

H,Ct ta'

il

HrC' ca"

il

&,r

Five unique hydrogens labeled a-e in a 3:2:2:2:3 ratio.

e" b' d Five unique hydrogens labeled a-e in a 3 : 1 : 3 : 2 : 3ratio.

oarrlcH?)cH2cH3 -

m!

ffim

o
H^Ct 'a" /lt

o
ocHrcH(cHj)?
b- c d--

mh

@!

Four unique hydrogens labeled

oc1arr.,;. ' b''' at Two unique hydrogens labeled

tH^Ct

lt

m
@

M m

dh

a-dina3:2:7:6ratio.
Figure 2-30

aandbinal,:3ratio.

dM

W
]b,fr

The first two esters, n-butyl acetate and sec-butyl acetate, each have six unique 1HNMR carbons, of which five hav-e hydrogens. gach exiribits five signals in its 13CNMR spectrum. Isobutyl acetate has five spectrum and six signals in its unique carbons, but only four contain hydrogens. This means that isobutyl acetate exhibits four signals in its I HNMR spectrum and five signals in its 13CNUR spectrum. Tertbutyl acetate has four unique carbons, but only two contain hydrogens. This means that tertbutyl^acetate exhibits only two signals in its 1HNMR spectrum and four signals in its IsCNMR spectrum. The presence o{ only two signals in an NMR spectrum makes it easy to identify tertbutyl acetate.
The comparison of symmetry between isomers is highly useful, particularly with benzene derivatives. Figure 2-31 shows three structural isomers of methyl anisole, each of which has different symmetry.

ffim

qm

H
f

H
c

H
c

H
e

q q
OCH"

l"'

OCH?

H
b

H
f

1""

CHs
b

.T,

H
e

H
c

CHs
d

H
d

H
d

Four unique hydrogens labeled a-d

Six unique hydrogens

Six unique hydrogens

Iabeled a-f

labeled a-f

ina3:2:2:3ratio.

ina3:1:3:1:1:1ratio.
Figure 2-31

ina3:3:1:1:1:1ratio.

Para-methylanisole has six unique carbons, of which only four contain hydrogens. This means that para-methylanisole has four types of hydrogens and Copyright O by The Berkeley Review

t26

The Berkeley Revieu

Organic Chemistry

Structure Elucidation

Spectroscopy and Analysis

methylanisole also has eight unique carbons, of wtrich six contain hydrogens. This means that ortho-methylanisole also has six types of hydrogens"and therefore six signals in its proton NMR. The structures in Figur e-2-Br"are para-, meta-, and ortho-methylanisole, respectively.
You may have noticed that many problems are often just variations on a single theme. In Figure 2-32, compounds with comparable i\MR readings are shoirn hydrogens. Figure 2-32 shows two sets of three isomers that can be distinguished irom one another using IHNMR by simply looking at the number of "signals. Butanol and pentanal each have five unique types of hydrogens. The hydrigens are in exactly the same ratio on both compounds, so thelr lHNMR spectra strong similaritie^s. Each has five signals in its 1HNMR spectrum, ilthough they "r,f,6it iave different 13cxMR spectra arie to a different nrr-L", of unique ciruor,r. rhe HNMR spectra of the two compounds can be distinguished from one .1 another by the shift values of the respective signals. Methylpiopyl ether and.2lentanone each have four unique types of hydrogens. rhe hydrogens are in exactly the same ratio on both compounds, so their 1HNMR spectra exhibit strong similarities. Each has four signals with the same relative area in their -HNMR spectra, but at different shift values. Diethyl ether and 3-pentanone each have mirror symmetry and thus have similar carbons and similar hydrogens due io this symmetry. There are two unique types of hydrogens in both diettyl ether :ld 3-pentanone. This means that diethyl ether and 3-pentanone have only two 1HNVn. 'ignals in their Butanol

therefore four signals in its proton NMR. Meta-methylanisole has eight unique carbons, of which six contain hydrogens. This means that meta-meiirylanlstle has six types of hydrogens and therefore six signals in its proton NMR. ortho-

side by side to demonstrate similarities in the distribution of lheir unique

H .r

./o--CH, . CH; e-CF:. -CHz


bd-

o
il

Pentanal

Fir-e unique hydrogens labeled a-e irr a 7 :2 :2 :2: 3 ratio.

T-t-.f {'!'-cu1cF:
Five unique hydrogens labeled a-e in a 1 : 2 : 2: 2 : 3 ratio. 2-Pentanone

Methylpropyl ether

H,C/ dr

o-a*t,-tF'b.

a". -;'j

il

Four unique hydrogens labeled

a-dtna3 2:2:3ratio.
Diethyl ether

CH. b' d' Four unique hydrogens labeled


a"

-C. H'rCt - CHr'

CH. t '\

a-dina3:2:2:3ratio.
3-Pentanone

F'c-cni o-..,; tF'


bb
-

fl

T"- c+{c-ctlr-cYu
b

rvo unique hydrogens labeled

aandbina3:2ntio.

Two unique hydrogens labeled

aandbina3:2rctto.
Figure 2-32

-.rpyright O by The Berkeley Review

127

Dxclusive MCAT Preparation

Organic Chemistry

Structure Elucidation

Spectroscopy and Analysis

The comparison of butanol to methylpropyl ether and diethyl ether is similar to the comparison of pentanal to 2-pentano-ne and 3-pentanone. For instance, the

presence of only two signals in the t HNMR spectrum makes it easy to distinguish 3-pentanone from 2-pentanone and pentanal in the same way it is easy to distinguish diethyl ether from methylpropyl ether and 1-butanol. You may see this theme repeated several times, so it is better to know basic trends rather than specific examples.
In the case of alcohols, such as butanol, the protic hydrogen can be distinguished from other signals by its broadness. Broadening results from hydrogen-bonding in solution. Hence, alcohols are easily distinguished from ethers by the presence of a broad. peak in their 1HNMR spectrum. In the case of NMR, the local environment of equivalent hydrogens undergoing hydrogen-bonding is not equal, so they appear at slightly different shift values. The degree of hydrogenbonding varies, so the effect is also varied, causing the signal to be a broadened. Hydrogen-bonding causes the broadening of peaks in all types of spectroscopy. Broad peaks are a dead give-away for protic hydrogens.

lrer,
-i

illl

f,*::

within a molecule. You will section. The key to predicting do this over and over throughout the spectroscopy an NMR pattern for a compound is to understand the symmetry of the molecule.
These examples were designed to look at symmetry

You must be able to identify unique hydrogens and then determine their respective features. This is where we shall start our analysis. The features we shall focus on primarily are the integration of the peak, the splitting pattem (shape) of the peak, and the shift value (measured i. pp*) of the peak. Be sure
that you understand the importance of each of these features and the factors that can produce changes in them.

lnteg:
-ra '^''
---^-! _.AL tL

!: L

: ack-r,
_5OU:

either
hvdro,

shou-li

Copyright

by The Berkeley Review

The Berkeley Revieu

Copr:

Organic Chemistry
1H Nuclear Magnetic Resonance

Structure Elucidation

Spectroscopy and Analysis

3-pentanone has two unique types of hvdrogens in a B:2 ratio, its spectrum has two signals with relative areas o{g,2. Figure 2-33 shows the signals from the 1Flrultn spectrum of 3-pentanone. Each p"ui i, explained in terms of its splitting, integral, and shift value. The unique hyhrogens aie labeled in the same fashion as they were in Figure 2-32.

P-":iy:: lHNMR

r)

tra\

CH. a'

zc\

ll

.zCHz
CH"

h'
Being next to a CH2 group yields a shift value between 0.9 and 1.5 ppm. Being next to a CH2 group yields a triplet.

Being next to a C=O group yields a shift value between 2.0 and 2.5 ppm. Being,next to a CH3 groupyields a quartet.

ppm

pp-

Figure 2-33
The zero reference is ignored for analytical purposes, because it is there just to set lhe scale correctly. The integral is not drawn on the spectrum in this example. In ntost cases, you will be provided with the relative areas of the peaks, or you will :e given a summation line to evaluate the relative areas. Either way, you must :e able to apply the relative areas of the peaks to the quantity of hydrogens that each peak represents. This is the start of NMR analysis.

Spectrum Analysis

iVe shall start off with how to anaryze the three basic components of the graph: :'r.tegral (determined by the number of hydrogens making up a signal), splitting :attern (derived from the coupling between hydrogen neighbors), and shift aalue determined by the local magnetic field caused by either lone pair electrons in rrotion or the electronic density associated with electronegative atoms). Each piece is equally important. At times, one piece of information may be a little more enlightening than the rest, but on the whole, every bit of data counts. Integral

for a signal can be integrated, meaning that the area under the curve :an be summed up, and set directly proportional to the number of hydrogens :hat the signal represents. For instance, a CH3 group has a signal with a relitive area of 3 compared to a CH2 group with a signal of relative area 2, working rackwards from the integration to the structure, it is possible to deduce the Sroup from the integration. For instance, a relative area of 5 can be attributed to either 2 equivalent CH3 groups or 3 equivalent CH2 groups. Further inspection should reveal which of the two scenarios is responsible for the six equivalent
The peaks

hydrogens.
11

Copyright O by The Berkeley Review

129

Exclusive MCAT Preparation

Organic Chemistry
Splitting Pattern

Structure Elucidation

Spectroscopy and Analysis

)r_

The splitting pattern, also referred to as coupling, corresponds to the number of hydrogens on a neighboring atom. Like electrons, nuclear particles have spin that can be classified as either up or down. The magnetization caused by the nuclear spin of hydrogen can be felt by the hydrogens on a neighboring atom. Because the spin can be either of two ways, the magnetic field may be additive or subtractive. The random distribution of spins is used to determine the number of hydrogen neighbors a group has. For instance, the CH3 group in 3-pentanone (labeled with an a in Figure 2-33) is next to a C}{2 (labeled with a b in Figure 233). The two hydrogens of the CH2 have one of four possible spin combinations:

t'
rl:

'1:

:::,L

:lll,::

iir:
yril
rit,r

It"

up/up, up/down, down,/up, or down,/down. Every CH3 group next to an up / up CH2 group has a slightly higher signal, while every CH3 group next to a down/down CH2 group has a slightly lower signal. Every CH3 group next to an
up/down or down/up CHZ group has a normal signal, because the opposite spins cancel each other. The result is that one out of every four times, the CH3
signal is slightly higher, two out of every four times the signal is unaffected, and one out of every four times the signal is slightly lower. This is why the CH3 signal in 3-pentanone occurs as a triplet (in a 1 : 2 : 1 ratio). Likewise, the CH2 group of 3-pentanone is next to a CH3. The three hydrogens of the CH3 have eight possible spin combinations: up / up / up, down/down/ up, dow n / up / down, up / down/ down, up / up / down, up,/ down / up, down / up / ap, or down/down/down. If all three spins are up (up/up/ up), then the net spin is +3/2. If only two spins are up, then the net spin is +7/2. There are three

liI

p
'llri

-*

It
tl rt

combinations where two spins are up and one is down (up/up/down, up / down/ up, down/up / up), so this is three times as frequent as the all spin up combination. The same thing can be done for the one spin up combinations (down/down/up,down/up/down, upldown/down) and the all spin down combination. The result is that a quartet is found to be in a 1 : 3 : 3 : 1 ratio.
This is why the CH2 signal in 3-pentanone occurs as a quartet (1 ; 3 : 3 : 1). There are eight outcomes, but three of them share one value and three of them share another value, so we see only four different outcomes. Working from a spectrum to a structure, it is possible to say that a 1 : 3 : 3 : 1 quartet is the result of the hydrogens on a carbon being next to three equivalent hydrogens, often due to the presence of a CH3 group as the neighbor. To determine the ratio of the peaks within an overall signal (like the 1 : 3 : 3 : 1 value for the quartet), you can use Pascal's triangle for binomial expansion to get the relative area of each peak

li'

fr,

:-.
j'.

i'

within the signal. Table 2-3 shows Pascal's triangle along with a brief explanation of what the relative numbers are expressing about the shape of the peak and the abundance of the signal. As the relative amount gets smaller, it is harder to distinguish a peak from noise in the baseline signal.
Neighbors
Signal Shape
Sinslet
Pascal's Triangle
1

Ratio of Peaks in Signal


1

0Hs

peak
:r:

1H
2Hs 3Hs 4Hs 5Hs 6Hs

Doublet

11

2peaks:L:1ratio

Triplet
Quartet Quintet
Sextet Septet

t27 1331 74641


1 5 1010 5 1 6 152075 6
1 1

:2: 1 ratio 4peaks:1:3:3:lratio


3 peaks: 1 5 peaks:
1, : 4: 6: 4 : 1 ratio 6peaks: 1 : 5 : 10 : 10 : 5 : 1 ratio

-:
.-

Tpeaks: 1 : 6 : 15 : 20 : 15 : 6 : 1 ratio

:.-

Table 2-3

Copyright

by The Berkeley Review

The Berkeley Review

ir

Organic Chemistry
Shift Value

Structure Elucidation

Spectroscopy and Analysis

The shift value is a diagnostic tool for assessing the locai electronic environment. it is measured in parts per million (ppm) relative to the magnetic field necessary

to detect a standard compound, tetramethylsilane ((H3C)aSi). It is used in wiih known shift values to determine what functional groups are neighboring. In the 3-pentanone example, the CH2 group adjacent to the carbonyl group feels the magnetic field of the electrons on a neighboring oxygen and thus requires a stronger external magnetic field to energize the spin levels than a CH2 group that is next to an alkyl chain. A larger shift value in the spectrum is thus observed than is typically observed for CH2 groups in hvdrocarbons. This is referred to as being shifted downfield, which indicates righer ppm values for the shift. To verify the presence of a C=O (carbonyl group), consult Table2-4 to find that a value of 2.1 to 2.5 ppm is expected for a r-rvdrogen on a carbon adjacent (alphn) to the carbonyl. Table 2-4 lists many .orrunon lUNVR shift values used for analyztng spectra. The bold hydrogen in each compound in Table 2-4 is the one of interest (to which the shift vaue
conjunction ;orresponds) and the shift value is measured in units of ppm. Hydrogen Atom
RCHc RCH2R (acyclic) RCH2R (cyclic) R3CH R2C=C(R')CH3 RCOCH3 (ketone)
D

(ppm)

Hydrogen Atom
R2C=CH2 RCH=CRz RNH2

d (ppm)
5.0 - 5.8 5.2 - 6.4

0.8 - 1.0
1.3 - 1.5 1.5

1.8

1-3
2.0 - 3.2 3.5 - 5.0

r.5 - 2.0
1.8 - 2.2

RNHCH3

ArNH2
RCONHR (amide) ROH (alcohol)

_)

5-9
1 - 5 (broad)

,\rCH3
RC=CH
R.OCH3 (ether)

2.2 - 2.5 2.5 - 2.6 3.5 - 4.0 3.0 - 3.8 3.5 - 4.0

AIOH (phenol)

4 - 7 (broad) 7.0 - 7.4 9.0 - 9.8


10

ArH

(benzene)

RCH2X (X = Cl, Br,I) RCO2CHa (ester)

RCOH (aidehyde) RCO2H (acid)

-72

Table 2-4 Erample 2.15 lentanal can be distinguished from 2-pentanone by what lgNIrrtR feature? \. A 3H triplet at 1.5 ppm B. A 2H multiplet at 1.8 ppm C. A 2H triplet at 2.3 ppm

D. A 1H triplet

at9.7 ppm

Solution lentanal has an aidehyde hydrogen, while 2-pentanone does not. The shift value :-r an aldehyde hydrogen is found between 9.0 and 9.8 ppm. The peak shape is a :rp1et, because there are two equivalent alkyl hydrogens on carbon 2. The two - .'-drogen nerghbors couple with the aldehyde hydrogen to split it into a triplet in :e lHNMR spectrum. This makes choice D the best choice. 2-pentanone could :e distinguished by the singlet of relative integration 3 caused by the isolated :'.ethyl group adjacent to the carbonyl. Because the carbonyl carbon has no :.','drogens attached, the methyl group of carbon one is not coupled to any other

-opyright O by The Berkeley Review

l5r

Exclusive MCAT Preparation

Organic Chemistry
o
lt
(-

Structure Elucidation

Spectroscopy and Analysis

I
l f

hydrogens, which makes it appear as a singlet in the 1HNMR spectrum. The two structures are drawn below, along with their proton NMR features.

o
il
b c d
e

(-

& A
b c
cl

.-/*\

CH2CH2CH2CH3 1HNMR.

^ H:C

,./-\

CH2CHzCH3

r(

Il

Aldehyde hydrogens show signals around 9.7 ppmin the

Because there are no hydrogens on the neighboring carbon, the methyl

f,

group is a singlet in the

1HNMR.

lfi

ffin

Example 2.16
\A/hat signals are present in the

GT gM

lIlUVn

spectrum of chloroethane?

ffim

A. A downfield doublet and an upfield triplet B. A downfield triplet and an upfield doublet C. A downfield triplet and an upfield quartet D. A downfield quartet and an upfield triplet
Solution Chloroethane has two unique types of hydrogens. This results in two signals in its 1HNMR spectrum. The two hydrogens on carbon 1 are split into a quartet bv the three hydrogens on carbon 2. Equally, the three hydrogens on carbon 2 are split into a triplet by the two hydrogens on carbon 1. The lines split according to the neighboring hydrogens and project down to the spectra. The quartet is farther downfield than an ordinary alkyl group due to the electron density on the chlorine atom. This means that the triplet is upfield, making choice D the best
answer. There are two types of H, so there are two

lffi

ft

h
rf

lL

n n

l[ ffi
tfrr

lfr

1HNMR

signals.

ag

CIH

ffir

\^

nttuTt-t\
H

-.Lu

n: l[r fi.

fl,
are coupied to the three

n"

F{t

on the carbon 2, resulting in a quartetdownfield.

iii ,II iiI

on carbon 1, resulting in

M
ilhs

&

triplet upfield.
i

mM

l]lt tttt

tu
ffi
fiF

Actual srectrum
1

Copyright

by The Berkeley Review

132

The Berkeley Review

Organic Chemistry

Structure Elucidation

Spectroscopy and Analysis

Erample 2.17 lentanol can best be distinguished from ethyl propyl ether by which of the :,rllowing features in the lHNMR spectrum?

\. A 3H triplet at 1.2 ppm B. A 2H triplet at 3.5 ppm C. A broad 1H peak between 1.0 ppm. and 5.0 ppm D. The total number of signals in the ether is substantially less
Solution in alcohol is distinguishable from an ether by its broad peak between 1.0 ppm. :rd 5.0 ppm, so pick C for best results. The peak is broad due to the hydrogen:onding within the alcohol. The broadening of the peak makes it difficult to =-,-aluate the integration of the alcohol hydrogen peak. Both structures contain a :;{ iriplet at1.2ppm, so choice A is eliminated. Both structures contain a 2H :rplet at 3.5 ppm, so choice B is eliminated. Choice D is eliminated, because --:.ere is a small difference in the number of peaks between the two compor-rnds.
E-rample 2.1.8

',',hich

of the following is a common feature in the 1HNMR spectra of all methyl

retones?

B.
C. D,

\,

A A A A

triplet at 1.5 ppm (3H)


quartet at 2.3 ppm (2H) doublet at 2.3 ppm (3H) singlet at2.7 ppm (3H)

Soiution

methyl ketone has an isolated methyl group neighboring the carbonyl carbon '.;hich has no hydrogens). Having no neighboring hydrogens makes the peak a .nglet. The protons on a carbon alpha to a carbonyl are found between 2.0 and I 5 ppm. The signal has a relative integration of 3 hydrogens. The only answer :hat fits this is choice D, a 3H singlet at 2.1 ppm. The best choice is choice D.
Example 2.19 -- monosubstituted benzene has which of the following in its IUXVIR spectrum?

B. A peak at 5.3 ppm (5H) C, A peak at 7.2 ppm (5H) D. A peak at 8.1 ppm (5H)
Solution \lonosubstituted benzenes have a single peak around 7.0 ppm. The aromatic l'drogens appear as one singlet, despite the fact that they are not all equivalent :r'symmetry. The key to this question is not the integration or the peak shape, :ut the shift value. Choice D is just a little too high, so it is eliminated along with :hoices A and B, which are far too low" To do this problem quickly, you should :e familiar with the common peaks. Choice C is the best answer.

\.

A peak at 1.2 ppm (5H)

Be aware

of certain peaks and features that occur over and over. For instance, ,vhenever you see a triplet and quartet in a 3: 2ratio, you should conclude that there is an isolated ethyl group (H3CCH2-) in the molecuie somewhere.
Copyright
@

by The Berkeley Review

r33

Exclusive MCAT Preparation

Organic Chemistry

Structure Elucidation

Spectroscopy and Analysis


!

tr\lhenever you see a doublet and a septet

in a 6 : 1 ratio, you should conclude that there is an isolated isopropyl group ((H3C)2CH-) somewhere in the

molecule. Rather than looking at molecules to determine the spectra (going from structure to spectrum), it is important to work problems from the spectrum to the structure. By recognizing the combination of peaks, you will save time in determining the unknown structure. This is very conunon in NMR spectroscopy.

ft

We will use symmetric structures at first and then move on to more difficult examples. The MCAT has traditionally asked simple questions about this topic, but it's better to be safe than sorry, so we will present examples that are harder than the questions they have given on previous exams. In the following few questions are sample spectra from which you must determine the corresponding structures. It helps to solve for the units of unsaturation first. Once these are known, deduce possible functional groups that fit both the heteroatoms in the formula and the calculated units of unsaturation. For instancet zero units of unsaturation and one oxygen can be an aliphatic ether or an aliphatic alcohol. Take advantage of the multiple-choice format by eliminating wrong answers as you come across them. In the case of a compound with zero units of unsaturation and one oxygen, an answer choice of a ketone is eliminated immediately. Any structures with rings or n-bonds should be eliminated. This ability to eliminate wrong answers can be very useful in the multiple-choice format. To gain both insight and experience, try the following spectral problems:
Example 2.20 \A/hat is the name of the compound that has the following 1I-nuraR spectrum, and whose formula is C7H14O?

I (

Tffi

ffi

&

&

fr

il"

c.

l.

D.

&

tu ffi :-

fr

mm

ffiF

A. 2,4-dimethyl-3-pentanone B. 2,2,4,4-tettamethyl-3-pentanone C. 1,1,3,3-tetramethyl-2-propanone


D.
2,2- dimethyl-3-pentarone

Solution
The septet and doublet in a 1 : 5 ratio are a dead give-away for an isopropd group. Choice D is eliminated, because it does not have an isopropyl group" Choice B is eliminated, because it contains too many carbons (nine instead od seven). Choice C is eliminated, because the structure is misnamed. The bes,t answer and only remaining choice is A.

Copyright O by The Berkeley Review

134

The Berkeley Revieu

Organic Chemistry
Example 2.21
.The

Structure Elucidation

Spectroscopy and Analysis

s-hape-of the signal at 2.3 ppm best described as:

in the luNvtR spectrum in Example 2.20 is

A. a quartet. B. a sextet. C. a septet.


D.
an octet.

Solution lve counting seven apexes within the signal at2.3 ppm, and seven apexes (peaks) is referred to as a septet. check to see whether the ratio is 1 : 6 :rs :20: 15 : 6 : 1 to be sure. This seems reasonable, so the best answer is choice C. If you already know the structure, then you can see that the septet results from six equivalent hvdrogen neighbors on the methyl groups neighboring the alpha carbon.
Example 2.22 The ratio of the areas under the peaks

within

a quartet is:

-{.7:2:2:1.. B. 1 :3:3: 1. C. 2:5:5:2. D. 1:4:4:'J..


Solution
Bv using Pascal's triangle, you can easily determine the ratio. It is a good idea to know the ratios of the more common peaks such as a doublet, a triplet, and in --}'ris case a quartet. A quartethas a ratio of 1 : 3 : 3 : 1. Choice B is correct.

Example 2.23
'r\'hat is the IUPAC name of the compound represented by the following spectrum, whose molecular formula is C4H3O?

lHNltR

c4H8o

B. Butanone C. Ethyl ethanoate D. Methyl propanoate


Solution
Because the formula has only one oxygen, the two esters (choices C and D) are Jre compound is an aldehyde or ketone.

-{.. Butanal

immediately eliminated. The question is now reduced to determining whether An aldehyde would have a peak in the

Copyright O by The Berkeley Review

135

Exclusive MCAT Preparation

Organic Chemistry

Structure Elucidation

Spectroscopy and Analysis

I
E"r

proton NMR between 9.0 and 9.8 ppm. There is no peak in that range, so choice A is eliminated. Only choice B remains. To be certain, butanone (CH3COCHZCHS) has three types of hydrogens and thus three peaks in its proton NMR spectrum. The peaks are a singlet (3H), a quartet (2H), and a triplet (3H). This is what the spectrum shows, so choice B is correct.
Example 2.24 \iVhat is the IUPAC name of the compound represented by the following lHirll,tR spectrum, whose molecular formula is C4H3O2?

-.
:n
"4t ffi

C.
lL-il

-.

50

][jr:

ta
t

l,/{1/,1

:-:

ppm

ppm

A. B. C. D.

Butanal Butanone Ethyl ethanoate Methyl propanoate

Solution
This question is similar to the previous question, except the ketone and aldehyde are eliminated, because there are two oxygen atoms in the molecular formula. Choices C and D show identical peak shapes and integrals in their 1HNMR spectra" The distinguishing feature is the shift value of each signal. The ethyl ethanoate (structure shown on the left below) exhibits a quartet near 4.0 ppm, making choice C correct. The structure of methyl propanoate is shown on the right below.
"0,

ffi

Srntr
-_E fri j

OO

,.4.,/tT'---,, o
3H
at

T" l'T' 1'Tt


Singlet
2.1

T,t-.-A o o/'\,
I

trr0g:

*Ltxn:

I.EI

ppm

2H Quartet 3H at - 3.8 ppm at -

Triplet 1.0 ppm

3H

Triplet
1.0

2H
at

at

ppm

Quartet 3H Singlet 2.3 ppm at - 3.5 ppm

:I: -*
l_:

a.

Copyright O by The Berkeley Review

The Berkeley Review

! =

JIl

Organic Chemistry

Structure Elucidation

Spectroscopy and Analysis

Example 2.25 Hydrogens on a carbon adjacent to two equivalent CH2 groups show which type of signal in a IHNMR spectrum?

A. A1:3:3:lquartet B. A1:4:4:lquartet C. A1:3:5:3:lquintet D. A1:4:6:4:Lquintet


Solution Having two equivalent CH2 groups adjacent to the site of interest results in a total of four equivalent hydrogen neighbors. Four equivalent hydrogens split a signal into a total of five (4 + 1,) peaks. This makes the signal a quintet, which according to Pascal's triangle (or binomial expansion of any sort) has a ratio of L : 1:6:4: 1. The best answer is choice D.
Example 2.26

\Vhat is the common name of the compound represented by the following HtWtR spectrum, whose molecular formula is C6H16O2?

-A.. Para-ethoxy phenol (H3CH2COC6HaOH)

B. Ortho-ethoxy phenol (H3CH2COC6HaOH) C. Para-methoxy anisole (H3COC6HaOCH3) D. Ortho-methoxy anisole (H3COC6HaOCH3)


The symmetry in the 1HNVR spectrum is associated

Solution

with a structure that is also >r'mmetric. The only way to get two types of hydrogens on a disubstituted lenzene is to have two equal substituents on benzene para to one another. This eliminates choices B and D. Based on the formula, this molecule has two rLethoxy groups para to one another on the benzene. All of the benzene hvdrogens are equivalent, which explarns why only a singlet is observed. The best choice is thus answer C. Choice A would exhibit more than two peaks, so it
Ls

eliminated.

Copyright

by The Berkeley Review

r37

Exclusive MCAT Preparation

Organic Chemistry

Structure Elucidation

Spectroscopy and Analysis

Recognizing Special Structural Features Recognizing special structural features requires knowing some general shift values (6-values) from memory. You should know that a carboxylic acid hydrogen falls in the 6 = 1.0 - 12 ppm range and that the signal is broad. An aldehyde hydrogen falls in the 6 = 9 - 10 ppm range, aromatic hydrogens fal in the 6 = 7 - 8 ppm range, vinylic hydrogens fall in the 5 = 5 - 6 pp* range, alkoxy hydrogens fall in the 6 = 3.5 - 4pp- range, and alphahydrogens fallin the E=2pp* range. Figure 2-34 shows a molecular structure and its corresponding ?.5 lHNMR spectrum that includes many of these key peaks.

ftm

mm

ffio

.f'

du
fu@

al
,fillfl

ffi

o
oHa

3H

3H

3H

m.

o
b 11H
A

2H
c

2H
d

1H

A1H

12

ppm

10

ppm

ppm

6 PPm

4 ppm

PP*

ppm

Figure 2-34
Be certain that you can match the peaks in the spectrum to the hydrogens in the

structure drawn above it in Figure 2-34. This can be done on the exam using a chart of values if one is given, but it is not a bad idea to know the values from memory.

Example2.27
\A/hat is the IUPAC name of an unknown compound with the molecular formula C3H6O, an IR absorption at 1722 cm-l, and three 1HNVR peak,; one at 9.7 ppm (1H), one at 2.3 ppm (2H), and one at 1.4 ppm (3H)?

A. Propanoic B. Propanal
C. D.
Propanone

acid

Methyloxyrane

Solution
There is an excess of information in this question beyond what is needed to aswer it. The one and only piece of information you need is the peak at9.7 ppm, which makes the compound an aldehyde. Pick choice B and move on quickly.

Copyright

by The Berkeley Review

The Berkeley Revier

Organic Chemistry
Distinguishing Disubstituted Benzenes

Structure Elucidation

Spectroscopy and Analysis

Integrals teil us the number of equivalent hydrogens in a signal and are often employed to determine the position of substituents on disubstituted benzene rings. Structures that are highly symmetrical have more equivalent hydrogens than asymmetrical structures. A 1,4-disubstituted benzene ring (referred to as para") shows the fewest peaks in the aromatic region of the spectrum of all disubstituted benzenes, due to its mirror symmetry. Both a 1,2-disubstituted and a 1,3-disubstituted benzene ring (referred to as "ortho" and "meta" respectively) have four unique hydrogens in the aromatic region of the spectrum. Figure 2-35 shows two sets of disubstituted benzenes, one set of three with identical groups and another set of tlrree with two different groups on the benzene.
Case 1: The two substituents on benzene are equal:

Ha

X
Ha U rlb

Hb

;f.
X Hb
3 different Hs in a 1 :2:lratio

Ha

Ha

Ha

Ha

inal:lratio

different Hs

All

Hs are

equivalent

Case

2: The two substituents on benzene are not equal:


Hb
Ha

Hd

Hc

{ different Hs in

a1:1:1:lratio

4 different Hs

in

a1:1:1:lratio
Figure 2-35

2 different Hs

inal:lratio

:rara substitution is the easiest arrangement to distinguish of the three possible -.:uctural isomers, because it has a doublet of doublets. The dissimilar heights of -:s peaks can be attributed to a mathematical phenomenon whereby peaks, as --:.ev near one another, begin to coalesce. Figure 2-36 shows the aromatic region

: a l HNMR spectrum if u puru substituted benzene ring, where the two .:bstituents are nonequivalent. Para coupling is a highly recognizable feature.
Enlargement of the aromatic region shows that the splitting is a doublet of doublets, corresponding to a para-substituted benzene

ring.

X
Ha

Ha

Hb

7.8ppm 7.6ppm 7.4ppm 7.2ppm 7.0ppm

Figure 2-36

-;.pyright

by The Berkeley Review

Exclusive MCAT Preparation

Organic Chemistry
Example 2.28

Structure Elucidation

Spectroscopy and Analysis

o
rL"AlI

TE

what is the common name of the compound that has the formula C6Hgo, an IR absorption at7722 cm-], and three notable lHNMR peaks at9.7 ppm (1H, s), 2.3 ppm (4H. dd), and 2.2ppm (3H, s)?

trs':
&,f,1r

rli J
&e"r

A. B. C. D.

Ortho-methylbenzoic acid
Para-hydroxyacetophenone

Ortho-methylbenzaldehyde
Para-methylbenzaldehyde

J,ffi

*-"

Mfl ,ILfu.

Solution The compound has only one oxygen, so neither a carboxylic acid (methylbenzoic acid) nor a hydroxy ketone (hydroxy acetophenone) is possible. Choices A and B are eliminated. we know that the compound must be an aldehyde from the choices that remain, so the lriNvtR peak at 9.7 ppm and the lR absorption at 1722 crn-ldo not help our efforts to identify the compound. The trqrulan signal at 7.3 ppm is a doublet of doublet (dd), which indicates para-substitution. This makes choice D the best answer.
Deuterated Solvents for 1HNMR Because the solvent is in substantially higher concentration than the solute, it is imperative that the solvent not have any hydrogens. If the solvent has 1H nuclei. then it would produce the largest signal in the spectrum, eliminating integration and causing the other peaks to disappear into the baseline. To avoid this problem, solvents are chosen that have deuterium (2H) instead of the standard isotope of hydrogen (1u;. one potential problem occurs when protic compounds are dissolved into deuterated protic solvents, such as D2O. protic hydrogens can undergo exchange with the protons of the solvent, if the solvent is protic. Although the dissociation constant (Ku) may be small for compounds such as alcohols, over enough time all of the hydrogens can be released and then are able to reform their bonds. rf D2o is present in the solution, then deuterium will gradually replace protic hydrogens capable of undergoing exchange. This causes the signal for the protic hydrogen to disappear gradually.

*."'
[i

*-

:mb{:.

tnlm

1me

$fft

Example 2.29

which of the following compounds does Nor lose a luNIvtR signal after D2o
has been added to a solution containing it?

A. B. C. D.

Carboxylic acid Cyclic ether Primary amine


Secondary alcohol

Solution

If a compound contains a protic hydrogen, then it loses a peak from its 1HNMR spectrum when D2o is added to the solution. Primary and secondary amines have a hydrogen bonded to nitrogen, so thev are protic. This eliminates choice C. All alcohols have a hydrogen bonded ,o s11:gen, so all alcohols are protic. This eliminates choice D. A cirboxylic acid hasl dissociable proton, so it readily exchanges with deuterated water. choice A is eliminated. An ether, whether cyclic or not, has all of its hycirogens bonded to carbon, so it is aprotic. When
D2O is added to an ether, no exchange transpires. The best answer is choice B.

,-

i,:,:

Copyright

by The Berkeley Review

The Berkeley Review

-:r:

Organic Chemistry

Structure Elucidation

Spectroscopy and Analysis

13CNMR Spectroscopy Carbon-13 NMR focuses on the carbons of a compound rather than the hydrogens. The NMR cannot detect carbon-12, because it has no nuclear spin. However, carbon-13 has an odd number of nucleons (particles in the nucleus), so ii has a nuclear spin. For that reason, NMR can be used to detect carbon-13. Because carbon-13 constitutes only about 1"/' of all carbon atoms, a 13CNMR requires maly more scans to obtain a spectrum than 1HNMR. This is why the 'raseline in 13CNMR spectra is noisy (scattered and messy). l3CNltR is used to rdentify the number of unique carbons in a compound. A typical application of t'hrs technique is to distinguish the substitution of disubstituted benzenes. Figure l-37 shows the proton-decoupled (ali singlet spectrum) C-13 NMR spectra for -,2-dimethylbenzene, 1,3-dimethylbenzene, and 1,4-dimethyibenzene. What is :reant by "proton-decoupled" is that the compound is irradiated constantly similar in concept to Dolby noise^reduction) with the coupling energy, so that iere is no coupling between the IJC and IH atoms. This makes all of the peaks =inglets, as drawn.

80

#
CH"
BO

ppm

140

120

100

60

40

20

Figwe 2-37 llhe shift values for 13CNMR are simple to recall. They come in blocks of 50 rpm. An sp3-hybridized carbon next to a carbon has a shift value between 0 and 50 ppm. An sp3-hybridized carbon next to an electronegative atom has a shift
Copyright
@

by The Berkeiey Review

t4t

Exclusive MCAT Preparation

Organic Chemistry

Structure Elucidation

Spectroscopy and Analysis

relaxation times, but instead will focus only on the application of information extracted from the spectra.

value between 50 and 100 ppm (usually less than 75 ppm). An sp2-hybidized, carbon next to a carbon has a shift value between 100 and 150 ppm. A carbonyl carbon with sp2-hybridization has a shift value between 180 u"a zso ppm. The shift value increases as the substitution increases, meaning that 3' > 2" > f in shift value. Quaternary carbons do not show up on the 13cruvR very well, due to their long relaxation times. \44-rat is meant by a "long relaxation time" is that the excited energy state of carbon takes longer than a few seconds to dissipate the energy into solution and relax back to a lower energy level. We will not focus on

tr:

peaks are from sp2-hyb.idized carbons and one is from an sp3-hybridized carbon. For the meta-substituted and para-substituted compounds, a similar relationship between spectrum and unique carbons in the structure is observed.

In the three spectra shown in Figure 2-37, you'lrnote that the number of peaks in each spectrum corresponds to the number of unique carbons in the compound the spectrum represents. For the ortho compound, there are four carbons in a 1 : 1:1:1ratio. Thespectrumshows fourpeaks ina 1:1:1:1ratio. Three of the

using spectroscopy data, you should be able to solve any problem following a systematic procedure. It is best to use l3ctlVR to get-the symmetry oI a structure and to identify selected functional groups (in a manner similar to IR spectroscopy). Don't make much more out of i3cNun spectroscopy than this.
Be sure to use your degrees of unsaturation in structure elucidation problems!

Stn
1.

Sper

1.

Copyright

by The Berkeley Review

t42

The Berkeley Review

copl

Organic Chemistry
lsomerism

Structure Elucidation

Section Summary

Key Points for Structure Elucidation (Section 2)

1.

Isomers (compounds with the same type and number of atoms but different spatial arrangement due to bonding, connectivity, or molecular contortion)

a) Structural isomers (isomers with different connectivitv ii.


different bonding) i. Have different IUPAC names

because of

Can be ciassified as skeletal isomers, positional isomers, or functional isomers

iii. Their number b)


arrangement)

lengths and connectivity

can be determined by evaluating possible chain

stereoisomers (compounds with the same bonding, but different spatial

i.

structure (leading to eclipsed and staggered conformations, with groups gauche and anti to one another) iv. Maximum number of possible optical isomers is 2n, where n is the number of chirai centers in the compound c) Nlewman projections (front view of moiecule) d) Cyclic molecules i. Three- and four-membered rings have ring strain that makes them highly reactive ii. Five- and six-membered rings are stable, with six being the more stable of the two iii. Cyclohexane (and six-membered rings in general) assume chair conformation, with groups equatoriai (more stable position) and axial
Structural Insights

ii. iii. Conformational isomers are formed by rotating or contorting

Can be classified as configurational isomers (geometrical and optical) or conformational isomers Have same IUPAC root, but a different prefix
a

1.

Structural symmetry a) Plane symmetry (mirror plane in molecule splits molecule into equal
halves) b)
c)

Point symmetry (molecule has an inversion point at its center of mass) Units of unsaturation i. Determined from excess bonding electrons divided by 2

ii. Unirs of unsaturation - 2(#C) + (#N) - (#H)- (#X) + 2 iii. Describes the number of n-bond., ur-ra .i-rg, in a molecule
Spectroscopy

1.

IR spectroscopy (used for vibrational excitation) a) Ranges from 1000 cm-1 to 4000 cm-1 (about 3 kcai/mole to 10 kcal/mole) b) Correlates bond-stretching and bond-bending to absorbance c) Used to identify functional groups

d)

Key peaks: C=O around 1700 cm-1, O-H around 3400 cm-1 (broad), and C-H around 3000+ cm-1 (varies with hybridization-- ,p > ,p2 > ,p3j
@

Copyright

by The Berkeley Review

t43

Exclusive MCAT Preparation

Organic Chemistry
e) 2.

Structure Dlucidation

Spectroscopy and Analysis

spectrophotometer uses salt plates to hold sample, because salt plates have ionic bonds and therefore do not interfere rn'ith the simpte
molecule's absorbances

Ultraviolet-visible (uv-vis) spectroscopy (used for electronic excitation) a) Ranges from 200 nm to 800 nm, increasing in wavelength as conjugation
increases b)
c)

Typicallv used for analyzing compounds with n-bonds, especially


conjugated systems Peak intensity and wavelength increase as the amount of conjugation
increases

3.

NMR spectroscopy (the basics of I HNMR analysis) a) IUXVIR Integration (Quantitative analysis using relative area under the
curves) i. Area under the curve for each signal is proportional to the number of hydrogens responsible for the signal ii. Connectivity can often be deduced from the integration ratio

b) lUXUR peak shape (coupling and J-values) i. The number of peaks within a signal equals the number ii. iii.

of

neighboring hydrogens plus 1 Hydrogens coupled to one another have the same j-values The ratio of the area of the peaks within a signai can be determined using Pascal's triangle

c) lUNVtn shift value (eiectron-rich

environments affect shift values by exerting a magnetic field) i. Common signals include 9-10 ppm for an aldehyde and around 7 ppm for hydrogens on benzene ii. "Upfield" refers to shifts at lower ppm values iii. All shifts are referenced against si(CH3)a, which is assigned a value

d)

e)

of 0 ppm 1HrultR special features (effects of deuterium and structural symmetry) i. Exchanging of deuterium for protons (peak disappearance) ii. Para substitution pattern (symmetric benzenes have unique spectra) iii. Solvent choice (solvent must be invisible) 13CXUR and symmetry i. The number of signals in 13CNVR spectrum is equal to the number of unique carbons in the molecule ii. Alkene carbons have signals between 100 ppm and 150 ppm, while carbonyl carbons have signals between 180 ppm and 230 ppm iii. 13C is a rare isotope, so 13CNMR spectra require more scans and have more noise in their baseline than IUXVn spectra

Copyright

by The Berkeley Review

r44

The Berkeley Review

Stnrcture Elucidation
Passages
14 Passages
l OO Questions

Suggested schedule: I: After reading this section and attending lecture: Passages I, V, VI, VIII, & XI Qrade passages immediately after completion and log your mistakes. II: Following Task I: Passages III, IV XII, & Xlll (27 questions in 55 minutes) Time yourself accurately, grade your answers, and review mistakes. III: Review: Passages II, VII, IX, X, XlV, & Questions 98 - lOO Focus on reviewing the concepts. Do not worry about timing.

REru{ffiNY
l)n.n.v.r.-E.w'
Speci altztng

in MCAT Preparation

rsftfficffirrui1ii1iruffiilil#tflff

lil

{tndh
mm@{

@m)

I. Chair Conformation II. Rotational Energy Diagrams III. Deuterated Cyclohexane IV. Infrared Spectroscopy V. Unknown Alkene Determination VI. Structure Elucidation VII. NMR and IR Spectroscopy VIII. NMR Data Table IX. Alkene-Coupling Experiment X. Unknown Compound Identification XI. Carbon-l5 NMR XII. Distinguishing Isomers Using lnnmR XIII. Structure Elucidation Using lHnMn and IsCNMR XIV. Proton NMR of an Unknown
Questions not Based on a Descriptive Passage

(r -7)
(B

ry
mc

r,Mm

ttAmrl

14)

lffi,Gdli'r

lhm

(r5 - 22)
(23 - 2e) (50 - 56) (37 - 43) (44 - so)
dffi
,@mo

,t5m

fr

qn*

(5r - 57)
(58 - 64) (65 - 71) (72 - 78) (7e - 84)
(B5 , eO)

(er - e7)
(eB

- l oo)

Structure Elucidation Scoring Scale


Raw Score
MCAT Score

84 - 100
66 47
B5

15-15

lo-12
7 -9

65

34-46

4-6

t-33

t-3

Passage

(Question 1 - 7)

carbon-deuterium bond is shorter than a carbon-

Cyclohexane is not a planar molecule, but in its mosl


stable conformation, fbur of the six carbons that make up the

ring are coplanar. Studies using I HNMR and X-ray :rystallography demonstrate that the most stable
:onformation of the molecule has carbon-carbon-carbon bond -rngles of approximately 107.5' and 1l l' in the ring and that .nere are two types of hydrogens present, axiaL and equatorial. The axial hydrogens are bonded directly above and directly :elow the ring carbons. The equatorial hydrogens lie away :rom the cyclohexane ring.

l-rydrogen bond. Using this idea, how many deuterium atoms assume axial orientation in the most stable conformation of the following molecule?

A low-temperature lHNVR stucly was conducted

to

A. 0 B.
1

::termine the equilibrium constant for the conversion from


:i axial- 1,2-dimethylcyclohexane to diequatorial- 1,2-dimethyl-

D.
.

c.2
3

:..clohexane by way ol a ring-f'lip plocess (Ksq-1,2). This .llue is directly comparable to the equilibrium constant fbr

::e conversion fiom diaxial-1,4-dimetl-rylcyclohexane

to
3

-.equatorial-1,4-dimethylcyclohexane by way of a ring-f1ip :rocess (Keq-l,a), given that they are both trcns-substituted :,,clohexanes. The difference between their equilibrium , rnstants is attributed to the gauche and anti orientations : -,ssible with the methyl substitucnts at the 1,2 positions. j:udies have shown that lalger substituents prefer the .:uatorial orientation of the so-called chair conformation. .qure I summarizes the findings of the study. CHr
I

What is the value of K.O- 1,4 for the conversion of trans-1,4-dimethylcyclohexane from its diaxial
conformation to its diequatorial conformation?

A. B.

0.0029 2.16 4.31 345

D.
4,

c.

-/--_/
*-H.

nttt
t\eq = ll'J

H,c{{ \
I

Cis-i,4-dimethylcyclohexane, in its most stable chair conformation, CANNOT have which of these
interactions?

9H..

A. B. C. D.

CH3/H gauche

CH3/H anti
CH3/CH3 gauche

H/H anti

s,cG4, '
K"o+ +.Zl
,

\--l-{/'cH\
5

.l:
Figure 1 Ring-flipping of trans-dimethylcyclohexane
The values fbr the equilibrium constants can be applied Jetermine the relative steric hindrance of one substituent rpared to anolher. A bulkier group exhibits greatet' steric rdrance, so the equilibrium lies more towards the more

Which of the following accurately describes the value of K"O for the conversion from one chair conformation [o the other chair conformation for the compound cis-t,2dimethylcyclohexane?

, "

A. Less than or equal to 0 B. Greater than 0 and less than I C. Equal to D. Greater than
1 1

.rle of the two possible chair conl'irrnations.

Hence,

.-3ater equilibrium constant irnplies that there is a greater ---ee of steric hindrance in the less stable confblmation.

6.

Which of the following is the most stable orientation of


a substituent on a cyclohexane molecule?

The value of K"O for the conversion of cls-1,3dirnethylcyclohexane from its diaxial conformation to
its diequatorial conformation is:

, B. C. D.
A

Axial orientation ol a chair conlormation


Equatorial orientation of a cl.rair conlotmation Bridge orientation of a boat conlormation Oar orientation of a boat conlormation
@

. B. C. D.

less than 0.22. between 0.22 and greater than 4.31.


1.

between I and 4.3 i.

-:pyright

by The Berkeley Review@

147

GO ON TO THE NEXT PAGE

7.

Which three-dimensional conformation corresponds to


the 3-hydroxy-cls-decalin, shown below?

Passage

ll

(Questions 8 - 14)

qlr"'

Alkanes are hydrocarbons that contain all sigma bonds. Sigma bonds have linear electron density (electron density that is localized between the two nuclei of the bonding atoms). This allows for free rotation about a sigma bond.

Rotation about sigma bonds is continually occurring

at

&F
ffi*f=,"
D.

temperatures above absolute zero, although the rate of the rotation varies. However, the rotation does not necessarily complete a full 360' cycle about the sigma bond.
OH Some conformations encountered during rotation are of high energy (due to steric repulsion) and others are of low energy (due to minimal steric interactions). The most stable conformation occurs when the largest groups are as far apart as possible. When two groups are as far apart as possible, the conformation is referred to as staggered, and the bulkiest substituents are said to be anti to one another. The least stable conformation occurs when the largest groups interfere with one another. This is known asfully ecLipsed. Drawn in

Figure

I is an energy diagram for the counterclockwise

rotation about the C2-C3 bond fbr R-2-methyl-1-butanol.

bo

b
+.1

g
Figure

l8o
Degrees displaced from initial state

360

Energy during rotation about the C2-C3 bond

The three apexes occurring at 60', 180' and 300' on the graph are not of equal energy. In 2-methyl-1-butanol, carbon 2 is a stereocenter. Because of this asymmetry, none of the eclipsed or staggered conformations are equal in energy. Al1

visual projections show asymmetric steric interactions.


Although the molecule is constantly rotating about its bonds,

it
8

assumes its most stable conformation most

of the time.

Which

of the following

structures represents the


1?

molecule at the 240" point on the graph in Figure


B.

H.C

CH.

Hf{,,cH:oH
C.

D.

H:C. H

sr,'HH J\

\CH2OH CHr

TF{::""

Copyright

by The Berkeley Reviewo

GO ON TO THE NEXT PAGE

9.

The 60'point on the graph in Figure 1 represents the


structure when it is:

3.

The reason there is no rotation diagram for trans-2butene is that:

A. B. C.
D

eclipsed and the methyl substituent on carbon-2


interferes with carbon-4.
eclipsed and carbon-l interferes with carbon-4.

staggered and the methyl substituent on carbon-2


interferes with carbon-4.
staggered and carbon-l interferes with carbon-4.

A. gauche is not favorable for alkenes. B. anti is not favorable for alkenes. C. steric hindrance does not affect alkenes. D. rotation about a n-bond requires energy in excess of
room temperature.

10. Which of the following structures


A.
B.

represents the
1?

14.

molecule at the 330" point on the graph in Figure

Which of the following compounds contains a single bond about which complete rotation is not possible?
A. B. C. D. Dipropyl ether
2-butanone 2,3-butanediol

9Hr

Methylcyclopentane

H
CHzOH HOH2C

CH2OH

H CHc

H:C

l.

Which of the following statements BEST explains why the MOST stable conformation of 2-amino-1-ethanol is
gauche?

A. B. C. D.

Hydrogen bonds are strongest when the two


substituents have gauche orientation.

Hydrogen bonds are weakest when the two


substituents have gauche orientation.

H is bulkier than NH2, due to the inductive effect. H is bulkier than NH2, due to resonance.

2.

The strongest hydrogen bond occurs between which of


the following?

A. B. C. D.

A A A A

lone pair on nitrogen bonded to an H on nitrogen lone pair on oxygen bonded to an H on nitrogen lone pair on oxygen bonded to an H on oxygen lone pair on nitrogen bonded to an H on oxygen

-opyright

by The Berkeley Review@

149

GO ON TO THE NEXT PAGE

Passage

lll

(Questions 15 - 22)

6.

Addition of D2 gas and palladium metal instead of shown what ratio

H2

A researcher wishes to determine the relative stability of axial orientation versus equatorial orientation for deuterium and hydrogen on cyclohexane. To do so, she treats benzene (COHO) with D2S04/D2O at 100"C for rhirry minutes to synthesize monodeuterobenzene (COHSD), which is then treated with H2 gas and palladium metal under 90 psi of
pressure to yield monodeuterocyclohexane (C6H11D). For monodeuterocyclohexane (C6H11D), there are two possible

gas and palladium metal to deuterobenzene would have

of equatorial hydrogens to

axial

hydrogens in its most stable chair conformation?

A.6:5 8.5:2 C.3:2 D. 2:3

chair conformations, one with the deuterium having axial


orientation and the other with the deuterium having equatorial

orientation. At room temperature, the interconversion


between the two chair conformations is too rapid to study and all eleven hydrogens appear as a singlet 1.38 ppm in the

17. The addition of H2 gas and platinum metal ro


chlorobenzene (C6H5CI) leads to a product whose most stable conformation is:

IHUUR. At low temperatures, the interconversion through

ring-flip from one chair-conformation to the other is slowed greatly, so that axial and equatorial hydr-ogens generate different signals in the IHNMR. As a resulr, the ring-flip process can be monitored using IHNMR spectroscopy.

A IHNMR

was recordecl at -89'C in deuterochloroform

. B. C. D.
A

boat with chlorine anti. boat with chlorine gauche.

chair with chlorine axial. chair with chlorine equatorial.

orienrarion shows a IHNMR shift of 5 = i.25 ppm. Integration shows that the relative area of lHNIr,lR signals is 1.12: l in favor of the 6 = 1.25 ppm signal. Because a larger amount of equatorial hydrogen is obser-ved than axial hydrogen, the deuterium has axial orientation in the more favorable chair confbrmation.
The researcher proposes that a diff'erence in bond length between the C-H bond and the C-D bond, rather than a difference in atomic size between hydrogen and deuterium, accounts for the equatorial preference of hydrogen over deuterium. The difference in bond length is attributecl to the gleater relative mass of deuterium compared to carbon versus the lesser relative mass of hydrogen compared to carbon.
Because the center of mass remains constant when a bond is stretched, the greater difference in mass between hydrogen and carbon than deuterium and carbon makes the carbon-hydrogen

solvent, A hydrogen with axiai orientation shows a lUNIrrtR shift of 6 = l.5l ppm, while a hydrogen with equatorial

18. The D-C-H bond angle about the deuterated carbon is


closest to which of the following values?

A. 90' B. 109.5' c. 120' D. 180"

19.

The most stable form of cis- 1,3,5-trimethylcyclohexane


has the chair conformation with:

bond stretch more asymmetrically than a carbon-deuterium bond. A carbon-hydrogen bond stretches more than a carbondeuterium bond, and thus occupies a greater amount of space. Despite the differences in bond length, the bond angles in cyclohexane remain similar, between 107.5' and I 1 l'.

Hsc

ctl

A. B.
C D

15,

The researcher reached the ultimate conclusion that a bond between carbon and deuterium is shorter than a bond between carbon and hydrogen, based on the tact
that:

three methyl groups in the equatorial position and no methyl groups in the axial position. two methyl groups in the equatorial position and one methyl group in the axial position.

. .

A. B. C. D.

the deuterium favors the equatorial orientation. the deuterium f'avols the axial orientation.

one methyl group in the equatorial position and two methyl groups in the axial position. no methyl groups in the equatorial position and three methyl groups in the axial position.

the interconversion between the two possible chair conformations of the deuterocyclohexane molecule through ring flip is rapid a[ room temperature. the lgNVtR shift at 6 = 1.25 ppm is fbrrher uplield than the IHNVIR shil'r ar 6 = l.5l ppm.

Copyright O by The Berkeley Review@

GO ON TO THE NEXT PAGE

0.

How many units of unsaturation are there in C6H5D?

Passage

lV

(Questions 23 - 29)

A. 3 B. 4

In most research laboratories, fourier transform infrared

c. 5

(FTIR) spectrophotographers are used to obtain infrared


spectra. The FTIR spectrophotographer works by passing an electromagnetic pulse of multiple frequencies through a
sample and then collecting and analyzing outgoing radiation.

D.

The difference between the output signal and a reference signal is digitized by computer and broken down into a set of
component sine waves (this is the Fourier transform process).

11. The reason that the diaxial orientation for cis-3hydroxycyclohexanol (a cis-1,3-diol) is preferred over the diequatorial orientation is that the hydroxyl groups:

The signals are processed and recorded to yield the same


spectra as those obtained using outdated variable frequency IR spectrophotorneters.

A.

are smaller than hydrogens, so they exhibit no pref'erence for the less hindered equatorial
orientation.

B. are larger C. D.
pref'erence
orientation.

than hydrogens, so they exhibit

preference for the less hindered axial orientation.

that it is not possible to focus on one peak by using a monochromatic light pulse. Focusing on one peak with a monochromatic beam can be done in rate studies, although the IR has a rapid shutter speed, faster than any reaction
(including diffusion controlled reactions).

One advantage of the FTIR machine is that the wave' number for each signal is given precisely. A disadvantage is

are larger than hydrogens, so they exhibit a for the more hindered equatorial

can form an intramolecular hydrogen bond from a 1,3-diaxial orientation, while they cannot form an intramolecular hydrogen bond from the 1,3diequatorial orientation.

The IR information is most useful if certain peaks are understood. For instance, an O-H bond in a compound can be recognized by the broad peak it displays around 3300 cm-1, although the exact value varies with the degree of hydrogenbonding. A carbonyl bond is found around of 1700 cm-1. If

a carbon has spj-hybridization, the bonds it forms to hydrogen are found just below 3000 cm-l. All of this information combines into a nice packet of data used to deduce the structure of a compound. Figure 1 shows the IR
spectrum for 2-heptanone:

Which structure represents the MOST stable form of


cis- 1,4-ethylmethylcyclohexane?

A.

cH:

B.

H,CN ' >'--L

CH:CH-1

CH2CHj
C

D.

4000 3600 3200 2800 2400 2000 1800 1600 1400 CH2CH3

1200

1000

HCh
CH2CH3

Figure

Infrared Spectra for 2-heptanone

CH:

The information extrapolated from the IR spectra can be coupled with NMR (nuclear magnetic resonance) data to form a powerful combination. For instance, aromatic hydrogens are found in the 7 to 8 ppm range in an 1HNMR spectrum.

23, An

absorbance between 1700 would NOT be present in:

cm-l and 1740

cm-1

A. ethyl propanoate. B. butanal. C. 2-pentanone. D. diethyl ether.

The Berkeley Reviewo

l5l

GO ON TO THE NEXT PAGE

24.

The IR spectrum for a straight chain monosaccharide


has all of the

8.

following absorbance values EXCEPT:

Which of the following isomers of C4H6O would NOT have an IR signal at 1715 cm-1?

A. B. C. D.

3300 cm-l. 2980 cm-l. 2300 cm-1.


1715 cm-1.

A. 2-methylpropanal B. Butanal C. Butanone D. Tetrahydrofuran

eill@

']rMt,"[ m!&m
:6r1ttlriu

{@fu
ifi'mrrrrlrm

qnmliftr

tfupr
2

5.

Which of the following isomers of C3H6O2 exhibits broad IR signal around 2850 cm-l?

29.

WMffi ilil&,m

A.oB.

c.o

"oA.H2cH3

H3co

Hydrolysis of an ester could be supported by which of these IR spectroscopic data?

Ml

cHs

D' ,4.. oo

A. B. C. D.

,6&Lm

The appearance of a signal around 1700 cm-l The disappearance of a signal around 1700 cm-l The appearance of a signal around 3300 cm1

The disappearance of a signal around 3300 cm-l

H,cnrcoAn

6.

Which of the following pairs of compounds could be distinguished by their splitting patterns in the proton NMR region between 7 and 8 ppm?

. . C. D.
A
B

Methylpropanoate from ethylethanoate


3-methyl-2-hexanone from 2-methyl-3-hexanone

l,4-methylphenol from l,4-ethylphenol


1,4-methylphenol from 1,3-methylphenol

27

To distinguish a tertiary alcohol from a primary alcohol (the tertiary alcohol exhibits more steric hindrance to hydrogen-bonding than the primary alcohol does), it would be best to focus on which of the following IR
features?

. B. C. D.

The width of the peaks near 3300 cm-l The length of the peaks near 3300 cm-l

The width of the peaks near 1700 qm-l The length of the peaks near 1700 cm-1

Copyright

by The Berkeley Review@

152

GO ON TO THE NEXT PAGE

Passage

(Question 30 - 36)
mass

1. Which of the following I UNUR shifts would


observed for Compound A?

An unknown alkyne with a molecular


-:

of

122.2

mole is treated with H2lPd and BaSO4 to convert it into a ..s-alkene. The cis-alkene is isolated in high purity. The cis ..rene is then treated with high-pressure ozone (O3) gas and

A. 12.0 ppm B. 9.7 ppm


C. 7.5 ppm D. 5.5 ppm

nc metal to convert both of the alkene sp2-hybridized :.rbons into carbonyl carbons. Because the cis-alkene was , -ned by hydlogenation of an alkyne, it is disubstituted with =.:h alkene carbon holding one hydrogen. This means that -: products of ozonolysis are both aldehydes. Two unique - rducts are isolated from the product mixture. The two --known products are designated as Compound A (C3H6O) --J Compound B (C6H1gO). An lHNIrrtR spectrum is ,.ained for Compound B and is shown in Figure I below. . -: lHNMR is cariecl out using CDCI3 as the solvent.

32. Which of the following IR


observed for Compound B?

absorbances would

A. 1125 cm-1 B . 2200 cm-l


C. 1620 cm-l D. 3550 cm-l
3.

What is the molecular formula for the original alkyne?

A. C9H14 B. C9H16 C. C9H1s D. C9H26

Figure

1HNMR spectrum of Compound B

34. Which of the fbllowing IR


indicative of an alkene?

absorbance values is

For Compound A, spectral data were obtained from an IR :::r,rum using pure Compound A in liquid form between " . -:lates. Table 1 lists the key IR absorbances collected for

-rfound A

A. B. C. D.

3550 cm-l 2200 cm-1


1"725 cm-l

1620 cm-1

Shilt(cm-11

Inrensir.y

2962 2912 2106 1126 1212


Table

strong medium mediurn strong strong

35. Which of the following


Compound A?

structures corresponds

A.oB.

all other peaks are irrelevant

1 IR absorbances

of Compound A

-r

The structures of Compounds A and B can be deduced qreat accuracy from the spectral data in the passage.
C.

t,"A.r,

HicH2c

A,,
CHr

engaging in structure elucidation, some information is useful than others. As a general rule, NMR data are -:d last, as they have the most information.

-\ compound with one degree of unsaturation and two


r\ygens CANNOT
be:

A. a cyclic ketone. B. a cyclic ether. C. acarboxylicacid. D. an ester.


: . right
@

O
153

by The Berkeley Review@

GO ON TO THE NEXT PAGE

6. Which of
A.

the

lollowing structures corresponds

to

Passage

Vl

(Questions 37 - 43)

Compound B?

'{x:,

.o

An unlabeled bottle containing an unknown compound is found in a lab storage locker. The compound is an odorless liquid that does not evaporate rapidly when the bottle is left uncapped. A lab technician labeis the bottle Compound T. Compound T exhibits three signals in its proton magnetic resonance spectrum. The three signals are listed in Table 1.

oA

shift
6 = 2.5 ppm (broad)

Integration
1H 1H

Shape
triplet
singlet
doublet

J-Value
3Hz
NA

6 = 3.1 ppm 6 = 4.3 ppm

2H

3Hz

Table

1HNUR signals of Compound T

Shorthand for describing nuclear magnetic resonance spectra describes the chemical shift value (6), the number of

hydrogen atoms (relative area under each signal), and the coupling along with the respective coupling constant, J.
Signal shapes can sometimes be described by single letters. such as s = singlet, d = doublet, m = multiplet, q = quarterand t = triplet.

For the unknown compound, Compound T, three bands at 6 50 (0, 73.8 (d), and 83.0 (s) appear in the carbon-li magnetic resonance spectrum. The splitting in the l3CNMR is due to the hydrogens directly bonded to the carbon. The important absorbances in the infrared spectrum of Compounc T are found at 3350 cm-l (broad),2988 cm-l, 2116 cm-1 , anC 1033 cm-1. The spectral data can be applied to determine the symmetry of a compound as well as the functional groups ot the compound. The IR peak at2116 cm-l is indicative of a triple bond, indicating the presence of either an alkyne or a nitrile in Compound T. There is no spectral evidence tc suggest that a nitrogen atom is present in the compound, sl the most logical assumption is that there is a carbon-carbori triple bond present in the structure.
From the spectral data, the lab technician concludes tha: the compound contains no carbonyl functionality. However"

,#!

"

the compound contains a functionality that is involved ir hydrogen-bonding, explaining the relatively slow evaporatiorof Compound T. Using chemical tests and a polarimeter, the lab technician determines that there is no mirror symmetry i: the molecule. This implies that the number of signals in the

l3CNtvtR spectrum

is also the

number

of

carbons

ii

Compound T.

37.

The shift at 1033 cm-l in the IR is caused by the C-C bond of:

A. an ether. B. an ester. C. a carboxylic acid. D. an alcohol,

Copyright

by The Berkeley Review@

GO ON TO THE NEXT PAGE

38.

The absorbance at2116 cm-l in the IR implies that the compound is an:

43. the

l3CNIr,tR peak at 50 ppm can be attributed to the:

A. B. C. D.

alkane. alkene.

A. CofaC=O. B. CofaC-O.
C. D.
H next to C=O. H next to C-O.

alkyne. alcohol.

39.

The l3CNIrrtR doublet ati3.B ppm can be attributed to


which type of carbon?

. B. C. D.
A

The terminal C of a terminal alkyne The internal C of a terminal alkyne Cofanaldehyde

Cofanketone

tt).

The number of carbons in an asymmetric molecule that shows seven peaks in the 13CNMR is:

A. 3. B. 4.

c. 7.

D.

14.

1.

Hydrogens that are coupled to one another have the


same:

A. B. C. D.

shift value.
peak integration.
area under theirpeaks. same J-value.

Compound T is which of the following compounds?

A. H-

C: C- Ct-te \

B.

OH

H- C= C- Chtz .H '\ ,/ c
il

c.
HO-

C=

"-

o\

D.

H-C=C-cH^
- cH,

\o,

-:'pyright

by The Berkeley Review@

GO ON TO THE NEXT PAGE

Passage

Vll

(Questions 44 - 50)

6.

According to IUPAC nomenclature convention, what is


the name of Compound III?

Fa

From the leaves of a tree that grows wild along the coast

ofEcuador, three esters were extracted using ether. The three esters were separated by fractional distillation and purified by column chromatography. Once isolated and purified, a IUNVIR spectrum was obtained for each compound. The three spectra are shown in Figure 1 below. All three spectra were obtained using the same NMR spectrometer in CDCI3 solvent from the same bottle. A dead give-away for an ester is the shift for the hydrogens attached to the carbon bound to the oxygen of the alkoxy group. These hydrogens are found
between 3.5 ppm and 4.0 ppm. Compound I has a molecular formula of CgHgO2, Compound II has a molecular formula of CgH1602, and Compound III has a molecular formula of

A. Methyl ethanoate B. Methyl propanoate C. Ethyl methanoate D. Propyl methanoate

7.

Which of the following IR absorbances would you NOT expect for an ester?

I.

CaHs02.
Compound I

A. 3500 cm-1 B. 2980 cm-1 C. 1685 cm-1 D . 1300 cm-1

lrr 6 pom

i'"
4

ppm

4
2 ppm

8.

Which of the following structures represents Compound


tr?

Compound

II

A.

B.
cH2cH3

o
ocH 2cH 3
ill I

lm
ll2H
illi
6

il
2 PPm

oa
III

ppm

DDm

Compound

C.

D.

:1
llil{:

lr"
il,'"
I

ilr
I I

d:,.
49.

Oclr*"

,T!*

llr
rll
rl-

'r t]
gj,

3 pDm

2 ppm

1 ppm

Figure
4

1HNMR spectra of Compounds I, II, and

III
For the spectrum of Compound III, the three 1HNMR signals are which of the following, respectively?

*.!n-:'

4.

Which of the following structures is Compound I?

i,]"

C.

d
.o
O
@

B.

.i[[,u:-

ctl

v
D.

..A,,/ocH.

A. Triplet, doublet, triplet B. Triplet, quartet, triplet C. Singlet, doublet, triplet D. Singlet, quartet, triplet

o*.
4

o"oYo'

0.

Which of the following would NOT work as an NNfR solvent for an ester?

5. It
A

important that each sample use the same solvent in

order to:

. ensure the same reactivity in the magnetic field. B. increase the pH of the compounds in solution. C. view any common impurities between samples. D . allow fbr extraction of protic compounds.
by The Berkeley Review@

A. CDC13 (chloroform) B. C6D6 (benzene) C. D3CD2COD (ethanol) D. D2CO (formaldehyde)

Copyright

r56

GO ON TO THE NEXT PAGE

Passage

Vlll

(Question

5t - 57)

53.

Pentanal is BEST distinguished from 2-pentanone by a


peak:

Nuclear magnetic resonance spectl'oscopy is a powerful :ool for distinguishing isomers. By cornparing shift values measured in ppm), coupling consrants (J-values), and rntegration ofpeaks (the area under the curve), it is possible io deduce a structure with great accuracy. The coupling lonstants help us deduce which hydlogens are neighbors of ,lne another. For instance, a carbon con taini ng two

A. B. C. D.

above 1700 cm-l in the IR. berween 2.0 to 2.3 ppm in rhe IHNMR.
near 3.5 ppm in the IHNMR. near 9.7 ppm in the IHNMR.

:quivalent hydrogens influences the signal for hydrogens on


::re neighboring carbon to foln.r a

triplet. The integration is

4.

To confirm the presence of an alcohol,


to add:

it would

be best

,re area under the curve that is directly proportional to the -.umber of hydrogens within that signal. Table 1 is a general ,sting of proton NMR shift values.

A. B.
C

D2O with NaOD and observe whether the broad


peak grows.

Hydrogen Shift (ppm)


-RCH3 -COCH3
-OCH3 0.8
2.1

Hydrogen
RCH2R,
-COCH2R

Shift

(ppm)

D2O with NaOD and observe whether the broad


peak disappears.

0.9 2.3

H2O with HCI and observe whether the broad peak


grows.

3.5 5.5 7-8

-OCH2R -CH=CHr -CH2CH=CH2


-CO2H
5.3 2.3

D.

H2O with HCI and observe whether the broad peak


disappears.

-CH=CH2

\H
CH=O Table

9.7

to-12
hyclrogen types
5

1 IHXVR shift values fbr common

5. A methyl ketone always has which of the following


proton NMR absorbances?

l:;
:
--

When IHNMR informarion is coupled with IR shifr .lles, it is possible to narrow the structure down quickly to : possibility. Important IR absorbances are 3500 crl-1 iO': . 1700 cm-l 1C=O;, anct 1600 cm-l 1C=C;. There are .:::r absorbances, but from these three values, it is possible ,, estimate many other values. As the strengLh of a bond r:reases, it exhibits a higher absorbance value in the IR.
-:e
heavier the atoms in the bond, the higher the absorbance IR. This intbrmation makes it possible to predict shift
5

. A 3H singlet between 2.0 to 2.3 ppm B. A 3H triplet between 2.0 to 2.3 ppm C. A 3H singlet between 3.5 to 3.8 ppm D. A 3H triplet between 3.5 to 3.8 ppm
A

6.

:is than 1600 cm-I in absorbance in the IR,


-

res lor other bonds. For instance, a C-C bond must be

because a i.,-:on-carbon single bond is weaker than a carbon-carbon


"

What is the common name for the compound with the fbrmula CaHgO, an IR absorbance at Il21 cm-l , and three NMR peaks at 9.7 ppm (lH),2.2 ppm (lH), and
1.0

ppm (6H)?
Tetrahydrofuran 2-methylpropanal Butanal Butanone

-cle bond. However, the irnportance o1


]-Methyl-3-pentanoneconrains:

NMR

:E.Iroscopy is greater than that of infrared spectroscopy.

i l.

-\. 3 non-equivalenthyclrogens in a9 : 2 : I ratio. B. 4 non-equivalenthydrogens in a4 : 3 : 3 : 2 ratio. C, 4 non-equivalenthydrogens in a6 : 3 : 2 : 1 ratio" D. 5 non-equivtrlent hydrogens in zi 3 : 3 : 3 : 2 : I


ratio"

A. B. C. D.

7. A

compound with one oxygen in its formula, one degree of unsaturation, and no IR absorbance between 1600 cm-l and 1750 cm-I must be:
a ketone.
an aldehyde.

il"

What in the proton NMR is a dead give-away for


isolated isopropyl group?

an

{. A 6H doublet and a lH septet B. A 6H singlet and a lH sextet C. A 6H septet and a lH doublet D. A 6H sextet and a lH singler
-

A. B. C. D.

an alkene.

cyclic.

::', nght O by The Berkeley Revie',v@

157

GO ON TO THE NEXT PAGE

Passage

lX

(Question 58 - 64)

A 1HNMR was run for the cis and trans isomers of the molecule shown in Figure 1 below. The spectra obtained for the two geometrical isomers are also shown in Figure 1. The cis compound can be distinguished from the trans compound using the coupling constants for the vinylic hydrogens. The trans species has a larger coupling constant due to better transfer of magnetization in the trans orientation. The shift values on the spectra are listed in parts per million (ppm) of
the frequency of the spectrometer. The hydrogen on the carbon adjacent to benzene can be tbund downfield from the other vinylic hydrogen due to the presence of oxygen. "Downfield" implies higher shiti values.

60. Which of the following compounds has the same number of signals in their carbon-13 NMR as there are
carbons in the compound?

P@

A. An ortho substituted benzene with two identical


substituents

B. A meta substituted
substituents

benzene with two identical

.A

meta substituted benzene with two different benzene

substituents

. A para substituted
substituents

with two different

n,.-G
Spectrum I

cH- cH]H

ocHj
7.25 5.37 4.88 3.76 2.25

61.

Where do the vinylic hydrogens appear in IHNMR?

ppn (4 H)

pptn (l H) ppm (1 H) ppm (3 H) ppm (3 H)

A. Above 7.0 ppm B. Between 4.5 and 7.0 ppm C . Between 2.0 and 4.5 ppm D. At less than 2.0 ppm

rllr
ilfl
]l]i

llLl
6
3
H

PPmz65432lo
Spectrum

2.

II

3H

4H

7.25 5.37 4.88 3.76 2.25

ppm (4 H)

Which of the following absorbances wouid be found the IR spectrum for the compound in Figure I ?

rn

ppn

ppm (l H) ppm (3 H) ppnt (3 H)

(l

H)

rllr
iltiltil

IHIH

ilil 1
1HNMR spectra of the cis and trans isotners

A. B. C. D.

3500 cm-l 2220 cm-l 1120 cm'l


1640 cm-l

PPmz6s432lo
Figure

63.

The spectra as drawn are different enough to distinguish the trans and cis compounds fiom one another. The shilt values does not help to distinguish the cis lrom the trans compound, the coupling does.
5

Had the compound had meta substitution rather than para substitution, the ratio of the benzene hydrogens would be which of the following?

8.

Spectrum I is associated with the:

A. B. C. D.

A. 1 :1 B. 1 :3 C" 1:2:l D. 1:1:1:1

ill:'tn:1

trans compound, because the J value is smaller. cis compound, because the J value is smaller. trans compound, because the J value is larger. cis compound, because the J value is larger.

FitE'ru

64.

Had the compound had an ethyl group on benzene rather than a methvi group, which of the following would be observed in the proton NMR?

iu.

*"'"
I Jr

lrl

intmr

i:
1.i"

9.

The methyl group on benzene appears in the IHNMR


at:

B. -5.37 ppm. C. 3.76 ppni. D. 2.25 ppm.


Copyright O by The Berkeley Review@

"

7.25 ppm.

. Doubiet (2H) and triplet (3H) B . Doublet (3H) and triplet (2H) C . Triplet i2H) and quartet (3H) D . Triplet (3H) and quartet (2H)

)illi;IlL,I
ll,il T1:.
[:ifL

.i

l[

l:iiL'tt:ti

L
t'

iud

t,i

l,nr;'

,il]fiI'rl',':r:

pr|trnTI- '

r58

GO ON TO THE NEXT PAGE

Passage

(Question 65 - 71)

5.

The shift at 1738 cm-l in rhe IR can be attributed to the

:u'o singlets in its proton magnetic resonance spectrum. One shows a shift of 6 1.42 ppm and the other shows a shift
-.96. ppm,

An unknown compound, labeled Compound B, has only


of
6

C=O of:

:tre

with relative intensities (from the integration of 1HNMR


spectrum) of 3

: l.

The decoupled carbon-l3

:lur signals (with one of greater intensity ihan the other 'lree) with shifts at 6 22.3 ppm, 2g.l ppm,79.9 ppm, and - r0.2 ppm. The signal at ljo.2 ppm is atributed to a :rrbonyl carbon. For carbons of equal hybridization, the :rgher the shift value in the carbon-l3 NMR, the greater the :-ectronegativity of the atoms bonded directly to that carbon. :igure I shows both the 1HNMR ana l3CNUR spectra of

:ruclear magnetic resonance spectrum for Compound B shows

A. an aldehyde. B. a carboxylic acid. C. an ester. D. a ketone.

66.

The molecular ratio of hydrogens in the structure is


which of the following?

mpound B.

A. 3:1 B. 6:2 C. 9:3 D. 12: 4

7.

The l3CNUR peak at 79.9 ppmcan be attributed to: A. an H bonded to C adjacent to C=O.

B. C. D.

an H bonded to C adjacent to C_O.

the C in rhe C=O bond. the C in the C-O bond.

8.

Which of the following compounds would NOT show l3cNnaR peak above 100 pfm?

A. B. C. D.

Methyl benzoate
3-methyl-2-hexanone 3-methyl-2-penranol 3-methylpentanal

Figure

lHNltn

and I3CI.Ur,IR spectra of Compound B

The carbon-13 NMR information leads to the conclusion


fibi.: there are four nonequivalent carbons tn the compound. Tre important bands in the infrared spectrum of Compound B iurir,: found at 1738 cm-l , 1256 cm-l, and ll73 cml1. The innr:d at 1738 cm-1 is attributed to the stretching of a carbonyl

69.

The lgNVtR peak at 1.96 ppm can be artribured to


hydrogen bonded to a carbon:

rr:alns two oxygen atoms. The spectral data, in r.:unction with mass percent values from elemental umn-vsis, can be combined to determine the structure of
inpound
,niur*orbance

*n'r:d. Elemental analysis of Compound B shows that it

A. adjacent to a C=O bond. B. adjacent to a C-O bond. C. ofaC-Obond. D . of a C=O bond.

1738 cm-l and a l3CNUR signal can be used to verify aspects of the structure.

B.
at

Overlapping spectral data, such as an IR

at

170.2

::vright @ by The Berkeley

Review@

GO ON TO THE NEXT PAGE

70. in IHNMR,
A

a singlet is explained as the evidence of

Passage

Xl

(Questions 72 - 78)

hydrogens of the signal being coupled to:

. B. C. D.

equivalent hydrogens on all adjacent carbons. non-equivalent hydrogens on all adjacent carbons.

Carbon-13 NMR can be used to determine the number ol r,rnique carbons in a compound. Carbon-13 NMR is similar to hydrogen-1 NMR in that it generates separate peaks for

only one hydrogen on an adjacent carbon. no hydrogens, because there are no hydrogens on
any of the adjacent carbons.

each unique isotope, in this c"s. 13C. Many I3CNMR spectra are recorded without coupling (known as decoupled spectra), so all peaks appear as singlets. Table I lists the approximate shift value for selected types of carbons. It is a brief guide to determining the types of carbon represented bi each I3CNMR peak. The range for the ppm of each signal is approxirnate and on occasion, a peak may fall outside of the
range.

1.

Compound B is which of the fbllowing?

A.oR'
Hrc
CH:

Carbon
R2C=O RHC=O R2C=CH2

ppm
205 - 220
185 - 200

Carbon
.C-OH -C-CI -C-NH2
-C-Br
F{3C-C=O

ppm

t20 - 140
r 10

HIC
C.

CH:

H:C
D.

CH:

R2C=CH2

120 85

RC=CH

12

50-70 40-45 35-45 25-35 20-35

"''K)rn .XY
HrC- - OH:C CHr
HsC CH:
O O

RC=CH

65 -10

R2CH2
values

l0-30

-CH-r

Table

1 l3cxltR shilt

should be noted that the peaks fbr carbons with r: hydrogen directiy attached are less intense than other carbo: peaks. Carbonyl carbons and quaternary carbons therefb:. generate shorter peaks than those for other carbons in t1::

It

spectrum. For peaks that represent rnore than one carbon, th: intensity increases, but not in a way that is easily integrate; Integration is generally not carried out on 13CNMR spectra.

A researcher used I3CNMR to distinguish two structur-isomcrs formed when toluene (methylbenzene) was acylate: using acetyl chloride by comparing the I3CNMR spectra :t the two compounds. The molecular tbrmula fbr both isome:i is CgH160. Spectra fbr both compounds were recorded usi:'s the same NMR instrument for the same period of time in lt:l': same concentration. The only clifTerence between the t''n: samples invoived the isomers themselves. The spectral d:-l tbr the two isomers are listed below:
Ison-ier

I: lsonierll:

2-1,34, 110,

ll1,

123, 130,206

31, 35, 108, 115, 119,123,126,131,?t-';

The dift'erence in the number of 13CNUR signals lisl:r for each isomer is cause<l by the lack of symmetry betse:r the two isomers. Isomer I has nine carbons but shows ol"' sevcn peaks in the l3CNMR, so it must have some type :$i symrnetry that equates carbons within the structure. B'-;tr isomers ale benzene rings with a n-rethyl substituent anC ar acetyl group attached to carbons on the benzene ring.

Copyright

by The Berkeley Review@

GO ON TO THE NEXT PAGE

72. Which of the following reactions CANNOT be monitored by a change in the shift values in the
l3CNvtR spectra?

76. How many signals

would be seen

in the carbon-l3

NMR of para-methoxy benzaldehyde?

. Oxidation of a secondary alcohol to a ketone B . Nitration of ethyl benzene to para-nitroethylbenzene C . Deprotonation of a carboxylic acid D . Reduction of an alkene to an alkane with hydrogen
A
gas and nickel catalyst

A. Four B. Five C. Six D. Eight

77

What is true about the units of unsaturation for the two


isomers?

3. Which of the following


structure of Compound I?

structures represents the

A.

B.

ov.+t

II,

B.

Isomer I, because it is more symmetric than Isomer has more units of unsaturation than Isomer II.

Isomer

I, because it

C.

D.

d il
cH2cH

Isomer

II,

has fewer units

has less unique carbons than of unsaturation than

Isomer II.

C. Isomer
Gl3

and Isomer

II

each have three units of

unsaturation.

D. Isomer I and Isomer II each have five units of


unsaturation.

78.

For the compound 2,2-djmethylbutane, which carbon


shows the peak of lowest intensity?

r
s

t !
D

Which of the following functionalities are NOT present in either of the two isomers?

A. Carbon B. Carbon 2 C. Carbon 3 D. Carbon 4


1

f,

A. Aldehyde B. Alkene C. Ketone D. Methyl

How can A

it

the spectrum for Compound

be explained that the least intense peak in II is found at209 ppm?

Carbons with sp2-hybridization show low intensity l3CNMR peaks. Carbons with spj-hybridization show low intensity l3cNuR peaks.
peaks.

B.

C. Chiral carbons show low intensity l3CNVtR D. Carbons with no hydrogen atoms
low intensity l3CNUR peaks.
attached show

)pyright

by The Berkeley Review@

l6l

GO ON TO THE NEXT PAGE

Passage

Xll

(Questions 79 - 84)

9.

Which of the following is the common name for


compound represented by Spectrum I?

the

Coupling in proton NMR is used to determine the relative positioning of hydrogens within a compound. Hydrogens are considered to be coupled when they are on neighboring carbons. Their respective magnetic fields influence the signals for one another. The effect is mutual,
so the coupling interaction is equal for all coupled hydrogens. This can be seen in terms of identical J-values. A J-value is also known as the coupling constant and is the distance between adjacent peaks within a proton NMR signal.

A. Ethyl acetate B. Acetone C. Methyl acerare D. Isopropyl formate


80. A triplet in the proton NMR is associated with
hydrogens:
t-be

spectrum for two isomers of C4H3O2 are collected under identical conditions using the same NMR machine. Figure 1 shows the spectrum for Isomer I, while Figure 2 shows the spectrum for Isomer II.
Spectrum

the lftNMR

A. B. C. D.
8

of a CH2 group.
on a carbon next to a CH2 group.

of a CH3 group. of a carbon next to a CH3 group.

13

What type of compound is represented by Spectrum H-t

A. B. C. D.
82. A

An ethyl ester A methyl ester An ethyl ketone A methyl ketone

lgNirAR signal in the range between 6 and


a hydroxyl proton.
an aldehyde hydrogen.

indicates the presence of which of the following?

4ppm

3ppm

2ppm

lppm

Figure 1 1HNMR spectrum of Isomer I

A. B. C. D.

abenzenehydrogen.
a

carboxylic acid proton.

83. Which of the following features in the proton


spectrum CANNOT be used to distinguish an ester

[h
,ct-

of a carboxylic acid when the two compounds have


same molecular formula?

A. B. C. D.

A sharp peak between 2.0 and 2.5 ppm A sharp peak between 3.5 and 4,0 ppm A sharp peak between 10.0 and 12.0 ppm
The observation that no peak integrates to a re

D-.

il. 0-,

ratio of

1.

3ppFigure

2ppm

lppm

84.

How many unique proton NMR signals are expected


4-heptanone?

frdmmm

2 IHNMR spectrum of Isomer II

Both of the isomers are esters. The exact connectivity of the esters can be deduced from the shift values of the singlet and quartet. IHNVIR signals around 3.5 to 4.0 ppm are due to alkyl groups bonded to an oxygen while alpha hydrogens typically show shift values between 2.0 and 2,5 ppm. The

A. Two B. Three C. Four D. Five

A-m

f-p

,lU- s

integration information verifies the substitution


carbon in the ester.

of

each

Copyright

by The Berkeley Review@

162

GO ON TO THE NEXT PA

Fassage

Xlll

(Question 85 - 90)

7.

What can be concluded from the data obtained using UV-visible spectroscopy?

A chemist sets out to determine the structural identity for fih;ee structural isomers, using data from NMR spectroscopy,
Ltu.L

:iviolet-visible spectroscopy, and mass spectroscopy to :tify each cornpound. The following spectral data are :l:rved for three separate compounds, all with the forrnula : Ht oo.
:-"pound I:

A. Compound I is a conjugated diene. B. Compounds II and III are carbonyl compounds. C. Compound I is a ketone. D. There is no n-bond in Compounds I and II.

r-:CNMR'

:HNMR:

211 ppm (1), 31 ppm (1), 22 ppm ppm (1), 1l ppm (1)

(i),

17

2.32 ppm (triplet, 2H),2.08 ppm (singlet, 3H), 1 .12 ppm (multiplet, 2H), 0.96 ppm (triplet,3H)

8.

Which of the fbllowing is NOT a valid conclusion from


the spectral data for Compound

III?

--\'-Visible: 268 nm and 189 nm


,".iound II:

A.
13 ppm (2)

rCNMR: HNMR:

The absence of a broad peak in the 1HNMR means the compound cannot be an alcohol.

12 ppm (1),24 ppm (2),

B. C. D.
ppm

3.71ppm (multiplet, iH), 1.88 ppm (broad,

The absence of a peak above 175 ppm in the l3CNltR means the compound cannot be a
carbonyl.

1H), 1.33ppm (multiplet,4H), 1.14 ppm (triplet,4H)

,f--Visible: No intense peaks above 180 nm


:,
:cLutd

The absence of an absorbance above 180 nm


confirms that the structure must contain a ring for its unit of unsaturation. The large number of peaks in the 13CNMR means
the compound is an ether with little symmetry.

III:

'CNMR: 68 ppm (2),21ppm (2), l0 ppm (1) FI-vMR: 3.58 ppm (triplet, 4H), 1.28
-,-"'-Visible:

(multiplet, 4H), 0.92 ppm (multipl et, 2H) No intense peaks above 180 nm

1.ll NMR spectra are obtained using deuterated .::form as solvent. The mass spectrometer show an l-'e peak at 86 amu for all three isomers, contirming their
':ular fbrmula. Elemental analysis shows that carbon
:'" drogen are

9.

What is the IUPAC name for Compound I?

not the only atoms present.

ilompound II is what type of compound?

A, An alcohol B. An ether C . A ketone D. An alkene


90.
^arbon-13 NMR is useful for determining all of the '.llowing EXCEPT the:

A. B. C. D.

Pentanal
2-Pentanone 3-Pentanone

3-Methylbutanone

The integration of a signal in a proton NMR is useful


for determining the:

. . C. D.
A
B

number of unique carbons in a structure.


presence of a carbonyl group.

. B. C. D.
A

local magnetic field experienced by a hydrogen"

neighboring hydrogen atoms.


presence of an atom other than hydrogen or carbon.

relative quantities of unique hydrogens in the


compound.

substitution of a benzene ring.


geometry about a double bond.

right

by The Berkeley Review@

163

GO ON TO THE NEXT PAGE

Passage XIV (Questions 91 - 97)


Proton NMR is a valuable tool used to deduce the of unknown organic compounds. It helps one distinguish between structural features of two similar
structure
compounds. The three useful components of the spectral data fbr extracting structural information are the shift value (6, measured in ppm), the coupling and coupling constants (peak shape), and the integral (the area under the curve of a signal). From the shift value, infbrmation about the electroncgativity of adjacent atoms may be obtained. Coupling is used to determine the number of hydrogens on any atoms bonded to the atom bound to the hydrogens producing the signal. The integral is directly proportional to the number of hydrogens

91.

Which of the following is the IUPAC name for


compound represented by Spectrum A?

rhe

A. B. C. D.

Phenylmethane Toluene Methylbenzene Orthoxylene

92.

In Spectrum B, what is the ratio of the areas under


three peaks?

in the compound, so it can be used to find the ratios of hydrogens in the cornpound. Figure I and Figure 2 show the l gNVtR spectra for two simple organic structures, Compound A and Compound B, along with the molecular
tbrmula of the compounds they represent.

A. 3 :2:3 B. 1:1 4
C. 2:3
5
3

D. 1:2

Spectrurl A
CzHrc
9

3.

What peaks are expected for 2-bromopropane?

5H

3H

A. A 6H sextet and a 1H singlet B. A lH sextet and a 6H singlet C. A 6H seprer and a lH doublet D. A 1H septet and a 6H doublet

ppm

ppm

ppm

ppm

0 pprn

94

Figure

IHNMR spectrum of Compound A


Spectrum B C3H602

Which of the following compounds would NOT har; quartet in its proton NMR spectrum?

A.
(H3C)2HC

B.

cH2cH3

H3CH2C(H3C)2C

I
o ppm

C.

(H3C)2HCH2C

D.

CHe

H3CH2CH2C

A
ha"'e

95. Which of the following compounds would NOT


iL) ppm

5 pprn

doublet in its proton NMR spectrum?

Figure

lHNMR spectrum of Compound B

Structures can often be deduced usins onlv some of the information, such as the coupliig information and the integration. Often, the coupling infbrmation is the most important of all the data. The coupling constants can give intormation about the connectivity of the structure, as well as irints about the three-dimensional orientation of atoms within

lUlltR

A. 2-methyl-1-pentanol B. 3-methyl-1-pentanol C. 4-methyl-1-pentanol D. A11 isomers of methyl-1-pentanol


their proton NMR spectrum.

have double-;

:ne n.iolecuie.

f.-,pvright O by The Berkeley Review@

GO ON TO THE NEXT P.{

'r

6.

The iodoform test involves the addition of hydroxide anion and iodine to a carbonyl compound. If a carbon contains three alpha hydrogens, then the iodine will react with the carbonyl compound to yield a yellow precipitate. A compound with a positive iodofonn test would likely have which of the following signals in its proton NMR?

Questions 98 through 100 are


descriptive passage.

NOT

based on

8.

The following two molecules are best described as:

A. B. C. D.

Singlet Doublet

Triplet
Septet

. B. C. D.
A

a
-structural isomers. geomelrical isomers. optical isomers.
the same molecule with altered spatial orientation.

The broadness ofthe signal around 10 ppm in Spectrum B is explained as a signal caused by hydrogens:

A. on a carbon involved in resonance. B. coupled to more than eight hydrogens. C. on a carbon involved in hydrogen bonding. D. involved in hydrogen bonding.

9.

The following two molecules are best described as:

. B. C. D.
A

structural isomers.
geometrical isomers.

optical isomers.
the same molecule with altered spatial orientation.

10

0.

How many structural isomers of C5H12 are possible?

A. 3 B. 4

c. 5

D.

l.B 2.C 3.D 4.C 5.C 6.D 1.8 8.C 9.A 10.A 11.A 12.D 13. D 14. D 15. B 16. C 1',7. D 18. B 19. A 20. B 2r. D 22. D 23. D 24. C 25. A 26. D 21. A 28. D 29. C 30. A 3r. B 32. A 33. A 34. D 35. B 36. C 31. D 38. C 39. A 40. C 41. D 42. A 43. B 44. B 45. C 46. B 41. A 48. B 49. D 50. C 51. C s2. A 53. D 54. B 55. A 56. B 57. D 58. C 59. D 60. C 6i. B 62. D 63. D 64. D 65. C 66. C 61. D 68. C 69. A 10. D 11. D 72. C 13. C 74. A 15. D 16. C 77. D 18, B 19. A 80. B 8i. B 82. C 83. A 84. B 85. A 86. D 87. C 88. D 89. B 90. D 91. c 92. D 93. D 94. C 95. D 96. A 91. D 98. B 99. C 100. A
- ::.t O by The Berkeley Reviewo ENOUGH CHEMISTRY... FOR NOW!

Structure Elucidation
L.
a

Passage Answers
s

Choice B is correct. The most stable form of the cyclohexane ring is the chair conformation. The most position on the chair form is referred to as equatorial. Combine these two facts and the result is choice B.

Choice C is correct. Because the three deuterium atoms are cis with respect to one another, they cannot all axial nor all be equatorial. The most stable orientation (most stable chair confirmation) has as many deute atoms with axial orientation as possible. However, because the deuterium atoms are all mutually cis to another, the structure must have at least one deuterium with equatorial orientation. The best choice ( consequently your choice) is C, which is drawn below. D 2 axial deuteriums and 1 equatorial deuterium

D
1 axialdeuterium 2 equatorial deuteri

11

J.

Choice D is correct. As the reaction is written, the value of Kgt-1,4 must be greater than 1, because the prod more stable than the reactant. The reaction is favorable in the forward direction as written. This elimi choice A, a value less tiran 1.

The question now focuses on whether the conformational change with Ksq-1,4 is more favorable than conformational change with K"O-1,2, because the value of Kqq-1,2 is 4.31. In the case of the 1,2-disubstih compound, there are both diequdtorial versus diaxial interactions as well as gauche versus anti interactions consider. The diequatorial orientation is better than the diaxial orientation, because with diaxial there eclipsed interactions with the axial hydrogens. Howevet, the anti orientation of the methyl groups is than the gauche orientation. Overall, diequatorial preference over diaxial is a more important factor than a preference for gauche over so the value of Keq-l,Z is greater than 1. The 1,4-disubstituted compound has no gauche-versus-anti ini between the methyi groups to consider, because the carbons are far apart. Thus, the conformational pre purely an effect of the diequatorial orientation being preferred over the diaxial orientation. This makes value of Keq-1,4 greater than the value of Keq-1,2, making choice D the best answer. Drawn below is one worth (apprbximately equal to 1000 words worth) of explanation.
Methyl groups are apart from Methyl groups are close to one Methyl grouPs collide with one another when diaxial. another when diequatorial. axial Hs when diaxial. Methyl groups do not collide one another when diequatorial

CH.

I,CH1
rr3L

9Hs

I
cF{3

oiulgl = !,"guubtl -

Diaxial < Diequatorial cFI3


No anti vs. gauche factor

-H.C, Anti > Gauche

1,2-diaxial orientation results in anti orientation, while 1,2-dieqatorial orientation results in gauche orientation. The equilibrium still favors product (the right), but not as much as the L,4-equilibrium does.

1,4-diaxial orientation results

in steric hindr
1

from diaxial interactions with hydrogens, w l,4-diequatorial has no eclipsing steric hindr
Equilibrium favors the products more than with

4"

Choice C is correct. In order for substituents to be gauche or anti to one another, they must be bonded to ca that are connected to one another. In the case of 1-4-dimethylcyclohexane, the methyl groups are not adjacent carbons, so the two methyl groups cannot be gauche or anti to one another. This makes choice C There are hydrogens on every carbon, so H can be gauche to methyl. For the H to be anti to a methyl, the group must assume axial orientation. The oniy possible chair conformation of cis-1,4-dimethylcyclohexane one methyl group axial and the other in an equatorial orientation. This makes choices A, B, and D valid.
@

Copyright

by The Berkeley Review@

166

Section II Detailed Exp

0 (products and reactants always have some positive quantity), so choice A is an absurd answer.
HsC

Choice C is correct. Both of the chair conformations possible for cis-1,2-dimethylcyclohexane are equivalent in energy. In both of the chair conformations, one methyl substituent assumes axial orientation and the other methyl substituent assumes equatorial orientation. The equilibrium constant for the ring-flip process is equal to 1, because the energy levei of the product is equal to the energy level of the reactant. This makes choice C correct. Do the correct thing and pick C. As a point of interest, the value of KuO can never be less than or equal to

HsC

Both structures have one methyi equatorial and the other methyl axial

Choice

between the two methyl groups makes the diaxial orientation less stable than the trans-1,2-diaxial orientation. This decrease in stability in the conformation drawn on the reactant side results in a greater value for Ksq as written. This means that K"O-1,3 ,K"q_I,2, which is choice D.
Methyl groups are apart from one another when diaxial. Methyl grouPs are close to one Methyl groups collide with Methyl groups do not collide with another when diequatorial. one another when diaxial. one another when diequatorial.

D is correct. With cis-1,3-dimethylcyclohexane, the cis-1,3-diaxial interactions (steric repulsion)

CH. IJ

.r, a, fn, In,

S^

cF{"

1,2-diaxial orientation results in anti orientation, while 1.2-dieqatorial orientation results in gauche orientation. I?re equilibrium still favors product (the right), but not as much as the 1,3-diaxial-to-diequatorial equilibrium does.

1,3-diaxial orientation results in increased. steric hindrance. This has an effect on the equilibrium by shifting it heavily to the right. 1,3-diequatorial has no eclipsing sieric hindrance.

Choice B is correct. The key to this problem is drawing the two hydrogen atoms on the bridging carbons cis to one another. When the hydrogens are cis to one another on adjacent carbons, one hydrogen assumes axial orientation, while the other assumes equatorial orientation. As a consequence, the carbon-carbon bonds to the left ring must also be axiai for one and equatorial for the other. The structure is drawn below with the hydrogens noted. Choice B is the best answer.

Equatorial

Note that the two hydrogen atoms and the hydroxyl group are all cis and up.

OH

Axial

Equatorial--+11

Choice C is correct. Because 240' is at the nadir (low point) of the graph in Figure 1, it correlates with the staggered structure. This eliminates choices A and B. The compound has R stereochemistry at carbon two which makes the correct answer choice C and eliminates choice D, which has S stereochemistry. Choice A is correct. Because 60" is at a local apex (high point) of the graph in Figure 1, it correlates with an eclipsed structure. This eliminates choices C and D, which have the compound in its staggered confirmation. The 60' point is not the highest point on the energy diagram, so it does not involve the largest groups (carbon 1 and carbon 4) interfering with one another. This eliminates choice B. The best answer is choice A, with the two methyl groups eclipsing one another.

-:-:vright O by The Berkeley Review@

t67

Section II Detailed Explanations

10.

Choice A is correct. The 330" point on the graph is near (30" away from) the most stable conformation (nh: has anti orientation of the CH2OH group and the CH3 group of carbon 4). This eliminates choices C and Because of R stereochemistry, the correct choice is A. Stereochemistry is difficult to see in the Newrr projection and can be seen more easily in other projections. Drawn below is a way to convert the Neu,c: projection back to the stick-wedge drawing and a subsequent evaluation of stereochemistry.
CHs

H:9
H cH2oH
11.

CH.OH

H\\"t"'

Choice A is correct. Conditioning may cause yoll to respond automatically that the best orientation is the ane with the fewest repuisive interactions. This is often true, but it does not tell the entire story in this case. The, most stable orientation can also be the result of the strongest attractive interactions. Hydrogen-bonding betn' the hydroxyl and amine groups occurs only from gauche orientation, where the two gtorpi are close enough bond. Hydrogen-bonding cannot occur between substituents with anti orientation. This *uk"r choice A correct.

1'

Choice D is correct. Hydrogen bonds have some acid-base character to them, so the most favorable pro:: transfer reaction is a good indicator of the strongest hydrogen bond. Because the amine is more basic than i hydroxyl group, the nitrogen is the lone pair donor. Likewise, the hydroxyl is more acidic than the amine the hydroxyl is the hydrogen donor. This makes choice D correct.
Choice D is correct. Because of the planar nature of electron density in a n-bond, rotation about a double bc: requires that the n-bond be broken. This is not observed under thermal conditions. To convert a cis n-bond int; trans n-bond, UV light is needed. Pick D to tally big points. The drawing below shows that a 90' rotation abc the C-C breaks the r-bond. It requires substantial energy to break a rc-bond.
ru-bond
R1

13.

90" rotation ---------------R2

R,
R3

1.4.

Choice D is correct. The only single bond about which rotation is not possible is a single bond between two ar in a cyclic compound. The only cyciic compound of the answer choices is methylcyclopentane, choice D. The answer is therefore choice D. One item of notable interest is that both n-bonds and rings lower the entropl compound by lowering its degrees of freedom (i.e., its ability to rotate freely).

1.5.

Choice B is correct. The most stable form of cyclohexane is the chair conformation (as opposed to the i: conformation), with the smallest substituents (determined by bond length) in the axial position. The inteq from the proton NMR shows that the ratio of peaks for 1H is 1.12 : 1 in favor of the 6 = 7-.25 ppm shift, the due to the equatorial hydrogen. This indicates that more hydrogens are located in the equatorial position= i 1.25 ppn) than the axial orientation (6 = 1.51 ppm). Consequently, the preferred conformation has deuteriu-cr the axial position. For this reason, choose B. Drawn below are the two chair structures and their equilibriurc" 6 = t.st ppm ----->H L{ D
6 = 1.25

ppm

--->H

,0

t[

rU

6axiaiH:5equatorialH (IHNMRintegral 7.2:7)


Copyright @ by The Berkeley Review@

H
5

rfi

axialH:6equatorialH llUruVRintegral 1: Section II Detailed Explanat

Actual ratio is I.1.2:1, rvhich shows H prefers the equatorial position, so the right structure is more stable.

,fl

l6a

Choice C is correct. Using D2 gas rather than H2 gas would have produced C6H5D7. The addition of the D2 is in syn addition for all 6 deuteriums that are added, producing two possible conformational isomers, one having three hydrogens equatorial and two hydrogens axial (choice C) and the other having two hydrogens equatorial and three hydrogens axial (choice D). Choice C is more stable, because hydrogens prefer the equatorial orientation. Choose C if you want to be a star. The structure is drawn below:

.equatoriaiH D

3axialH:2equatoriaiH 3 axialH:2

2axiaIH:3 equatorialH

H prefers the equatorial position, confirming that the structure on the right is more stable. The K"O for this ring flip is > 1. The ratio is 2 axial: 3 equatorial.

This question could have also been answered without knowing the exact chemistry. The passage infers that hvdrogen prefers the equatorial position over deuterium. This means that in the most stable orientation, there are more hydrogens with equatorial orientation than axiai orientation. This eliminates choice D. Because there are only five hydrogens present (you start with five on deuterobenzene), the sum of the two numbers must be five. This eliminates choices A and B and makes choice C the best answer. Learn to answer questions as quickly as you can, whether you use organic chemistry knowledge or common sense.
Choice D is correct. The addition of H2 gas to chlorobenzene results in the hydrogenation of the benzene ring

and the formation of CH11Cl (chlorocyclohexane). For chlorocyclohexane, there are two possible stable conformational isomers, one having a ring structure in a chair conformation with the chlorine equatorial (choice D) and one iraving a ring structure in a chair conformation with the chlorine axial (choice C). Chlorine is larger than hydrogen, so it prefers to be in the equatorial orientation. Choice D is more stable so choose it.
Clroice B is correct. The hybridi zatron of carbon remains sp3, whether it is bonded to deuterium or to hydrogen' The angle therefore should be around 109.5". It is stated in the passage that the bond angles are between 107.5" and 111", which are both nearest to 109.5" of all the choices. Pick B for best results on this question. Choice A is correct. The cis form of 1,3,5-trimethylcyclohexane allows for all three methyl groups to assume identical orientation in terms of axial or equatorial. This means either choice A or choice D is the best answer. Equatorial orientation is more stable than axial orientation, so choice A is the best answer. The conformation is shown below:

H
H

Hsc
H

Choice B is correct. Like benzene, C6H5D has three n-bonds and one ring. The best answer is choice B. If you recall the formula for units of unsaturation, (2(#C) - (#H) + 2)/2, you can calcuiate the units of unsaturation, knowing that D behaves the same as H. This would also yield a total of 4. Choice D is correct. A hydroxyl group, OH, is larger than a hydrogen, so choice A should be eliminated. The axial orientation is -or" iritld"redin terms of sterics, so choice B is invalid. Choice C is a good explanation for why the two hydroxyl groups would be found in a diequatorial orientation, but the question is looking for an

expianation foi why diaxiai, rather than diequatorial, is the preferred conformation. This eliminates choice C. When the two hydroxyl groups are both in axial orientation, they can exhibit 1,3-diaxial interactions. This is normally considered to be steric repulsion; but in the case of two hydroxyl grouPs, there exists the ability to form hydrogen bonds. This makes choice D the best answer.
Revierv@

::',right O by The Berkeley

r69

Section II Detailed ExPlanations

,,

Choice D is correct. Only in choice C and choice D do the rings have their substituents with cis orientation. Choices A and B are eliminated, because the substituents are trans to one another. The more stable conformer has the larger substituent (the ethyl group) in the equatorial position. This describes choice D.

23.

Choice D is correct. An IR absorbance between 1700 cm-1 and 7740 cm-1 is the result of a carbonyl (C=O) group, as stated in the passage. Of the choices, only the_ether doesn'tcontain a carbonyl group, so it is the ether thai does not have an IR absorbance between 1700 cm-1 and.7740 cm-1. Pick D for best t"rrttr.

24.

Choice C is correct. A straight-chain monosaccharide has O-H groups, a C=O functionality and C-H bonds. These groups have IR absorbances for bond stretching of 3300 cm-r,l7I5 cm-1, and 2980 cm-1 respectively. This makes choice C the best choice. You are required to know common values for the IR absorbances. No commonvalue peak is found around 2300 cm-1, so thit should lead you to answer choice C.
Choice A is correct. A broad signal near 2850 cm-1 indicates the hydroxyl group of a carboxyiic acid (O-H witin hydrogen-bonding). A hydroxyl group exhibits hydrogen-bonding, so the signal is broad. Because the O-H bond of a carboxylic acid is weak, its absorbance is lower than that of standard hydroxyl groups. Of the ansirer choices, only choice A has a carboxylic acid functionality, let alone a hydroxyl gio,rp. plit a for happiness.
Choice D is correct. The region between 7.0 and 8.0 ppm in the proton NMR can be attributed to hydrogens on aromatic ring. This immediately eliminates choices A and B, which have no aromatic rings associated rri them. The difference between ethylphenol and methylphenol can be demonstrated by eithei the total numb of hydrogens (as shown in the integration) or by the coupling of the alkyl portion of the compounds. The grouP exhibits a 2H quarter and a 3H triplet, while the methyl group exhibits a 3H singlet. The extra group associated with the ethylphenol compared to methylphenol is found in the 2.0 to 2.5 ppm region, not aromatic region. This eliminates choice C. To decide between the two compounds in choice D, one can look the coupling of the hydrogens on the benzene ring, found in the range between 7.0 ppm and 8.0 ppm. splitting for the hydrogens on a benzene ring with para-substitution is symmetric, while the splitting for hydrogens on a benzene ring with meta-substitution is asymmetric. This is the distinguishing iactor betu the para-substituted and meta-substituted phenols in choice D. The best choice is answer D.

25.

26.

,ii

iu

fr

,n

Choice A is correct. Because the broadness of hydroxyl peaks is associated with hydrogen-bonding, decreased hydrogen-bonding of a tertiary alcohol is reflected as a narrower absorbance in the IR spectru-n:. compared to a primary alcohol. The alcohol peak is found weli above 3000 cm-1, so the best answer ii choice
Choice D is correct. An IR absorbance at 1775 cm-1 is rndicative of a carbonyl group (the stretching of a C bond)' Aldehydes and ketones contain a carbonyl group, so choices A, B, and C are eliminated. Only the (tetrahydrofuran) does not have a carbonyl functionality. Pick D and bask in the glow of correctness.

'@

rffi

28.

29.

Choice C is correct. The hydrolysis of an ester results in the formation of a carboxylic acid and an a Both the ester and the carboxylic acid have carbonyl groups, so each has an IR absorbance around 1700 c This means that both before and after hydrolysis, there is a signal around 1700 cm-1, eliminating choices A B. An ester has no hydroxyl group, so initially there is no signai around 3300 cm-1. Following hydrolysis, an alcohol and carboxylic acid are formed, so a signal for the hydroxyl group appears. In particular" hydroxyl group of an alcohol shows an absorbance around 3300 cm-1. This means that during the course cdhydrolysis of an ester, an IR signal appears around 3300 cm-1, making the best answer choice C.

30.

Choice A is correct. Because a ring takes one degree of unsaturation and a carbonyl takes one d unsaturation, a cyclic ketone has two degrees of unsaturation. The compound has only one unsaturation, so it cannot be a cyclic ketone. The correct answer is choice A. A cyclic ether has one ring and bonds, so it has one degree of unsaturation. This eliminates choice B. A carboxylic acid has no rings and bond, so it has one degree of unsaturation. This eliminates choice C. An ester has no rings and one n-bond. has one degree of unsaturation. This eliminates choice D.

Copyright @ by The Berkeley Review@

t70

Section II Detailed Expl

Choice B is correct. Ozonolvsis cieaves the double bond of an aikene, converting it into two carbonyl compounds (either an aldehyde or a ketone). In this reaction, the products formed are aldehydes, as mentioned in the passage. Aldehvdes can be identified by their peak around 9.7 ppm in the 1HNMR. Choose B for best results.

Choice A is correct. Compound B, like Compound A, is an aldehyde. Like Compound A, absorbar-ice near 7725 cm-1 for the stretching mode of the C=O bond. This makes choice A correct.

it

shows an IR

Choice A is correct. Compound A has one degree of r-rnsaturation (due to the carbonyl), and Compound B has two degrees of unsaturation (one due to the carbonyl and the other due to a ring). The second degree of unsaturation has to be from a ring, because there can be no double bond in the product from ozonolysis (had there been a n-

bond, the ozone would have reacted with it.) This means that the original alkyne must have had a ring and a triple bond, which in turn means that the original alkyne had three degrees of unsaturation. The alkyne had nine carbons (the sum of the carbons from Compounds A and B). A nine-carbon compound with no units of unsaturation has the formula CSHZO. For every unit of unsaturation, two hydrogens are removed from the formula, so a hydrocarbon with three units of unsaturation must have the formula C9H1.4, choice A. You can verify tlris with the molecuiar mass, which is roughly 122 g/mole, as stated in the passage.
Choice D is correct. An alkene is distinguishable by an IR absorbance between7620 cm-1 and 1660 cm-1 for the stretching of the C=C bond. You should know that i carbonyl C=O bond absorbs at around 1700 cm-1, and being that a C=C bond is slightly weaker than a C=O bond (they are both double bonds, but a carbonyl bond is slightly shorter), it takes siightly iess energy to stretch the C=C bond than the C=O bond. This means that a C=C bond has an absorbance slightly less ihar-r 1700 cm-1. Only choice D is less than 1700.*-1, so choice D is tl-re best answer. The test requires that you have some iR peaks in your memory; but for ones you don't recali, estimate them by comparison to the values you know. Choose D in this question for the feeling of correctitude.
Choice ll is correct. The products from the ozonolysis of the alkene are both aldehydes, so Compound A must be an aldehvde, which makes choice B the best answer. All of the choices have the correct units of unsaturation, so the bcst answer rnust be determir-red from the presence of the aldehyde proton.

Choice C is correct. As stated in the passage, Compound B must be an aldehyde, which eliminates choice A. The con'rpound cannot contair a double bond (the product from ozonolysis cannot have a double bond between carbons), so choice B is eliminated. The lHXVR integration shows a peak ratio of 7:1:4:4, which indicates that there are no methyl groups in the compound. A methyl group would have shown a relative ratio in the integrai of 3. This eliminates choice D. Choice D can also be eliminated, because it contains too many carbons. This narrows it down to choice C, which does in fact show four nonequivalent hydrogens in a ratio of 1:1:4:4. It is vital that you solve this question by a muitiple-choice elimination process rather than structural deduction, because in a multiple-choice format, eliminatir-rg invalid structures is faster than elucidating the correct one.

Choice D is correct. The question here is not what type of bond is causing the shift, because all four answer choices are C-O bonds. The question is: "What type of molecule is Compound T?" Because of the broad IR absorbance at 3350 cm-1, tire compound has H-bonding, so it must be an alcohol. The correct answer is therefore choice D. Choices B and C should be elimilated, because the passage states there is no carbonyl. Choice A can be eliminated, because ethers exhibit no hydrogen-bonding. Choice C is correct. An absorbance at 2\76 cm-1 implies that the compound is an alkyne. This is trivial knowledge for the most part, but the passage provided the information, in case you don't have carbon-to-carbon triple-bond IR data committed to memory. You can deduce the IR absorbance vaiue for an alkyne knowing that a C-C boncl has an IR absorbance between 1100 cm-1 and 1300 cm-1, and that a C=C bond has an IR absorbance between 1620 cm-1 and 1680 cm-1. As the bor-id strength increases (or bond length decreases), the IR shift value incre;rses. Choices C is the best answer. The answer is given at the end of the tl-rird paragraph in the passage.
Choice A is correct. A doublet in the 13CNMR is due to a carrbon with one hydrogen attached to it. Because a ketone carbon and an internarl alkyne carbon do not have hydrogens attached to them, they would be singlets rather than doublets. This eli.mi.nates both choice B ancl choice D. There is no peak in the IR spectrum, inciicating that there is a C=O present (no peak around 1700 cm-1), so choice C is eliminated. The only choice left is choice A, the best choice.

,:vright O by The Berkelev Reviervo

t7l

Section II Detailed Explanations

40.

best results.
47.

Choice C is correct. If a compound has no symmetry, then thereis a 13CNMR peak for every carbon in -*-* compor-rnd. The number of carbons in a compound exhibiting seven 13CWR peaks is therefore r"rr"rr. pick C :.r

Choice D is correct. When hydrogens are coupled to one another, they have the same coupling constants : values)' This makes choice D the best answer. Two different hydrogen groups have different shift values, .,: choice A is eliminated. Choices B and C are the same answer, so they shouldboth be eliminated. This lear'= only choice D as the best answer.
Choice A is correct. From tl-re NMR data, we know that Compound T has three carbon atoms and four hydrog.: atoms, so the structure must contain only three carbons. Choices B, C, and D al1 have four carbonr, ,o thuy .-,= eliminated without involving the spectroscopic data. The only choice that fits the formula is choice A. Or^.:: this question is resolved, it can help to solve some of the other questions. This happens often on the MCAT. Choice B is correct. The 13CNMR peak at 50 ppm is due to a carbon, not a hydrogen. This eliminates choices and D. A carbonyl shows a 13CNMR peak around 200 ppm, so choice A is eliminafed. Only choice B remains, you sl-rould pick it. The absence of an IR peak around 1700 cm-1 should reaffirm that a carbonyl is not present '.,: Compound T. This means that choice B is the best answer.

42.

m
hllL.

43.

Choice B is correct. The total number of hydrogens in the compound, according to the NMR is eight, of whr:: five are on the benzene ring. This eliminates choice C. The pisug" states thal the compound is an ester,:. choice A, a ketone, is eliminated. The best answer is choice B, because the methyl shift is around 4.0 ppm. Th,is tough to know without an NMR chart. In the passage, it is mentioned that the alkoxy group hai a shift : the 1HNMR between 3.5 and 4.0 ppm. if you can'f recali a fact, search for it in the passage.
45.

Choice C is correct. The three separate IUXMR spectra are to be compared to determine the compounds : solution. It is important that nothing varies between samples, so the same solvent should be used i1 each ca.s= Any impurity peak wouid be common to all of the spectra and thus could be eliminated. This makes choice C the best answer. A common solvent has no effect on the pH or the behavior of a protic species in the lF{Nlt: Although lHUltR invokes an externai magnetic field, the solvent has no bearing on the iielcl.
Choice B is correct' This question is typical of spectra-to-structure-to-name questions. There are four carbons : Compor,rnd III, so choices A and C are elj.minated, bec;ruse they have only three carbons, according to t:nomcnclature. Translating the narrles of cl-roices B and D yields the following two structures:

46.

H3CH2C

ocH3

H:C

OCH2CH3

Methyl propanoate

Ethyl ethanoate

Both A anci C would have identical peaks of a 2H quartet, 3H triplet, and a 3H singlet. The key feature is ti. 3H singlet at approximately 3.5 ppm. This indicates that the methyl group is attached to the ester oxygen :i in choice B. You should pick B to score.

47.

Choice A is correct. The four answer choices are the result of the following bonds: O-H (3500 sp3C-"^-t), (2980 cm-l), C=O (1685 cm-1;, and C-O (1300 cm-1). An ester contains an sp3-f-g bond, a C=O bond, and a C-i bond, but it does not contain an O-H bond. This means that there will be no O-H stretcl-r (thus no IR peak at i: near 3500 cm-1; associatecl with the IR spectra of an ester. Cl-roose A and be a stellar achiever at choosr-r.:
correct answers^

48.

Choice B is correct. Compound II has two oxygerls in its formula, so choices A ancl C are out immediately. T:'. question here is reciuced to placing the a1ky1 group either on the ox\1gen (as in choice B) or on the carbon., carbon (as in choice D)" Because the peak for the two hydrogens is around 4.0 ppm, the ethyl group must'r= attached to the oxygen, making choice B the best answer. You should note that when there is a jH quartet ar.: a 3H tripiet in the spectra, there is an isolatec-l ethyl group ir-i the n-rolecule.
@

Copyright

by The Berkeley Reviewo

172

Section II Detailed Dxplanations

+Y.

Choice D is correct. Taken sequentially, the signals are a singlet (one apex), a quartet (four apexes), and a triplet (three apexes). This makes choice D the correct choice. The shape of a peak, you should recall, is due
to the presence of hydrogens on the neighboring carbon.

Choice C is correct. An NMR solvent should be inert and show no peaks in the spectra. All of the compounds are deuterated, (contains 2H rather than 1H) so they show no peaks in the 1HNMR spectrum. The most reactive solvent listed is the alcohoi. The problem with an alcohol is that when it is heated or given enough time, it can undergo a transesterification reaction and change the ester. This makes choice C the best choice. An ester should be soluble in all four of the solvents. It is a general rule that protic solvents are bad choices for the NMR, because they exchange for hydrogens and are typically more reactive than aprotic solvents.

Choice C is correct. 2-methyl-3-pentanone has four nonequivalent hydrogens, according to the symmetry of the molecule. The structure of 2-methyl-3-pentanone is drawn below: H*C

"l bl

6 a hydrogens
c

A../tt\ ,/t'=-.ff,
o

'll

b hydrogen

2 c hydrogens 3 d hydrogens

This resuits in a ratio of 6:3:2:l for the four nonequivalent hydrogens, which makes choice C the best answer.

Choice A is correct. An isolated isopropyl group has two equivalent methyls groups (six hydrogens total) adjacent to a CH group (one hydrogen). The six hydrogens of the methyl groups are expressed as a doublet (being adjacent to one hydrogen), and the one hydrogen of CH are expressed as a septet (being adjacent to six equivalent hydrogens). This results in a proton NMR with a septet (1H) and a doublet (6H). This is choice A, so choice A is the best of all possible choices,
distinguished from other compounds by a peak near 9.7 ppm for the aldehyde proton. Choices A and B should be eliminated, because both pentanal and l-pentanone have a peak in the IR just above 1700 cm-1 for the C=O bond and a peak in the proton NMR between 2.0 and 2.3 ppm for the alpha hydrogens. Neither comPound has a proton NMR peak between 3.5 ppm and 4.0 ppm, because that shift value is attributed to hydrogens on a carbon bonded to oxygen. Only choice D, a peak near 9.7 ppm, is for a peak that is unique to the aldehyde, so the presence of the peak confirms that the compound is pentanal, while the absence of the peak supports that the
compound is 2-pentanone.

Choice D is correct. Pentanal is an aldehyde, while 2-pentanone is a ketone. An aldehyde is best

Choice B is correct. An alcohol hydrogen is rnildly acidic. This means that when a base is added to an alcohol, ihe protic H on oxygen can be removed. If the alkoxide formed is placed in deuterium-labeled water, then the alkoxide can remove deuterium from water to form a deuterium-labeled alcohol. Deuteriums do not appear in the 1HNMR, so the peak for the H on oxygen disappears, making choice B the best answer.

R_d:\ r\ "\/

e +oo1
H----/
e +on1

D-\

+Ro
H

D-dl u "\/

+DO
D

D---/

Choice A is correct. A methyl ketone has an isolated CH3 group adjacent to a carbonyl. Because the methyl group has no hydrogen neighbors, it has no coupling, and thus is a singlet between 2.0 and 2.5 ppm. Pick A.

-::r-right

by The Berkeley Review@

173

Section II Detaited Explanations

56.

Choice B is correct. A 1HNMR peak at 9.7 pprr- indicates that the compound is an aldehyde. This eliminat.s choice A (an ether) and choice D (a ketone). The ratio of 1,:7:6 lor theirea of the signali indicates that the:: are two equivalent methyl groups (that accounts for six equivalent hydrogens) and two unique hydrogens on +;,: compound. Only choice B has two equivalent methyl groups, so choice B is the best answer. Theie are fc:: unique types of hydrogens on butanal, so butanal would show four signals in its 1HNMR, not just three signals. Choice D is correct. Having no IR absorbance between 1600 cm-1 and 1750 cm-1 indicates that the compound h.:-. neither a C=C bond nor a C=O bond in its structure, both of which have IR signals between 1600 cm-1 and 1li, cm-1' This eliminates all of the choices except choice D. The compound ,rr.rit b" cyclic to account for the o:* degree of unsaturation.

ry
tr5.

57.

Choice C is correct. In Spectrum I, the distance between the peaks in the aikene region (coupling constant :; greater than it is in Spectrum II. A larger coupling between vinyiic hydrogens is attributed to the tra:= compound. This is choice C.
59.

Choice D is correct. The methyl group has three hydrogens, so the signai for the methyl group on benze:. cannot be the peak at either 7.25 ppm and 5.37 ppm, because neither of those peaks contains three hydroge:-This eliminates choices A and B. The alkoxy methyl group on the oxygen is found farther downfield than f * methyl on benzene. Thus from the values on the spectra, the methyl on benzene (3H singlet) comes at 2.25 pp:: which is choice D.
Choice C is correct. In order for the disubstituted benzene compound to have the same number of peaks in -:* carbon NMR as it has carbons, the compound must not have symmetry. Symmetry results in equivallnt carbc:-. resulting in fewer 13CXUR signals. the only asymmetric iompound u-orlg tire answer choices is the n.-:.l substituted benzene with two different substituents. Choice C is correct. The choices are drawn below:

60.

A.

B.X
I

C.X
I

D{
au
1"

cc /Cb,'

c,/cu.i-'
-

?Cr CcX
6 Carbons 6 Carbons

C.\

il

t Cr--. ^ udY

'iiT"'f.
c.1 lzC.
$'
6

3 61.

l3cltttR

Carbons

Carbons

peaks

13CNMR

peaks

13CNMR

peaks

13CNMR

peaks

Choice B is correct. The 1H peaks (the vinylic hydrogens) in the spectrum fall between 4.88 and 5.37 py=". implying that the hydrogens on the alkene are found in this range. Hydrogens on an alkene are referred tc m vinylic hydrogens. This means that vinylic hydrogens are found in the range of 4.5 ppm to 7.0 ppm. This ma..:s
choice B the best choice. Choice D is correct. The compound contains a C=C double bond, which has an IR absorbance between 1620 c::'') and 1660 cm-1. The best choice is 1640 cm-l, choice D. An IR absorbance of 3500 cm-1 is due to a O-H bond, ar. Ji absorbance of 2220 cm-1 is d"ue to a C=C bond, and an IR absorbance of 7720 cn-7 is due to a C=O bond.

63.

Choice D is correct. With meta-substitution, all of the benzvlic hydrogens would be different, because --:* benzene compound would be asymmetric. There would thus be four nonequivalent hydrogens in a 1:1:1:1 ra-r:, Picking D is a beautiful thing on this question.
Choice D is correct. An ethyl group on benzene is composed of a CH3 group next to two equivaient hydrogi--r,., (making it a 3H triplet) and a CH2 group next to three equivalent hvdrogens (making it a 2H quartet). I:
describes choice D.

64.

Copyright @ by The Berkeley Review@

t74

Section II Detailed Explanatio

Choice C is correct. Because the compound contains two oxygen atoms, choice A (an aldehyde) and choice D (a ketone) are both eliminated. There is no peak in the IR spectrum that is indicative of an O-H bond, so the compound is not a carboxylic acid. This eliminates choice B. The only choice left is choice C.
,ffi.

Choice C is correct. The molecular ratio of hydrogens cannot be 3:1 in that region of the proton NMR spectrum, given the shift values. In order for there to be only one hydrogen on a carbon, there must be other groups ittached to that carbon. If the other group was caibot'r-based, ihere would be more signals in the IHNVIR spectrum than two and more signals in the 13CNMR spectrum than four. If the groups contained no hydrogen, then they would affect the shift value, making it farther downfield than 2.0 ppm. They are alkyl hydrogens, so the ratio is most likely 9:3, caused by the presence of equivalent methyl groups. Any alkyl group besides methyl would show coupling and not singlets. The structure must therefore have a tertiary butyl group (nine equivalent hydrogens forming a singlet) and an isolated methyl group (three equivalent hydrogens forming a singlet). Choose C for best results.

Choice D is correct. A 13CNMR peak is for carbon, not hydrogen, which eliminates choices A and B. The passage states that the carbon of the C=O bond is found at170.2 ppm., which eliminates choice C. The best answer is choice D. Carbons bonded to an electronegative atom that arc spJ-hybridized typically show a signal between 60 and 90 ppm.
tr!"

Carbonyl carbons are found between 770 and220 ppm in the 13CNMR. Choice A is an ester, choice B is a ketone, and choice D is an aldehyde, of which all three contain a carbonyl group. This means that choices A, B, and D can be eliminated, because they all exhibit a signal in the l3CNltR that is greater than 100 ppm. An alcohol has a peak around 70 ppm to 80 ppm for the alcohol carbon, but not greater than 100 ppm. The best answer is
choice C.

Choice C is correct. As stated in the passage, the peak at 170.2 ppm is attributed to a carbonyl carbon.

Choice A is correct. The shift value for hydrogens on an alpha carbon (the carbon adjacent to a carbonyl) is found to be between 2.0 ppm. and 2.5 ppm. The alpha hydrogens are described in the answer choices as hydrogens on a carbon adjacent to a carbonyl (C=O) bond. This is choice A; your choice for a question like this. Choice D is correct. A singlet in a 1HNMR spectrum occurs when there is an isolated hydrogen (or group of equivalent hydrogens), with no hydrogens on the neighboring atoms. This eliminates choices A, B, and C, and
makes choice D the best answer.

Choice D is correct. Compounds A and C are eliminated, because they do not contain two oxygen atoms, as is stated in the passage. Choice B is eliminated, because the CH3 of the methoxy group would show a shift value between 3.5 ppm and 4.0 ppm. The peak for the lone methyl group is found near 2.0 ppm, which indicates that the methyl group is adjacent to the carbonyl. The correct structure is choice D.

:a

Choice C is correct. The oxidation of a secondary alcohol to a ketone results in a carbon that goes from a i3CIrIptR signal of roughly 75 ppm to a signal of roughly 200 ppm. This conversion can be monitored.easily by 13CNMR, so choice A is valid. Nitration of ethylbenzene results in a carbon that changes from a TTCNMR signal of roughly 115 ppm to a signal of roughly 140 ppm,.lecause the nitro group changes the immediate chemical environment. This conversion can be monitored by 13CNVR, so choice B is valid. Deprotonation of a carboxylic acid. does not change the immediate environment (adjacent atoms) of any carb.oSr so no drastic change in change in any l3CNltR sifral is observed. This conversion cannot be monitored Uy isg*tR, so choice C is invalid, and thus is the best choice. Reduction of an alkene to an alkane results in a carbon that goes from a 13CNMR signal of roughly 100 ppm to a signal of roughly 20 ppm. This conversion can be monitored by l3CNVln, so choice D is valid.
Choice C is correct. According to the reaction in the passage, the compound is an aromatic ketone. The peak at 206 pprn, according to the data in Table 1, confirms that Isomer I is a ketone. Choice D (the aldehyde) is thus eliminated.. Becau-se there are only seven peaks for the 13CNMR, the compound must have symmetry, so choices A and B are eliminated. The best answer (one that contains only seven unique carbons) is choice C.
Review@

:i

J:n-right O by The Berkeley

175

Section II Detailed Explanations

74.

Choice A is correct. An aldehyde, according to Table 1, is found between 185 and 200 ppm, and no peak is fou_i.: in this range for either structural isomer. Choice A may be the corect choice. It stated in the pu5ug" that c,: occasion, the shift may not fall exactly in the range given. An alkene, according to Table 1, is found beiween 1- and 140 ppm, and there are peaks found in this range for both structural isomers. Choice B can thus i. eliminated. A ketone, according to Table 1, is found betr.r'een 205 and 220 ppm, and peaks are found in this rans= for both compounds. Choice C is thus eliminated. A methyl group, according to Table 1, is found between 10 a-r; 35 ppm, and there are peaks found in this range for both structural isomers. Choice D is thus eliminated. Tf.best answer is choice A.

/J.

Choice D is correct. According to the passage, the least intense peak is caused by a carbon with no hydroge:,. attached. This makes choice D the best answer. The carbons with no hydrogens attached in a disubstitute: benzene derivative with a carbonyl are two of the benzene carbons (the beniene carbons with a substituer: eliminates choices B and C. While it has sp2-hybridization, that is not the cause of its low intensity. Chor.: D is a better answer than choice A. This is a question that rewarded the test taker who sifted through t:information in the passage. This will happen on occasion, because even the science sections are reading u*u*..

attached) and the ketone carbon. It has sp2-hybtrdization, not sp3-hybridization, and

it is achiral.

Th-"

76.

Choice C is correct. Para-methoxybenzaldehyde has a total of eight carbons in its structure, but the molecu-. contains a mirror plane that reflects two pairs of equivalent carbons. Using symmetry, this means that the:= are only six unique carbons, so the best answer is six signals, choice C. The structure is shown below:

o\^-t
!u
I

.c/-\

CD

ilt oC-..

c.
Cd

eC

-Z

ocH3
Choice D is correct. Because the two compounds are isomers, they have the same molecular formula. Havrr: the same molecular formula results in having the same units of unsaturation. This eliminates choices A and E Each compound has a benzene ring and a carbonyl group. The benzene ring has three n-bonds and the one rir-r: There are four ur-rits of unsaturation due to the benzene ring alone, so choice C is eliminated. When the carbor.-. n-bond is accounted for, there are five units of unsaturation in the molecule, so the best answer is choice D.
-

lr

E
&F

78.

Choice B is correct. It is stated in the second paragraph of the passage that quaternary and carbonyl carbor.. generate peaks of low intensity. There is no carbonyl in the compound, but carbon 2 of 2,2-dimethylbutane ha-. four other carbons attached, making it a quaternary carbon. The best answer is choice B, carbon 2.

79.

Choice A is correct. Spectrum I contains a quartet (2H), a triplet (3H) (this combination is a dead give-an-a: for an isolated ethyl group), and a singlet (3H) (a dead give-away for an isolated methyl group)- The t',= question here is whether the ethyl group or methyl group is attached directly to the oxygen of the ester grou; The degrees of unsaturation (1) and number of oxygens (2) tell you the compound must be either an ester or : carboxylic acid. The lack of a broad peak between 10 - 12 ppm eliminates the possibility of the compound bef : a carboxylic acid, so the compound must be an ester. Because the quartet is so far ciownfield (at a higher sh,:value), the ethyl group is attached to the oxygen. This makes the best choice an ethyl group on the meth'. ester, whose common name is ethyl acetate. Choose A for best results.

80.

Choice B is correct. A triplet is the result of coupiing to the neigl-rboring hydrogens (there are two hydrogens c. the adjacent carbon in the case of a triplet). The integral (quantitr. of hvdrogens for the signal) has no effect c: the shape of the signal, meaning that the peak shape does not tell vou any information about the hydrogens cthe signal, only about the neighboring hydrogens. This eliminates choices A and C. A triplet is therefore th= result of neighboring a CH2 group. Choose B for a grade A, genuine, altogether correct, best answer.

Copyright @ by The Berkeley Review@

176

Section II Detailed Explanations

Choice B is correct. Spectrum II contains a quartet (2H), a triplet (3H) (this combination is a dead give-away for an isolated ethyl group), and a singiet (3F{) (a dead give-away for an isolated methyl group). Because the 3H singiet is found around 4.0 ppm, the methyl group is attached to the oxygen. This makes the best choice a methyfgroup on an ethyl ester. Choose B for the happiness of another correct answer. The drawing below lists how the name and structure are determined for the compound.
Singlet in the 3.5 to 4.0 PPm range

H;C-.
--J

/il

cHzcHj
propanoate

methyl

Nomenclature rules state that the alkyl group on oxygen is named first, followed by the ester chain' This makes this compound methyl propanoate.
Choice C is correct. This is one of those trivial facts that you should know. A peak in the neighborho od ol 7 ppm is indicative of aromatic hydrogens, which are found in benzene compounds. Pick C, to score bigl
Choice A is correct. A carboxylic acid has one proton that forms a broad peak between 10 ppm and 12 ppm in the 1nruVtR. The hydrogen in question is the acidic proton of the carboxylic acid. Because there is only one proton, the peak between 10 ppm and 12 ppm has an integration value of one hydrogen, so choices C and D are eliminated. An ester nui at'r alkyl group attached to the noncarbonyl oxygen of the ester. Protons on the first carbon from the oxygen have a peak between 3.5 ppm and 4.0 ppm. This eliminates choice B, leaving only choice A as the possible answer. The peak between 2.0 ppm and 2.5 ppm is the result of alpha hydrogens, which ot" preser-ri in both an ester and a carboxylic acid. 'lhis means that a peak between 2.0 ppm and 2.5 ppm cannot be used to distinguish an ester from a carboxylic acid. The correct answer is choice A'

Choice B is correct. 4-Heptanone is a seven-carbon ketone with the carbonyl directly in the middle- The structure is symmetric, so there are many equivalent carbons and hydrogens. There are four unique carbons, of which tl-iree contain hydrogens. This resulti in three signals in the 1HNMR for 4-heptanone. The best answer
is choice B.

Choice A is correct. For Compound II, the absence of a 13CNMR signal between 180 ppm and 230 ppm supports rHNMR spectrum the idea that it has no C=O group. This eliminates choice C. No peak above 5 pp* in the unsaturation in the confirms that there is no double bond. This eiiminates choice D. This means that the unit of 1HNMR spectrum suPPorts,the idea compound must be the result of a ring. The presence of a broad peak in the IrCNMR that the compound is an alcohol, eliminating choice B and making choice A the best al1swer. The cyclopentanol or 2,3spectrum shows that there is great symmetrv in the structure. The choices are either dlmethylcyclopropanol. The integral of the proton NMR says that there are mostly CH2 groups present, which iurro., ryclopentanol over 2,3-dimethylcyclopropanol. Cyclopentanol is drawn below:

Broad------>1tO

(1H)

H<\ /
CH"

Multiplet
(1H)

13cNMR,
There is a mirror planecutting throughthe molecule, so there are three unique carbons, resulting in three different signals.

H"C

,zt\

42c-

\,2 (4H) Tripiet

MuitiDiet
(4FU

Copyright

by The Berkeley Review@

Section II Detailed ExPlanations

86.

Choice D is correct. Each unique carbon within a molecule exhibits a unique signal in the 13CNMR spectrum, so choice A is valid. This eliminates choice A. A carbonyl carbon has a slgnaiaround 180 ppm in a 13CNVR spectrum, which b d+lf:=llom other signals in the 13CNMR spectru-. ih" presence of a carbonyl group can be identified using l3CNl'tR spectroscopy, so choice B is valid and thus elirninated. The substitution of a benzene ring affects the symmetry of the molecule. For instance, if the substitution is para, then there is a mirror plane in the molecule. This results in fewer unique carbons, which is seen with 13CNMR spectroscopy. This makes choice C a valid statement, and therefore it is eliminated. The geometry about a doulle bond, cis versus trans, does not express itself in l3CNltR spectroscopy. Typicaily, geJmetry is determined by looking at constants of the vinyiic hydrogens in 1HNMR spectroscopy. bhoice D cannot be determined using ll:::_"-p_t*g

E
91"

IrCNMR spectroscopy, so choice D is the best answer.

87.

and III cannot be carbonyl compounds, according to their UV data. If there was a carbonyl gro.rp on the molecule, there would be a n-to-n* transition around 190 nm and an n-to-n* transition around jOO n*. this eliminates choice B. Because Compound I has two UV absorbances and only one n-bond, it must be a carbonyl species of some sort, While it is not possible to decide between an aldehyde and ketone based on this information, choice C is a solid answer. Because Compounds II and III show no UV absorbance above 1,75 nm, there is no n-bond present. However, choice D refers to Compounds I and II, not II and III, so choice D is eliminated. Choice C is the best answer.
88.

Choice C is correct. According to the data in the passage, only Compound I has a UV absorbance above 175 nm. This means that only Compound I has a r-bond. There is only one unit of unsaturation, so Compound I can have, at most, one n-bond. This means that Compound I cannot be a conjugated diene, which eliminites choice A. If it were a conjugated diene, there would be a UV absorbance above 200 nm for the n to rc* transition. Compounds II

Choice D is correct. For Compound III, the absence of a broad peak in its 1HNMR spectrum confirms that there is no alcohol in the compound. This makes choice A a valid statement, and thui the incorrect choice. The absence of a peak around 180+ ppm in its 13CNMR spectrum confirms there is no C=O present. This makes choice B a valid statement, which eliminates it. The absence of an absorbance above 180 nm in the UV-visible spectrum implies that there is no n-bond ir-r the compound, confirming that the structure must be cyclic to account for the one unit of unsaturation. This makes choice C a valid statement, which eliminates it. The I3CNIrAR shows very few signals (only three), which implies that there is great symmetry in the structure. It must be a symmetric cyclic ether. This makes choice D an invalid statement, making it the best answer. Choice B is correct. Key features from each spectrum must be extracted. From the molecular formula, we know there is one unit of unsaturation and one oxygen. This means that the compound must contain either a ring, a C=C bond, or a C=O bond. All of the answer choices fit these criteria, so we must use the spectroscopic dita. the l3CNVtR data show a peak at 211 ppm (1), and that no two carbons are alike. This means that the compound is a carbonyl, which does nothing to eliminate any choices. If the compound were 3-pentanone, it would show only three signals in the 13CNVR due to its symmetry, which eliminates choice C. The fact that no two carbons are alike also eliminates choice D, which has two equivalent methyl groups. the 1HN\4R data shows a 3H singlet at 2.08 ppm, and no peak between 9 and 10 ppm. This means that the iompound is a methyl ketone and not an aldehyde, which elj.minates choice A and choice C. The ratio of the hydrogen signals in the lHNltn (3 :2 :2: 3) supports choice B. The structure is shown below:

89.

n"r/'T-.r'/
1HNMR,
13CNMR,

rrJptet

t' I t'

tV',.
t

ll

:v-vis:

18e

nm and 268 nm
PPm

CNMR:

21 1

(3H)

v,,rlipturrrlpt"t
(2H) (2H)

silgr"t

All five carbons

(3H) are different, so there are five different signals.

There can be no branching, because the proton NMR shows that the integral values are 3,2,2,3. This implies

that the structure is linear.

Copyright

by The Berkeley Review@

t7a

Section II Detailed Dxplanations

90.

Choice D is correct. Integration is used to determine the reiative quantity of hydrogens within a signal by looking at the area of the signal. Integration does not change with magnetic environment, so choice A is eliminated. The neighboring hydrogen atoms affect the splitting, not the integral, so choice B is eliminated. Integration does nothing to determine the presence of atoms other than hydrogen, so choice C is eliminated.
The best answer is choice D.

91.

Choice C is correct. Five hydrogens constituting a singlet with a shift value between 6.0 and 8.0 ppm indicates that the compound is a monosubstituted benzene. The three remaining hydrogens make up a methyl group. This now becomes a nomenclature question, rather than a spectroscopy question. The correct name for a methyl group attached to benzene is methylbenzene. The common name for methylbenzene is toluene. Choose C for optimal results. Choice D is correct. The formula contains six hydrogens in all, so the sum of the ratio values must equal 6. The first peak is shortest, the middle peak is the second tallest, and the last peak is the tailest. This means that the values must be ascending. The only combination of ascending values adding to 6 is 1:2:3. Choice D is the
best answer.

92.

93.

Choice D is correct. The compound 2-bromopropane has two unique types of hydrogens, so it has two peaks in its are seen as one signal with an integration of 6. The middle carbon (carbon 2) has one hydrogen, so it has a signal with an integration of 1. The peak shape is determined by adding 1 to the number of hydrogens on the adjacent carbons. The six equivalent hydrogens have one hydrogen neighbor, so there is a doubiet of integration 6. The one hydrogen has six hydrogen neighbors, so there is a septet of integration 1. Choice D is the best ansr.ver.

IUNUn spectrum. The two terminal methyl groups are equivalent, so they

9'1.

Choice C is correct. A quartet is the result of the observed hydrogens being coupled to three equivalent hydrogens. This is often the result of hydrogens that are adjacent to a methyl group on one side and no other protons on the other. The quartet hydrogens are in bold face, and the neighboring three hydrogens are boxed in the drawing below. Choice C is the oniy structure that shows no quartet in its proton NMR spectrum.
A. (H3C)2HC
C.

A cHr@

B.

(H3C)2HCH2C
95.

fnrburclHrcyrc o D.
H3CH2CH2 C

CH:

cHg

A cHr@

Choice D is correct. A doublet is the result of hydrogens on a carbon that neighbors a carbon with only one hydrogen attached (most easily recognized as a tertiary carbon). In each of the first three answer choices, the methyl group attached to the interior of the carbon chain is bonded to a carbon with only one hydrogen (a tertiary carbon), which results in every compound having a doublet with an integration of three hydrogens. This leaves choice D as the best answer.
Choice

q5.

A is correct. A positive iodoform test, as stated in the question, is caused by a compound with three aipha hydrogens on one carbon. This means that the iodoform test is positive for a methyl ketone, which

would have a CH3 group adjacent to a carbonyl (there are no hydrogens on a carbonyl). With no hydrogens on the neighboring carbon (carbonyl), there is no coupling and thus a the peak is a singlet. Pick A for the pleasure of correctness. The iodoform test works by removing an alpha hydrogen to form an anion. The anion subsequently attacks iodine, adding an iodide to the alpha carbon. This is repeated two more times, until there are three iodides bonded to the alpha carbon. The CI3 group is a great leaving group, and it forms a yellow, oily compound when protonated.

-opyright

by The Berkeley Reviewo

Section II Detailed Explanations

97.

multiplet made of many sharp peaks, not a broadened signal, which eliminates ciroice B. Choice C can be eliminated, because hydrogens on carbon do not form hydrogen bonds. the hydrogen forming the hydrogen bond
in this molecule is bonded to oxygen. This makes choice D the best answer.

Choice D is correct- The hydrogen responsible for the broadness is the carboxylic acid proton. It is bondecl to an oxygen, making choice A an incorrect answer. The couphng to eight or more other hvdrogens would result in a

Choice B is correct. Because the compound on the left has cis orientation about ttrre internal double bond, while the compound on the right has trans orientation abor-rt the internal doubie bond, the two compouncls must be geometrical isomers of one another. Pick choice B, and you won't be sorry. In case you were considering choice C, the two compounds have the same absolute configuration, so they cannot be optical isomers.
99.

Choice C is correct. Both double bonds are trans, so choice B is immediately eliminated. Both have the same connectivity, so that eliminates choice A. The last thing to check is the chiral centers, and each has just one chiral center. The compound on the left has R chirality, while the compound on the right has S chiralitr. This makes the two structures optical isomers, choice C. If you flip the structure on the right to align with the structure on the left, it can be seen that the chirai center has changed" A change in a chiral center results in an optical isomer.

,ars".s
100. Choice A is correct. The best way to do this probiem is the systematic counting of carbon backbones, starting with the iongest carbon chain possibie (five carbons). The tally for each possible carbon backbone is drar,r,r beiow. There are only three possible structures: pentane, 2-methylbutane, and 2,2-dimethylpropane. Tlr-c structures that are in fact structural isomers must have different IUPAC names. Pick A to be a correct anst'e:
picker person.
C
I

C-C
I I

Copyright O by The Berkeley Review@

Section II Detailed Dxplanations

Section III
Stereochemistry
by Todd Bennett

Molecule fits template! Rcenlelill>Y>a>l

Molecule does not fit template! Scenterilr>v>a>.J

Speciahztng in MCAT Preparation

TIEBITEIEY l) g. p- y.1.-6. u/'

Stereochemistry Section Goals


oB
Be able tq identify stereocenters and chiral com A stereogenic center (often referred to as a chiral center) is most commonly made up of a central atom (gsually carbon) wlth four unique substitutents aitached. The stere6centei ii'identified as R (lf,t1l : or S (Latin,. sinister) to define its orientatio" i.rpuce. g";;ti;;ilI t"qiiir" :,it-h.* lec,tusl that you identify the number of chiral centers and often label them accordini to convention.'The cite.d example of a,non-t1.pical situation involves allene, which has sf-hybridized carbons lSguentlY
that can be chlral. An example of allene enantiomers is drawn below:
H
H

&
CI

Ntrt,,,-

ct/

-i,n_^_^

cl

a?

Be able to and stereoisomers. Youmustunderstandthedifferencesbetweenenantiomersu''da op.tica] inactivity 9"*""*.'.fl_1":"_.::Tf?.yf:ji9T lh"toanh epimers- M<istand structure. iL" casesinvol,vi"g such as anomers importantly, when :"g?.t: be able to identify

rnlJi't i-'o,' special giu; irvo their relationship if they are'stereoisomers" of onestrucLures, another.

Be familiar with common exam of chiral molecules. It should be as second nature to yog that sugars occur naturally in the D form, *fli.f-r defined bv having the penultimate carbon wittr R sterJochemistry. The iypical exception ir ;;;;ith ;t;A " Vpes where one of the sugars in the antigenic determin-ant is L-fticose. lt shbuld be as second nahure oicur naturally "in the L form, which is defined by il;ing the;lpd;";il; l.o,y,o:1la1a1ino.acids ylti-::1"^r:o-:,1:iil]V Jexqep,t for cysteine). A typical exception is seen wi[h transiriptidase enzymes
wnere the acttve amtno acrd is D-alanine.

@?

an unknorun JustIiketheboilrngpointandthemeltingpoint,theopticalrotationi'upnyffi uyrrldr rurdr.rurr ls d pftyslcal pl.opeftv tnat can l"'""'; _"': :"'::.,6 Deused to Identlty a molecule. The optical rotation is a measurement of the rotation ol .r nlan,,d df t'he rotalion o1, pti""J ::,T:9P,i1""l,ilt: i?],"jq"ofIhe opti"cal,rotation .o*po.rtld. A;.-;;;;;ppli;"ti;"'il;h; p,."hlt::1,!,qht bi a solution the cipticatly active ldentrtlcatron an unknown susar. sugar.
rrrL rrrcrrrrrS PUrrrr, .rrE

Be able to use

rotation data to i

Be able to disti

ish

ic substitution mechanisms.
)il[*

There are two mechanisms for nucleophilic-substitution that you must know. fft" fitst ir tt-r" Slryf ancl and the second is the S1x2. They are deTined bv the number of leactants in the rate cletermininc, step Sg2. aredefined ieactants determining sfen of the mechanism. You.must 6e able io predict the reaction from the l.ltiri.."ailio"s, t"co"g"ir'" the reaction from the intermediate or trahsition state, and identify the reaction it;;ih6 p;;f,;;r. The differences include solvent, strength.of leaving group, sterii hindrance, ability to slabilize a carbocation intermedia te, and stereoch"emistry.

llili:li

ilit i:

lh*:

:!*

iX!:tlt

Be able to recognize typical nucleophiles and leaving groups.


You must recognize what makes a go-od leaving grgup, and what effect this has on the reaction. The strength of a lehvi leaving group is d dependent onthe solvent and can be qredicted from the acidity of the conjugate acid of tlieleaving gtg*p: Equally, the strength of anuileopl-rile can be predictdd from p the Daslclty ot rne basicitv of the nucleophilel Agpin, dolvaiion and st&ic hindrance nucleoohilel
a
:tLlt

nucleophile.

it"y;;;l";ih;;;"dil;l

tllgli

r*

&r& -[gfi!

$Ima:
|]t&i0![

ilUMll'r tumrrn

Organic Chemistry

Stereochemistry

Introduction

stereochemistry involves the asymmetry of a molecule. we can consider the asymmetry as a whole or the asymmetry about specific atoms in the molecule, most often carbon. If there is asymmetry within a molecular structure, the compound's reactivity, physical properties, and stability are all impacted. The study of stereochemistry has direct implications in the biological applications of
molecules.

In this particular section we shall address the concept of configurational


with ail isomers, configurational isomers have the same atoms within the molecule, but they differ in some manner so that the molecules are not
superimposable on one another. No matter how a compound is rotated or contorted, it is not superimposable on its configurational isomer. As a consequence of their different configurations, one configurational isomer may have the correct arrangement of atoms to offer minimal steric hindrance in a chemical reaction with another asymmetric molecule, while another configurational isomer proves to be too stericaily hindered on one side to undergo reaction. This is frequently seen with enzymatic chemistry, where enzymes have several stereogenic centers and are highly specific about which configurational isomer can bind and undergo a reaction.
isomerism and the many different classifications of configurational isomers. As

Configurational isomers can be categorized as either optical isomers or geometrical isomers. optical isomers rotate plane-polarized light while geometrical isomers are structures with limited rotation. In addition to that categofization, configurational isomers can also be categorized as either enantiomers or diastereomers. Enantiomers are nonsuperimposable mirror images while diastereomers are nonsuperimposabie structures that are not mirror images. The two categorizations are not mutually exclusive; meaning a pair of configurational isomers could be enantiomeric optical isomers, diastereomeric optical isomers, enantiomeric geometrical isomers, or
diastereomeric geometrical isomers.

In this section we shall also address nucleophilic substitution. We will consider the two mechanisms for nucleophilic substitution: the S511-mechanism and the Sp2-mechanism. In a nucleophilic substitution reaction that proceeds by an SN1mechanism, the leaving group leaves to form a carbocation intermediate before the nucleophile attacks. An S511-reaction has a unimolecuiar rate-determining step. In a nucleophilic substitution reaction that proceeds by an Sp2-mechanism, the nucleophile attacks the electrophile from the opposite side of the leaving group and forces the ieaving group off of the electrophile. An Sy2-reaction has oniy one step, a bimolecular step. We will compare and contrast the conditions and features of an Sp1-reaction with that of an S52-reaction to establish a set of criteria you can use when deciding which mechanism (Sp1 or Sp2) is applicable for a given nucleophilic substitution reaction.

We will consider the impact of stereochemistry on reactant interactions, transition state formation, and product distribution. We will present the basic tenant that if the reactants are optically active, then the product mixture is likely
optically active, and at the very least enantiomerically rich in one configurational isomer (possible a geometrical isomer, which is optically inactive.) We will also consider enantiomeric distribution in a product mixture and discuss ways to increase the optical purity.

Copyright

by The Berkeley Review

r85

Exclusive MCAT Preparation

Organic Chemistry

Stereochemistry

Configurational Isorners

Confi gurational trsorners


Stereochemistry Stereochemistry centers around the formation, orientation, and reactivity of molecules with stereogenic centers, referred to as stereoisomers. The molecules we shall consider in this section are configurational isomers.

Configurational Isomers Configurationai isomers have identical bonds, but they have a different spatial arrangement of their atoms, no matter how the structures are contorted. Common examples, with which you are familiar, are optical isomers. Opticat isomers, due to their asymmetry, rotate piane-polarized light. This is used as a diagnostic test to identify a specific configurational isomer. We shall first look at asymmetry and chirality, as configurational isomers are based on chirality. Asymmetry A molecule with asymmetry has a site about which there is uneven distribution of the bonded atoms. Analyzing symmetry is critical, because at least one of two stereoisomers must be asymmetric in some manner if the two structures are not superimposabie. To understand stereoisomers, it helps to be familiar with mirror plane symmetry and chirality (molecular asymmetry). Figure 3-1 shows the
asymmetry of carvone and the symmetry of 2,2-dichloropropane.
C1

-H5

Hsc
No plane of symmetry, so the compound is chiral. Figure 3-1

CHs

Plane of symmetry, so the compound is achiral.

Chirality
Chiral is the term assigned to a molecuie with no plane of symmetry, therefore a chiral molecule has an asymmetric structure. Simply put, chirality is the "left and right handedness" of a molecule. From our perspective (keeping at the leve1 of this test), a chiral molecule has at least one stereogenic center present. A stereogenic center is an atom within the chiral compound^that has asymmetrrabor.rt it. For our needs, chiral (asymmetric) carbons are sf -hybridized carbons with four unique substituents attached. Within 2-chloropentane there is one stereocenter (chirai carbon), as emphasized in Figure 3-2 below. Figure 3-2 shows the two configurational isomers (enantiomers) of 2-chloropentane in such a manner that the two structures are mirror images of one another. The plane mirror reflects a configurational isomer as its image.

mlrror

-;---plane

-'CHTCH2CH3

I '
I

-_____;;__.v,1:
I-ug" cl
sp3-hyb.idirotitor-t

'tLl

:tl

vith four different groups attached.

Figure 3-2

&n

Copyright O by The Berkeley Review

The Berkeley Revieu

Organic Chemistry

Stereochemistry

Configurational Isomers

What makes this important is that an atom with four unique substituents attached has two possible ways that the substituents may be connected (which ate mirror images of one another). The two structures, mirror images that have
identical bonds, are stereoisomers that may exhibit different chemical properties despite identical physical properties as one another. The biological ramifications of chirality are important. For instance, humans digest only D-sugars (D refers to one of the two possible stereogenic orientations associated with the penultimate carbon within a sugar backbone), because enzymes bind and react only with Dsugars. If this seems unclear, the following everyday analogies may help: a) A soew with right-handed threads does not fit into a left-handed nut. b) Your right hand does not fit rnto a left glove. :) A key with a groove on only one side and its mirror image do not open the
same iock.

Exampie 3.1 shows some examples of pairs of butane moiecules that have four 'l1ique substituents on carbon two. The goal of the question is to develop skills :or quickly recognizing when two structures represent conformational isomers ciscussed in section II) versus when they represent configurational isomers.

Example 3.1

,ihich of the following structurai pairs represents the same molecule and not a :air of configurational isomers?

\.
F{3CH2C
D D.

&

F{rCHrC&I
H

OH

irL !Br
i

&

cH2cH3

""-&rx,.,,,
cH2cH3
CHg

-,./YcHrcH3&
D.
cH.cH"

CH"Cl
&

:ul2clY/H
! olution

l'J

H3cH2clY/H

t-

- :u must decide whether the compournds are either enantiomers or the identical ,:r-Lpound with different spatial orientation. Rotate the molecules in your mind ' - see if the atoms overlay. If you do this successfully, then you will see that they =:e identical only in choice C. However, if this is hard for you to visualize, try a .=: of models. A shortcut you may recognize is that when two of the substituents ':e interchanged, the chirality of that stereocenter changes. In choices A, B, and l' two substituents have interchanged, making them enantiomeric pairs.

-:pyright

O by The Berkeley Review

r85

Exclusive MCAT Preparation

Organic Chemistry
We

Stereochemistry

Confi

gurational Isomers

will start with a traditional approach to stereochemistry probiems and then slowly work our way into shortcuts and visualization tricks. The first pair of molecules in Example 3.1 are mirror images of one another, because if you rotate either structure by 180" about its carbon 1 - carbon 2 bond, the structures are not superimposable. According to our shortcut, when tlvo of the substituents interchange their locati.ons (H and I in this case), because there is only one chiral center, the tr,r'o molecules are mirror images of one another. Mirror images that are non-superimposable are defined as enantiomers. The substituent exchange shortcut shoulcl make it easier to recognize enantiomers.
The molecules in choice C in Example 3.1 are identical. If the left structure is rotated counterclockwise by 120" about the carbon 2 - carbon 3 bond (as shown in Figure 3-3), the identical structure and orientation are formed. Note that when a structure is rotated by 720" about a bond, the three other substituents interchange their locations on the molecule. The conclusion from this is that when three substituents are different from one structure to another, those two structures represent different conformational isomers (orientations) of the same compound.
C1
I

zGH:-

BrlYcH2cH3
Figure 3-3

Rotating molecuies in your mind becomes easier with practice, although if the never fully developed, you can stil1 answer stereochemistry questions br following a few simple rules. If two groups are interchanged and the rest of the molecr,rle remains in place, then the two structures are configurational isomers. If there is a mirror plane between two molecules and no mirror plane within the molecule, then the two structures are configurational isomers.
ski11 is

In addition to the symmetry of a compound with respect to another molecule.


there is also internal symmetry to consider. For internal symmetry, you can look for either a plane within the molecule that reflects equal halves, or an inversiorr. point. An inversion point is a point at the molecule's center of mass through which a line passing through that spot will intersect the same atom at the same distance, no matter which direction you proceed (positive or negative direction on the 1ine. Within asymmetric molecules, each stereogenic center is assigned a letter, R or S, to describe its stereochemistry.

lx

;
1i

l
s
llr

lr

Determining Absolute Configuration The identification and naming of a chiral center is based on nornenclature convention. There is a set of guidelines, the Cahn-Ingoid-Prelog rules, to follol for determining R and S for a stereocenter (chiral carbon). The Cahn-IngoldPreiog rules to determine the stereochemical orientation (R or S) are as follows:

"i
'11

J
:tr"r

1) First, you must prioritize (from heaviest to lighiest) the substituents that

are

2) 3)

attached to the carbon of the stereocenter according to the atomic mass of the atom directly bonded to the chiral carbon. (Get it from the periodic table) Second, you must orient the molecule in such a way that the substituent wil;. priority number four points behild the plane of the molecule' Third, you must draw a circular arc from substituent 1 through 2 and on to l If the arc is clockwise, it is referred to as R. If the arc is counterclockwise, it r.

$."
l:t

lill lr ,

iut!1,

zu.

referred to

as S.

;i.*

Copyright O by The Berkeley Revierv

ra6

The Berkeley Revieu

'l,r

Organic Chemistry

Stereochemistry

Configurational Isomers

R is from the word rechts, which is right in Latin, while S is from the word sinister which is left in Latin. If you point your thumb in the direction of substituent number four on a compound with R-stereochemistry, the fingers of your right hand will curl from one to two and on to three. Thus, R-chirality can be thought of as right handedness. The same holds true for your left hand with an s-center.

Figure 3-4 shows a generic R molecuie and a generic s molecuie as oriented according to convention. The steps of rotation presented, take the structure from a standard view to a view with the fourth priority substituent eclipsed, to the Newman projection from which the stereochemical identity is derived.
1

Reposition by

rotating by 90" about C.6irot-C1 bond

Redraw to
see

_______>

chirality

Clockwise: R
Reposition by rotating by 90' about
C.51.o1-C1 bond 3

Redraw to
see

chlrality

---------->
Counterclockwise:
S

Figure 3-4

Knowing the terminology is key; it is recognition, not recail, that is emphasized on a multiple-choice exam. Here are some nndified definitions of common terms. Chiral Center (stereochemical center): A carbon with four unique substituents attached. Any carbon with four unique substituents has two different orientations that it can assume (R and S). What is neant by "unique substituents" is not four different atoms, but four unique $oups including the atoms attached to the four atoms bonded directly to carbon. For example, carbon two of 2-chloropentane (see Figure 3-2) is chiral, because it ras a chlorine (priority 1), a propyl group (priority 2), a methyl group (priority it, and a hydrogen (priority 4)bonded to it. These four substituents are different :rom one another, therefore they are four unique substituents.
R-center:

-\ carbon center that when you look down the bond from the chiral carbon to the :ourth priority substituent (usually a C-H bond) in a way that you can't see the iourth priority substituent, the remaining substituents form a clockwise arc ,r-hen moving from priority one to priority two and on to priority three according io the priority rules. This can be thought of as a right-handed molecule when llacing your thurnb in the direction you're looking and curling your fingers to natch your right hand to the structure.
S-center:
-1,

carbon center that when you look down the bond from the chiral carbon to the :ourth priority substituent (usually a C-H bond) in a way that you can't see the :ourth priority substituent, the remaining substituents form a counterclockwise :rc when moving from priority one to priority two and on to priority three :ccording to the priority rules. This can be thought of as a left-handed molecule '";hen placing your thumb in the direction you're looking and curling

your

:rngers to match your ieft hand to the structure.

Copyright

by The Berkeley Review

ta7

Exclusive MCAT Preparation

Organic Chemistry

Stereochemistry

Configurational Isomers

Prioritizing Substituents to Determine R and S To prioritize, first you must look at the four atoms directly bonded to the asymmetric carbon. You then rank those atoms according to their atomic mass with the heaviest atom taking ihe highest priority. if two atoms are equal (as is often the case n,ith carbon) you must continue down the molecule following the bonds outward from the chiral center. Figure 3-5 shown below presents examples with the priorities labeled on the molecules.
3

f', Ir
n4$'r sI
1

Tt'
n,

23

!n'
CH2CH3

T"'
C-C-C-C C{

"r'*'it

H,cH,tA';;, "'t-t. "tY:.d ::,"=


Br>C=C>H t CHs
C=O > C=C >
>

I>Br>C>H O>C>D>H

C-H

Figure 3-5 Shortcut to Determine R and S As with so many other topics in organic chemistry, such as nomenclature, R and S questior-rs become easy and redundant with time. Once they become easy, there are useful quick tricks to help you to identify chiral centers as being either R or S. For instance, when the fourth-priority substituent is sticking out from the rnolecule, the molecule must be rotated. To save time, it is easiest to first solve for the arc using the structure as it is, and then take the opposite chirality for the center. In the interest of saving time, this works well for use on tl're MCAT. Substituent number four can either be behind the plane, in front of the plane or in the plane. In each case, there is a technique to apply to arrive at the chiral center easily. Many techniques shall be presented, so choose your favorite. Figure 3-6 shows how to get the chirality easily when the structure is drawn irr the conventional manner.

,l,.i(f-},.t-,,
t, Ho
H
H in back of plane .'. Take whai you observe as is. Clockwise Arc = R

'(')t4

H?CH2

H"

H in front of plane

.'.

Reverse what you observe. Clockwise Arc reverses to S

H3CH

CHs H in plane close to back group


Take what you observe as is. Ctlunterclockwise Arc = S Reverse what you observe. Clockwise Arc reverses to S

iY)
HO

.'.

Figure 3-6

Copyright

by The Berkeley Review

The Berkeley Review

L{._

Organic Chemistry
Example 3.2

Stereochemistry

Configurational Isomers

The following molecule has what type of chiral orientation? -

H"C ,\

Ho$tyc
H

cH2cH3

A.R B.S C. The molecule has no chiral center. D. The compound is meso.
Solution
The compound has one chiral center, so it cannot be meso (to be meso requires an even number of chiral centers). The compound is chiral, because carbon two has

four different substituents attached it. The molecuie is therefore either R or S. The priorities are oH > CH2CH3 > CH3 > H. Correct arignment of the
in front, the arc should be reversed. Pick A for best resuits.
Priority +S HsC
Priority #2

substituents shows that the compound has an R chiral center. A counterclockwise arc connects priorities 1,,2 and 3. Because the H (priority 4) is

Priority#1

HO\\)
H
Priority

CH2CH3 Priority #4 in front


Counterclockwise = R
#4

r'\4rat is the chirality of the triol below according to the Cahn-Ingold-preiog rules?

Example 3.3

CH"OH

OH

OH

{.

2R,35,45,6R B. 2R,3R,4R C. 2R,3R,43 D. 2R,35,45

Solution larbon six is not a chiral center, because there are two methyl groups present lhis eliminates choice A. The chiral centers are 2R, 35, 45 (choice D), as shown.

CH"OH

V<

cH2oH
4S

oH oH

CHs
Counterclockwise Arc and #4 in back .'. take as S Clockwise Arc and #4 in front ,'. reverse to S

Clockwise Arc and #4 in back .'. take as R

-,:pvright

by The Berkelev Review

ra9

Exclusive IvICAT Preparation

Organic Chemistry

Stereochemistry

Confi

gurational Isomers

Example 3.4 Which of the foilowing compounds have R orientation?


C1
I

HgN*

HOH2C
H

nrcnrc4/;cn,
Compound I

Compound II

CH2OH

Compound III

HOH

A. B. C. D.

Compound T only Compound II only Compound III only Compounds I and III only

Solution
Hydrogen points out in each of the compounds. Whichever arc is seen from this view must be reversed to get the arc that would be seen from the correct view. The priorities in Compound I are: Ci > CH2CHg > CHe > H. Compound I has an R chiral center. The priorities in Compound iI are: NH3+ > COZ- > CH2OH > H. Compound iI has an S chiral center. The priorities in Compound III are: OH > CHO > CH2OH > H. Compound III has an R chiral center. Choice D is best. Compound I
(2-chlorobutane)

Compound II
(Serine)

tcl

Compound III (D-Glyceraldehyde)

o
3

HOH2

H3CH2

H3

oClockwise Arc and #4 in fiont .'. reverse to S Counterclockwise Arc and #4 in front .'. reverse to R

Counterclockwise Arc and #4 in fiont .'. reverse to R

Example 3.5 What is the stereochemical orientation of the foilowing molecule?

"\-

r,.J)y'-'a.,
A. B. C. D.
2R,3R 2R,35

Eil

.CH,

23,3R
2S,3S

Solution
For the first chirai center (carbon 2), the fourth priority (hydrogen) is in the plane close to the group in back (a reversing position). An arc from priority one to two and on to priority three is counterclockrvise. However, because H is in a reverse position, the chirality is R. The second chiral center (carbon 3) has H in front of the plane, so it is in a reversing position too. An arc from priority one to two and on to priority three is clockwise. Horr-ever, because H is in a reverse position, the chirality is S. The best answer is thus 2R, 35 which makes choice B correct.

:i
-s

lt*
Il

i,:
lr:

Copyright O by The Berkeley Review

r90

The Berkeley Review

Organic Chemistry
If Priority #4 is in the plane far from group in back, then take arc as is

Stereochemistry

Configurational Isomers

Figure 3-7 shows a summary of the tricks presented in Figure 3-6 and applied in Examples 3.2 through 3.5.

If Priority #4 is in the plane near group in back, then reverse arc.

If Priority #4 is in front,
reverse the arc to opposite.

If Priority #4 is in back,
then take arc as is.

Figure 3-7

An alternative short cut in determining the chirality of a compound involves interchanging substituents to generate an easy structure to solve, and then
assigning the opposite chirality to the original molecule. This method is based on the idea that it is easiest to solve for chirality when the fourth priority substituent is in back and that when two substituents are interchar-rg"a, tn" chirality is rnverted. Figure 3-8 shows the application of this method.

HOH2C<

cHs

HoH2r<

H. J

'@
Priority #4 not in back
...

,
Priority
#4

'HO

lHtz

H*
.'.

now in back of plane

interchange #4 and group in back

New structure has a ciockwise arc, so it is R The original compound must be S

CH"OH
J

H2OH

Priority #4 not in back

...

lnterchange #4 and group in back

Priority #4 now in back of plane .'. New structure has a clockwise arc, so it is R The original compound must be S

Hgcl ,
HOH2C
Priority #4 not in back
.'.

Hscj
2

1oH

HOH2c
Priority #4 now in back of plane .'. New structure has a clockwise arc, so it is R The original compound must be S

lnterchange #4 and group in back

Figure 3-8
These are two-dimensional tricks that may be done on paper. There are other ticks that involve contorting your hand to model the molecule. No one method is more accurate than another is, so once you find the one you prefer, hone it in and use it. We will cover the three methods in class. One involves pointing in the direction your eye should be looking, and forming an arc that goes from oriority 1 to 2 to 3. Another involves using your fingers to represent the bonds in the molecule. And as mentioned earlier, one method involves matching your

thumb and curling fingers to Copyright


@

a molecule.

by The Berkeley Review

l9r

Exclusive MCAT Preparation

Organic Chemistry

Stereochemistry

Confi

gurational Isomers

Optical Rotation Optical rotation is a physical measurement of the rotation of plane polarized light by a solution with a chiral molecule. A soiution containing a pure compound of known concentration (dissolved into solvent) in a standardized cuvette rotates piane-polarized iight the same amount each time. Consequently, the specific rotation (optical rotation under specific condiiions) is a physical measurement (like melting point and boiiing point) that may be used as a diagnostic test for the identity of a compound. This is common with sugars which have multiple chiral centers. The direction of the rotation is signified by either (+) or (-) orientation foliowed by the degrees of rotation. The (+) denotation describes clockwise rotation of light while the (-) denotation describes counterciockwise rotation of light by the molecule. If the R-enantiomer of a compound lotates the light in a positive direction by X", then the S-enantiomer of the cornpound rotates light by X' in the negative direction. The measurement is taken with a polarimeter which is made from a sample tube sandwiched between two polarizing plates' The plates are rotated in a manner to allow for the greatest amount of iight to pass through. If the plates are rotated 90' away from the orientation with highest intensity, then the new orientation of the plates does not allow for any light to pass through. A sample polarimeter is shown in Figure 3-9 below.
Solution of Chiral Compound

light enters

light rotates

rotated light exits

^nd porarlzet z

Figure 3-9

Chiral molecules may be assigned a "+" or "-" preceding their name to indicate the direction that the compound r,r'il1 rotate plane polarized light. For instance. (+)-2-butanol rotates tight in a clockwise direction while its mirror image, (-)-2butanol, rotates light in a counterclockr.r'ise direction. There are compounds with R-stereochemistry that rotate plane polarized light in a clockwise fashion anC other compounds with R-stereochemistry that rotate plane polarized light in a counterclockwise direction. Consequently, R stereochemistry does not necessarily correspond to either (+) or (-) rotation of plane polarized light. Ir biochemistry, the designation of D and L is based on threose, where (+) was originally assigned to D-threose and (-) rvas assigned to L-threose. Flowever. because of the vast multitude of sr-rgars, there is no correlation between D and L designation and (+) or (-) rotation oi pl3ns polarized light. Copyrighi O by The Berkeley Review

t92

The Berkeley Revieu

Organic Chemistry

Stereochemistry

Configurational Isomers

Having chiral centers does not always result in the rotation of plane polarized light. Meso compounds have opposing chiral centers that cancel one another out, resulting in no net rotation of plane polarized light. Meso compounds are
therefore opticnlly innctiue, meaning they have a specific rotation of zero. Polarimeters measure the optical rotation of a soiution. The specific rotation of a compound, [a]p, is calculated from the optical rotation using Equation 3.1.

[o]5

observed rotation in degrees

(length of sample in dm)(concentration of sample it-r

ItuTt)
mL

(3.1)

As seen in Equation 3.1, the standard cell length of a cuvette is 10 centimeters (one decimeter), and the standard concentration is 1 gram solute per milliliter solution. The superscript T refers to temperature as measured in Celsius and the subscript D refers to monochromatic light from a sodium lamp, known as the Dband. Specific rotation is the optical rotation observed under specific conditions. If the solution is too concentrated, then the rotation of tight is greater than it should be. If the solution is less concentrated than standard conditions, then the rotation of light is less than it should be. For this reason, the observed optical rotation is converted to specific rotation to determine purity. Types of Configurational Isomers Until now, we have focused on the chirality of molecules and their stereogenic centers. A moiecule is a chiral molecule when it is asymmetric. However, often times, a molecule can be asymmetric in more than one way. There are terms that describe the relationship between two stereoisomers. The relationship requires determining whether the structures are mirror images and whether they are superimposable. Configurational isomers can be classified as either enantiomers or diastereomers. Enantiomers and diastereomers have the same bonds, but a different spatial orientation of their atoms. Enantiomers are configurational isomers that are nonsuperimposable mirror images (reflections that you can't overlay). Diastereomers are configurational isomers that are nonsuperimposable and that are not mirror images. Thus, to classify a pair of configurational isomers as either enantiomers or diastereomers requires evaluating whether the structures are mirror images and whether they are superimposable. There is a wonderful short cut for determining whether two compounds are
enantiomers or diastereomers. Consider a molecule that has two chiral centers, R and R. To be mirror images, ail of the chirai centers must differ, because each chiral center must switch when it is reflected (just as a left hand in the mirror turns into a right hand). If you were to place an R, R molecule into the mirror it rvould reflect an S, S molecule, so the S, S molecule is the enantiomer of the R, R

molecule. This means that to be enantiomers, all of the chiral centers must differ between the two configurational isomers. If no chiral centers differ, then the two structures are identical (the same molecuie). If one of two stereocenters differs, then the two compounds are neither mirror images of one another nor the same molecule. This makes the two compounds diastereomers. If only a few, but not all chiral centers differ, then the compounds are diastereomers. Listed below are the modified definitions of enantiomers and diastereomers. Enantiomer; Enantiomers are configurational isomers in which centers in each molecule are different from one another.

all of the chiral

Diastereomer: Diastereomers are configurational isomers in which at least one, but not all of the chiral centers in each molecule is different from one another. Copyright
@

by The Berkeley Review

r93

Exclusive MCAT Preparation

Organic Chemistry

Stereochemistry

Configurational Isomers

These modified definitions should prove to be easier to use than the traditional definitions. Recall how these definitions were derived and they will be easy to remember and apply. Figure 3-10 shows two pairs of enantiomers and Figure 311"

shows two pairs of diastereomers.

HgC.

8,,'f Vott
Enantiomeric pair because both'of the chiral centers are different

\J

/cF{2cH3

HsC

cH2cH3

rl

&

H'v

ffi

Enantiomeric pair because both bf the chiral centers are different Figure 3-10

{m

s
lt

cH2cH3 &

Diastereomers, because only one of the chiral centers (the left one) is different

A,

ocH2cH?

OCH"CH" J
-.--

Ht

rt

Diastereomers, because only one of the chiral centers (the right one) is different

Figure 3-1L

After practice (and thus on your exam), you should be able to just scan structures to look for interchanged substituents (chiral centers that are different.) First, look for any chiral centers (sf-carbons with four unique substituents). From that point, compare the comparabie chiral centers in the two structures. If ttr structure is oriented in a similar fashion, but two substituents are in differsfr positions, then the chiral center is different between the two compounds. If ttrc structure is oriented in a similar fashion, but three substituents are in differeril positions, then the chiral center is the same between the two compound.* Finally, it is a matter of deciding i-f a1l, some, or none of the chiral centers on tlu two molecules are different and then determining their relationship. Copyright
@

by The Berkeley Review

t9,4

The Berkeley Revien

Organic Chemistry
Example 3.5

Stereochemistry

Configurational Isomers

Ihe following pair of compounds is best described

as

which of the following?

\-JYrort H.,c$\'/
HH A. Diastereomers B. Enantiomers C. Identical achiral compounds D. Identical chirai compounds
Solution

cl.

c(cH3)3

"v
Ci

H1C " \___./

C(CH.):

'

\r"
OH

\Vhen the orientation of the molecule remains constant and three substituents :hange their location, this implies that the compound has been rotated about that chiral center. The left chiral center is just rotated between the two compounds, thus it has the same chirality. When two substituents interchange their location, this implies that the chiral center changed. The right chiral center has changed, because the H and aldehyde group have interconverted. This means that only one out of the two chiral centers has changed its orientation between the two structures. The two compounds are therefore diastereomers of one another,

naking choice A the best answer.


Example 3.7 F{ow can the relationship of the following pair of molecules be described?

CHs

-{. Diastereomers B. Enantiomers C. Identical achiral compounds D. Identical chiral compounds


Solution
ln this example, the mirror plane between the two molecules can be seen easily
as

lhey are drawn. So without rotating or counting chiral centers, the two :ompounds can be identified as enantiomers of one another. Enantiomers are

nonsuperimposable mirror images. Choice B is the best answer. Example 3.8


The following pair of isomers is best described as which of the following?

HO

HHO.H
&
OCH3

Hrcrsf \
H

\-,)cH,

f",.", ocH3

A. A pair of anomers B. A pair of constitutional isomers C. A pair of diastereomers D. A pair of enantiomers

Copyright

by The Berkeley Review

r95

Exclusive MCAT Preparation

Organic Chemistry
Solution

Stereochemistry

Confi

gurational Isomers

The two structures are not aligned in an equivalent fashion, so one of the trt'o structures must be rotated into a structure equivalent to the other structure.

J";
H"C

#'
OCH?

H\

Itwo 60' rotations)

"t.&--J" / poc",
H
CHe

ro'&_/- -z
H

&
H

\ao.tt, CHs

ocH3

After rotating the structure, it is easier to see that the left chiral center has tn'c substituents thut huu" interchanged between the two structures, thus it ha-' changed chirality. The same is true for the right chiral center after rotation, thus it toJhas changed chirality. This means that both chiral centers have changed, sc the compoundi are e.rut iio*"ts. There is no need to determine the chiralitv c: the stereocenters (R or S) within molecules to determine whether they are enantiomers or diastereomers. Deciding whether two molecules are enantiomers or diastereomers is as easy as asking whether aII of the stereocenters or just sonr of the stereocenters have changed their orientation between the two compounds' (All centers differ = enantiomers; Some centers differ = diastereomers)
Example 3.9 The following molecules are best described as:

(I

r(

Hsc
Br

CHs

HeC

CHs

H
CI CHc

&

Br
C1

H H

A. diastereomers. B. enantiomers. C. identical achiral comPounds. D. identical chiral comPounds.


The structures are drawn as Fischer projections, which represent the top vierr the all eclipsed form of the molecule. In a Fischer projection, the side gloups coming olrt ut you in the three dimensional perspective,. The two isomers h

Solution

two c(iral ."r-tiuru. Only the chiral center with the chlorine is different r'r' comparing the two compounds. The chiral carbon with bromine has changed, t".urrr" the subitituents have not moved' Only one out of the
the best answer.

chiraf centers di{fers, so the compounds are diastereomers. This makes choice

Copyright

by The BerkeieY Review

r96

The BerkeleY

Organic Chemistry
Example 3.10

Stereochemistry

Configurational Isomers

Ihe physical properties of the follolving stereoisomers relate in what way?

cH?oH

H.c{ou

CH:
&

Br-+H
I

CH:

CHs

\. Same boiling point; different melting points B. Same density; different boiling points C. Different boiling points; different melting points D. Different densities; same boiling point
Solution
The chiral center on carbon 2 is different between the two structures, because the

CH3 and OH are interchanged. The chiral center on carbon 3 is not different

:etween the two structures, because the H, CH3 and the Br have all ,nterconverted. When three substituents interconvert, the chiral center is not :hanged. This means that one out of two chirai centers differ, so the two :ompounds are diastereomers. Diastereomers have different physical properties rcluding melting point, boiling point, and density. Pick C for best results.
Example 3.11

lhe following pair of isomers is best described by which of the following terms?

H:cr'''1)''\$oH

r,.r,"'Q
\.
B.

",tt

aH

Anomers

Diastereomers rl. Enantiomers D. identical meso compounds

Solution -ne two compounds are mirror images of one another, so they are enantiomers, :.::oice C. To see the mirror plane relationship, one of the compounds must be ::tated 180'. The drawing below shows the right structure berng rotated.

nr.r,,O..rr\oH

"r@
t""O"rr\oH
nirror ima

"or'O".,,tCHg
I

- -.pyright @ by The Berkelev Review

Exclusive MCAT Preparation

Organic Chemistry

Stereochemistry

Confi

gurational Isomers

Meso Meso compounds are individual structures which contain a mirror plane slicing through the middle of the compound and an eaen number of chiral centers

symmetrically displaced about the mirror plane. The net optical rotation of a meso compound is 0". It is zero, because the opposing chiral centers on each haU of the molecule cancel one another out, Ieaving no net rotation of plane polarized light. Meso compounds are referred to as optically inactiae. Remember the phrase, "Me so inactive", a high-energy rap lyric that describes your physicai state while studying for the MCAT. Figure 3-12 shows a meso compound (it has been rotated into a side view to see the mirror plane more easily).

top view

side view

Figure 3-12

A meso compound may be identified by an inversion center in the middle of the molecule. Figure 3-13 shows two conformational isomers of a meso compound. one in its most stable conformation (where it has an inversion point), and the other in its least stable conformation (where it has a mirror plane of symmetry. A meso compound has the same number of R-stereocenters as S-stereocenters.
Inversion point

CHa

I !

Figure 3-13

Example 3.12
The reflection of a meso compound can be classified as which of the following?

A. Identical to the original compound B. An enantiomer of the original compound C. A diastereomer of the original compound D. An ameso compound from the fifth dimension where evil lurks and the sock
that disappear from laundry loads in our world gather.

,[

Solution

A meso compound when viewed in a mirror reflects the identical compounc This makes choice A the best choice, although choice D is a close second. A:
example of a meso compound and its reflection is drawn below.

5
'n

"u"l H
HOO \,/
mirror plane
198

"aI

.i:

ln

tr

--t

Copyright

by The Berkeley Review

The Berkeley Revieu

Organic Chemistry

Stereochemistry

Configurational Isomers

Stereoisomerism As mentioned before, stereoisomers are compounds that have identical bonds but their atoms differ in spatial orientation. \Mhen a molecule contains more than one chiral center, the maximum number of stereoisomers increases exponentially with each new chiral center according to the equation 2n, where n is the number of chiral carbons in the molecule. There are less than 2n stereoisomers, if one of the possible structures is meso. If there are an odd number of chiral centers, the structure cannot be meso, so there is exactly 2n possible stereoisomers. For example, consider the compound 3,4,5-trimethyloctane, which has three chiral centers and thus eight possible stereoisomers. 3R, 4R, SR-trimethyloctane is just one of the eight possible stereoisomers. Table 3-1 shows the possible stereoisomers for compounds with a variable number of chiral centers.

Chiral Centers
1

Maximum
Stereoisomers
2 4
8

Stereoisomers

RorS
R& RS, SR, or
SS
OT SSS

RRR, SRR, RS& RRS, SSR, SRS, RSS,

Table 3-1
lVe are most concerned with stereoisomers when a molecule contains more than rne chiral center. If there is only one chiral center, there cannot be diastereomers. Stereoisomerism is important in biological sciences, because only a very few

riological compounds have just one chiral center (some amino acids being
amongst of these few) with some proteins having in excess of 200 chiral centers.

E-xample 3.13

I{ow many stereoisomers are possible for the following structure? OH OH

o
-\.
4
72 16

CHs

CH:

ts.8

c.

D.

Solution

lhe molecule is drawn in a way to make you mistakenly see four chiral centers if - ou don't pay close attention, but the number of chiral centers is only three. larbon five is not a chiral center, because there are two methyl substituents ::'iached to it (one that is drawn as a methyl substituent and the other that is j:awn as carbon six of the longest chain). By having two methyl groups .::ached, it does not have four different substituents attached, thus it is not a ::riral carbon. This means that only carbons two, three, and four are chiral. The :-aximum number of stereoisomers is derived using the equation 2n, which is ::rs case is 23. Because there is an odd number of chiral centers, the compound ::mot be meso, so there are eight stereoisomers. The correct answer is choice B.
by The Berkeley Review

-:,:vright

r99

Dxclusive MCAT Preparation

Organic Chemistry
Example 3.14

Stereochemistry

Configurational Isomers

The relationship of the following pair of compounds is best described as:

CHs

OH

A. B. C. D.

Anomers
Diastereomers Enantiomers Structural isomers

Solution

hydroxyl group is on carbon 2. This makes the two compounds structural isomers, which makes choice D the best answer. you may recall that if the two
compounds have different IUPAC names, then they are structural isomers. By virtue of the hydroxyl being in a different position, the two compounds in question have different IUPAC names.
Example 3.15

The hydroxyl group is attached to different carbons in the two structures. In the left structure the hydroxyl group is on carbon 4 while in the right structure, the

Which of the following compounds islare optically inactive?

L 2R,3S-dibromopentane II. 2S,3R-dichlorobutane III. 1R,2R-diiodocyclopentane A. Compound I only B. Compound II only C. Compound III only D. Compounds I and III only
Solution
To be optically inactive, the compound must either be achiral or meso. All of the compounds iisted have stereocenters, so achiral is not a possibility. The questio:.

is whether or not each structure is meso. To be meso, the compound must b= symmetric and have an even number of chiral centers equally displaced abou: the internal mirror plane (i.e., R on one side and s on the other). Compound I s eliminated, because it is not symmetric about the a plane (the mirror plane wou.l; have to slice through carbon three to break the five carbon species into halves To be symmetric, carbons two and four would have to have the sam" substituents, which they do not. Compound I is chiral, so it is optically acti,,'= and therefore eliminated. Compound III is eliminated, because it does not havr opposite chiral centers (they are both R). To be meso, carbons one and trt: would have to have opposite absolute configuration, which they do nc: Compound III is chiral, so it is optically active and therefore eliminated. Tl: only choice left is Compound II, which is symmetric, because it has a mirrir plane that slices through the bond between carbon two and carbon three. Th:
best answer is choice B.

:
T]f
1l!18

mrt

,&-

il"
C.
l!i't

Copyright

by The Berkeley Review

The Berkeley Revie*

Organic Chemistry
Stereoisomer Formation

Stereochemistry

Stereochemistry in Reactions

The

utility of stereochemistry lies in the selectivity of chiral reactants for one another. \Mhen reactions involve chiral reactants, they are often selective for one stereoisomer over another. This is a staple of enzymatic selectivity. Howevet, when a reaction involves reactants without any chirality, the formation of stereoisomers is random and follows basic probability. Most reactions in organic chemistry produce stereoisomers. The type of stereoisomers formed depends on the chirality of the starting reagents. \44ren a symmetric nucleophile can attack a planar species from either side, there usually are two enantiomers formed in equal proportion. When a symmetric nucleophile is hindered from attacking a planar species from one side more than the other (due to a chiral center in the electrophile that creates greater steric hindrance on one side than the other), there are two diastereomers formed in unequal proportions. Stereoisomers can result from electrophilic addition reactions with alkenes as well as substitution reactions. Figure 3-14 shows an example of an electrophilic addition reaction that forms two enantiomers.

ll ,rrc'c\nrcr,,
Planar carbonyl

j*",

t,to\

.. O

O..

& ,C-. .rC-. CH2CH3 HsC CH2CH3 HsC


I I

l"o

t\ St",

""\ c-. ft z
HsC

HO. OCH,

nocu"

& cH2cH3

\l'._-,
,rC-.

"

HsC

cH2cH3

Enantiomers

Figure 3-14

Example 3.16
The addition of alkyl magnesium bromide (RMgBr) to a carbonyl in ether adds a :iew alkyl substituent to the carbonyl carbon, resulting in conversion of the :arbonyl into an alcohol. The addition of H3CMgBr to R-2-methylcyclohexanone n diethyl ether yields which products? -A.. One meso compound

B. Two diastereomers C. Two enantiomers D. Two epimers

,opyright

by The Berkeley Review

Exclusive MCAT Preparation

Organic Chemistry
Solution

Stereochemistry

Stereochemistry in Reactions

In this reaction, the methyl group can add to either the top or bottom of the planar carbonyl group. This results in a new chiral center that can be either R or S. However, there is already a chiral center present in the reactant that is not involved in the reaction, which retains its original chirality. The chiral center present in the reactant does not change during the course of a reaction, so the products cannot be enantiomers. This eiiminates choice C. The compound is not
a sugar, so choice D is eliminated. It is not meso, so choice A is eliminated. One of the two chiral centers differs between the two stereoisomer products, so thev are diastereomers. Choice B is correct. Due to steric hindrance from the ethll group on carbon two, the product mixture of the two diastereomers is not 50/50.

HO
.*rCHe

+ HrCMgBr

----->

.'rrCHs

.,tfCHe

When two enantiomers are formed, they are formed in equal quantity, and the product mixture is said to be rncemic. When the two enantiomers are equalhpresent, there is no net rotation of plane poiarized light. When tu'o diastereomers are formed, they are formed unequally, so the product mixture has a major and a minor product. When the two diastereomers are present ir.

unequal amounts, there can be a net rotation of plane polarized light.


Enantiomers can be formed in an unequal ratio if a chiral catalyst is present. This leads to the concept of enantiomeric excess, used to analyze product distributions from reactions with a chiral catalyst (most often an enzyme).

:i:

.4_

Example 3.17 \Atrich of the following reactions produces no optically active compounds?

,I
D

ffi

A. B. C. D.

2-Butanone treated with NaBH4 in ether followed by acidic workup. (Z)-Z-butene with KMnO4 in base (sgr addition of two hydroxyl groups). Reduction of HN=C(CH3)CH2CH3 using LiAiH4 in thf solvent. S-2-Bromobutane treated with NaCN in ether solvent.

i:

::i

Solution For a compound to be optically inactive, it must either be meso or achiral. In choice A, NaBH4 adds a hydrogen to the carbonyl carbon from either side, resulting in a racemic mixture of alcohols. Choice A is eliminated. In choice B, KMnO4 adds a pair of hydroxyl groups to ihe alkene carbons to from a syn vicinal diol. Because the alkene is svmmetric to begin with, symmetric addition results in a symmetric product. The product is a meso diol, so choice B is the best answer. In choice C, LiAIH4 adds a hydrogen to the imine carbon from either side, resulting in a racemic mixture of ami.nes following workup. Choice C is eliminated. In choice D, a good nucleophile attacks an alkyl halide, resulting in inversion of stereochemistry. One chiral species is formed, so choice D is out.
Choice

B\ \-*/

+ KMnOr
Symmetric -+ additioi-r

Symmetric about plane

OH OH HO Symmetric product: meso vicinal diol

HO

-t'

Copyright

by The Berkeley Review

The Berkeley Review

Organic Chemistry

Stereochemistry

Stereochemistry in Reactions

The only way to get a meso compound from an addition to an alkene, is to have a

symmetric addition to a symmetric alkene or an asymmetric addition to an asymmetric alkene. The best answer is choice B. Choices A and C involve racemization which is common when a nucleophile attacks an sp2-hybridized carbon. Choice D is the result of an SN2-reaction, which inverts the chirality. Because the compound starts optically active, inversion generates an optically active product.

Enantiomeric Excess Enantiomers rotate light in opposite directions of one another, but with equal magnitude. When both enantiomers are present in equal quantities in soiution (a
50-50 mixture), the solution exhibits no net rotation of plane-polarized light. Based on this idea, when a mixture is not in a 50-50 ratio, then the net rotation of light by the solution is not zero. The farther the value deviates from zero, the greater the difference in concentration of the two enantiomers. From the observed rotation of the solution, the percentage of the enantiomer in excess can be derived. Equation 3.2 shows how to determine the enantiomeric excess from the observed specific rotation. The enantiomeric excess is the difference in

percentage between the more abundant enantiomer and the less abundant
enantiomer.
'oee (enantiomeric excess) =

measured specific rotation specific rotation of the pure enantiomer

700% (3.2)

Example 3.18

i\4rat enantiomeric distribution would account for a specific rotation of +13.6' ',Jre pure enantiomers have specific rotations of +27.2" and -27.2' respectively?

if

-\. (+)-enantiomer =25ok and (-)-enantiomer B. (+)-enantiomer = 33oh and (-)-enantiomer C. (+)-enantiomer = 67oh and (-)-enantiomer D. (+)-enantiomer =75o/o and (-)-enantiomer
Solution
Because the

=75%
= 67.h

= 75% =ZS%

net rotation is positive, the (+)-enantiomer must be in higher concentration than the (-)-enantiomer. This eliminates choices A and B. To determine the exact quantity, Equation 3.2 canbe applied.
"/oee

=+13'6 x

27.2

100%

=!x
2

100%

50'/oinfavorof the(+)-enantiomer

The exact ratio is found using the following reiationship:

'i'(+)-enantiomer + %(-)-enantiomer = 100% and (+)-enantiomer - (-)-enantiomer = Bjok (+)-enantiomer = 75o/o and (-)-enantiomer = 25"/o, choice D

Copyright

by The Berkeley Review

203

Exclusive MCAT Preparation

Organic Chemistry

Stereochemistry

Nucleophilic Substitution

Nubledphiliru1i;$fib$$.flfi ffi,bn
Nucleophilic Substitution One application of stereochemistry is in nucleophilic substitution reactions. Nucleophilic substitution involves the attack of an electropositive carbon by a nucleophile (Lewis base) to dislodge an atom or functional group (referred to as the leaving group). This is a recurring reaction in organic chemistry and it involves the substitution of one functional group for another. Nucleophilic substitution can proceed by more than one reaction pathway. It can proceed bv the two-step 51111 mechanism, or it can proceed by the one-step 5512 mechanism. Nucleophilic substitution reactions are based on the fundamental chemistrv concept that negative charge seeks positive charge. The electron pair of the nucleophile hunts for an electron deficient carbon to attach to. It will be important to understand the steps of the reaction for both mechanisms, as reactants proceed to products. Figure 3-15 shows an example of a nucleophiJic substitution reaction.

fi

lli

ri[

rl

..-..o

.1.

H.CO

,M

d
Figure 3-15

will discuss the mechanism of this reaction shortly, but for now, there are with which to be familiar. Listed below are the most important definitions. Each definition is followed by some generecomments about the relative reactivity of the species and how to discern i5
We
some fundamental definitions reactive strength.
Nucleophile

The species donating an electron pair in a nucleophilic substitution reactic'n (Lewis base). As its name implies, it loaes (philes) a positiae charge (nucieo' Nucleophiles must have an available pair of electrons to share. Nucleophile strength is closely approximated by its base strength, although steric factors (nucleophile size) affect nucleophilicity. Small nucleophiles are generally better nucleophiles. This is to say that steric hindrance plays a larger role in nucleophilic substitution reactions than proton transfer reactions. The strength of the nucleophile does not perfectly correlate with base strength, but it is close enough to say that it parallels. A short list of nucleophilic strength in water solvent is as follows: SH-> CN- > I-> OR- > OH-> Br-> NHg > C6H5O- > CH3CO2- > Cf > F- > ROH > HzO It should be noted that if the base is too strong, an elimination reaction can occur (as is the case with OR- and OH-). The solvent also has an effect in that nucleophiles that can hydrogen bond are hindered in protic solvents, because they are solvated. The solvation by water binds the electron pair of the nucleophiie and reduces its nucleophilicity. This phenomenon explains why SH-, CN- and I- are stronger nucleophiles in water than OH- despite being weaker bases than OH-. In aprotic solvents and the gas phase, nucleophilicity more closely parallels basicity. The big difference in nucleophilicity is that size of the anion is not as important as it is with basicity.

ffii

.Lq

I (

f,

Copyright O by The Berkeley Review

The Berkeley Review

Organic Chemistry
Electrophile

Stereochemistry

Nucleophilic Substitution

The species accepting an electron pair in a nucleophilic substitution reaction (Lewis acid). The electrophile holds the reactive .urborl and the leaving group. The weaker the bond between the leavrng group and the carbon, the better the electrophile. Electrophile strength can be ipproximated by the stability of the leaving group once it is off of the electrophile. Electrophiiic carbons typically nave a partially positive charge.
Leaaing group

The functionai group that dissociates from the eiectrophile in a nucleophilic substitution reaction. The more stabie the leaving group, the weaker it ii as a rase. This means that the strength of a leaving gio"p can be predicted by the .trength (pKu) of its conjugate acid. The theory is that the more stable the -eaving group, the less basic the leaving group, and thus the more acidic the :onjugate acid of the leaving group. The strengih of a leaving group increases as :he pKu of its conjugate acid decreases. This is most true in wafer, but can also be seen in organic solvents. group strength increases as the strength of the f-eaving :ond between carbon and the leaving gro.tp J""r"ur"r. This is why iJdine is a retter leaving group than fluorine.
?.scemic mixtrn.e

'+ product mixture that has an even distribution of enantiomers, 507u of each =nantiomer, in the product mixture. A racemic mixture is the observed product -'""hen the mechanism involves an intermediate where the reactive site is'an sp2-^,., bridized carbon (like a carbonyl or carbocation) and the molecule is symmeiric -:ras r-lo other chiral centers). There is no such thing as a racemic mixture of ..iastereomers, because diastereomers have at least two chiral centers associated ''ith them, and a chirai center present in the reaction hinders attack of one side of -: e electrophiie relative to the other, causing the distribution to not be fifty-fifty.

nucleophilic reactions involve a nucleophile, an electrophiie, and a leaving some, but not all, nucieophilic reactions generate a racemic mixture. a racemic mixture is generated or not depends on the reaction pathway. ",hether lhe fundamental question in a nucleophilic subititution reaction is, ;does the - ;cleophiie come in first, or does the leaving group leave first?,, This is the basic -..Jference between the sry1 and the slrJ2 mechanisms. The nucleophile attacks ::st in an S]N2 reaction mechanism, while in the leaving $oup leaves first in an :';1 reaction mechanism. we will look at these two scenarios in more detail.
--.1

;:oup.

Erample 3.19 -, iavorable nucleophilic substitution reaction has all of the following EXCEpT: d. a good leaving group. l. a reactant with a weak bond to the leaving group. :. a strong Lewis base as the nucleophile. f . a weak Lewis base as the nucleophile.

nvright

by The Berkeley Review

20s

Exclusive MCAT Preparation

Organic Chemistry

Stereochemistry

Nucleophilic Substitution

Solution A favorable nucleophilic substitution reaction is one that forms a stronger bond than the one broken. A good nucleophile is one that forms a stronger bJnd with carbon than the bond between carbon and the leaving group. A good leaving group is one that forms a weak bond with carbon, thus minimal eneigy is needeJ to break the bond between it and carbon. when the leaving group ii strong, ihe reaction is said to be favorable, so choice A is a valid statemet-,i. choic" e is eliminated. A weak bond to the leaving group makes it a good leaving group, so choice B is also a valid statement. This eliminates choiie B. The n.t"t"optl1t" should be a strong Lewis base, so choice C is a valid statement and choice Dls an invalid statement. This eliminates choice C and makes choice D the best answer. You might note that to determine the favorability of a nucleophilic substitution

,i

, "".rt"", t"" """,

leophile and leaving group.

sru2

In an sjr]2 reaction, the nucleophile attacks prior to the ieaving group leaving. lru:1"1.9, the nucleophile comes in from the backside and pushes ine teaving gio.rp off of the electrophile. An important factor to consider is the transition state tha: forms during the reaction. Transition states cannot be viewed directly (thei: lifetimes are too short), but evidence in the product (inversion of configuration ai a chirai carbon) infers they exist. Backside attack by a nucleophile causes thr-. inversion at the chiral carbon. Certain features in the reactants (nucleophile an; electrophile) and the product (if it is chiral) indicate that the reaction proceedec by an slr]2 mechanism. These are idiosyncrasies of the reaction, and they dictatr the pathway the reaction chooses. Each property favors one of the trr l mechanisms. They can be used to distinguish an slr]2 reaction from an slreaction. Figure 3-16 shows a generic mechanism for an sl.I2 reaction.
Trigonal bipyramidal Transition state

rm

II

&
,&riil

R+
I

s,
.G.

Nuc:

L'G'--+
H

Nuc---f---'L HH
l'==

------> Nuc

L.G.-

G
ln"

Figure 3-16 The reaction takes place in one step, so the rate of an sry2 reaction depend.s

begin to fold in the direction of the less hindered side of the mblecule (l hindered because the leaving group has left). The hybridization finishes as #Table 3-2 lists some key features associated with an sp2 reaction, according observation order. What is meant by observation order is that the first featr:: (features of the reactants) are observed before the reaction begins, the ser features (features of the transition state) are observed during the reaction, the last features (features of the products) are observed after the reaction end-sCopyright
@

electrophile. The nucleophile initiates the reaction by attacking the electrophie and forcing the bond between the carbon and the leaving group to stretch a:rd weaken. At the same time that the nucleophile approaches the electroph:-rc carbon, the electron density of the nucleophile repels the substituents on tw electrophilic carbon and thus they form the trigonal bipyramidal transition sta:eAs the leaving group begins to leave, the substituents on the electrophilic carb,cnmr

both the concentration of the nucleophile and the concentration of t:e

.-mr

by The Berkeley Review

206

The Berkeley

Organic Chemistry
Reactant Features

Stereochemistry

Nucleophilic Substitution

Course of Reaction Features

Product Features

rhe preference for an 51112


::,.echanism is 1' > 2" > 3" in :erms of electrophiles

An

51112

mechanism forms a five'

ligand transition state during


the middle of the reaction

A single enantiomeric product is formed (No racemic mixture)

-{n

The S-ligand transition state is 5512 reactions exhibit second mechanism is favored the highest energy state and it order kinetics i'.-ith a good nucleophile (rate = klNucllElectl) exists for just a split second
S11i2

ln
-l

5512 mechanism is favored Steric forces destabilize the Sl12 reactions are one-step poiar, aprotic solvents such transition state by forcing bond reactions, so they have fast rates of formation ethers and ketones angles to values less than 109,5'

Table 3-2

lie reaction in Figure 3-17 proceeds by an Sy2 mechanism, because the :-ectrophile is primary and it has a good nucleophile. With a primary
=-ectrophile, the reaction must proceed by an 5512 mechanism. The ether solvent polar and aprotic, which further favors the Sp2 reaction pathway. -.

H"C ,\
NC:@ +

CH"

Good

H'''y

r --+ Et2O NC-JFigure 3-17

\ucleophile

D 1" Electrophile

Y,o"
D

+I@
Stable

Anion

Inversion Product

E"xample 3.20

-ll of the following are associated with an 51112 reaction EXCEPT: A. backside attack of the electrophile by the nucleophile. ts. inversion of stereochemistry. C. nucleophile concentration affecting the reaction rate. D. rearrangement of alkyl groups from reactant to product.
5olution ohis question focuses on the fundamentals of an S1.tr2 reaction. For an St12 :eaction to proceed, the nucleophile must attack the electrophile from the --rpposite side as the leaving group in a collinear fashion reiative to the bond to :he leaving group. This is referred to as backside attack, so choice A is valid, and --nus eliminated. Backside attack results in inversion of stereochemistry if the tlectrophilic carbon is a chiral carbon. This makes choice B valid, which eliminates it. Because there is just one step in an 51112 reaction, the rate depends rn all of the reactants, including the nucleophile. This makes choice C valid, and thus eliminates choice C. Rearrangement can occur when there is a carbocation present, because carbocations lack a bond. Carbocations are associated with 51111 reactions, not 51112 reactions, so choice D is invalid and thus the correct answer to
:his question.

Copyright

by The Berkeley Review

Exclusive MCAT Preparation

Organic Chemistry
sp1:

Stereochemistry

Nucleophilic Substitution

essence/ the nucleophile waits

intermediate has a long enough lifetime to be detected using spectroscopy. Both rearrangement (hydride shifts and alkyl shifts) and a mixture of stereoisomers (formed from the spl-intermediate) are observed with sNI1 reactions. The nucleophiie is free to attack from either side of the carbocation intermediate, if the carbocation is symmetric. As a result, a racemic mixture of enantiomers is formed as the product mixture. Figure 3-18 shows a generic mechanism for an
5111

room to attack. The slr]1 reaction rate does not depend on nucleophile concentration. once the leaving group has dissociated, a planar caiionic intermediate forms. Evidence for the intermediate comes from kinetics data as well as stereochemicai evidence provided by the products. The carbocation

Lr an s|J1 reaction, the leaving group leaves before the nucleophile attacks. Lr until the leaving group has left, allowing it more

reaction.

,onf"$G"
R"

->

720'A ,."(

Nuc attacks

R
109.5"

Trigonal planar Carbocation Intermediate Figure 3-18

R,,

Vf ffi;=-.
R,
from the

fromrhe,9r__ Nuc

right

109

'[r
R>..
R"

-R;

L.c.

first step, the bond between the carbon and the leaving group breaks. As the leaving group begins to leave, the substituents on the carbon foia in the direction of the less hindered side of the molecule, allowing the-bond-angles to increase from 109.5' to 120" as the carbon re-hybridizes from ry3 to sp2. ihis results in a slight increase in stability, accounting for the intermediate being at a lower energy level than the first transition state in the energy diagram. In addition to re-hybridization, the planar cation is solvated, which also increases the stability
of the intermediate. The nucleophile can attack the carbocation intermediate and displace the solvent from either side of the carbocation intermediate. This displacement of solvent and the re-hybridization from sp2 back to sp3 causes a decrease in stability from the intermediate to the second transition state. Finally, as the new bond is formed, the energy level decreases until it reaches the level of the products. Bond formation is an exothermic process. The hybridization of the central carbon finishes atsp3. Table 3-3 lists features associited with the sx11 reaction. Like in Table 3-2, the features are iisted according to observation order. Reactant Features
The preference for an 5511 mechanism is 3" > 2' > 7" in terms of electrophiles

The reaction takes place in two steps, where the first step is the slowest. In the

Ll

.:
D

-,':

.q,.

Course of Reaction Features

Product Features

Steric hindrance pushes the A racemic mixture forms leaving group off of the when the electrophile electrophile has chirality

An
An

S11 1 mechanism is The intermediate is a planar, SN 1 reactions exhibit favored in a protic solvent three-ligand carbocation with first order kinetics such as alcohol sp2-hybrrdtzatton (rate = k[Elect])

Sry1 reaction is seen An intermediate is observed in Sry1 reactions are slow with a poor nucleophile addition to transition states two-step reactions

Table 3-3

-:pvright

O by The Berkeley Review

The Berkeley Review

Organic Chemistry

Stereochemistry

Nucleophilic Substitution

The Sxyl reaction can be complicated by rearrangement, because of the carbocation intermediate formed. If a secondary carbocation (R2CH+) is formed, it can rearrange to form a tertiary carbocation (RgC*), if a tertiary carbocation is possible. For alkyl carbocations, the relative stability is 3" > 2' > 1", the same preference that is observed with free radicals. The features in an S5i1 reaction are opposite of those in the Sp2 reaction. These features of a reaction can be used to predict whether a reaction will proceed by an 51111 or 51112 reaction mechanism. The reaction in Figure 3-19 proceeds by an Sp1 mechanism, because the electrophile is tertiary and it has a good ieaving group.
H3CH2CHTC

'\

H?CH2CH2C

H3N:
Average

H3CH2C

H3c\f

NH3

Nucleophile

tr.o'YF
H3CH2C
3" Electrophile

I+&

*Io
stable

ol ,,*_\,,.,r,
Figure 3-19

gH2cH2cH. ' ' |

Anion

cH2cH3
Racemic Mixture of Products

The reaction is favorable, because the leaving group is a good leaving group and the nucleophile forms a stronger bond with carbon than the leaving group.

Example 3.21

Ihe addition of ammonia to R-3-iodo-3-methylhexane at low temperature would


vield:

A. one product with R configuration exclusively (retention of stereochemistry). B. one product with S configuration exclusively (inversion of stereochemistry).
C. D.
two products in an enantiomer mixture. two products in a diastereomeric mixture.

Solution First, we must determine whether the reaction proceeds by an S1r11 or 51..12 mechanism. The electrophile (R-3-iodo-3-methylhexane) is tertiary, so the reaction proceeds by an S1'tr1 mechanism. The chiral center is lost with the formation of the carbocation intermediate, because the intermediate is pianar with symmetric sides. This results in a racemic product mixture of two
enantiomers. Choice C is a swell answer for this question.

Copyright

by The Berkeley Review

209

Exclusive MCAT Preparation

Organic Chemistry

Stereochemistry

Nucleophilic Sutistitution
cases where the reactive carbon is

In Figure 3-19, the electrophile is tertiary. In

secondary, an slr]1 reaction can be complicated by rearrangement. This is shown in Figure 3-20.

H.C

"\

(H3C)2HC

Hv

,,L--r

\'
-

H,.C "l
hydride

H"C

shift
/\ HsC

"l

/\ HsC CHs 2'carbocation


H3CH2C
HAN:

-([tH "-c

"'h c*

CHe 3" carbocation

HlCH,C

" 'l

\HsC

li,CHa

urcsf.NH'
Figure 3-20
HsC achiral product

3" carbocation

Rearrangement is rapid, because it is an intramolecular process. L:r the example in Figure 3-20, the secondary carbocation rearranges to form a more stable

tertiary carbocation before the ammonia nucleophile attacks the carbocation intermediate. This results in a tertiary product. The halide leaving group is not basic enough to deprotonate the ammonium cation formed from the substitution reaction, so the product remains as a cation. If the electrophile has a chiral center at a site other than the electrophilic carbon, an Str11 reaction will form both a major and minor product. The major product results from the transition state with least steric hindrance.

YYYY

CHICH?

CH,,CH"

CH,CH"

CH.CH,

OCI

-.>

Attack from the backside is


more favorable than frontside attack due to steric hindrance.

e.'C*

ry.s

O'"?:il; Ort:il;
Major Product

Minor
Product

Figure 3-21 In the example in Figure 3-27, the ethyl group in front of the plane interferes with the attack by the nucleophile, which results in an uneven distribution of diastereomers as the product mixture. The major product is formed when ammonia attacks the less hindered face of the carbocation (backside attack in this example). The minor product is formed when ammonia attacks the more hindered face of the carbocation (front side attack in this example).

Copyright O by The Berkeley Review

2to

The Berkeley Review

Organic Chemistry

Stereochemistry

Nucleophilic Substitution

Distinguishing an Sp2 reaction from an SpL reaction The first thing to look for when determining the mechanism by which a nucleophilic substitution reaction will proceed, is the substitution of the electrophile. Tertiary and allylic (adjacent to a n-bond) electrophiles wiil proceed by an Sx11 mechanism while methyl and primary electrophiles will proceed by an 5512 mechanism. This is the first factor to view. If the electrophile is secondary, then the reaction can proceed by either mechanism. After considering the substitution of the electrophile, the next feature to consider is the nucleophilic strength. The stronger the nucleophile, the more likely the reaction will proceed by an 5512 mechanism. The better the leaving grosp, the more likely the reaction will proceed by an 51111 mechanism. Lastly, you should consider the solvent. If the solvent is protic (capable of forming hydrogen bonds), the reaction wili have a tendency to proceed by an 5111 mechanism. If the solvent is aprotic (not capabie of forming hydrogen bonds), the reaction will have a tendency to proceed by an Sry2 mechanism. These factors can be applied when looking at the
reactants.

If rate data are given, then the mechanism can be inferred without ambiguity. The rate law associated with an 5111 mechanism is shown in Equation 3.3, while the rate iaw associated with an Sp2 mechanism is shown in Equation 3.4.
S1.tr1

Rate = k [Electrophile]

(3.3) (3.4)

5512 Rate =

k [Nucleophile][Electrophile]

changes as the nucleophile concentration is varied, the reaction is proceeding by an Sp2 mechanism. Conversely, if the rate of a reaction does not'change as the nucleophile concentration is varied, the reaction is proceeding by an 5511 mechanism. Because the solvent can affect the strength of a nucleophile, solvent and nucleophile are often considered together. The rates of both reactions vary with a change in electrophile concentration.

If the rate of a reaction

The energy diagrams for the two mechanisms also differ. There is no intermediate associated with an 51112 reaction, only a transition state. There is ar"t intermediate and two transition states associated with an 5511 reaction. Figure 322 shows the energy diagrams for the one-step S52 reaction (on the left) and the

two-step Sp1 reaction.


sru1

Transition state

Intermediate

Reactant
bo H
G)

F]

Product

Reaction co-ordinate ------->

Reaction co-ordinate

-+

Figure 3-22

Copyright

by The Berkeley Review

2tl

Exclusive MCAT Preparation

Organic Chemistry

Stereochemistry

Nucleophilic Substitution

attack. It is important to be able to recognize these diagrams and apply the information they contain to conceptuai questions.
Example 3.22

The energy level increases at the start of each energy diagram, because a bond is being broken. In the case of the S1r11 reaction, the intermediate is of lower energy than the transition state, because the carbocation can rehybridize to the leil crowded sp2-center rather than an sp3-center and the intermediate can be solvated in a protic solvent. The increase in energy from the intermediate to the second transition state is associated with rehybridization to the more crowded sp3-center and the intermediate losing solvition to allow the nucleophile to

The addition of sodium methoxide to s-2-bromohexane at low temperature would yield:

A. B. C. D.

one product with R configuration exclusively (retention of stereochemistry). one product with S configuration exclusively (inversion of stereochemistry). two products in an enantiomeric mixture.

two products in a diastereomeric mixture.

r" r^" trt-",t"" t


Example 3.23

mechanism. The electrophile (S-2-bromohexane) is secondary so the reaction can proceed by either an sl.J1 or Sry2 mechanism. The nucleophile is a strong nucleophiie, so we can assume the reaction will proceed via an Sp2 mechanisml This results in inversion of the chiral center and the final product having R stereochemistry, so choice A is the best answer. The low temperature is important, so that there is little to no E2 product formed. An E2 reaition results

Solution First, we must determine whether the reaction proceeds by an sIrI1 or sry2

The following reaction shows what relationship between nucleophile


concentration and reaction rate?

+ H,CCHTBT -----+ H3CCH2SCHTCH, + BrA. The reaction rate increases in a linear fashion with increasing nucleophile
H?CCH2Sconcentration.

B. The reaction rate C. D.

increases in an exponential fashion with increasing nucleophile concentration. The reaction rate does not change with increasing nucleophile concentration. The reaction rate decreases in a linear fashion with increasing nucleophile
concentration.

Solution
The reaction has a primary electrophile and a good nucleophile, which favors an s512 mechanism. The rate equation associated with a reaction proceeding by an s}..J2 mechanism is rate = k lElectrophile][Nucleophile]. The equation shows that

the reaction rate is directly proportional to the nucleophile concentration. The rate increases in a linear fashion wiih increasing nucleophile concentration, as stated in choice A. The best answer is choice A. Choice B should be eliminated, because the rate of a nucleophilic substitution reaction does not depend on the concentration of any species in an exponential fashion. Choice D should also be eliminated, because the rate will not decrease with additional nucleophile. It will either increase in a linear fashion or not change.

Copyright

by The Berkeiey Review

212

The Berkeley Review

Organic Chemistry
Example 3.24

Stereochemistry

Nucleophilic Substitution

A transition state with no intermediate is associated with which of the following


reactions?

A. H3CCHZO- + H3CCH2BT ----t> H3CCH2OCH2CH3 + BrB. H3CCH2OH + (H3C)3CBr ------> H3CCH2OC(CHa)3 + HBr C. (H3C)3CSH + (H3C)2CHOHz* + @3C)3CS+HCH(CH3)2 + H2o D. NH3 + (H3CH2C)3CBr --.> (H3CCH2)3NH3+ + BrSolution
As shown in Figure 3-27, no intermediate is associated with an sx12 mechanism, so we must find the reaction most likely to proceed by an sN2 mechanism. The reaction most likely to proceed by an Sl.]2 mechanism should have a good nucleophile and ideally a primary electrophile. A low temperature is important here so that there will be little to no E2 product formed. Choice C is the ieaction of a secondary electrophile with a poor nucleophile, so it will likely proceed by an S51 reaction mechanism. This eliminates choice C. Choices B and D involve tertiary electrophiles, therefore they definitely will proceed by an SlrJ1 reaction mechanism. This eliminates both choice B and choice D. Choice A has a primary electrophile and a good nucleophile, which makes it the most likely to proceed by an sg2 mechanism, and therefore makes it the best answer. The ethoxide anion is also a strong base, so elimination is possible in choice A. Despite the competition with the E2 reaction, choice A is still the best answer. Reaction Kinetics The rate'of a reaction depends on several factors. The rate depends on the available energy for the molecules to collide, orient, and break the necessary bonds. The rate depends on the likelihood of the molecules colliding. For an sirl2 reaction, the rate depends on the availability of nucleophile, while it does not depend on the nucleophile concentration in an Sry1 reaction. Consider the 5512 reaction shown in Figure 3-23 with an ethoxide concentration, [CH3CH2O-], of 0.01 M a 2-bromopropane concentration, [CH3CHBTCH3], of 0.01 M and a k1* of 2.53 x 10-2 L.mol-1.s1 at 2gBK. HqCH?CO:

Figure 3-23
The concentrations are low, so the reaction is very slow. Plugging the values into Equation 3.4 yields a rate of 2.53 x 70-6 M per second. The reaction rate may be increased by increasing the reactant concentrations, increasing the temperature, or by adding a catalyst. A catalyst stabilizes the transjtion state complex and lowers Eu.1. Transition states are short-lived complexes. In the course of the reaction, reactants collide with the correction orientation (from backside attack) to form the transition state complex, when eventually splits io generate the products. Figure 3-24 represents the species of the S5tr2 reaction in Figure 3-23 at different stages in chronological order over the duration of the reaction.

Copyright O by The Berkeley Review

213

Exclusive MCAT Preparation

Organic Chemistry

Stereochemistry
CH" ..4- | - ..5 ----+----Bri

Nucleophilic Substitution

H,cH"co
Reactants draw close to start bond formation and form the Early Complex (C"u.1r).

CH3

Transition State

Transition State starts to split as the bond breaks to form the Late Complex (C1u1s).
H3CH2CO.- - - /::'

H3cH2co.

H.C ..6- '\

;f

---

..6Br!

..5-

..6Bri
CHs

HsC
Ceurly

\"

Cl"t.

H.CH"CO J L ..
Reaction co-ordinate

Yg" -/'n'

Figure 3-24 Physical Properties of Stereoisomers Enantiomers have identical physical properties (boiling point, melting point, and density to name a few), while diastereomers have slightly different physical

properties. Because they have slightly different physical properties,

diastereomers are easier to separate than enantiomers. Enantiomeric mixtures

are difficult to purify, because a racemic mixture often has stronger

intermolecular forces than the pure enantiomer. Table 3-4 lists the physical properties of the two enantiomers and the racemic mixture of 2-butanol.
Form (+)
(-)

Chirality
S

CTD

Boiling Point
99.4'C

Density
0.808 0.808 0.840

Index of Refraction
1.398 1.398
1..442

+ 13.5"

-t-J.5

99.4'C t01.2"c Table 3-4

(r)

R/S

0'

lrom the data in TabIe 3-4, it can be seen that a racemic mixture allows the molecules to get closer together. This can be thought of when considering your
hands, where a left and right hand fit together nicely. It is common that a racemic solid mixture has a higher melting point and greater density than either
enantiomer.

Copyright O by The Berkeley Review

The Berkeley Review

Organic Chemistry

Stereochemistry

Nucleophilic Substitution

Separating Stereoisomers One of the most chailenging tasks a synthetic organic chemist faces is the separation of stereoisomers. If a reaction generates a new chiral center in the product, then it will be complicated by stereoisomerism. To generate a pure stereoisomer as a product, chirality must be invoked at some point. From biochemical examples, we know that enzymes (chiral polypeptides) orient molecules in a specific fashion, allowing just one stereoisomer to form, The chirality of the enzyme helps to select for the desired product. In organic chemistry, there are compounds known as chiral nuxiliaries, which introduce chiraiity to, or exaggerate existing chirality within, a reactant molecule. Chiral auxiliaries serve in a similar fashion to an enzyme. When aiming for one specific stereoisomer, it is often easiest to select for it in the reaction. If not, a mixture of stereoisomers is formed and chirally selective separation techniques must be applied.

Chirally selective separation techniques come in two types. The first involves employing an enzyme (or chirally selective molecule) to react specifically with one stereoisomer within the mixture. By reacting and therefore introducing a new functional group to only one stereoisomer, the two enantiomers now have different physical properties and can easily be separated. Once separated, the same enzyme can be employeci to return the compound back to its original form. An example of such an enzyme ts porcine renal acylase, which selectively acylates the N-terminal of L-amino acids. By acylating the L-amino acid, it is no longer a zwitterion at neutral pH, while the D-amino acid is a zwitterion. Because one carries a net charge, it is easily separated from the other.
The second chiraily selective separation technique involves invoking chirality in an existing separation technique. For instance, a column chromatography gel can be made from a pure stereoisomer. If the column is made with an R-alcohol for instance, then when a racemic mixture of alcohols is added, the S-enantiomer has a greater affinity for the column and thus has a greater elution time. This is the basic principle behind affinity chromatography in biochemistry, where an antibody is bound to the column so that it can selectiveiy bind an antigen. Chiral columns in organic chemistry are not as specific as enzyme columns and they

hinder solutes, but do not actualiy bind them. In theory, chiraiity could be invoked in any organic chemistry separation technique, including distillation, but only it is chiral columns that are commonly used.

Copyright

by The Berkeley Review

215

Exclusive MCAT Preparation

Organic Chemistry

Stereochemistry

Summary

Summary This section involved a few basic concepts. From this particular section, you should be able to identify R and S chirality, determine the relationship between stereoisomers, understand the nuances of nucleophilic substitution, distinguish between the slri1 and sIN2 reactions, and apply stereochemistry to organic lab techniques and biochemistry.
Determining R and S: When a structure is drawn in dash-and-wedge style, if priority number four is in back or two substituents away from the substituent in back, then take the arc as is and assign the corresponding chirality (Clockwise = R; Counterclockwise = S). If priority number four is one substituent away from the substituent in back, then determine the arc and assign the opposite chirality

of the arc.
Enantiomer: Enantiomers are stereoisomers which are nonsuperimposable mirror irnages (reflections that you can't overlay). They have the same bonds, but they have a different orientation of atoms in space, as do all stereoisomers. Enantiomers can be thought of as stereoisomers in which all of the chiral centers have different orientation between the two molecules. Enantiomers have

identical physical properties.


Diastereomer: Diastereomers are stereoisomers which are nonsuperimposable and that are not mirror images. They too have the same bonds and a different orientation of atoms in space. To be diastereomers, the compounds must contain a minimum of two chiral centers. Diastereomers are stereoisomers in which at least one but not all of the chiral centers have different orientation between the two molecules. Diastereomers have close, but not identical, physical properties,

sy1 Reactions: Preferred when an electrophile is tertiary, when the solvent is polar and protic, and when the electrophlle has u good leaving group, Sx1 reactions form a planar carbocation intermediate that can undergo rearrangelnent to form a more stable carbocation. Sru1 reactions result in a
SX11 reaction depends on only the electrophile and not the nucleophile (rate = k[Electrophile]).

The rate of an

racemic mixture when the reactive carbon is the only chiral center in the reactant.

5672 Reactio;,zs: Preferred when an electrophile is primary, when the solvent is polar and aprotic, and when the nucleophile is good. 5112 reactions form a transition state complex as the nucleophile forces the leaving group off from the electrophile. SN2 reactions result in inversion when the reactive carbon is a chiral center. The rate of an 5112 reaction depends on both the electrophile anC

nucleophile (rate = k[Nucleophiie][Electrophile]).


Stereoisomer

Mixtures: A fifty-fifty mixture of enantiomers is said to be a racemr:

mixture. The diastereomers in a product mixture formed from a chemica,


reaction are referred to as major and minor, because they do not occur in a fiftvfifty ratio. Separating one diastereomer from another is easier than separatirrg one enantiomer from another. To separate enantiomers, chirality must b. incorporated into the separation technique.

Copyright O by The Berkeiey Review

216

The Berkeley Revien

Organic Chemistry
Chirality and Asymmetry

Stereochemistry

Nucleophilic Substitution

Key Points for Stereochemistry (Section 3)

1.

Chiral Molecules a) Have asymmetry within their structure due to atoms that are unevenly substituted i. Stereogenic carbons are sp3-hybridized carbons with four unique substituents ii. Chiral moiecules are predominant in many organic reactions

b)

Stereogenic carbons are assigned an absolute configuration of either R or S to describe their chirality

i. Priorities are assigned according to atomic mass of the atoms ii.


attached to the stereogenic center. If two atoms are identical, then you proceed along its connectivity until there is a difference when priority #4 is in back, a clockwise arc connects priorities #1, #2, and #3 in R-stereogenic centers. When priority #4 is in back, a

counterclockwise arc connects priorities #I, #2, and #3

in

S-

iii.

stereogenic centers. To determine whether a center is R or S, you can place your thumb in the direction of substituent #4 and curl your fingers from priority #1, through priority #2, and on to priority #3. Only one of your hands can do this. If it is a right hand that does this, the stereogenic carbon has R-chirality. If it is a left hand that does this, the stereogenic carbon has S-chirality.

c)

short cuts for determining R and s involve the positioning of priority #4

i. If priority #4 is in back, then the arc determines the chirality


(clockwise for R and counterclockwise for S). If priority #4 is in front, then the arc must be reversed to determine the chirality (a
clockwise arc is reversed to represent S and a counterclockwise arc is reversed to represent R). If priority #4 is drawn in the plane close to the group going back, then the arc is reversed to determine the chirality. If priority #4 is drawn in the plane far away from the group going back, then the arc as is determines the chirality. \Ahenever two groups are switched, the chirality reverses

ii. 1.

Configurational Isomers

a)

Same connectivity, but different spatial arrangement of atoms Can be categorized as either optical isomers or geometrical isomers

i. ii. iii. b)

Optical isomers rotate plane-polarized light Geometrical isomers differ about a feature in the molecule about which rotation is not possible (n-bond or ring) Optical isomers are identified by a standard rotation value

Can be categorized as enantiomers or diastereomers i. Enantiomers are nonsuperimposable stereoisomers that are mirror images

ii. iii. iv.

Diastereomers are nonsuperimposable stereoisomers that are not mirror images Enantiomeric optical isomers are better thought of as stereoisomers where all of the chiral centers differ Diastereomeric optical isomers are better thought of as stereoisomers where some, but not ali, of the chiral centers differ

Copyright O by The Berkeley Review

217

Exclusive MCAT Preparation

Organic Chemistry
7.

Stereochemistry

Section Summary

Stereochemistry in Reactions Stereoisomers are formed when


moiecule in multiple ways

a nucleophile attacks an asymmetric

a) b)

Racemic mixtures form when there is no preexisting chirality

Diastereomers are formed in a major/minor distribution when one of the reactants has a chiral center at a non-reactive site

i. ii. iii. c)

Mixtures are resolved by using chiral reagents or lab techniques that invoke chirality. Enzymatic reactions use a chirai catalyst to cause the reaction to drastically favor the formation of one stereoisomer over all other possible stereoisomer products When two enantiomers are present in unequal amount, there is said to be an enantiomeric excess. Enantiomeric excess is used to describe the success in a stereoselective synthesis

Enantiomers have identical physical properties as one another while diastereomers have different physical properties.

Nucleophilic Substitution
1.

Proceeds by either an Sp1-mechanism or Sp2-mechanism


a)

In an Sg1-mechanism, the electrophile is highly substituted, the solvent is protic, and a carbocation intermediate is formed because the leaving group leaves in ihe first step. There is potentially rearrangement and the product mixture is often racemic. The reaction rate only depends on the
electrophile.

b)

In an S52-mechanism, the electrophile is minimally substituted, the solvent is polar and aprotic, and a transition state is formed because the nucleophile attacks to force the leaving group off in the only step. There is inversion of chirality so the product mixture is often the opposite

chirality of the reactant and the reaction is fast. The reaction rate
depends on both the nucleophile and the electrophile.

51
Copyright
@

by The Berkeley Review

2ta

The Berkeley Review

Stereochemistry Passages
15 Passages
I OO

Questions

Suggested schedule: I: After reading this section and attending lecture: passages I, II, vll & x Grade passages immediately after completion and log your mistakes.

II: III:

Following Task I: Passages IV V IX, & XIII (28 questions in 56 minutes) Time yourself accurately, grade your answers, and review mistaKes. Keview: Passages III, VI, VIII, XI, XII, & Questions 92 - IOO Focus on reviewing the concepts. Do not worry about timing.

SpeciaLtztng in MCAT Preparation

#fi

il

rtffi-iffit

:lffiffings.
(r -7)
(B

I. Isoleucine and Threonine II. Unknown Stereochemically Active Compound III. Classification of Isomers IV. Stereoisomers and Optical Activity V. Enantiomers from Diels-Alder Reaction VI. Proposed S51 SYnthesis VII. Nucleophilic Substitution vlil. Nucleophilic substitution of chlorocyclohexane IX. Leaving GrouP Strength X. Reaction Rates of Nucleophilic Substitution XL Elimination versus Substitution Experiment XII. Enantiomeric Excess XIII. Stereochemistry in Synthesis
Questions not Based on a Descriptive Passage

14)

(15 - 21) (22 - 28) (2e - 35) (36 - 42) (43 - 4e)
(5O - s6)

(s7 - 63)
(64 - 70)

(7r -77)
(78 - 84)
(Bs -

lI:r
lll.r:

.lr-:
rlltut illl:,"

el)

rLllJ

(e2 - lOO)

Structure, Bonding, and Reactivity Scoring Scale 84 - 100

Passage

(Questions

'1

- 7)

of eight essen.tirrl amino acids. The essential is applied to amino acids that humans cannot ::rduce, and therefore must take in through diet. Isoleucine :, .sts as a zwitterion in aqueous solution. The natural form
Isoleucine is one

::n

The amino acid glycine has an H as its side chain. What would you predict for the optical rotation for naturally occuning glycine?

A. B.
D

(.+)32'
(-) 32'

isoleucine has the same chirality as other naturally -:urring amino acids at carbon number two. Isoleucine is - lnd naturally in the L form. Naturally occurring amino
.,-ds have an S chiral center on carbon two (except cysteine). -..rleucine has a second chiral center in addition to the one at -'rbon two. Seventeen of the amino acids we code for have : ,.rctly one chiral center, with glycine (which has no chiral 'rter) and threonine (which has two chiral centers) being the ::er exceptions. The second chiral center of L-isoleucine -:s fixed chirality, so a diastereomer of isoleucine varying at . : side chain is not a biological substitute for L-isoleucine. . .sure 1 shows the zwitterion form of L-isoleucine.

c. 0'
.

More information is needed. (Never choose this!)

. If L-isoleucine A. B.
-62'

were found to have an optical rotation of -62' from plane-polarized light studies, what would you

predict for the optical rotation of its enantiomer?

D.

c. -31'

+62'

+31'

CH 2CH

r
6

oL-Isoleucine

Which of the following is the side chain of threonine? A. -CH(CH3)2

Figure

Zwitterion form of L-Isoleucine

B. -CH(oH)CH3 C. -CH2CH2OH D. -CH2OH

The only other amino acid coded fbr by human beings The .ireonine side chain contains an alcohol functionality. Like .oleucine, the side chain chiral center must be specific for .ie amino acid to be biologically incorporated. Threonine is ..so an essential amino acid. The diaslereomer of threonine .:at varies at the side chain is known as "allo-threonine."

.:at has a chiral side chain attached is threonine.

Only twenty-five percent of synthesized isoleucine can be used biologically. This is best explained by which of
the following explanations?

What is the stereoconfiguration for isoleucine?

A. B. C. D. 2. If
A

A. B.

Only 25Vo exists as a zwitterion in the body.


15Vo

of synthetic

isoleucine does not have the


the two chiral centers

2R. 3R

correct side chain.

2R, 35

C. In synthesizing isoleucine, D. In synthesizing isoleucine,

25,3R 25, 35

result in four stereoisomers being formed. Only one of the fbur is biologically usable.

the side chain chiral center were changed, the new An enantiomer of isoleucine. A diastereomer of isoleucine. An epimer of isoleucine. ldentical lo isolcucine.

structure would be which of the fbllowing?

the two chiral centers result in eight stereoisomers being formed. Only two of the eight are biologically usable.

. B. C. D.

3.

Most naturally occurring amino acids have which


stereochemical orientation
?

A. R B. S
C. D.
E

Copyright

hy The Berkeley RcviewG

GO ON TO THE NEXT PAGE

Passage

ll

(Questions 8

14)

10.

What is the stereochemistry of Compound P?

In two controversial laboratory studies, Compound P, shown in Figure I below, has been determined to reduce constipation in the Southern European Red-Eared Jumping
Lizard during mating season. The effects are less significant during courting periods and does nol occur at all during the first three days following the New Moon. The two studies compared this particular stereoisomer, along with other stereoisomers and structural isomers of Compound P, to determine the effects of the drugs. The dosages used were

A. 35, 45 B. 3R, 45
C. D.
35, 4R 3R, 4R

1. The following structure relates in what way to


Compound
P,

the Southern European

Red-Eared

held constant between compounds. The disagreement


between researchers came in determining the binding activity ofeach stereoisomer and its subsequent reactivity.

Jumping Lizard' s constipation medication?

HOH
cH 2cH
3

;"

H H HO

Ct-12CHs

-!: *rt

OH

Figure

Compound P

A. It is an enantiomer of Compound P B . It is a diastereomer of Compound P C. It is identical to Compound P D . It is a meso structural isomer of Compound P
12.

The exact mechanism for the constipation reducing behavior is not known, but it is speculated to work in conjunction with sex hormones to induce muscle relaxation. In other studies, the compound has been applied to the
abdomen of the Saskatchewan Green-Nosed Squatting Frog to test for similar effects. To date, no solid conclusions have been formed as to the effect of Compound P on constipation in other organisms. Being such an important chemical to the bowel process of reptiles and amphibians everywhere, a great deal ofresearch is destined to be under way. Researchers have

jr-

::i*.

What is the stereochemical orientation of the followins


compound?

c,-

"1::r'
- i'

lr:
-"&

:.;

continued to develop other structural isomers that will hopefully show similar effects in creatures such as the very
rare Yellow-Tongued Sabertooth Crotch Cricket. 8

. If treated with PBr3, the OH groups can be converted


into Br groups through a reaction which proceeds by an 5512 mechanism. The dibromo product formed from Compound P would show which of the following chiral
assignments?

A. B. C. D.

15, 25

:Ft

lR,
lR,

25
2R
*'it:r: lEri

15,2R

";

13. Enzymatic active sites are all of the following


EXCEPT:

lf-:r:
-i:r':

A. B. C. D.
9

35, 45
3R, 45 35, 4R
3R., 4R

A. B. C. D.
14.

chiral speciiic.
size specific.

L:,

functional group specific. isotope specific.

Assuming Compound P is made by treating an alkene with KMnO4 in basic water, what is the geometrical orientation of the alkene precursor to Compound P? (KMnOa adds two hydroxyl groups with syn orientation)

The enantiomer of Compound P has:

A. E. B. Z. C. Either, the reaction has no stereoselectivity. D . Neither, the reaction has no stereoselectivity.
Copyright @ by The Berkeley Review@

A. B. C. D.

the same boiling point as Compound P a higher melting point than Compound P. A lower density than Compound P. The same specific rotation as Compound P.

GO ON TO THE NEXT PAGE

I.lrr"

Passage

lll

(Questions 15 - 21)

16.

Which of the following explanations does NOT account

Isomers are compounds with identical formulas but a different arrangement of atoms in space. There are two basic

for an observed optical rotation greater than that of

types

of

isomers: structuraL isomers (also known

as

constitutional isomers) and stereoisonters (which can further be categorized as either configurational isomers or conformational isonters). Structural isomers are defined as raving different connectivity of atoms, also referred to as different bonds. Ethanol and dimethyl ether are an example of
structural isomers.

positive literature value for the pure species? A . Both enantiomers are present in an unequal amount. B. The solution is too concentrated. C . The cell for the polarimeter is too long. D . The wavelength of light being used is greater than that of the standard sodium light used.

Stereoisomers are isomers which have the exact same 'londs, but diff'er by the position of their atoms (substituents)

'.rithin space. Stereoisomers can be classified as either -tptical isomers or geometrical isomers. Optical isomers

17. How many

degrees of unsaturation are present compound with formula C11H21NO2?

in

rave different spatial arrangement due to asymmetry about an

.tom with in the molecule. A good example of optical .somers are the R and S enantiomers of a given molecule. 3eometrical isomers have a different spatial arrangement of :ioms with fespect to a molecular plane. Cis and trans
:utene are a good example of geometrical isomers.

4.4 B. 3
D.
I

c.2

A common, and often challenging, task in an organic :remistry laboratory is to separate and distinguish the )omers formed fiom a chemical reaction. Of all isomeric :ixtures, it is easiest to separate and distinguish structural somers, which have varying physical and chemical
::operties. Geometrical isomers also have different physical
::operties, and can be separated readily. They can often be ::stinguished from one another by their melting or boiling : rints. Of isomers, optical isomers are the most difficult to ,:parate. To separate optical isomers, a pure chiral material -"n be used in a chemical reaction, which is then followed by ::ecipitation or extraction. Once separated, the specific ::tical rotation value may be used to identify the enantiomer
'-.3t was isolated.

18. How many structural isomers are possible for


formula C6Hla?

the

A. rl B. 1
D.

c.

6 5

19. The chiral

centers

for the following molecule are:

(note: OH is located on carbon 2)

,:ploy chirality to help separate optical isomers from one ..other. As a general rule, the distinguishing features
:

Other traditional laboratory techniques can be modified to

-'lween isomers can be used to separate them in a laboratory ::ocedure, although some are considerably easier than others. -:r identifying a purified isomer, spectroscopy and physical r: lperties are most often employed.
-

A. 2R, 3R B. 2R, 35
C. D.
25, 35 25, 3R

5. The pure R

enantiomer of some compound has a specific rotation of +32.2'. A sample you make in lab has a specific rotation of +16. l'. This is best explained by which of the following mixtures?
100Vo R enantiomer present.
7

A. B. C. D.

5Vo

R R

and 25Vo S enantiomers present.

20. Which of the following CANNOT form


isomers?

optical

50Vo 25Va

R and 50% S enantiomers present. and l5Vo S enantiomers present.

. B. C. D.
A

A four carbon gem diol A four carbon vicinal diol


A fourcarbon secondary amine A four carbon secondarv alcohol

,pyright

by The Berkeley Review@

223

GO ON TO THE NEXT PAGE

L.

How many stereoisomers are possible for the following


structure? OH

Passage lV (Questions 22 - 28)


Stereocenters are important features in chiral organic compounds. Stereocenters are responsible for physical properties, chemical reactivity, and biological function. An example of the correlation between physical properties and chirality is shown with the stereoisomers of tartaric acid. Drawn in Figure 1 is the generic structure of tartaric acid. and a data table of the physical properties of the stereoisomers: HO

4.32 B. 64

(oH) (oH)

c.

128

D.256
Figure

o 1
Tartaric Acid

Table 1 shows the physical properties of the thre; stereoisomers of tartaric acid and the physical properties c: the enantiomeric mixture.
Form (+)
(-)

m.p.
168-170 168-170

0D

Density 1.7598 1.7598

H2O sol. @ 20'C


139

12" 12' 0"

g/l00ml

139 g/100ml 125

meso

t46-148
205-207

r.5996
1.7880

(t)

0'

g/l}0mL 2l g/l}0mL

Table

The differences in physical properties can be attributed :: lattice formation in the solid phase. The compound can pa;r. most easily when it is symmetric. This can be seen in t:* density and the melting point of each stereoisome: Examples of biological activity are numerous. A comni:,'r example involves the digestion of D-sugars. Our bod,v car metabolize D-glucose yet it cannot metabolize L-glucose i::e enantiomer of D-glucose). A recent example involves ne compound L-Dopa. L-Dopa is used as an anti-Parkinson dr-r4 while D-Dopa has no effect and can in fact be toxic in larg enough doses. L-Dopa is drawn in Figure 2 below.

{
ts

HO

NHz

Figure
22

L-Dopa

Which of the following physical properties would hi'*u the same value for morphine and a diastereomer ui{
morphine?

-{"
E"

A. Melting point. B. Density. C . Molecular mass. D. Specific rotation.


Copyright @ by The Berkeley Review@

I
D"

GO ON TO THE NEXT P.{

How can a compound with an optical rotation of


rotation of -121 .0"2

+233.0'be discerned from a compound with an optical

6.

B.

C. The larger the absolute D.


fiom one another.

posilive optical rolation. The sample with +233.0" oprical rotarion when diluted to half of its original concentration would show an optical rotation of +116.5".

The intensity of the light is greater with the

value

rotation, the greater the density of the compound. It is not possible to distinguish the two compounds

of the optical

What can be concluded about the packing of molecules in the crystal lattice of the stereoisomers of tartaric acid? A. The meso compound packs most tightly while the (+) enantiomer and (-) enantiomer pack the same. B. The meso compound packs least tightly while the (+) enantiomer and (-) enantiomer pack the same" C . The meso compound packs most tightly of all of the stereoisomers. The (+) enantiomir packs more

D.

tightly than the (-) enantiomer. The meso compound packs most tightly of all of the stereoisomers. The (+) enantiomer packs less tightly than the (-) enantiomer.

Given that the specific rotation of D-Glucose is +52.6.,

epimer is a diastereomer that varies at onlv one


stereocenter.

what can be said about the specific rotation of D_ mannose (the C-2 epimer of glucose)? Note: An

27

How many stereocenters are present in the molecule camphor which shows an optical rotation of +44.3"? cHs

^vo
u-f
I

oH

so-FH
I

H-1I

oH oH

CH:

-1I

A. 0 B. I

c.2
3

CH2OH
D_Glucose

D.

. It cannotbe +52.6', -52.6', or 0'. B. It must be either +52.6', -52.6', or 0'. C. It is 0'. D. It is -52.6 ".
A

28. Which of

the following statements best explains why


S

an R/S enantiomeric mixture has a higher melting point

than pure samples of either the R enantiomer or the enantiomer? How many stereoisomers are possible for penicillin V? H

It requires more energy to break the hydrogen bonds within a pure compound than within a mixture of
compounds.

H'coc6Hsl/
I

t\a.r,
-CHr
co2H

B.
C

Enantiomers readily form covalent bonds with one


another.

o
A. 8,2 c" 4
X

,h.y

The covalent bonds are weaker when the material is one pure stereoisomer then when it is a mixture of two or more stereoisomers.

D.

The R-stereoisomer packs more tightly with its


enantiomer than it does with itself.

D.

:vright

by The Berkeley Review@

GO ON TO THE NEXT PAGE

Passage V (Questions 29 - 35)


The Diels-Alder reaction of isoprene upon itself at 100'C yields two enantiomers of limonene in a fifty-fifty ratio. The diene of one isoprene molecule reacts with the less hindered

31.

The lemon flavored isomer has what stereochemisr,


associated with it?

double bond of another isoprene molecule to form the product. Because isoprene is a planar molecule, there is an equal chance fbr the reaction to occur on the top face as the bottom face of isoprene. The Diels-Alder reaction that
synthesizes limonene is shown in Figure 1 below.

. One chiral center with R stereochemistry B. One chiral center with S stereochemistry C. Two chiral centers with R, R stereochemistry D. Two chiral centers with S, S stereochemistry
A

32.

Which of the following physical properties is MOSI likely different for the two enantiomers?

Figure 1 Diels-Alder Condensation Reaction of Isoprene

The two enantiomers that are formed have similar physical properties, but have different applications in the flavoring agent business. One enantiomer has a lemon scent while the other has an orange scent. The two enantiomers of limonene are shown in Figure 2 below.

A. Boiling point B. Density C. Alcohol solubiiity D. Optical Rotation

3.

When hydroborane (BH3) reacts with limonene, ir a.rj boron to the less hindered carbon of both alkenes ::n hydrogen to the more hindered carbon of both alken::;, What is the major product when hydroborane reacts $:17
R-limonene?

lemon

odor 2

orange odor

Figure

Enantiomers of Limonene

The percentage of each enantiomer in the product mixture can be altered only with the addition of another chiral reagent

involved in the transition state. To isolate either the lemon flavoring or the lirne flavoring, the chosen enantiomer must be separated from the product mixture by one of several possible laborator"y techniques that involve chirality.

ca
"-u\.4
34

^"-&""&
p, n-u-=A

29.

Which of the following techniques does NOT work ro

isolate one enantiorner from the plesence


enantiomeric mixture?

of

an

A. B. C.
D

Adding the mixture to a chiral gel in a column


chromatography.

Distillation of the product mixture. Crystallization of the mixrure with the addition of
an enantiomerically pure compound.

What can be concluded about the olfactory receptors

A. B.
C

They are symmetric (achiral) because thel


distinguish between enantiomers.

;:anr

Filtering through an enzymatic membrane.

They are asymmetric (chiral) because ther :m


d

istinguish between enantiomers.

30.

Which of the following will lead to a product mixture composed of more than fifty percent of one of the
enantiomers (the product mixture not being racemic)'/

They are symmetric (achiral) because they c.::nm.


distinguish between enantiomers.

D.

They are asymmetric (chiral) because they


distinguish between enantiomers.

c-rmum

A. B. C. D.

Carrying the reaction out with a chiral solvent. Carrying the reaction out with a chiral catalyst. Carrying the reaction out at a lower temperature. Carrying the reaction out at a higher concentration.

Copyright @ by The Berkeley Review@

226

GO ON TO THE NEXT P{

5. If the following molecule

were the reactant rather than isoprene, how would the results differ?

Passage Vl (Questions 36 - 42)

A chemist intended to study the effect of periphery chirality on a nucleophilic substitution reaction. To 50 mL of methanol at 45'C, the chemist added 0.10 moles of
(l S,2R)-2-methylbromocyclopentane in the hopes of carrying out Reaction 1, which is shown below.

HrC Br
. B. C. D.
A
The product mixture would no longer be racemic. The product mixture would still be racemic. The products would be achiral. The products would have four stereocenters each.

IJ O

HrC

CHroH'

Li O

OCHr

"'u''
minor
product

&

major product

Reaction

The reaction was carried out for one hour at 45"C in an aqueous solution buffered at a pH of 5.0. Excess methanol

was removed from the product mixture by fractional distillation under reduced atmospheric pressure. The
atmospheric pressure was reduced to lower the boiling point of methanol, in the hopes that additional reactivity would be minimized at a lower temperature. The optical rotation for the crude product mixture was found to be 0', which is contrary to what the chemist had expected. The chemist had expected that the crude product mixture would exhibit optical activity, although the exact value would be different than that of the reactant. The chemist reevaluated the proposed reaction, Reaction

1, and decided that the temperature and the pH should be changed. Under the reaction conditions used, the proposed reaction proceeds by a mechanism that is susceptible to rearrangement. The chemist also failed to consider other reactions that compete with nucleophilic substitution at elevated temperatures, such as elimination. Under different conditions, the chemist found that optical activity could be
retained.

36. The chemist


reaction?

attempted

to carry out what type of

A. B.

Syl
S52 Ez

c. Er

D.

7.

Which of the fbllowing reaction mechanisms wouid explain a 0' optical rotation? A . An Spl reaction with inversion of configuration

B. C. D.

An Sy2 reaction with rearrangement An Sy2 reaction with inversion of configuration An E1 reaction with rearrangement

:r'right

by The Berkeley Review@

221

GO ON TO THE NEXT PAGE

8.

The two products from the proposed reaction are related in what manner?

42.

The following distribution of products can best explained by which ofthe explanations?

L,'e

Pas

. B. C. D.

They are enantiomers of one another. They are diastereomers ofone another. They are identical to one another.

",u,s.",
HrC

_:_
:rtu:

::[:
":

They are different orientations of the same meso


compound.

t/

CHr

su'
cHroH .->
maJor

:::::

\,)

"'ry,
minor

-u

":ira

J:i

'fui:

.F--

9. To

increase the amount of substitution reaction observed, the chemist would likely change the
temperature and pH in what way?

"tr

."

MI;:

. B. C. D.
A

Increase the temperature and decrease the pH Decrease the temperature and increase the pH

. B.
C

The intermediate undergoes a hydride shift.

\,
tr:
r,:

The methyl group on the carbon adjacent to


carbocation influences the attack of methanol. The methyl group on the carbocation influences attack of methanol. The intermediate undergoes a methyl shift.

Increase both the temperature and the pH


Decrease both the temperature and the pH

D.

0.

The small amount of substitution product isolated was


found to have both the OCH3 group and the CH3 group both on the same carbon. This can best be explained in
what way?

\r

A. B.
C. D.

First an elimination reaction took place followed


by
a

Markovnikov addition reaction.

First a Markovnikov addition reaction took place


lbllowed by an elimination reaction. A hydride shift occuned. A methyl shift occured.

llllus

.,:

unit;
urrTI
ill"ri-ullJ

41.

PBr3, when added to an alcohol, converts an OH group into a Br substituent with inversion of configuration at the carbon. The mechanism is an Sy2 substitution of the Br for the OH. What alcohol can be used in this reaction to produce the starting reagent in the proposed synthesis in Reaction 1?

. B. C. D.

(1R,2S)-2-Methylcyclopentanol (1R,2R)-2-Methylcyclopentanol (1S,2R)-2-Methylcyclopentanol (1S,2S)-2-Methylcyclopentanol

Copyright @ by The Berkeley Review@

228

GO ON

TO THE NEXT P{

f,assage

Vll

(Questions 43

49)

45.

Reaction

of

(2R,3S) 2-bromo-3-methylpentane with

Nucleophilic substitution is the process by which -nctional groups on an alkane can be exchanged. It is ,:mmonly viewed as a reaction which can proceed by one of '.o paihways. The first of the pathways is named S51 I for . rate dependence on one reactant. The second pathway is -.-erred to as Sp2 1br its rate dependence on two reactants.
-

ammonia yields which of the following products?

. (2S,3S) 2-amino-3-methylpentane B. (2R,3S) 2-amino-3-methylpentane C. (2S,3R) 2-amino-3-methylpentane D . A racemic mixture of A and C.


A

.:.e data in the tables were gathered for reactions in which the -:leophile was varied in two different experiments involving

,'r different electrophiles (one primary and the other .::iary). The reactions were monitored using UV

46. A reaction in which the specific

:3rtroscopy wl-rere the magnitude of the rate of disappearance : the peak corresponding to the electrophile varies directly ,h the reaction rate.

rotation of the starting material is + 32" and the product (which still contains a chiral center) is 0" is which of the following?
Sp1
SNr2

\ucleophile Reaction rate w/ CH3CH2CH2CI


H3CNH2 4.7 x 10-2 M/sec 4.2 x l0-2 M/sec 2.1 x 10-2 M/sec
8.2
47

A. B. C. D.

S5E1

SpE2

NH:
H3CSH H3COH

10-3 M/sec

Which of the following electrophiles is the best choice to react with NaOCH3 to yield the following ether? H

Table

Reactions with n-Propylchloride

\ ucleophile
H3CNH2

Reaction rate w/ (Hf C)f CCl


3.3 3.6

x l0-4 M/sec x x
10-4 M/sec

H:C-o . B. C. D.
A
(R)-2-chlorobutane (S)-2-chlorobutane (R)-2-aminobutane (S)-2-aminobutane

NH:
H3CSH H3COH

{r'"""'

3.3 x 10-4 M/sec


3.5

10-4 M/sec

Table

Reactions with t-Butylchloride

The reaction rates listed in Table I and Table 2 represent

: initial rate ol'cach reaction. All other variables that can :ct the reaction rate besides the nucleophile, such as
-Derature and concentration, were held constant between
.;S
.

48. Which of

the following graphs BEST represents the Clconcentration as a function of time for the reaction of ammonia with 1-chloropropane?

From the data presented in the passage, which of the

fbllowing is thc best nucleophile?

A. H3CNH2 B. NH3 C. H3CSH D. H3COH


1

Time

--> . A nucleophilic substitution A. B.


1'
2"

Time+

reaction proceeds MOST rapidly with the leaving group on what type of carbon?

c.

3'

D.

The reaction rate is independent of the degree of


substitution.

Time

---------_->

Time

---+

rvright

by The Berkeley Revier.v@

GO ON TO THE NEXT PAGE

49.

Monitoring the following reaction by optical roration


would yield which of the fbllowing graphs?

Passage

Vlll

(Questions 50 - 56)

HjCH2C
tz-

Br

+ HqCSH -+

HjCH2CH2C

-/

CH:

It is possible to exchange one functional group or. I substituted alkane for another by performing a nucleoplu.,: substitution reaction. There are two versions of nucleophi,r: substitution, known as S,y1 and S,rr'2. The number in ea:r reaction describes the rate dependence. The rate of an S.; reaction depends only on the electrophile concentration ;-; not on the nucleophile concentration. The rate of an S.,l reaction depends on both the concentration of nucleophile cr; the concentration of electrophile. The difference between ;* two mechanisms boils down to the sequence of the steps. -:
Sp1 reactions, a leaving group first leaves followed :'
nucleophilic attack of the carbocation intermediate. In S.,reactions, the nucleophile attacks the electrophilic carb':i

Time

------->

Time

mechanistic study, a secondary alkyl chloride w4s rsa1ei r,il two different nucleophiles to get the same ether product. ; j( rate data were collected for each.

forcing the leaving group off of the molecule.

I: ,

----------------

NaOEt

HOEr.0'

OEt

Reaction
L-l--->HOEI

Time

Time

_-_=>

pH=5,0"
Reaction
2

Tables I and2 show the initial rate data for three trials lbr each of the two recctions.

Reaction
Rate (M/s)
1.32

ICH3CH2ONa]
0.05 M 0.10 M 0.10 M

IC6H11Cl]
0.0-5

10-2 10-2

2.63

0.05 M 0.10 M
1

5.25 x l0-2

Table

Initial Rates for Reaction

Reaction
Rate (M/s)
1.93 1.95

ICHTCHzOHI
0.20 M 0.40 M 0.40 M

lCoHrrCll
0.05 M
0,0.5

x l0-3

x l0-3 3.83 x 10-3


Table

0.10 M
2

2 Initial Rates for Reaction

Based on the data presented, the nature of the mechan,n: (whether it follows Sy1 or Sp2 kinetics) can be determl:.:r The key to the analysis is to observe the change in rate a! 'rrw nucleophile concentration changes. As a rule, Sy2 reaci,,rn, are fil\tcr than S5 l reactions.

Copyright @ by The Berkeley Review@

230

GO ON TO THE NEXT

PAG.M

0.

Reaction 1 and Reaction 2 are best described as what


type of reactions?

5. Which of the following

reactions

is most likely

to

proceed by an Sy2 mechanism?

A. B. C. D.

Reaction Reaction Reaction Reaction

I is an Syl; Reaction 2 is an Sp2 I is an Sp2; Reaction 2 is an Spl I is an El; Reaction 2 is an Sp2 I is an Spl; Reaction 2 is anE2

A. H3CO- + (H3C)3CBr -+ B. H3COH + (H3C)3CBr -+


C. D.
H3CO- + (H3C)2CHCHBTCH3 -+ H3COH + (H3C)2CHCHBTCH3 -+

1. All of

the following are associated with Reaction

2
5

EXCEPT:

6.

Which of the following energy diagrams corresponds to


an exothermic Sp2 reaction?

. B. C. D.
A

inversion of the chiral center.


a

carbocation intermediate.

A.

B.

the rate depending on the electrophile.

a greater rate when a protic solvent is used than


when an aprotic solvent is used.

2. A

product mixture from a nucleophilic substitution

Rxn Coordinate

Rxn Coordinate

reaction on an enantiomerically pure compound that yields a product distribution of 87Vo R and 13% S can best be explained by which of the following?

A. B. C. D.

The reaction goes purely by an 5112 mechanism.

The reaction goes mostly by an Sp2 mechanism with some Sp1 mechanism transpiring. The reaction goes mostly by an Syl mechanism with some 51,12 mechanism transpiring.
The reaction goes purely by an SNI1 mechanism. Rxn Coordinate

3. If

bromine were used as the leaving group from the cyclohexane in lieu of chlorine, what effect would you expect on the rate? (Note that a C-Cl bond is stronger than a C-Br bond)

. B. C.

Both the S51l and Sy2 rates would increase.

The

Spl

rate would increase, while the Sp2 rate

would decrease.

The Sy1 rate would decrease, while the Sp2 rate


would increase. Both the Spl and Sp2 rates would decrease.

-1. Which of the following is the BEST nucleophile?

A. B. C. D.

tteCOH3COH

ClHCI

Sopyright @ by The Berkeley Review@

GO ON TO THE NEXT PAGE

Passage

lX

(Questions 5Z - 63)

58. Which of the following

compounds

is the MOST

::::tive electrophile,

- -ucleophilic substitution reactions, the reactivity of a: ..::,rophile dictates the reaction rate. The reactivity ofthe ..=::-rphile is correlated to the str.ength of the leaving group. ,..: :lectrophile with the better leaving group is the more
and thus reacts faster and undergoes the

reactive when treated with cyanide nucleophile?

favorabie nucleophilic substitution reaction. A good .::r ing group is stable once it has left the electrophile, so a
s:able leaving group does not readily donate its electron pair. Thts implies that a good leaving group is a weak base. The ueaker the compound is as a base, the stronger its conjugate acid is. As acid strength increases, the pKu of the icicl decreases. This ultimately means that the lower the pKu of the conjugate acid o1'the leaving group, the more reactive the electrophile.

::re

A. CH3CH2F B. CH3CH2OC6H5 C. CH3CH2SCH3 D. CH3CH2BT


59. Which of the following is the MOST srable leavin.
group?

A. HCN B. CN. C. H3CCH2SD. H3CCH2SH


60. The best explanation of why NaSCH3 is a bette: nucleophile than NaSCH(CH3)2 is which of ri_:
fbllowing?

Based on the pKs values, it is possible to predict the relative reactiviry of various electrophiles. The favorability of a nucleophilic substitution reaction can be approximated by comparing the pK2 value of the conjugate icid of the nucleophile with the pKu value of the conjugate acid of the leaving group. Equation 1 can be ernployed to approximate a reactivity constilnt, C, for the reaction: C = 10[pKr(H-Nucleophile) - pKo(H-Leaving group)]

. B. C. D.
A

Inductive effect of methyl is weaker than isopropr


Resonance affects only three carbon fragments

Equation

Hybridization of carbon varies with substitution Steric hindrance is less with the methyl gr<_rup

A reaction can be classified anywhere front very fbvorable to very unfavorable. The C value can be used as follows to approximate the tavorability of a reaction:

61.

The reaction of sodium cyanide with 2-iodobutane is: A. very favorable.

li:

If C >

103, then the reaction is very favor.able

If 103 > C > l, then the reaction is slightly favorable If I > C > l0-3, then the r-eaction is slightly unfavorable If 10-3 > C, then the reaction is very unfavorable
lists pKn values 1br the conjugate acids of sorne common leaving groups.
1

B. C. D.
62.

slightly favorable. slightly unfavorable.


very unfavorable.

Lll a

t\,,

Table

The reaction of methylthiol (CH3SH) wirh R-2-buta::, is which of the following?

a_

Acid
HI HBr

pKa -r0.5
-8..5

Acid
HCN
C6H5OH H3CCH2SH

PKa
9.1

10.0
10.5 15.1

A. B. C. D.

Very favorable. Slightly favorable. Slightly unfavorable. Very unfavorable.

HCI

-1.0
3.3

HF

Hzo

Table I
The pK6 data given in Table I can be userJ to pr.edict the

63. How can it be explained that fluoride, F-, is a bei:.r nucleophile than iodide, I-, in eiher solvent but a uc:l;l
nucleophile in alcohol solvent?

favorability of a nucleophilic substitution reaction. The


accuracy is within experimental error fbr substitution stuclies. Equation I does not hold well in protic solvents due to variations in nucleophile strength from hydrogen bonding.

B.
C

57. Which of rhe fbllowing


electrophile'l

compounds

is the

BEST

D.

A. B. C. D.

(CH3)3CI

In a protic solvent such as alcohol, F- is hinde::,rr by hydrogen bonding, and cannot migrate as we.. In an aprotic solvent such as ether, F- is hinde,:r by hydrogen bonding, and cannot migrate as wel. In a protic solvent such as alcohol, F- exhibits : I hydrogen bonding, so it cannot migrate as we1l. In an aprotic solvent such as ether, F- exhibirs - I hydrogen bonding, so it cannot migrate as we11.

r iir"i:

LLtt]tri

r.4

(CH3)jCBr
(CH3)3CCI (CH3)3CF

Copyright @ by The Berkeley Review@

GO ON TO THE NEXT

PACM

Passage X (Questions 64 - 70)


The strength of a leaving group can be deternined by

65

What difficulty arises if Reaction 1 is carried out using a secondary propyl electrophile instead of the methyl
electrophiie?

:ramining the reaction rate of either the S51 or Sp2 reaction. The rates fbr both reaction mechanisms show a linear jependence on the concentration of the electrophile. It can be .nferred from kinetic data that an electrophile with a better
.eaving group undergoes nucleophilic substitution at a faster .ate than an electrophile with a worse leaving group. A
:esearcher designed a study that varies the leaving group on an

A. B. C. D.

The electrophile exhibits more steric hindrance. The electrophile exhibits less steric hindrance. There is the chance of an elimination side reaction. The chance for an elimination side reaction is
reduced.

:lectrophile while keeping all other factors constant. Factors .hat influence the rate include temperature, nucleophile ,trength, solvent, and concentration. In a valid study, all of
.hese

66. Generally, nucleophiiicity and basicity run parallel

to

factors should remain constant between trials.

one another. What can be said about the correlation between the leaving grouprs basicity and the strength of the leaving group?

A difficulty that can arise with this

experiment is

A
B

. . .

:ompetition between substitution reactions and elimination

:eactions. To alleviate the problem of the competing :limination reaction, a one-carbon electrophile is chosen.
i\-hen the electrophile has only one carbon, the substitution :eaction must proceed via an S52 mechanism. Reaction I .hown below is a generic reaction representing each of the

D.

:irteen triai runs.

The less basic the leaving group, the better it is as a leaving group. The less basic the leaving group, the worse it is as a leaving group. The more basic the ieaving group, the better it is as a leaving group. There is no observable correlation between basicity and leaving group strength.

Nuc + CH3-X -+ Nuc-CH3 + X Reaction


1

o/. Which of the following


A
B

does NOT directly aff'ect the

strength of the nucleophile?

There were fbur nucleophiles and fbur leaving groups -sed in the experiment to account for sixteen combinations. lable I is a matrix showing the four nucleophiles ancl lbur :aving groups used in the generic reaction. The vaiue listed r each box in the table is the log of the reaction rate.
)JucJ

. The nature of the solvent. . The basicity of the nucleophile. C . The steric bulkiness of the nucleophile. D. The quality of the leaving group on the
electrophile.

\x \

-+

OSO3CH3 -3.82
1.39

I
-3.31

CI
-4.11

OCHr

H:N
CN.
H3CS-

68. Which of
No Rx
No Rx No Rx No Rx

1.tl
1.92

L92
)Aa
-5.10

the following could NOT be determined from a similar experiment carried out with an Sy1 reaction instead of the Sy2 reaction'?

-2.16
-5.21

H3COH

-4.80

Table

. B. C. D.

The strength of the leaving group. The strength of the nucleophile. The ellect of temperature. The effect of varying solvent.

The pKu values fbr the conjugate acids of the leaving ::oups can be used to estimate reactivity. The relative pKus ':e pKulg.6gCHr)

69. The best explanation tor the lack of any

observed

PKa(HCl) > PKa(HOSOTCH:) > PKa(HI)'

reaction when NH3 was added to H3COCH3 is that:

The rate for the reaction is measured in rnolar per second, :erefore the less negative the log value of the initial reaction

:.rte, the faster the reaction. Because the rate of an S52 -:action depends on both the nucleophile and the electrophile, .ris experiment can be used to determine the strength of both - -rcleophiles and leaving groups.

A. B. C. D.

NH3 is a poor nucleophile. NH3 is a poor leaving group.


-OCH3 is a poor nucleophile. OCH3 is a poor leaving group.

: J . The relative strength of the nucleophiles is best described by which of the fbllowing relationships?

70. A nucleophile can also be classifled


following?

as which

of

the

A. B. C. D.

CN- > CH3S- > NHr > HOCH3 CN- > CH3S- > HOCH3 > NH:
CH3S- > CN- > NH3 > HOCH3

A. B. C. D.
233

ALewisacid. A Lewis base.


An oxidizing agent. A reducing agent.

CH3S- > CN- > HOCH3 > NH3

Jopyright @ by The Berkeley Review@

GO ON TO THE NEXT PAGE

Passage

Xl

(Questions 71 - 77)

2.

It is theorized that under identical conditions, while rarying only the temperature of a solution, it is possible to
convert a reaction that yields purely substitution product (a

When a secondary alkyl bromide is treated with stronl base at 60'C, what type of reaction occurs?

A. B.

Sp1
S1q2

a greater activation energy than the competing substitution reaction). Elimination is carried out in the presence of eithel a strong bulky base (82) or a strong acid (Et) in solution. This implies that weak bases at low temperature react as nucleophiles rather than as bases. Reaction i, drawn below, was designed to verify this theory.
has
Br

substituted alkane) into a reaction that yields purely elimination product (an alkene). Studies have shown that favorable conditions for an elimination reaction involve higher temperatures (elimination is endothermic and often

c.

Er
Ez

D.

3.

The chirality of the reactant can BEST be described which of the following?

a.

A. 2R, 3R B. 2R, 35
C. 25, 3R D. 25, 35

D H$\l H:C

+ Nuc

--->

Product

CH:

Reaction

I
7

The idea was ro monitor the reaction by the optical rotation of plane polarized light. Only the SyZ reaction shows retention of some optical activity although the exact value of the specific rotation (tolp) is not predictable. Table I shows the final specific rotations for each of the six trials of Reaction 1, where either the nucleophile or temperature were varied. The initial specific rotation for the deuterated alkyl bromide is +24'.

4. In Trial II, the -28" optical rotation for the prodr_:l


mixture can BEST be explained by which of ::,:
following explanations'/

. The reaction goes purely by an 5612 mechanism. B. The reaction goes purely by an E2 mechanism. C . The reaction is an Sp 1 reaction with sor:r*
competing elimination reaction side
present. produ:::t

Trial
Trial I Trial II Trial III Trial IV Trial V Trial VI

Temperature

l0'c
60"c

Nucleophile [cr]p products NH: -JO NH: -28"


NaOH NaOH
-38"
0"

D.

The reaction is an Sy2 reaction with so:rr competing elimination reaction side proc::1.
present.

l0'c
60'c r0"c 60'c

NaNH2 NaNH2

0' 0'
7

Table

5.

Which of the following statements is true regarding


interchanging of stereoisomers for the reactant?

:u

The two cornpeting reactions when a good nucleophile is present in solution, in the absence of an acid, are thi E2 and 5512 reactions. The internal alkene is the predominant

When the enantiomer of the reactant is used. ::u


same geometrical isomers are formed, so either:nc reactant or its enantiomer may be used.

It is fbund that deuterium is less acidic than a proton due to the cleuterium isotope effect rooted in the shorter bond length associated with the deuterium-carbon bond. An E1 r.eaction can compete if the leaving group is a good leaving group and it is situated on a tel'tiary carbon.
product when elimination is observed.

B.
C

When the enantiomer of the reactant is us:'L different geometrical isomers are formed, sc ir
enantiomer cannot be substituted for the reactan:

. .

When a diastereomer of the reactant is used.

::r'c

71.

Based

predominantly what type ol reaction?

on the data listed in Tabie l. Trial I

same geometrical isomers are formed, so either :lrw reactant or its diastereomer may be used.

is

A. B.

When a diastereomer of the reactant is usac different geometrical isomers are formed. s:
diastereomer cannot be substituted for the reacta:r-

ur

Spl
5512

c. Er
Ez

D.

Copyright @ by The Berkeley Review@

GO ON TO THE NEXT

P.{GM

An elimination product for this reaction would have


what optical rotation?

Passage

Xll

(Questions 78

84)

A. B.

+24'
0"

D.

c.

-24' -42'

Not ali stereocenters are chemically reactive. When a reaction is carried out on a molecule with an unreactive stereocenter present, there exists the possibility that
diastereomers

will be formed in unequal quantities, due to the asymmetry in the molecule. This influence is referred to as

stereochemical control. Reaction


demonstrates this principle.

l,

drawn below,

CH2CHj
1. BHr(etrO)

CH2CHj
..t\OH

To support the theory that an E,2 reaction mechanrsm rs taking place, it would be best to use chiral centers on
which carbons of a deuterated 2-bromobutane?

---->

cHs oH (aq)
[cr]o +48' [o]o = +48'

2'fuoz

,'r/CH3

A. Carbon2only B. Carbon 3 only C. Both calbon 2 and carbon 3 D. Carbons l, 2 and 3

[cr]l = +18"

Reaction
The two products, Compound

A and Compound B,

are

nonsuperimposable and are

not mirror images. In

hydroboration, the hydroxyl group prefers to add to the less


hindered carbon of the n-bond, so the reaction is referred to as anti-Markovnikov. The hydroborane prefers to add to the less

hindered fbce of the molecule, which means that the two products are present in unequal amounts. Their percentages can be fbund using Equation 1

below. The percenlages

determined using this equation can be referenced against the quantitative values obtained using GC analysis.

Oobs=xacxa+(l -xu)cx6

Equation

uo6, is the observed optical rotation for the mixture and xu is the mole fraction of component a in the mixture.

The same phenomena can be observed any time a nucleophile is attacking an ,p2-hybridized carbon of an asymmetric molecule. This means that unequal amounts of diastereomers may be observed with Sy 1 reactions, electrophilic addition reactions, and carbonyl addition reactions. Reaction 2, shown below, is an Sp1 reaction
involving the forrnation of two diastereomers.

o
.$\

NHr

:NH:

...'St'

___l-

[s]o = +62' Reaction


fbr the product mixture?
2

lulo = +30"

78. For Reaction 2, what is the observed specific rotation


A
B

. . C. D.

Greater than 62" Between 46' and 62" Between 30" and 46' Less than 30'

:yright

by The Berkeley Review@

GO ON TO THE NEXT PAGE

9.

What would be the specific rotation for Reaction 1 Compound A is 807o of the diastereomeric mixture?

if

84. How many stereogenic centers (chiral

carbons) at:

rPa

present in the alkene reactant in Reaction 1?

A. 48' B. 42" c. 33" D. 18'

A. 0 B.
1

D.

c.2
3

0.

The two products in Reaction 2 are best described as:

. B. C. D.
A

enantiomers. epimers.
diastereomers.

identical.

Tlll_t L\.

1.

The products of Reaction 1 can be distinguished from one another by all of the following methods EXCEPT:

. B. C. D.
A

specific rotation. melting point. retention time on a GC.

IR spectroscopy.

82.

Which of the following structures best represents the most stable conformer of product A of Reaction 1?

A'

M.o ""# />cHt


,GH.CH3
OH

B'Hn

ay

HrcH2i
9Hr

C.

CH2CHj

cH2cH3

3.

What are the orientations of the three chiral centers in the reactant in Reaction 2, starting with the chirai center on which the iodine is attached and moving clockwise
around the cyclopentane?

A. B. C. D.

S,R,S

R,R,S
S,S,S R,S,S

Copyright @ by The Berkeley Review@

GO ON TO THE NEXT P{GmI

Passage

Xlll

(Questions 85 - 91)

85

All of the following reagents, when added to Compound 3a, result in a product with more asymmetric carbons
than compound 3a EXCEPT:

chemist sets out to perform a multistep synthesis.


step, Reaction 1, is a standard Diels Alder reaction.

lhe first

oo

q"ri,. "..\.
Compound 1 Compound 2

A. B. C. D.
8

Br2(l)/CCla(l)
Hz(e)/Pd(s)

cold KMnO4(aq) at pH =

l0

NH3(l)

Compound

6.

Which of the following is Compound 3b?


A.

o
..rr(

B.

Reaction I Compound 3 represents a mixture of enantiomers. The :irture undergoes a chirally specific laboratory technique to .oiate Compound 3a, shown below, from Compound 3b.

,,,,,(
HgC

o
HeC

o ..r( o

o
C. D.

HsC

o
Compound 3a In the second step, Reaction 2, Compound 3a is treated
rth meta-chloro peroxybenzoic acid, mcpba, in ether to form !ompound 4.
,'.

87

What laboratory technique would be MOST effective in obtaining a pure enantiomer from a racemic mixture?

o o
HgC

A. B.

Adding the mixture to a chromatography column filled with a gel with both enantiomers bound to it. Adding the mixture to a chromatography column filled with a gel with just one of the enantiomers
bound to it.

Compound Compound
Compound 5.

o
Reaction

C. Distilling the mixture using a vertical column filled with beads that contained both enantiomers

4 then undergoes o

3 to

bound to their surface.

form

D. Using a chirally pure carrier gas in a


chromatography experiment.

gas

o
8

-.:e("HicNH2H3;iq" o o
Compound 4
Reaction
3

8.

What is the major product, most abundant stereoisomer, formed in Reaction 2?

A.

B.

Compound

o
HrC"

In Reaction 4, Compound 5 is hydrolyzed using water at 90"C to form Compound 6.

H3CHN

HO Hec
Compound
6

OH OH

Copyright @ by The Berkeley Review@

237

GO ON TO THE NEXT PAGE

89. All of the steps in the overall synthesis shown in the


passage generate an optically active product mixture

Questions 92 through
descriptive passage.
92

EXCEPT:

A. Reaction I B. Reaction 2 C. Reaction 3 D. Reaction 4

The following molecule has which of the lollou i:g


stereocherhical orientations
?

ocH3

90. What is a likely side product of Reaction 3, if


amine is used?

excess

-oH A. 2R, 3R B. 2R, 35


C. D.
25,3R 25, 35

A.

B.
H3CN
HsC

93. The following molecule has which of


stereochemical orientations
?

the

NCFI3

CHr

C.

H?CHN

ocH3

HO
HsC

A. 2R, 3R B. 2R, 35
C. 25, 3R D. 25, 35
NHCFL NHCr-t

D.

H"CHN
HO
HsC

94.

The following pair of molecules can best be described

which of the following?

CHr

t?a HHO
91.
The final product mixture following Reaction 4 can best
be described as:
and

CH2CH3

HOH

A. B. C. D.

up = Q'. a diastereomeric mixture with ap = Q". a diastereomeric mixture with up * 0". a meso mixture with up * 0".

an enantiomeric mixture with

\ts

cH2cH3

CH:

. Diastereomers B . Enantiomers C. Epimers D. Anomers


A
Copyright @ by The Berkeley Review@

GO ON TO THE NEXT PAGI

f,r:

5.

How many stereoisomers are possibie for the molecule


1,2,3-trifl uoropentane?

100. Which of the following

compounds does NOT show

any optical rotation of planar light?

4.2
B.
4
6
8

c.
D.

A. B. C. D.

2R, 3R dibromobutane 2R, 35 dibromobutane 2R, 3R dibromohexane 2R, 35 dibromohexane

,6.

Addition of KMnO4(aq) at pH = l0 generates a vicinal diol with syn stereoselectivity. What does the addition of KMnO4(aq) at pH = l0 to E-2-butene would yield
which of the following?

A. B. C. D.

Two enantiomers Two diastereomers Two meso compounds (not identical)


One meso compound

7.

Which of the following compounds is optically active?

A. B. C. D.

2R,3s-dibromobutane 2R,4s-dibromopentane 2R,4s-dibromohexane


cis-1,3-dichlorocyclohexane

8. Which of the following


optically active?

compounds CANNOT be

A. 2-chlorocyclopentanol B. 2-chlorocyclohexanol C. 3-chlorocyclohexanol D. 4-chlorocyclohexanol

9.

Only twenty-five percent of synthesized isoleucine can be used biologically. This is best explained by which of the following explanations?

. B.

15Vo of synthetic isoleucine does correct side chain.

Only 25Vo exists as a zwitterion in the body. not have the

C. In synthesizing isoleucine, the two chiral centers D.


resuit in four stereoisomers being formed. Only one of the four is biologically coffect. In synthesizing isoleucine, the two chiral centers result in eight stereoisomers being formed. Only one of the eight are biologically correct.

1.D 1.C 13. D 19. B 25. D 31. A 31. D 43. A 49. A 55. C 61. A 61. D 73. B 19. B 85. D 91. C 91. C

2.8 8.C 14. A 20. A 26. B 32. D 38. B 44. A 50. B 56. B 62. D 68. B 14. D 80. C 86. A 92. A 98. D

3.B 4.C 5.8 6.8 9.B l0.B 11.B 12.4 15. B 16. A 17. C 18. D 21. B 22. C 23. B 24. A 21. C 28. D 29. B 30. B 33. A 34. B 35. A 36. A 39. B 40. C 41. B 42. B 45. A 46. A 41. A 48. A 51. A 52. B 53. A 54. A 57. A 58. D 59. D 60. D 63. A 64. A 65. C 66. A 69. D 70. B 1r. B 72. D 75. A 76. B 77. C 78. B 81. D 82. A 83. C 84. B 87. B 88. B 89. A 90. C 93. D 94. A 95. B 96. A 99. C 100. B

Copyright @ by The Berkeley Review@

LIFE BEYOND CHEM STARTS NOW

Stereochemistry

& Nucleophilic Subst'n Passage Answers


H:9
*HrN
H

arcs, the chiraiity for the two stereogenic carbons of isoleucine is determined as iollows:

Choice D is correct. Using the Cahn-Ingold-Prelog rules for substituent priority and drawing the appropria:.

Priority #4 in back (an as lsposition) Countercloskwise Arc .'. Chirality isS

Priority #4 in back (an as isposition) Countercioskwise Arc CH2CFI3 ChiralitY isS

"'

CozIsoleucine

Both of the chiral carbons have S chirality, which makes the compound 2S,3S. It wouid be swell of you:D. The chiral center on carbon two of an amino acid must be S according to the rules discussed in :: . Passage, so choices A and B could have been eliminated eariy. Regardless, the chiral center on the side cha-: of the amino acid must be solved for. Determining R and S ii actuiily rather simple when you get the hang .: it. The key is to find a method that works for you and hone it in by repeated .rr". '
choose
2.

Choice B is correct. If the side chain (carbon three of isoleucine) were to change the orientation of its chr.ot" o] the two chiral centers lyoL-: differ between the two compounds. The two stereoisomers (isoleucine and the other compound) would :: classified as diastereomers. The correct choice is B.
center while carbon 2 retained the orientation of its chiral center, then only

Choice B is correct. This question can be solved from straight memorization. Naturally occurring amino ac:-., are "L" as in life and natural. It is also stated in the passage that naturally occurring amino acids har'- : stereoconfiguration and that S stereochemistry is associated with L-amino acids. The besi answer is choice B

4.

Choice C is correct. With H as the side chain, carbon two of glycine (the alpha carbon) has two hydrog.. -, attached, thus there is no chiral center present on glycine. Neither of the two carbons in glycine have::-: different substituents attached. The absence of a chiral center in glycine results in an opticai rotation o: , Choose C and be happy. Choice B is correct. Enantiomers are nonsuperimposable mirror images, thus ail of the chiral centers ,:. different between the two structures. If all of the chiral centers are reversed, then the specific rotation shc*-: be completeiy reversed, which would lead to a value of +62" rather than -62". It would be terrific if you \{e:- :: choose B. Enantiomers always have the same absolute value for the specific rotation, only the sign (direc-* : of rotation) differs. Choice B is correct. As stated in the passage, the side chain of threonine is an alcohol (eliminating cholct .. and it is chiral (eliminating choices A, C, and D), all choices except answer B are eliminated. Only chor:t S contains a carbon that is asymmetric (chiral center).
Choice C is correct. Isoleucine contains two chiral centers, one for the alpha carbon and one in the side ch,*Plugging into the stereoisomer equation 2n where n is the number of chiral centers, there are four pos'-- -r stereoisomers for the isoleucine structure. Because isoleucine contains two chiral centers that must have s1.;- -,orientation, only one of the four stereoisomers wili have the correct chirality to be biologically usable. Ti.,-" ,. stated in the passage in two fragments. The best choice is C.

5.

6.

7'

8.

Choice C is correct. PBr3 converts the OH group of an alcohol ir-rto a Br group through an S1.;2 reaction. Bec: --,o the reaction is by way of an S52 mechanism, the chiral centers inverts. If you recall, chiral centers inr e :: Sp2 reactions, but not S1l11 reactions. This means the product shows stereochemistry of 35, 4R. Choose C fc: --:': sensation of correctness and satisfaction.
-Ln

Copyright @ by The Berkeley Review@

240

STEREOCHEMISTRY EXPLANATIC,i\]$

9.

Choice B is correct. If you rotate the original structure so that the hydroxyl groups are syn, then the two alkyl groups of the molecule also have syn orientation. The original alkene must therefore have the two alkyl groups cis to one another, resulting in Z geometrical alignment. Pick B for best results. HO

cH2cH3
h B{r

R"t"t

"

+
cH2cH3 Hydxroxyl groups are syn H
So, utt

cH2cH3

OH

"rlu*ust

have been cis

10.

Choice B is correct. Using the rules of priorities, the following is determined:


Prioritv #4 in back
Clockwise = R

74

cH2cH3

Priority #4 in front
Clockwise =
S

This makes the compound 3R,45. Choose B for optimal correctness and the satisfaction that goes with it.
11.

Choice B is correct. On the third carbon, the OH and H groups have interchanged, so that chiral center has changed. On the fourth carbon, the ethyl group, hydrogen and hydroxyl group have interchanged, so that chiral center has not changed. When only one out of two (some, not all) chiral centers change, it is not a mirror image, nor identical (superimposable), making the two compounds diastereomers. Pick answer B.

o
cH2cH3

HOH

il
M
mt

?.,

HH
Compound
12.

H
P

HO

CH2CH3

Mystery Compound

Choice A is correct. Using the rules of priorities, the following is determined.


H3C

H in front, so reverse chirality from R toS


m!

h
ile
:hr

H in back, so chirality is S as shown


This rnakes the compound 15, 25. Choose A.

[3.

&
for

hc

Choice D is correct. The active site of an enzyme carries out a highly specific function (reaction), so they must be highly selective in terms of reactivity. As implied by the passage, active sites are highly specific in terms of chirality. This eliminates choice A. Although it is not stated in the passage, you should know that the active site has specific dimensions, so it is size specific. This eliminates choice B. Active sites are highly specific for the functional groups involved in a chemical reaction, so choice C is eliminated. Because isotopes show the same chemical reactivity, enzymes are unable to distinguish isotopes. This means that enzyme active sites are nof isotope specific, making choice D the best answer.
@

lopyright

by The Berkeley Review@

241

STEREOCHEMISTRY EXPI,ANATIONS

't4.

Choice A is correct. Enantiomers have the exact same physical properties, such as boiling point, melting point, and density. This makes choice A correct and eliminates choices B and C. Enantiomers have the same magnitude for specific rotation, but with the opposite sign (in the opposite direction). This makes choice D an incorrect answer, and leave choice A as the best choice ini sea of many choices (weil maybe not many, but four.)

15.

Choice B is correct. A specific rotation for an enantiomeric mixture that is positive means that there is an of the enantiomer with the positive rotation (in this case the R enantitmer). If the two enantiomers were present in a fifty-fifty ratio, then the specific rotation would be 0". This eliminates answer choice C. Choice D is eliminated because an excess of th" S stereoisomer would result in an overall negative opticai rotation, making choice D invalid. If the mixture were in fact 100% of the R enantiomer, thei the specific rotation would be +32.2" ' This is not the case either, so choice A is eliminated. By the process of elimination. choice B must be the correct answer. The problem could have been solved mathematically as follows:
excess

%R (+32.2) + 7"5 (-32.2) = +L6.1. %R (32.2) - %S (32.2) = +16.1


32.2 (%R - "/oS) = 16.1 %R - %S = 0.5 = 50/o; and %R + %S = 700'/"

Thus2(%R-%S)=%R+%S 2%F.-2"k5=%R+%S
%R = 3 %S, so o/,R = 75oh and %S = 25"/o 16.

the value to be less than the literature value (given that the literature value is positive). Choices B and C both lead to an increase in the observed rotation. The standard rotation is based orrmonochromatic light from; sodium iamp. If monochromatic light from another source is used, it will interact differentlyiwith the compound, resulting in a different optical rotation. Whether the rotation is greater or lower is uniertain, bu: because it is possible that the rotation is greater, choice D is eliminated.
17.

Choice A is correct. Because one enantiomer aiways has the opposite optical rotation of the other enantiome: (one is negative and one is positive with the same magnitude), li Uotn enantiomers are present in solution, the absolute value of the rotation must decrease from the absolute value of the pure enantiomer. The observec optical rotation is an average of the enantiomers in solution. Therefore, choice A cannot account for an obser'ec specific rotation greater than that of the pure species, because the mixture of the two enantiomers would cause

Choice C is correct. The formula for units of unsaturation is Units of unsat = 2(#C)+1(#N)+2-1(#H)

For

compound with a formula of C11H21NO2, the calculation is: Unitsof unsat

2(77)+7(7)+2-7(27)

Pick C. Remember that oxygen is not included in the formula for degrees of unsaturation. Iflrou didn't rememte: the formula, then count the bonding electrons for the compound (44 for the eleven carbons, 21 for the hydrogerr. 3 for the nitrogen, and 4 for the two oxygens). There are a total of 72 bonding electrons, so the compound has lr bonds total. There are 35 atoms in the molecule, therefore only 34 bonds (minimum) are necessary to connect r;,e atoms and form the molecule. There are 36 bonds when only 34 are needed, thus there are two extra bonds. Thlong-winded path still leads to answer choice C.

22

- 4 -'

18.

Choice D is correct. The only way to do this problem is to count isomers systematically. Start with the longe=: chain (6) and list all the possible isomers. Then look at chain lengths of one less carbon (5) and list all of thopossible isomers. This continues until the possible backbones are depleted. For this question, the carbi: backbone is shown for all of the possible isomers. There are five isomers total, so pick D.
rongest chaln = 6 carbons rnnnact ^h.ilongest chain _ 5 ^-, = < carbons

longest chain = 4 carbons

c-c-c-c-c-c
Copyright @ by The Berkeley Review@

c-c-c-c-c ccccc

llrtt

c-c-c-c_c
242

\l
-

c_c_c_c c_c_c_c

STEREOCHEMISTRY EXPLANATIO\5

19.

Choice B is correct. It is stated that the OH substituent is on carbon 2. The OH is attached to an R chiral center while the Br is attached to an S chiral center. Pick B. The rationale is shown below:
I
Br

If the Br and H were

switched, the compound would be R. Because they are not switched, it must be an S.

,,F,,,Y

Li\

Looks like an S center from the arc, but because H is in

front, it reverses to R.

20.

Choice A is correct. A geminal diol has two hydroxyl groups on the same carbon (think of 'geminal' as being equivalent to gemini, meaning that the two OH groups are twins.) Because the two hydroxyl groups are on the same carbon, that carbon cannot be asymmetric. As a result, the molecule is achiral, so it cannot form optical isomers. This makes choice A the best answer. A vicinal diol has two hydroxyl groups on neighboring carbons (think of 'vicinal' as meaning vicinity or vicino, the Spanish word for neighbor). Because this ensures that the second carbon has a hydroxyl group, the second carbon must be asymmetric. As a result, the molecule is chiral, so it can form optical isomers. This eliminates choice B. Four carbon chains with a secondary functional group, whether it is an amine or alcohol, have a carbon with four different substituents (H, CH3, CH2CH3, and the group), so they are chiral. Being chiral, four carbon chains with a secondary functional group can form optical isomers. This eliminates choices C and D. Choice A is the best answer.

21.

Choice B is correct. In polycyclic systems, if there is no plane of symmetr1z, then all tertiary and quaternary carbons are stereogenic centers. In addition, any secondary carbons with a functional group are also stereogenic centers. The compound has two quaternary carbons, three tertiary carbons, and one secondary hydroxyl group. This means that there are six chiral centers. Given that there are two possible orientations at each chiral center and the compound is not meso, the total possible number of unique stereoisome rs ts 26, which is 64, Pick B, and feel the warmth of correctness.

22.

Choice C is correct. As indicated in Table 1 in the passage, diastereomers show different physical properties. The melting points of diastereomers are different, because the molecules pack into their respective lattice structures differently. The density is different between isomers, because the two diastereomers have different conformations that also pack into their respective lattice structures differently. The optical rotation of diastereorners must be different given the fact that they are identified by their differences in optical rotation. This eliminates choices A, B, and D. Because diastereomers are isomers, and they have the same molecular formula, thus they have the same molecular mass. The correct answer is therefore choice C. Choice B is correct. When using a polarimeter, an observed optical rotation of +233.0" and -1.27.0'would yieid the same reading (given that a full circle is 360"). To discern one optical rotation from the other, the sample should be diluted to reduce the rotation. If the actual optical rotation is in fact +233.0', then the lower concentration would show a rotation less than +233.0' (less clockwise). If the actual optical rotation is in fact -727.0", then the lower concentration wouid show a rotation less than -727.0" (less counterclockwise). If the solution concentration were cut in half for instance, the rotation would be either +116.5" or -63.5". The change in rotation can therefore determine the original rotation value. The only answer that indicates changing the concentration is choice B. Choice A is correct. If D-glucose has an optical rotation of +52.6' , then the enantiomer of D-glucose (L-glucose) must have an optical rotation of -52.6'. Mannose, the C-2 epimer of glucose (the diastereomer of glucose that only differs at carbon two) is neither of these two structures (L or D glucose), thus it does not show an optical rotation of either + 52.6' or -52.6". Mannose is chiral and not meso, so it cannot have an optical rotation of 0". The best answer is choice A. Choice D is correct. The number of stereoisomers (assuming that there is no meso structure), can be determined

23.

24.

25"

by raising 2 to the power of the number of chiral carbons (stereocenters). There are three chiral carbons (stereocenters) associated wiih penicillin V, thus there will be eight (23) stereoisomers for the structure of
penicillin V. The 2n formula represents the maximum number of stereoisomers. For every meso structure, you must subtract one frorn the total. Choice D is correct.
Copyright O by The Berkeley Review@
243

STEREOCHEMISTRY EXPLANATIONS

26.

Choice B is correct. The greater the density of a compound, the more tightly packed the compound is in its crystal lattice. This means that this question is a read-the-chart-to-fitr"a-tnu-d"r-rsity question. The mesc compound is less dense than the other compounds according to the data in the table, thus it must pack least tightly of all of the choices. The best answer is therefore choice B. Choice C is correct. Stereocenters can be identified quickly as sp3 carbons with four different substituents attached. In camphor, there are two carbons that fit this deicriptibn. The correct choice is answer C. Dral'n below is camphor with the two chiral carbons (stereocenters) labeled:
Top
Side

27.

CH:

28.

pure compound. This makes choice D the best answer.

Choice D is correct. If the hydrogen bonds in a pure compound were stronger than the hydrogen bonds in a mixture of stereoisomers, then the intermolecular forces would be greate"r in the p,rr"- .orripound. As a consequence, the pure compound would have the higher melting point, which is the exact opposite of the premise. Choice A is an untrue statement, and thus it is eliminated. Physical properties, such as'melting point result from intermolecuiar forces, not covalent bonds. Enantiomers are no -ore lit ely to form covaleni bonds with one another than diastereomers. Choice B is an untrue statement that does not explain the observei melting points. Choice B is eliminated. Covalent bonds are not affected when a .o*porr1d melts, so choice C should be eliminated. The best explanation is choice D, because when the moleculei pack more tightly, ther exert stronger forces on each other. Because the enantiomeric mixture (R with S) has a higher -"itlng po*, than the pure enantiomer (R with itself), the forces are in fact stronger in the enantiomeric mixture than the

]!

29.

Choice B is correct. To separate enantiomers from one another, the medium must be chirally pure (be chiral with only one enantiomer present). The best method is the use of chiral gei in colum. .hro^uiography. The two enantiomers exhibit different migration rates down the column, because the two adhere to thJcoiumn to a different extent. Choice A is a valid methocl. Distiilation r,vill nof separate the two enantiomers, so choice B is the correct answer choice. The mixture can be selectively crystallized with a pure R or pure S compound. ThL is often carried out with tartaric acid. Enzymes are chiral, so chiral .o-porrr.i, pass thiough an enzyme filter at different rates. This makes choice C and D vaiid.

30.

Choice B is correct. To prevent the product mixture from being racemic, chirality must be present in the transition state. A change in temperature does not affect the chirality of the products. Choice Cis eliminated. The concentration does_not affect ihe alignment of the molecules in the transition state, only the frequency witi. which the reactants collide to form the transition state. This eliminates choice D. The presence of a chiracenter in the solvent cloes not affect the chirality of the transition state unless the soivent is involved in the transition state. Choice A cannot be eliminated yet, but it is not a likely choice. The only change that vr'i-definitely affect the distribution of enantiomers is the addltion of a chiral catalyst which affects the transition state. This is the whole idea behind the activity and specificity of enzymes (chiral catalysts) i-r. biological reactions. The best answer is therefore choice B.
Choice A is correct' Both enantiomers have oniy one stereocenter, therefore choices C and D are eliminatec The lemon fiavored extract is the structure on the left, which has its lone stereogenic center in R configgration This eliminates choice B and makes choice A the correct answer.
.)At

31"

Copyright @ by The Berkeley Review@

STEREOCHEMISTRY EXPLANATIONS

enantiomer). The boiling point, density, and solubility in a given solvent does not vary between enantiomers. These physical properties can vary between diastereomers, but enantiomers are identical in their packing and intermolecular forces, unless the alcohol solvent is chiral and optically pure. The best answer is choice D.

Choice D is correct. The one physical property that definitely changes with the chiral center is the optical rotation. The specific rotation of a product mixture measures its enantiomeric purity (percentage of each

Choice A is correct. The regio-chemistry is correct in all of the answer choices (the boron has attached to the less hindered carbon of both alkenes). Hydroboration, you may recall, proceeds with anti-Markovnikov regioselectivity. The reaction calls for the R enantiomer, which has the alkenyl group sticking out of the plane (the structure on the left in Figure 2, labeled "lemon odor", has R stereochemistry). This eliminates choices C and D. Because the boron and the hydrogen add syn to one another, the methyl substituent on the ring must be trans to the bridging boron. This eliminates choice B and makes choice A the best answer.
nrC4,

First addition of BH3(Et2O)

Second addition

of BH"(EI2O)

H-B
R stereocenter

->

Trans addition to methyl

Choice A

Choice B is correct. The two enantiomers have a different flavor (and thus different smell), so they must bind the olfactory receptors differently. This eliminates choices C and D. Because they recognize the difference between the two enantiomers, they too must be asymmetric (and thus chiral). The correct answer is choice B.

Choice A is correct. The isoprene molecule has no chiral centers, so the product is a racemic mixture. The molecule listed in the question as an alternative reactant has a chiral center present that will influence the orientation in the transition state. The product would contain three chiral centers (one new chiral center formed and one each present in the two reactant molecules). The product would be present in a mixture of diastereomers. Diastereomers cannot be present in a racemic mixture, therefore the product mixture would not be racemic for the new reaction. The best answer is choice A.

Choice A is correct. The proposed product is a mixture of diastereomers formed from a nucleophilic substitution reaction. Because two diastereomers are formed, the proposed reaction must have been predicted to proceed by an Sp1 mechanism. The best answer is choice A.

Choice D is correct. Any nucleophilic substitution reaction that proceeds with either inversion of a chiral center or retention of a chiral center remains chiral, and thus is optically active. This eliminates choices A, B, and C. The product from elimination is an alkene, which has no chirality, and thus no optical activity. The correct answer is elimination, choice D. The rearrangement clause, although true, has little bearing. i8.
Choice B is correct. There are two chiral carbons present on each compound. The tertiary carbon retains its chirality between stereoisomers, but the methoxy carbon has different chirality in the two stereoisomers. This means that one out of two of the chiral centers differs, making choice B the best answer.

39.

Choice B is correct. To increase the amount of substitution product that forms, the amount of elimination product that forms must be reduced. The elimination reaction is by way of an E1 mechanism, because no strong base is present. To reduce the amount of E1 product, the amount of acid should be reduced, and the reaction should be carried out at a lower temperature. This means that the temperature should decrease (elimination is favored at higher temperatures) and the pH should increase. The best answer is choice B.
Choice C is correct. There is a secondary electrophile, protic solvent, and a poor nucleophile present, so the substitution reaction takes place by an SX11 mechanism. The presence of the methyl and methoxy groups on the same carbon can be explained by rearrangement. There is a secondary carbocation formed when the leaving group leaves. When the hydride shifts, a tertiary carbocation forms. This is a more favorable intermediate, so the reaction proceeds via a hydride shift before the nucleophile attacks. The best answer is choice C.
245

40.

Copyright @ by The Berkeley Review@

STEREOCHEMISTRY EXPLANATIONS

41,

Choice B is correct. Given that the conversion of the hydroxyl group into a bromine goes with inversion o; stereochemistry, the alcohol must have opposite chirality at carbon 1 as the bromoalkane in Reaction 1. The passage states that the reactant is (1S,2R)-2-methylbromocyclopentane, so the alcohol precursor must har.e chirality of 1R and 2R. The best answer is B. The reaction and chirality is shown below.
H in back on C-2, so take as is: R

.)'rrOU
H in front on C-1,
so reverse it: R

42.

PBre .,rrr\OH ______*

Choice B is correct. The preference for the major product can be attributed to steric hindrance in the transition state (the transition state is asymmetric). The carbocation is sp2-hybridized,, so it is planar. The only steri: hindrance comes from the adjacent methyl group (which is above the ring). The adjacent methyl, by beir.e above the ring, influences the nucleophile to attack from below the ring. The best answer is choice B.

#
CFL

Br

I
illl'

43. Choice A is correct. The first

reaction in Table 1 proceeds by way of an 5512 mechanism, because the electrophile is a primarv alkyl halide. The second reaction in Table 1 proceeds by way of an 51'11 mechanisrr. because the electrophile is a tertiary alkyl halide. Only the rate of an 51112 reaction depends on the nucleophile, therefore to determine the best nucleophiie, the data from the reaction with the primary aik-, halide (first set of data in Table 1) should be used. The best nucleophile is the compound that has the fastes: reaction rate, which according to Table 1, is methylamine (H3CNH2). Choose A if you're a table believer.

44'

Choice A is correct. From the data in Table 1, the faster reactions are observed with 1-chioropropane an othe: electrophiles, so it is a safe and valid assumption that the reaction with the primary aikyl halide proceed. most rapidly. Choose A.

45.

Choice A is correct. The reaction can proceed by either an S511 or Sl112 mechanism with a secondary alk,,'halide as the eiectrophile. If the reaction were to proceed by an 51112 mechanism, then the product wouid be the 25, 35 stereoisomer. Only the stereocenter from which the bromine substituent left underwent a change r,.. chirality (inversion), thus only that stereocenter will show a change in its orientation. If the reaction were tc proceed by an S1r11 mechanism, then the intermediate would undergo rearrangement, and thus the ammoni" wouid attack the third carbon leaving an achiral product. Given the answer choices, choices B and C car.: form, and there is no achirai choice, therefore the reaction proceeded by an SNI2 mechanism. This means tha: inversion of the second carbon will transpire to yield 25,35. Choose A.
Choice A is correct. To have optical activity and lose it during the course of the reaction eliminates choice B because the 51'12 reaction proceeds with inversion (thus an optically active product is formed). By definitior the product mixture as described in the question is racemic. A racemic product mixture is associated with th.
51111

46.

reaction. Pick A.

47,

Choice A is correct. The ether product has S stereochemistry as drawn and was formed by a substitutic:. reaction using NaOCH3 as the nucleophile. Sodium methoxide (NaOCH3) is a strong base, and thus it is also a good nucleophile. Because the nucieophile is good, the reaction must have proceeded by an 51112 mechanisn'. Because the final product has S stereochemistry, the starting material (electrophile) must have had F. stereochemistry to form the S product from inversion. This eliminates choices B and D. For the substitutior. reaction to proceed, the electrophile must have had a good leaving group. Ammonia is not a leaving group therefore the chlorine leaving group is the better choice. This makes the correct answer choice A.
Choice A is correct. The reaction proceeds at the fastest rate during the first segment of the reaction becaus: initially the concentration of L-chloropropane (a reactant in the rate determining step) is greatest and bot:r reactants (nucleophile and electrophile) are depleted over time. Chloride anion is the leaving group, thus rls concentration will increase over time. A11 of the graphs show increasing concentration. Over time, the concentration of 1-chloropropane gradually decreases, thus the reaction rate decreases graduallyi this resulin a slower production of Cl- anion. Graph A best depicts this gradual decrease in reaction rate. Choose A anc
be a wunder sfudent.

48.

Copyright @ by The Berkeley Review@

246

STEREOCHEMISTRY EXPLANATIONS

49.

Choice A is correct. Because the electrophile is a tertiary alkyt halide, the reaction is an Sry1 reaction, which with racemization. The optical rotation of the product mixture following an 5511 reaction is 0". This eliminates choice B. Graph C (the schizophrenic graph) shows the correct final optical rotation, but no reaction will proceed with the erratic change in rotation. Choice D shows that the reaction proceeds at a constant rate until the reaction is complete. This would be seen with a zero order reaction, not a first order reaction' The SIrI1 reaction is first otder, so answer choice D is eliminated. Graph A best depicts the gradual loss of chirality. The optical rotation will never switch to the opposite sign unless there is inversion *hi"h it not possible with an sI.I1, reaction. Choose A and make your support group proud.
proceeds

50.

Choice B is correct. The ether product shown in both Reaction 1, and 2 is the result of a substitution reaction, not an elimination reaction, so choices C and D are eliminated. The data in Table 1 correlates to Reaction 1. Because the rate of the reaction varies directly with both the concentration of the nucleophile and the concentration of the electrophile, Reaction 1 must be an 5512 reaction. The data in Table 2 correlates to Reaction 2' Because the rate of the reaction varies directly with the concentration of the electrophile, but does not vary with the concentration of the nucleophile, the reaction must be an 51111 reaction. The rate of an 51111 reaction only depends on the concentration of the electrophile, and does not vary with the concentration of the nucleophile. This means that you must choose B to live up to your potential. Choice A is correct. Because the rate of Reaction 2 varies directly with the concentration of the electrophile, but does not vary with the concentration of the nucleophile, Reaction 2 must be an St'tr1 reaction. an S51t reaction undergoes racemization, not inversion, so choice A cannot apply to Reaction 2. In addition, Reaction 2 has no chirality, so choice A is invalid. The best answer is choice A. A carbocation intermediate corresponds with an Sp1 reaction, so choice B is valid and thus eliminated. All nucleophilic substitution .eactions, whether it is an 5111 or S1rJ2 rnechanism, have a rate that depends on the electrophile. Choice C is valid, and thus eliminated. A protic solvent helps to stabilize the carbocation intermediate and the leaving group, so a
5511

protic solvent increases the rate of an A is in fact the top dog of choices.
52.

reaction. This makes choice D valid, and thus eiiminateJ

it.

Choice

Choice B is correct. If the reaction were to proceed purely by an 5512 mechanism, the product would be 100% R, because the reactant is enantiomerically pure and the SNI2 reaction results in complete inversion. If the reaction were to proceed purely by an S1r11 mechanism, the product mixture would be 50% R and 5Ao/" S, because the reaction goes through a planar carbocation intermediate resulting in a racemic mixture. The mixture is 87% R and 13% S, which is closer to the products of an 51'12 mechanism than the products of an 51111 mechanism. It is not a pure reaction so the best answer is choice B. Choice A is correct. Because the carbon-bromine bond is weaker than the carbon-chlorine bond, it is more easily broken. This makes the bromine a better leaving group than chlorine. An alkyl bromide is therefore a more reactive electrophile than an alkyl chloride. With a better electrophile, the reaction is faster for both the Sp1 and the Sir12 mechanisms, because they both depend on the electrophile. This makes choice A correct. Choice A is correct. The strongest nucleophile is most willing to donate its lone pair to carbon. The answer choices include two conjugate pairs. The conjugate base is the better nucleophile of the pair, so choices B and D are eliminated. HCI is a strong acid while methanol is a weak acid, so methoxide is a stronger base than

53.

i4.

chloride. This means that methoxide is more willing to donate electrons than chloride, and therefore
methoxide is the better nucleophile. Pick A and see your score improve.
f5.

(and tertiary electrophiles proceed via the Sry1 mechanism). Choice C is better than choice D, because methoxide is a better nucleophile than methanol. Pick C for optimal results.

Choice C is correct. An SNJ2 reaction favors a primary electrophile over second aty ot tertiary electrophile, and a good nucleophile is required. Choices A and B can both be eliminated, because the electrophiles are tertiary

56,

5512 reaction proceeds by way of a one-step mechanism, which eliminates choices A and reaction has the products at a lower energy level than the reactants, which eliminates choice D and leaves choice B as the correct answer. The apex of the graph represents the transition state, and the absence of a valley on the graph implies that there is no intermediate for the reaction. Choice A is an exothermic 5111 reaction, choice C is an endothermic Sp1 reaction, and choice D is an endothermic Sp2 reaction.

C. An exothermic

Choice B is correct. An

Copyright @ by The Berkeley Review@

241

STEREOCHEMISTRY EXPLANATIONS

57.

Choice A is correct. The best electrophile is the compound with the best leaving group. The best leaving group is the leaving group with the strongest conjugate acid. In this case, iodide is the best leaving group, beciuse HI (hydroiodic acid) is the strongest conjugate acid of the choices listed. This makes choice A the best answer. Choice D is correct. This question requires that you identify the best electrophile. Again, the best electrophile is the compound with the best leaving group. The best leaving group has the strongest conjugate acid, which rn this question is the bromide leaving group. The ranking of the conjugate acids for the leaving groups are: HBr > HF > HOC6H5 > HSCH3. It is in your best interest to choose D.

58.

59.

Choice D is correct. A leaving group, once it has left an electrophile, must have at least one lone pair (as a result of the heterolytic bond cleaving). This stipulation eliminates choice A, because the carbon of the cyanic acid has no lone pair, and the nitrogen does not interact with a carbon to be a leaving group. The most stable leaving #oup is the weakest base. Of the three choices left, CH3CH2SH has the strongest conjugate acic (CH3CH2SH2+ is a stronger acid than CH3CH2SH and HCN), thus CH3CH2SH is a weaker base than CH3CH2S- and CN-. CH3CH2SH is the weakest base of the choices remaining, therefore it is the best leaving group. Choose D to score a point in this contest of point collecting.
Choice D is correct. The difference between the two molecules is the alkyl group. According to the question, the smaller molecule is the better nucleophile. The inductive effect would predict that the electron donating methyl groups would make the larger alkyl group more electron donating and thus more nucleophilic. This is the opposite of what is observed, so choice A is eliminated. Resonance is not a factor, because there is no rsystem. Choice B is eliminated. The hybridization is sp3 in both cases, so the difference in nucleophilicit',cannot be attributed to hybridization. This eliminates choice C. Steric hindrance predicts that the smaller nucleophile has less interference in the transition state, thus iL is a better nucleophile. In this case, steric hindrance plays a larger role than the inductive effect in the reactivity of the nucleophile. The correct ansn'er is choice D. Choice A is correct. Sodium cyanide is a good nucleophile and iodide is a great leaving group, therefore thi. reaction shouid be very favorable. The difference between their pKu values ts 79.6, which implies that the Ksq for this reaction is near 1919'6 = 4v1919. This defines a reaction that goes completely to product, whic-i. supports the evaluation that the reaction is very favorable. Do the correct thing, choose A. Choice D is correct. Hydroxyl groups are terrible leaving groups. Thiols are average to poor nucleophiles. The reaction between a poor nucleophile and an electrophile with a poor leaving group should be very unfavorab:e by intuitio.n. The difference between their pKu values is -5.2, which implies that the K"O for this reaction'. near 10-5'2 = 6 x 10-6. This defines a reaction that stays predominanlly as reactant, rirhich supports the evaluation that the reaction is very unfavorable. Choose D,

60.

61.

62.

63.

Choice A is correct. In protic solvents, there is hydrogen bonding, so choice C is eliminated. Equally, in a:, aprotic solvent, there is no hydrogen bonding, so choice B is eliminated. Hydrogen bonding affects fluoride an: not iodide, so the fact that fluoride is a worse nucleophile than iodide in alcohol implies that hydroger, bonding reduces the nucieophilicity of fluoride. This can be attributed to hindrance to migration caused t'i hydrogen bonding. The best answer is choice A. Choice D can be eliminated, because if it were true, then the opposite relative nucleophilicity would be observed for fluoride and iodide.

64.

Choice A is correct. The strength of a nucleophile can be measured by its reaction rate in a second-orde: nucleophilic substitution reaction (an SX12 reaction). The nucleophiles are listed in the first column of Table 1 Any other column can be used to determine the relative strength of the nucleophiles, because all other factors i the reaction are constant. The less negative the value in ihe table, the faster the reaction, and therefore th.r better the nucleophile. The CN- has the lowest value in all three columns, so the best nucleophile is CN-. Thri eliminates choices C and D. The question now is to determine whether the ammonia (NH3) or methanc(HOCH3) is the better nucleophile. The stronger nucleophile is ammonia, because in each column, the 1e.. negative value is associated with ammonia. The best answer is choice A, CN- > CH3S- > NHg > HOCH3.

Copyright @ by The Berkeley Review@

STEREOCHEMISTRY EXPLANATIO\,(

55.

Choice C is correct. As mentioned in the passage, the methyl electrophile is chosen to avoid the complication of the competing elimination reaction. It is not possible to form a dbuble bond with only one carbon in the reactant (at least two carbons are required for the formation of a double bond). This makes choice C the best answer. Steric hindrance increases when using the isopropyl electrophile in lieu of the methyl electrophile, but that is not necessarily a difficulty. The effect should be uniform u"ross the reaction chart, so choices A and B can be eliminated. Choice A is correct. The best leaving group is the functional group that takes electrons from carbon and retains them to the greatest extent. Retaining electrons can also be viewed as not sharing electrons. By not sharing electrons, an ion or molecule can be viewed as being a weak base. The strength of i leaving group is generally correlated (in a linear fashion) to the strength of the conjugate acid of the leaving group. aJ un icid becomes stronger, the conjugate base becomes weaker. This implies that it is valid to compare the strength of the ieaving grouP in a linear fashion with weakening base strength. This makes choice A, "The less basic the leaving group, the better it is as a leaving group,' the best answer. Choices B and C are essentially the same answer, therefore they should both be eliminated.

66.

67-

Choice D is correct. The strength of a nucleophile can vary with many reaction features. Depending on its nature, a solvent hinders a nucleophile to a varying degree. For instance, if a solvent is capable of fbrming hydrogen bonds, then it will hinder the attack of nucleophiles that are capable of forming hydrogen bonds-. This can be seen in the differing nucleophilic strength of halides as they are observed in apiotic and protic solvents. The fluoride is the strongest nucleophile of the halides in aprotic solvents whilelt is the weakest nucleophile of the halides in protic solvents. This eliminates choice A. The nucleophilicity of a compound can be correlated to its basicity in terms of a Lewis base. Generally, for a nucleopiile that is more basic than another, it is the better nucleophile of the two, with steric hindrance responsible for most deviations from that pattern. This eliminates choice B. The strength of a nucleophile reduces with increasing bulk. This implies that nucleophilicity can vary with steric hindrance, which eliminates choice C. The only answer choice left is choice D. The leaving group is independent of the nucleophile in nucleophilic substitution reactions. This makes choice D the best answer.
Choice B is correct. Because the rate of an Sp1 reaction depends only on the leaving group breaking free from the electrophile in the rate-determining step, the nucleophile is irrelevant .to the reaction rate f-or an Sy1 reaction. The strength of a nucleophile cannot be determined by a rate study in which the nucleophile does not influence the rate. The rate can vary with changes in the leaving group strength (which can be viewed as changes in the electrophile), the temperature (temperature always affects the rate of a reaction), and solvent. It is only the strength of the nucleophile that cannot be determined from the reaction rate data of an S1q1 reaction. Choose B for yet another chance to flash a happy "I just got another one right" smile.

68.

59.

Choice D is correct. Choice B can be eliminated immediately, because NH3 is the nucleophile and not the leaving group. Choice C can be eliminated immediately, because -OCH3 is the leaving group (if it were to react) and not the nucleophile. The data in Table 1 shows that no reaction was observed each time that the electrophile was dimethyl ether (CH3OCH3). O.t the other hand, the data in Table tr shows that ammonia (NHg) is a reactive nucleophile with the other three electrophiles used in the experiment (CH3OSO3CH3, CH3I, and CH3CI). This implies that the lack of reactivity can be attributed to the electrophile rather than the nucleophile. The leaving group in the cases where dimethyl ether is the electrophile is a methoxide anion (-OCH3). Pick D to prosper and score... well score at least. Choice B is correct. By definition, a nucleophile is a lone pair donor, which by yet another definition is Lewis base. This makes this question a freebie and the correct answer choice B.
a

70.

71.

Choice B is correct. From the low temperature of the reaction and the retention of optical activity in the product, it can be inferred that the reaction proceeds by way of an 51112 mechanism. Elimination and S1g1 reactions produce products that lose their optical activity. Choices C and D can be eliminated because the alkene products would show no optical rotation because they lose both stereocenters in the formation of the alkene. The products from a reaction proceeding by an Sry1 mechanism in this case would be a mixture of diastereomers (not enantiomers), which would lead to an optical rotation close to zero. Enantiomers are obtained if the reactant is symmetric. The best (although not perfect) answer is choice B.
249

Copyright @ by The Berkeley Review@

STEREOCHEMISTRY EXPLANATIONS

72.

Choice D is correct. At high temperature, the predominant reaction is elimination. This eliminates choices C and D. Because a strong base (NaOH) was used, the mechanism must have been an E2 rather than an E1 mechanism. The best answer is choice D. This can be verified by looking at Trial IV in Table 1, which shou' identical reaction conditions as the reaction in the question. The loss of optical rotation in the product impl,v that the reaction was an elimination reaction. A substitution reaction would yield some sort of optical activity. You have to know that base infers that the mechanism is E2. Choice B is correct. There are two chiral centers in the reactant, located at carbons two and three. This can be

/5.

inferred from the answer selections. The number four priority is hydrogen on both chiral centers and conveniently it is in the back in both cases. The first chiral center (carbon two) has priorities bromine > deutero ethyl > methyl which form a clockwise arc. The first chiral center is thus R. The second chiral center (carbon three) has priorities ethylbromide > methyl > deuterium which form a counterclockwise arc. The seconi chiral center is thus S. The solution is drawn below:
I

u'\.
2

n"}V
H
carbon 2 = R The correct answer is 2R,35, which makes choice B correct.
74.

carbon 3 =

Choice D is correct. If the reaction proceeded purely by an SNI2 mechanism, then the optical rotation would b,e the same as was observed for trial I(-36"), a purely 5512 reaction. This eliminates choice A. An eliminatior. reaction would yield an optically inactive product so the optical rotation observed for an elimination reactio:. would be zero. This eliminates choice B. The optical activity observed implies that an S52 reaction must har-e been occurring to some degree. The reduction in optical activity must be attributed to the Presence of sone impurities (from some side reaction). The best answer is choice D. Choice A is correct. Because the enantiomer is a mirror image of the reactant, it forms a transition state wher, -: is eliminating that is a mirror image of the reactant's transition state. This means that the products are ali: mirror images, but without stereogenic centers, they can be rotated to match as identical compounds. Th; symmetry presents itself in the product as an identical geometric isomer. Because a diastereomer varies at on-i one chiral center, it is not a mirror image of the reactant when it is eliminating. This asymmetry presents itse'' in the product as d"ifferent geometric isomers. This may not make sense in words so the drawing below shou s the products.

75.

BrH

Br rotation to correct alignment CH"

H
Elimnation

H$FT HsC
Reactant

H$tjl
HsC

'

CHs

"

rtrr"g

with b"* - HEC

CH:

Cis Methyls

Br

H
Elimnation

HsC

"T,>+"
Diastereomer

CHs

rotation to correct ---------=--+ alignment

H\ty
H:C

with -

a strong base

HaC
HeC

D Trans methyls

><

CH.

-..,i';*" H
Enantiomer
choice A.

CHs

rotation to correct aiignment

Cis Methyls

The enantiomer forms the same products while a diastereomer forms geometric isomers, making the best ansl,:s" Copyright O by The Berkeley Review@

250

STEREOCHEMISTRY EXPLANATION$

76.

Choice B is correct. The elimination product would be optically inactive, because it will lose both stereocenters (chiral carbons) upon forming the alkene. This means that the specific rotation of the alkene product is zero making choice B the best answer. Choice C is correct. An E2 mechanism involves the removal of a proton alpha to the leaving group which then shifts the electrons into a n-bond forcing the leaving group off. The mechanism is concerted, iiplying that both events occur simultaneously. This results in very predictable geometry with respect to the p.oirct a-lkene. To support the operation of the E2 mechanism, both carbons that become involved rn the n-bond should be chirally labeled to trace the reaction. If the mechanism is specific, then the final product is a specific geometrical isomer. This is why the reactant is deutero labeled at iarbon three. Therefore-, the best answer is choice C.

77.

78'

Choice B is correct. Because the electrophile is a tertiary alkyl halide, Reaction 2 proceeds by way of an S.u1 mechanism. The two products have specific rotations of +62' and +30' respectiveiy as writtln. A fifty / fifty mixture of the two products would yield an observed specific rotation of +46'" (the average of the values ior the two diastereomers). Because of steric hindrance from the bonds to the six membereJring, the nucleophile (It{Hs) prefers to attack from the backside of the molecule. This makes the major product Lompound A, the first product in Reaction 1. The -ujgt product has a specific rotation of +62', thus'the specific rotution of the mixture is closer to +62" than +30". The specific rotation is between 46' and 62', which -ik", choice B the best
answer.

79'

Choice B is correct. The specific rotation for the mixture is found by taking a weighted average of the specific rotations for the components in the mixture, which is essentially what Equition 1 does. Becarise there is more of the component with an ct = +48" than the component with an o = +18', the averaged value should be closer to +48' than +18'. However, because it is not purely one component, the specific rotation for the mixture must be less than +48" (within the range of +33" to +48'). The best answer is therefore +42', choice B. The exact can be determined mathematically as follows: 'alue
oobs = 80% (+a8") + 20'/" (+18') = 38.4 + 3.6

= 42'

80'

Choice C is correct. The two products formed in Reaction 2 both have identical bonds to one another and three chiral centers each. In comparing the two sttuctures, only one of the three chiral centers differs, making the two structures diastereomers. If some, but not all, of the chiral centers differ between two stereoiso^"rr,1h"y are not superimposable nor are they mirror images. This, by definition, makes them diastereomers. The best answer is choice 9 Jh" term epimers describes diastereomers that differ at one chiral center, but it applies specifically to the backbones of sugars. Choice D is correct. The two products for Reaction 1 are diastereomers of one another so they have different specific rotations' This means that they can be distinguished by their different specific rotation values. This

81'

eliminates choice

A.

Because they have different geometry (asymmetry),

molecules, therefore

respective solid lattices and thus they exhibit different melting points. Tiis eliminates choice B. Because of their varying asymmetry, they bind a gas chromatography differently (due to a difference in steric "ot.,*n hindrance) and thus they show different retention times on the gas chromatogiapher (GC). It can be inferred from the last sentence in paragraph two of the passage that the two products hurr" dlff"r"nt retention times on the gas chromatographer (GC), since the concentration values can be d.etermined (and thus verified in this example) using the GC. This eliminates choice C. Infrared spectroscopy measures the type of bonds in the

thef pack differently into their

spectroscopy' Diastereomers have identical bonds as one another. The best answer is choice D even if vou have no idea what infrared spectroscopy does.

it is difficult, if not impossible, to diitinguistr ihe two diaster"o*ers by

infrared

82"

Choice A is correct. The first product has the methyl and ethyl groups both up and the hydroxyl group down. To retain the chiral integrity shown in Figure 1, the chair confoimatior,, have ethyi and mjhyl Lp with the hydroxyl down. This eliminates choices B and C, which have the -.rrt opposite geometry (ethyl and exact methyi are down and the hydroxyi group is up). The most stable conformer hai the least steric repulsion, which idealiy occurs if the three substituents are in equatorial orientation rather than having the three substituents in axial orientation. Because the substituents are on adjacent carbons, they alternate upTdown/up, which allows all the substituents to be equatorial. This makes the best answer choice A.

Copyright @ by The Berkeley Review@

251

STEREOCHEMISTRY EXPLANATIONS

83.

Choice C is correct. The three chiral centers are determined as follows:

41

Substituent number four is in front,

clockwise to

so the arc must be reversed from


counterclockwise.

Substituent number four is in back, so the arc is correct as drawn. The

This makes the chiral centerS.

counterclockwise chiral center S.

arc makes the

Substituent number four is in back, so the arc is correct as drawn. The

counterclockwise chiral center S.

arc makes

the

The correct answer is S, S, S, which makes choice C the correct answer. 84.

Choice B is correct. In the alkene reactant in Reaction 1, only one carbon has four different substituent= attached, therefore only one carbon is chiral. The best answer is choice B. The chirality is specified in th= reactant for the ring carbon with the ethyl substituent attached.

85.

Choice D is correct. This question is asking to determine which reactions generate new chiral centers. Wher treating an alkene with bromine, two bromides are added, one to each of the double bond carbons. The result that the two double bond carbons become new chiral centers. This eliminates choice A. When treating a: alkene with hydrogen gas in the presence of a catalytic metal such as palladium, two hydrogen atoms aradded, one to each of the double bond carbons. The result is that the double bond carbon with the methyl grou: becomes a new chiral center. This eliminates choice B. When treating an alkene with permanganate in bas-: water, two hydroxyl groups are added, one to each of the double bond carbons. The result is that the two doub,i= bond carbons become new chiral centers. This eliminates choice C. When Compound 3a is treated with a goc: nucleophile like ammonia, the acid anhydride is cleaved. The result is that one carbonyl group becomes amide and the other becomes a carboxylic acid. Neither of these groups are chiral, so no new chiral centers a::

formed. The best answer is choice D.


86.

Choice A is correct. It is stated in the passage that Compound 3 represents a mixture of enantiomers, -: Compound 3b is the enantiomer of Compound 3a. Enantiomers are mirror images of one another, therefore ail --: their chiral centers differ. There are two chiral centers in Compound 3, so Compound 3b must have the opposr:: chirality of Compound 3a at these two chiral centers. This is true in choice A, so choice A is the best answer.

87.

enantiomers. This requires that there is chirality (asymmetry) in the technique. If the mixture tra\ = j through a column with one enantiomer attached, the other enantiomer is likely to adhere to the column as -: travels. This means that one enantiomer r,r'ill migrate quickly while the other travels slowly. This ma-.choice B a strong answer. If the column has both enantiomers bound, then both of the free enantiomer: : solution will be hindered by the column, slowing equally. This does not result in separation. Choice A ' eliminated. The same logic can be used to eliminate choice C . If the distilling column has beads with b - -: enantiomers, then both of the free enantiomers in solution will exhibit the same affinity for the beads, a:: will not separate as well. Choice C is eliminated. The compounds do not interact with the carrier gas in ;* chromatography to any notable extent. This means that choice D will do nothing to help separate --- { enantiomers. Choice D is eliminated.
Choice B is correct. Reaction 2 converts Compound 3a into Compound 4, using a peroxyacid. The oxirane ::L: form from either side of the n-bond. Flowever, because of the steric hindrance above the ring with the aLk.- * (caused by the six-membered ring), it is preferential to form the epoxide on the back side. Both bonds to oxr-;*nl in the epoxide must be on the same side, so choices C and D are eliminated. Because of steric hindrance, ci^ - -:*
B is a better structure than choice

Choice B is correct. To separate a compound from its enantiomer, the technique must select for one of the

t',"":

88.

A.

The best answer is choice B.


252

Copyright O by Thc Berkeley Review@

STEREOCHEMISTRY EXPLANATIO\$

89.

answer is choice A.
90.

Choice A is correct. It is stated in the passage that Reaction 1 generates a mixture of enantiomers. When enantiomers are formed, they are formed in a racemic mixture. The result is that the optical activity is zero, because the two isomers cancel one another out. This means that Reaction 1 generates an optically inactive product mixture. Once the enantiomer mixture is separated, isolating Compound 3a, the subsequent reactions all start with an optically active starting material, resulting in optical activity in the end. Reactions 2 and 3 form diastereomers and Reaction 4 cleaves the anhydride to retain the same chirality as the starting material. Because the materiai is optically pure at the start, the product of reaction 4 is also optically purel The best

Choice C is correct. Methyl antine, like all amines, is a good nucleophile. Tl-re first equivalent reacts with the most reactive electrophile, which in this case is the epoxide ring. However, an anhydride is also highly reactive. In the event excess amine is added, it can easily add to the carbonyi, cleaving the anhydride and generating a carboxylic acid and amide. Choices A and B can be eliminated immediateiy, becar-rse the threemembered ring is highly unstable and will not reform once broken. The OH group is a bad ieaving group, so it is not likely that an amine will substitute for the hydroxyl group, once the airhydride is cleaved, so choice D is elimrnated. The result is that choice C is the best answer. Choice C is correct. In Reaction 2, a mixture of stereoisomers is formed that is never resolved. As a result, the reactant in Reaction 4, Compottnd 5, represents a mixture of stereoisomers. Two of the chiral centers are the same in all of the stereoisomers, so they cannot be enantiomers (where all of the chirai centers differ). This eliminates choice A. The product is not meso, it does not have an internal mirror plane, so choice D is eliminated. Because the mixture starts with optical activity, and Reaction 4 does nothing to affect the chirality, the product mixture in Reaction 4, Compound 6, must be optically active. This makes choice C the
best answer.

91.

92.

Choice A is correct. If you are well versed ir-r r-rsing the thumb technique, then you can place your thumb in the direction of the C-H bond and curl your fingers from priorities 1-2-3 using a right hand for both stereocenters. This makes both centers R. For some, it may be easier to rotate the molecule to the side view rather than using the Newman projection to deterrnine the chirality. From the side perspective, it is much easier to see the three dimensional orientation of the molecule. As shown in the determination below, the molecule has 2Ii., 3R stereochemistry. Choose A; be a chem star.
CFI?

ocFi3 OH

H in back; R stereocenter

OH
H in back; R stereocenter

93-

represents an H coming out at you ln a three dimensional perspective. Hence, whatever arc you determine from the Fischer projection must sutsequently be reversed to get the chirality of the stereocenter. in this example,

Choice D is correct. With Fischer projections, you must remember that when an H is drawn on the side, it

both chiral centers generate ciockwise circles in a two-dimensional perspective. But after reversing ihe clockwise circles to counterclockwise becanse of the hydrogen atoms are in fiont, both centers have S chirility. Choose D and be a scholar.
CH"

1/l rF+T-

| '

Clockwise arc with H in front;S stereocenter

-Fs.,,.
\
ocF{3
Copyright O by The Berkeley Review@

\y

OH

Clockwise arc r,r'ith H in front;

S stereocenter

253

STBREOCHEMISTRY EXPLANATIONS

94.

Choice A is correct. On the second carbon, the OH and H groups have interchanged, so that chiral center differs between the two stereoisomers. On the third carbon, the OH and H groups have not interchanged, so tha: chiral center is identical in the two stereoisomers. On the fourth carbon, the ethyl and methyl groups hale interchanged, so that chiral center differs between the two stereoisomers. When two out of three (i.e., some but not all) of the chiral centers differ, the two stereoisomers are neither superimposable, nor are they mirro: images of one another, which defines diastereomers. Pick A and be jovial. Choice B is correct. In the molecule, carbons two and three are chiral centers. However, carbon one is not r chiral center, because it contains two equivalent hydrogen atoms. The maximum number of stereoisomers i. equal to 2n where n is the number of chiral centers in the molecule, which in this case is 2. This yields a total c: fot.l (22) stereoisomers. Be stellar and choose B. Choice A is correct. Syn addition of equivalent to a trans double bond in an alkene results in the formation c: two enantiomers (specifically the R,R and S,S enantiomers) as shown below. Choose A for best results. A mes: compound can be obtained from syn addition to a cis alkene, where the aikyl groups are equal on the alkene.

95.

96.

H.C \

Syt-,addition Trans reactant (asymmetric) (symmetric)


97.

/.,-, \ H CFL "

Fi

KMnOalaq) __-+

pH=10

. H:cr,l /-.,,,"H c' HrcY \"'*, ,,,Tt H HO


CH:

HO

\_J

OH

CFI"

OH

As).rnmetric products

Choice C is correct. If a compound has stereocenters but is not optically active, this implies that the compou:.: must be meso. To be meso, a compound must have a mirror plane in the molecule about which the chiral centef. are evenly displaced. This mirror plane slices the molecule into two equivalent halves. 2R,3S-dibromobuta:.= and 2R,4S-dibromopentane each have equivalent mirror halves (thus mirror symmetry) so they are both me>: compounds. Cis-1,3-dichlorocyclohexane has equivalent halves as wel1, thus it is a meso compound. Or-. 2R,4S-dibromohexane does not have two equivaient halves. The correct answer is thus choice C. Choice D is correct. 4-chlorocyclohexanol contains no stereocenters (chiral carbons), therefore it cannot rota.:= plane polarized light. The result is that 4-chlorocyclohexanol is not optically active. Choose wisely ar.: choose D. The other three compounds have two chiral carbons and therefore are all opticaily active. It:' possible for the compound (4-chlorocyclohexanol) to exhibit isomerism in the form of geometrical isomers r.-' and trans), but geometrical isomers do not rotate plane polarized light. Choice C is correct. Isoleucine contains two chiral centers, one for the alpha carbon and one in the side chaPlugging into the stereoisomer equation 2n, wl-rere n represents the number of chiral centers, there are tc..: possible stereoisomers for the isoleucine structure. Because isoleucine contains two chiral centers that mu-. have specific orientation, only one of the four stereoisomers has the correct chirality to be biologically use:..., Choose C to be a successful point collector.

98.

99.

100.

Choice B is correct. For a compound with chiral centers to be optically inactive, it must be meso. and tt-. contain an internal mirror plane of symmetry. The molecule must have R, S chiraiity to be meso. T-,-: eliminates choices A ar-rd C. The mirror plane must be through the C2-C3 bond according to the chiral centers -: the ansll'er choices. This mirror plane is possible only with butane, so the correct answer must be choice B.

Copr ri-sht O

by flie Berkeley Review@

STEREOCHEMISTRY EXPLANATIO\i

Section HV
Hydrocarbon Reactions
by Todd Bennett

Alkanes

a)
b)

Structures and Physical Properties i. Aliphatic and Cyclic Alkanes Alkane Reactivity i. Free Radical Llalogenation ii. Mechanism iii. Energetics iv. SelecLivity

llydrocarbon R.eactions a) Elimination Reactions

Geraniol

i. Ez Mechanism ii" E1 Mechanism iii. Carbocation Stability iv" Electrophilic Addition v. Regioselectivity vi. Steneoselectivity
vii.
Stereoisomer Formation

b) 1,Z-Additi<ln versus 1,4-Addition Me c) Pericyclic Reactions i. Diels-AlderReaction ii. Cope Rearrangement iii. Claisen Rearrangement
Limonene

Terpenes

b) c) d)

a)

Classification (Carbon Count) l'ormation and Synthesis Connectivity Spectroscopic Properties

Speciahzing in MCAT Preparation

Ilydroctrrbons and Keactions

Section Goals

Recoanize the nomenclature associated with the alkanes and dienes.


The Greek prefixes and suffixes associated with the hydrocarbons must be common knowledge. Know the terminology so that when names are presented in passages, you can draw the structure or recognize the structure in an answer choice. Be able to recognize the positioning of the n-bonds in conjugated n-systems.

aB

Know the general mechanism for free radical addition to an alkene. Free radical halogenation of an alkane follows a simple three-step process. A free radical halogen is generated by the initiation reaction, requiring heat or light. The free radical halogen starts the propagation reactions which undergo a continuous chain reaction until it is terminated. Termination i"uitidtlr involve two free radicals coming together to form a bond. The overall reaction is a sum
of the propagation steps.

@?

Know the mechanisms associated with elimination reactions.


You must know the E1 and pZ mechanism in detail. Know which reactant, solvent, and catalyst combination will result in which mechanism. As a rule, E1 occurs in acid and E2 occurs in strong base. Rearrangement can offer complications in an E1 reaction. E2 reactions require the acidic proton and leaving group to be anti to one another.

Know the general mechanism for electrophilic addition to an diene. There are several reactions which involve the addition of an electrophile to one of the n-bonds in a diene. You must recognize the diffrence between the 1,2-addition and 1,4-additions of haloacids
and water across a conilgated diene. Recognize the stereochemical results associated with electrophilic addition reactions of allienes.

"?

Know the general mechanism for electrocyclic reactions of polyenes.


The most common electrocyclic in organic chemistry is the Diels-Alder reaction. You must recognize it and be able to predict r6latir.e reictivity of diflerent reactions based on electronic factors"and steric hindrance.'You must also recognize that the six-electron transition state in the Diels-Alder

reaction is also found in sigmatropiclearrangements like the Cope and Claisen rearrangements. You should have a basic understanding of n-systems.

Recoqnize the isoorene subunits in natural terpenes. Terpenes are natural oils derived from a five-carbon species. Isoprene is 2-methyl-1,3-butadiene, altliough the biologically active molecule is a derivative. Terpenes can be extracted and/or distilled from natural products.'You should be able to identify the isoprene units in biologically common
terpenes and terpenoids.

Organic Chemistry

tlydrocarbons and Reactions

Introduction

features. Common .reaction types incrude acid-base reactions (section 1), nucleophilic substitution (section 3), free radical reactions (typically halogenation of an alkane or polymerization of an alkene), elimination, electrophilic Jdditlon, electrophilic aromatic substitution, electrocyclic reactions, oxidation-reduction reactions, and carbonyl substitution reactions (section 5). There are other types of reactions, name reactions, which we shall address by functiond grtlp. However, in this section, we shall address the reactions of alkaner ur,a tt-r" reactions of n-bonds, as classified by their mechanisms.
propagation, and termination. Whether we consider free radical halogenation of an alkane or free radical polymerization of an alkene, the first step of ih" pro.u5 is initiation. Initiation entails the homolytic cleavage of a weak'sigma bond in only a small number of the molecules present in tne reaction mixture. In halogenation reactions, the halogen-to-hilogen bond is cleaved to initiate the

The crux of organic.chemistry centers on reactivity. In organic chemistry there are many classes of reactions. Th"y are organized accoiding to mechanistic

Free radical reactions follow three basic mechanistic steps: initiation,

synthesized.

reaction. In free radical polymerization, the oxygen-to-oxygen bond of a peroxide initiator is cleaved to generate a low concentratiot of fr"e radical compounds. The product of a free radical halogenation reaction is an alkyl halide. An alkyl halide can und"ergo nucleophilic substitution reactions (as we saw in the stereochemistry section) and elimination reactions. Elimination reactions result in the formation of a n-bond, which opens the door to a plethora of new reactions. starting with an alkane, ,oori u.ry compound can be

Although the MCAT test writers do not require that you memorize the reactions of alkenes, as they have required in the past, you are expected to know the general classes of reactions that involve n-bonds. when the n-bond of an alkene ac-ts- as a nucleophile by attacking an electrophile, this starts an electrophilic addition reaction. The ru-bond is not limited to being part of an alkene, u, lt .u1 also be part of a diene, triene, or any other conjugaied system of multiple rcbonds. V\4ren there is more than one n-bond in the system, we must consider the possibility of the addition reaction occurring at different reactive sites. The nucleophiiic n-bond can also be found on benzene, although the aromaticity of the n-system drastically reduces the nucleophilicity of a r-bond. However, if the electrophile is strong enough, the n-bond oi benrene can attack it and eventually e>rqhange the electrophile for a hydrogen, retaining the aromaticity of the system. This is known as an electrophilic aromatic substitulion. The last reaction we shall consider for n-bonds is the Diels Alder reaction, an electrocyclic reaction involving the addition of an alkene to a conjugated diene to form a cyclohexene moiety.
From this section on, we shall focus on the reactivity of organic compound.s. To best prepare for the MCAT, your goal shourd not be to memorize every reaction,

but instead, learn a few simple, common mechanisms and have a conceptual picture of how they work. If you can summarize the contents of this secti,on in terms of the nucleophilicity of a n-bond, no matter what compound contains the rc-bond, then you have a solid grasp of the topic at the MCAr level. Reaction details will be provided in the passage, so from this point on, know the general reaction and work on your information extraction skills by reading graphs,
tables, and data charts,

Copyright O by The Berkeley Review

257

Dxclusive MCAT Preparation

Organic Chemistry
Alkanes

llydrocarbons and Reactions

Alkanes

Alkane Structure Alkanes contain only carbon and hydrogen atoms, and all of the carbons have spr-hybridization. AII of the hydrogens in aikanes use s-orbitals to form bonds. Alkanes are held together exclusively by o-bonds. In any alkane, there are only two types of bonds present: C-C bonds (which are osD3-s'3) and C-H bonds (which are oro3-r). Both types of bonds present in alkanis aie shown in Figure 41 as molecular orbitals.

cffic
orp3-tp3

@.
os-sP3

Figure 4-1

Alkanes can be classified as either aliphntic (straight chain form) or cyclic (containing a ring in its structure). Aliphatic alkanes hive a molecular formula of CnH2r, 12 while cycioalkanes with one ring have a morecular formula of C,.,H2r.,. For each additional ring in a cyclic alkane, the molecule has two less hydrogen atoms. Table 4-1 lists some common linear alkanes and cyclic alkanes up to-ten carbons, It should be noted that at least three carbons are needed to form a ring.
C^H2', * 2 for linear alkanes (no rings)
Methane
Ethane

CHa
CzH.e

Hexane Heptane Octane Nonane


Decane

CoHra

CzHrc
CgHrs
CsHzo

Propane
Butane Pentane

csHs
C+Hro

CsHrz

CrcHzz

C^H2., for cycloalkanes (with one ring) Cyclopropane Cyclobutane Cyclopentane Cvclohexane CsHo C+Hs Cycioheptane Cyclooctane Cyclononane Cyclodecane

CrF{u
CsHro CsHra
CroHzo

CsHro CoHrz

Table 4-1

Copyright

by The Berkeley Review

258

The Berkeley Revie*

Organic Chemistry

tlydrocarbons and Reactions

Alkanes

Alkane Properties The physical properties of concern associated with an alkane are its solubility features, its density, its melting point, and its boiling point. Alkanes are hydrophobic, nonpolar molecules. They can also be defined as lipophilic, which implies that they are highly soluble in oils such as the lipid membrane of a cell. As with all compounds, their physical properties vary with mass and branching. As the molecular mass increases, both the boiling point and melting points increase. As the branching increases, the boiling point decreases. Table 4-2 shows the physical properties of several aliphatic alkanes. From the data in Table 4-2, the effects of mass and branching on the physicai properties can be
observed.

Isomer
CH+ H3CCHe H3CCH2CH3 H3C(CH2)2CH3 H3C(CH2)3CH3 H3C(CH2)aCH3

Name Methane Ethane Propane Butane


Pentane

Boiling Point
-762"C

Melting Point
-183"C

Density

Mass
76.043 30.070
44.097 58.124

(g/mL) (g/mole)

-89'C
-42"C

-183'C -787'C -138'C -130'C


-95.3'C -754"C
118"C 0.557 0.659

0"c
36'C
69"C

72.151 86.778
86.1.78

n-Hexane

(HrC)zCH(CHD2CH3
H3CCH2CH(CH3)CH2CH3
(H3C)3CCH2CH3

2-Methylpentane 3-Methylpentane 2,2-Dimethylbutane 2,3-Dimethylbutane


n-Heptane n-Octane n-Nonane
n-Decane

60'c
63'C

0.654
0.676 0.649

86.778 86.178 86.778


100.205
1.1.4.232

50"c
58'C 98'C
126"C 151"C

-98"C -729"C
-90.5"C

(H3C)2CHCH(CH3)2
H3C(CH2)5CH3 H3C(CH2)6CH3 H3C(CH2)7CH3

0.668 0.684
0.703

-57"C

-54'C -30"c

0.778
0.7362

128.259

H3C(CH2)sCH3

174'C

142.286

Table 4-2
Example 4.1

What is the molecular weight of 2,2-dimethyl-4-propyl-5-cyclopentylnonane?

A, 228.21,grams,/mole B. 266.51, grams/mole C. 268.57 grams/mole D. 280.54 grams/mole


Solution First, we must determine the number of carbon atoms and hydrogen atoms. Dimethyl accounts for two carbons, propyl accounts for another three, pentyl accounts for another five, and nonane accounts for nine. The compound contains 19 carbon atoms totai. Because of the "cyclo" in the name, there is one unit of unsaturation. The one unit of unsaturation implies that there are 38 hydrogens (two less then the 40 that would be present in an aliphatic, linear alkane). The molecular mass is thus 19(12) + 38 = 266, choice B.

Copyright O by The Berkeley Review

259

Exclusive MCAT Preparation

Organic Chemistry

llydrocarbons and Reactions

Alkanes

Alkane Reactivity
Alkanes undergo a minimal number of reactions, and the few they do undergo involve free radical chemistry. The two reactions of concern are free radi&l halogenation (more specifically bromination, using Br2, and chlorination, using C12) and combustion. Reaction 4.1 is the free radical chlorination of methane.
CH+(g) + Cl2(g)

-il*

CH3CI(g)

+ CHrCtr(1) +

CHC13(I)

+ CCla(t) +

C2Cl6(s)

80%

70%
Reaction 4.1

minor

minor

minor

Free Radical Halogenation of Alkanes

A free radical halogenation reaction starts with the addition of activation energ\to cleave a halogen-halogen bond (the weakest bond in the reactants) to form two free radical halogen atoms. A free radical is an atom, such as a halogen or carbon, with one unpaired electron. According to the octet rule, most atoms wish to have eight valence electrons. In a free radical, there are only seven, so free radicals are highiy reactive. Figure 4-2 shows a 3-D perspective of four alkyl free radicals with the p-orbital filled with one electron. The three substituents are drawn slightly below the carbon atom, because the free radical molecule is slightly trigonal planar due the electrostatic repulsion from the single electron. Because the electron can exist in either lobe of the p-orbital, the substituents carr be angied up or down, so the average of the two trigonal pyramidal forms is planar' The stability of a free radical depends on its substitution. Tertiary free radicals are more stable than secondary free radicals, which in turn are more stabie than primary free radicals. Figure 4-2 shows the relative stability of the alkyl free radicals.

o-6Xii.* ' "-6*gF


3" Free

' o-6*9,# ' "Gx'+


1' Free

>H.

radical

2' Free

radical

radical Methyl free radical

Figure 4-2
Once a free radicai halogenation reaction has been initiated by the addition cr light (activation energy), a free radical halogen atom then attacks an alkane anc abstracts a hydrogen from the alkane to leave behind an alkyl free radical. The halogen free radical abstracts the first hydrogen it encounters, but because alkr. free radicals can react with other alkanes, over time the distribution favors the formation of the more stable tertiary free radicals. The conversion from a primary free radical into a tertiary free radical is shown in Figure 4-3 below.

t'.\
H*f
H:C

cH.cH,.

\-^*i
HsC

n,\
JQ_
cH"cH1----*

n,.\
cH"cH^

HaC
Figure 4-3

.,,,|

HsC

.|

cH^cH

Primarv Free Radical

Tertiarv Free Radical

The reaction in Figure 4-3 heavily favors the formation of product, because the tertiary free radical is substantially more stable than the primary free radical.

Copyright

by The Berkeley Review

The Berkeley Revieu

Organic Chemistry

flydrocarbons and Reactions

Alkanes

Free Radical Mechanrsm The mechanism for a free radical halogenation is a chain reaction process involving an initiation reaction, followed by propagation reactions, and ultimately a termination reaction. An initiaiion reaction breaks a covaient bond in a homolytic fashion to form two free radicals, so an initiation reaction goes from no free radicals to two free radicais. Homolytic cleavage is djfferent from heterolytic cleavage in that each atom gets a single eiectron (resulting in free radicals) as opposed to the more electronegative atom getting both bonding electrons (resulting in a cation and an anion). In halogenation, it is the haiogenhalogen bond that is broken. Propagation steps involve the abstraction of an atom from a neutral molecule by a free radical to form a new free radical. Propagation reactions include the consumption and formation of a free radical, so a propagation reaction goes from one free radical to one free radical. In free radical halogenation, there are two propagation steps. Termination occurs when two free radicals combine to form a neutral, stable molecule. Termination steps involve the consumption of two free radicals, so a termination reaction goes from two free radicals to no free radicals. There are several possible termination reactions in a free radical halogenation reaction, most of which form minor side products. The steps for a generic halogenation of an alkane are shown below as Reactions 4.2,4.3, and 4.4.

Tnitiation:

ila

--------->

X.+.X
R. + H-X
.X

Reaction 4.2 Propagation:

tGi*
-c?a
n! *
'.R

----->
Reaction 4.3a

-------> R-X +
Reaction 4.3b

Termination:

------->
Reaction 4.4a

R-R

nli.x

------->
Reaction 4.4b

R-X

*li.*
Overall

--------> x-x
Reaction 4.4c

Reaction: R-H + X-X

--------+ R-X + H-X


Reaction 4.5

The sum of the propagation steps for a free radical halogenation reaction gives the overall reactiort, Reaction 4.5. The initiation step is brief, for just a split second at the start of a free radical reaction. However, the propagation steps continue until the reaction is quencired or the free radicals are completely consumed in termination reactions. There are always multiple termination steps possible. One of the possible termination steps is the reverse of the initiation reaction. A termination step is any reaction that combines two free radicals to generate a stable compound. Termination steps are responsible for several minor side products.

Copyright O by The Berkelev Revieu

26r

Exclusive MCAT Preparation

Organic Chemistry
Example 4.2

tlydrocarbons and Reactions

Alkanes

Which of the following reactions represents an initiation step?

A. B. C. D.

+ Br. -) H3CCH2CH2BT + H3CCH2CH3 -+ H3CCH2CH3 + Br2 -) H3CCHBTCH3 + Br. HOBr --> HO. + Br.
H3CCHzCHz. H3CCH2CH2. (H3C)2CH. +

@3C)2CH.

Solution In an initiation reaction, free radicais are generated, so the product side has more radicals than the reactant side. Choice A is eliminated, because there are fewer radicals on the product side, making it a termination reaction. In choices B and C, there is the same number of radicals on both sides of the reaction, so they are propagation reactions. This elimrnates choices B and C. In choice D, the reaction goes from zero free radicals on the reactant side to two free radicals on the product side, so it is an initiation reaction.
Example 4.3

In the free radical chlorination of ethane, butane is a minor side product. Horr'
can this best be explained?

A. B. C. D.

An ethane molecule attacked an ethyl chloride in a nucleophilic substitutior, reaction to form butane. An ethyl free radical removed a hydrogen from an ethane molecule. Two ethyl free radicals combined to form a new sigma bond.
The carbon-carbon bond of ethane was cleaved during initiation to forrn methyl free radicals, which rapidly combine to form iong-chain alkanes.

Solution An ethane molecule has neither a lone pair of electrons nor an available pair o: bonding electrons to share (like a n-bond), so it is definitely not going to act as a nucleophile. This eliminates choice A. An ethyl free radical can definitel,,' remove a hydrogen from an ethane molecule. However, that does not resuit ir. the formation of butane, it simply regenerates the same molecules.

H3CCH2. + H3CCH3 -+ H3CCH3 + H3CCH2.


Choice B is eliminated. When two ethyl free radicals combine, they form a ne\^.' sigma bond between the two free radical carbons. Combining the two twocarbon fragments results in the formation of a four-carbon fragment. The resuL: is that butane is formed from the termination reaction of two ethyl free radicals, The best answer is choice C. Carbon-carbon bonds are not easily cleaved, so the activation energy added in the initiation step is not high enough to cleave a carbon-carbon bond. Even if it could cleave the carbon-carbon bond to forr.

methyl free radicals, there is iittle likelihood that four CH3 groups woulc combine to form C+H1O. The loss of two hydrogen atoms would not occur.

Copyright O by The Berkeley Review

262

The Berkeley Kevieu

Organic Chemistry

tlydrocarbons and Reactions

Alkanes

Free Radical Halogenation Energetics with each reaction, we consider how much product is formed (thermodynamics) and how fast the reaction proceeds (kinetics). AH and AG vaiues determine how much is formed while the activation energy (Eo.1) dictates the speed. For the chlorination of methane, Reaction 4.1, the foilowing enthalpy valuel apply:

Ci-Cl -) Cl.+ .Cl H3C-H + .Cl -+ H3C. + H-CI H3C. + Cl-Cl -) H3C-CI + .Cl
Reaction 4.1.

AHi.itiatic,n = 58 kcal
AH1 =

I ks.1

AH2 = -26kcaI

Figure 4-4 shows the energy diagram associated with propagation steps in

HCl + CH3

fr
CJ

overall AH = AHr

tr.l

OJ

CH3C1 +

Reaction Coordinate

Figure 4-4

The energetics of each free radical halogenation reaction is different. For instance, bromine has a lower bond dissociation energy than chlorine, so bromination requires less activation energy than chlorination. This is why chlorination requires light for initiation, while either light or heat can initiate a bromination reaction. F{owever, this does not mean that bromination proceeds faster than chlorination. The reaction rate is determined from the aitivation energy of the rate-determining step in propagation. Table 4-3 lists the energies for the halogenation of methane. From Table 4-3, you can determine which reaction is fastest and which reaction generates the most heat. The data shows that free radical iodination of methane is unfavorable and free radical fluorination of methane is too favorable, generatrng enough energy to explode.
AH values {Kcal/*o1") Halogenation Reaction

X=F
38

X=Cl
58
+1

X=Br
46

X=I
36

kritiation; X2

-+

2X.

X. + CHa -+ HX + H3C. H3C" + X2 --+ H3CX + X. CH4 + X2 -+ H3CX + HX

-32
-

+76 -z+ -8

+33
-20 +13

/t)

-26 -25

-102

Table 4-3 Copyright


@

by The Berkeley Review

Exclusive MCAT Preparation

Organic Chemistry

Ilydrocarbons and Reactions

Alkanes

Free Radical Halogenation Selectivity

the relative free radical intermediate stability, the relative abundance of equivalent hydrogens, and the rate of the reaction. For the chlorination reaction, the relative reactivity of 3" :2' : 1" carbons is 5 : 3.g : 1 at room temperature. At temperatures around 100"C, chlorination selects for tertiary over seiondary over primary by a ratio of roughly 4 : 2.5 : r. As the temperature increases, the reaction proceeds faster, and is therefore less selective. For the bromination reaction, the relative reactivity of 3' : 2" : 7" carbons is 1600 : 62 : r at room temperature. This means that the product of free radical bromination is almost exclusively tertiary, while the free radical chlorination reaction gives a more
balanced product mixture. Figure 4-5 shows the product disiribution for comparable chlorination and bromination reactions.

Because tertiary free radicals are more stable than other free radicals, halogenation occurs preferentially at tertiary carbons. When there is more than one unique carbon in the alkane reactant, the product distribution is a result of

cl" __>
c+Hro

hv

$.,
77.7"/o Secondary

28.3%

Primary

Br

C+Hro

Br,
hv

.t\-/
97.6"/o Seconda ty

$Br
2.4"h Prirnary

Figure 4-5

As a general rule, slower reactions are more selective than faster reactions,
than chlorination. In the slower bromination reaction, if a primary free radicai js formed, it has time to abstract a hydrogen from another alkane and form a ner!., possibly more stable free radical. If the new free radical is tertiary, it is likely to only react with a dihalogen molecule. Consider Reaction 4.6 and, the data in Table 4-4 corresponding to the distribution of mono-halogenated products.
because reactants have more time to select the best site for reacting. Chlorination reactions are faster than bromination reactions, so bromination is more selective

>\+X2grb.hx
Reaction 4.6

Trial
I

Halogen Br2

Temp (K)
298 298
J/J

Product A 03%
29.1%
0.4%
JJ.J
70

Product B
89.40/"

Product C Product D
70.1%
32.00/"

0.2%

II III IV V

Clz Btz Clz


12

243%
88.3%
aa ao/ LL.L /O

t4.6%
0.2%

1,L.1%

J/J
373

27.8%

No Rxn Table 4-4

No Rxn

No Rxn

T6]% No Rxn

Copyright

by The Berkeley Review

264

The Berkeley Revie*-

Organic Chemistry

tlydrocarbons and Reactions

Alkanes

The data in Table 4-4 confirms that bromination is more selective than chlorination for tertiary over secondary over primary. Bromination is slower than chlorination, but it should also be noted that bromination is more reversible than chlorination, so it is more likely undergo a reverse reaction from an unstable product to ultimately form the most stable product. The data in Table 4-4 also shows that temperature has an effect on the reaction rate and therefore on the selectivity. As the reaction temperature is increased, the reaction proceeds at a faster rate, resulting in the formation of products based more on random probability rather than selection for the most stable intermediate. In addition, Boltzmann's law states that as energy is added to a system, the distribution of compounds is shifted to the less stable compounds, to absorb the energy. Actually, who reaily knows if Boltzmann said it, or if it's even a law. The key thing is that as energy is added, less stabie compounds are formed.
Example 4.4 \44ry in Trial
B?

II of Table

4-4 is Product A formed to a greater extent than product

A. B. c. D.

Product A results from the more stable free radical, thus it is selected for. Product B rearranges to form product A. Product B is more stable, but there are more hydrogens that lead to product A, so overall less Product B is formed. Product A is more stable, but there are more hydrogens that lead to product B, so overall less Product B is formed.

Solution Product A results from a reaction at a primary carbon, so it proceeds via a primary free radical. Product B results from reaction at a tertiary carbon, so it proceeds via a tertiary free radical. This eliminates choice A. Rearrangement is seen with carbocations, but not with free radicals or carbanions, so choice B is eliminated. The best answer is choice C. It is often possible to answer a question without fuil analysis. The reason for the substantial amount of Product A is because there are six hydrogens that lead to Product A while there is only one hydrogen that leads to Product B. Although tertiary reactivity with chlorination is roughly four to five times greater than primary reactivity, the six-to-one abundance ratio outweighs the four or five-to-one reactivity preference, making the probability of forming Product A greater than the probability of forming Product B. This is makes choice C the best answer. Choice D can be eliminated, because there are more hydrogens available to form Product A than Product B.

Copyright

by The Berkeley Review

Exclusive MCAT Preparation

Organic Chemistry
Example 4.5

tlydrocarbons and Reactions

Alkanes

Why is there no reaction observed when iodine is used? A. Iodine cannot form a free radical. B. The iodine-iodine bond is too strong to cleave.

C. D.

Free-radical iodination of an alkane is too reactive. Free-radical iodination of an alkane is unfavorable.

Solution
Because chlorine and bromine form free radicals, we can be assume that another halogens, such as iodine, can also form a free radical. This eliminates choice A.

Iodine is lower in the periodic table than chlorine and bromine, so the iodineiodine bond is weaker than the chiorine-chlorine bond and the bromine-bromine bond. Because Cl2 and Br2 are cleaved, it is safe to assume that 12 is even easier to cleave. This eiiminates choice B. Iodine forms weak bonds to carbon and hydrogen, so the products are less stable than the reactants. Because the products of free radical iodination are less stable than the reactants, the reaction is unfavorable, so there is no reaction observed with iodine. This makes choice D the best answer and eliminates choice C. Fluorine is not used for completely opposite reasons. Fluorine forms strong bonds to carbon and hydrogen, and th! fluorine-fluorine bond is weak. The products are so much more stable than the

:j'"",,^"
Example 4.6

t'""

",

Why are there minimal di-halogenated products formed in the free-radical


chlorination of an alkane?

A. B. c. D.

The acldition of the halogen makes the alkyl halide less acidic than the alkane, so it is less reactive to subsequent halogenation reactions. The addition of the halogen makes the alkyl halide more acidic than the alkane, so it is more reactive to subsequent halogenation reactions. After the first halogen is added to the alkane, the carbon-hydrogen bonds grow weaker and thus more reactive. After the first haiogen is added to the alkane, the weakest bond is a carbonhalogen bond and not the carbon-hydrogen bond. As a result, it is easier to remove the halogen rather than the hydrogen from the mono-substituted alkyl halide.

-1.

-j

Solution
Halogens are electron-withdrawrng, so their presence on a molecule increases its acidity. This eliminates choice A. Choice B is invalid, because an increase in

.
ts

-,1L

reactivity would imply that more poly-halogenated products would form, not less. Choice C can be eliminated for almost the same reason. If the carbonhydrogen bond is weaker, and thus more reactive, then it would be easier to add a_second halogen than the first, making poly-haiogenation preferable. once an alkane is haiogenated, the weakest bond is the carbon-halogen bond, not a carbon-hydrogen bond. If a second halogen free radical ruuit, with an alkvl halide (rather than an alkane), it preferentiully r"rr,orr"s the halogen (breaking the weakest bond), forming a non-halogenated alkyi free radical. this is because the reverse halogenation reaction (in propagation) is more favorable than the removal of a hydrogen and subsequent additional halogenation reaction. Choice

(-

5-c

:._

:_

-:

"'*.
Iopyright O by The Berkeley Review

The Berkeley Review

:f

Organic Chemistry
Example 4.7

Ilydrocarbons and Reactions

Alkanes

To synthesize a primary alkyl halide from an alkane in highest yield, what


should be done?

C. Bromination at 100"C D. Chlorination at 100'C


Solution
so the best free radical halogenation reaction is the one with lowest selectivity. According to the data in

A. Bromination at25"C B. Chlorination at25'C

A primary alkyl halide is the least favorable product,

Table 4-3, chlorination is less selective than bromination and selectivity is reduced at higher temperatures. This means that choice D, chlorination at the highest listed temperature, is best.
Example 4.8

Using the data listed in Table 4.3, what percent of the mono-halogenated products is 2-bromo-2-methylbutane following the free-radical bromination of 2methylbutane at75"C?

A. 88.1% B. 88.7% c. 89.1% D. 89.7%


Solution
To answer this question, you need to read Table 4-3. In trials I and III, product B is 2-bromo-2-methylbutane. Therefore, we need to estimate how much Product B is formed at75"C. At25'C, there is 89.4% product B formed, while at 100"C there is 88.3% product B formed. This means that the amount of product B formed at 75'C should be between 88.3% and 89.4o/o, which eliminates choices A and D. The amount formed at75'C should be closer to 88.3% than89.4"h, so choice B is the best answer. Example 4.9

How many mono-chlorinated structural isomer products are possible when 2,5dimethylhexane undergoes free radical chlorination?

A.3 8.5

c.6
D.8
Solution

This question is asking for how many structural isomers there are for chloro-2,Sdimethylhexane. Because of the mirror plane through the carbon 3-to-carbon 4, bond, there are three unique carbons on 2,5-dimethylhexane. This means that there are just three carbons that can be chlorinated, so there are only three monochlorinated structural isomers. If stereoisomers were included, the value would increase to four, given that chlorination of the secondary carbon yields a stereogenic center. The best answer is choice A.

Copyright

by The Berkeley Review

267

Exclusive MCAT Preparation

Organic Chemistry

tlydrocarbons and Reactions

tlydrocarbon Keactions
,

Aydroiberbdh

Radtiddb

,,-,,.,*,,,,,,r',i,,,t,,,,,,,f,,,,

Elimination Reactions The reaction that forms an alkene from a substituted alkane is elimination. It is named from the fact that a functionai group and a hydrogen on adjacent carbons are eliminated in order to form a n-bond. The-reac-tion requires elevated temperatures to help overcome the activation energy and to p.rsh the reaction in the forward direction. Like the nucleophilic substiiution ,*u.tior,r, there are two reaction mechanisms, appropriately named E1 and E2. As with nucleophilic substitution, the two versions are named also for theiireaction orders (kinetic rate dependence). E1 is similar to sry1 and E2 is similar to slr]2, except that the product is an alkene. In an E1 elimination, the leaving gto.rp first ieaves and there is a carbocation formed. The empty p-orbital of thi c"arbocation eventuallrbecomes one of the two p-orbitais in the new n-bond. In an E2 elimination, a base removes an alpha hydrogen to force the leaving group off of the neighboring carbon. Elimination converts a functionarized alkyigroup into an alkenJ.
E2 Reaction (Carried out under Basic Conditions at High Temperature) The E2 reaction is corrcerted (one-step) rike the sNI2 r;action, with one major exception being that the E2 reaction occurs at higher temperatures (temperatuies above ambient temperature) than the S52 reaction. An E2 reaction also requires that the base be bulky. Because of the steric hindrance aisociated with a bulky base, it is.less apt to act as a nucleophile and thereby minimlze in;;;;fi;; Sp2 reaction. An important feature of the E2 reacti-on is that the substituenti being eliminated must be onti to one another (have a dihedral angle of 1g0.). The mechanism for an E2 reaction is shown in Figure 4-6.

(^
H

oR-

R:
H

o
I I

-----+
A

H H
X-

H,CV
H H3Ctt,,)
HgC

(trrH
H

Figure 4-6

In an E2 reaction, the compound may have to rotate to proper conformation before the reaction can proceed to form the alkene. This is to siy that the leaving

at one of the reactive carbons, a corresponding specific geometrical isomer product (either cis or trans) forms. Figure 4-7 shows an exampte where there are two alpha protons to choose between. One of the two protoni has been replacecl by deuterium, so that the structure can be monitorecl and the product can be used to support the idea that the elimination reaction o.c.rrr"d from the anti orientation. Figure 4-7 shows the rotation to anti and formation of both products.
Copyright
@

attacks the proton to start the reaction. whenever there are two alpha hydrogen,. that can be eliminated from the starting reagent, there are two posiible prodicts. The consequence is that when the starting material has a specific stereoChemistl'

group must be oriented anti to the hydrogen being eliminated before the base

by The Berkeley Review

268

The Berkeley Revien'

r.:

Organic Chemistry

Ilydrocarbons and Reactions

llydrocarbon Reactions

----1-

LINR^

DH \

EJ

/
CH:

^/\ H:C
Bt
H

H.C
LiNR,

CH:

--+

F<

cHs

Figure 4-7

If the reaction were not carried out through a mechanism that required anti orientation, then there would have been four elimination products total, rather than just two. The four products wourd include the cis and trans alkenes featuring the deuterium, and the cis and trans alkenes with no deuterium. Figure 4-B shows the two alkenes not formed,in the reaction in Figure 4_7.

HH HsC

><

CHs

CH:

Not formed in the elimination reaction of (2s, 3R)-3d-2-bromobutane, therefore the reaction must have proceeded by the anti orientation.
Figure 4-8

Multiple products can result when their are multiple hydrogens available for deprotonation. In Figure 4-7, onry the products with in iniernai n-bond are drawn. The internal n-bond is more favorable then the terminal n-bond by about 1.5 kcals, making 1-butene a minor side product in the reaction in Figure [-2. rn" relative stability of substituted alkenes ii shown in Figure 4_9.

\_J

R'''

\_-/
R''

R'

R'

R''

HR

\--/
H
Monosubstituted

Tetrasubstituted

Trisubstituted

Disubstituted

Figure 4-9 Terms such as anti-periplanar orientatiott should sound familiar and immediately make you think of an F2 reaction. Any time you see a strong bulky base, a hydrocarbon with a leaving group, and high temperature, you should think
E2 reaction.

of an

Copyright

by The Berkeley Revierv

Exclusive MCAT Preparation

Organic Chemistry

tlydrocarbons and Reactions

tlydrocarbon Reactions

the sI.J1 reaction. In both the E1 and sNJ1 reactions, there is (1) a carbocation intermediate formed after the leaving group leaves in the rate determining step of the reaction and (2) the possibility of rearrangement with the carbocation intermediate. A schematic for a typical E1 reaction is shown in Figure 4-10.
HsQ

E1 Reaction (Carried out Under Acidic Conditions at High Temperature) Tu E-t reaction also yields an alkene, but it goes through a different mechanism than the E2 reaction. The mechanism for the t1 reactioi is quite similar to that of

OH

-n ------> A

--+

oHz

,rg,
. Hb\

+ Hro

T'.';*1f\'" '
-HA
HJ

H"C
Hb
Figure 4-10

i
CH, H

CHs

Ha

><J

To recognize E1, as opposed io E2, it is easiest to look at the reaction conditions E1 reactions are best carried out under acidic conditions while E2 reactions require basic conditions. Both reactions proceed under thermal conditions (elevated temperature) and yield the most substituted alkene as the major product. There may be some of the less substituted alkene formed in smal1 quantity. The reaction in Figure 4-11 shows the formation of both the 2-alkene major product and the 1-alkene side product.

HO:

.atll*

H-'-'--.-H

*r"au"R
HsC
H

s- ::. ..S"1:.t, H3c$\'7-l -:


HrC /,
H

."O '\)

\SH2CH3
CHs

l,,rt\\

CH, cH2cH3

Urclrr,,rp-W,

,?
CH:

_-H
Figure 4-11

H,c7\ /F.", ts-{./


t

r.3

1t

rrrttl CH2C

H.

Copyright

by The Berkeley Review

The Berkeley Review

Organic Chemistry
Example 4.10

Ilydrocarbons and Reactions

Ilydrocarbon Reactions

What is the major product when 2-methylcyclopentanol is treated with


concentrated sulfuric acid at 50"C?

A.

B.

CHz

cHs

Solution At an elevated temperature in the presence of a strong acid, the reaction proceeds by an E1 mechanism, so the product is a highly substituted aikene. Choice A is eliminated, because it is a terminal alkene. Choice B is eliminated, because it is an alkane and not an alkene. Choice D is eliminated, because it is not the most substituted alkene and it formed via a secondary carbocation, a less stable intermediate than the tertiary carbocation. The best answer is choice C, because the n-bond is highly substituted and it includes the tertiary carbon, implying that it was formed via a tertiary intermediate.
Rearrangement is possible with carbocation intermediates, so E1 reactions are susceptible to forming rearranged products. Hydride and alkyl shifts are rapid,

intramolecular processes that occur before the soivent can remove the
neighboring proton to form an alkene. Carbocation Stability Because carbocations are common intermediates,

it is important to know the relative stability of the various carbocations. The relative stability of alkyl carbocations is 3' > 2' > 1' > Methyl. For carbocations conjugated to a t-bond, vinylic and benzylic carbocations, there is additional stability because of resonance. Carbocations can undergo rearrangement by having hydrides or alkyl groups shift, resulting in a different carbocation. For instance, if a secondary carbocation (R2CH+) is formed, it can rearrange to a tertiary carbocation (R2R'C+), if a tertiary carbocation is possible. Figure 4-12 shows the relative stabiiity of alkyl carbocations.

".. 'slx:
3"

Carbocation

'"{+H: '"" *= '"


2'Carbocation Figure 4-12

o:. )@..,,,trtu

:6"'"

1'Carbocation MethylCarbocation

The conclusion that can be drawn from the relative stability of alkyl carbocations is that methyl groups are electron donating to electron poor carbons, which can be extrapolated to say that alkyl groups are electron donating. Figure 4-13 shows three rearrangements where a less stable carbocation is converted into a more

Copyright

by The Berkeley Review

277

Exclusive MCAT Preparation

Organic Chemistry

tlydrocarbons and Reactions

llydrocarbon Reactions

stable carbocation via a hydride H


I

carbocation, which is more stable than alkyl carbocations, beiause of resonance.

shift. The third example forms an allylic


H

)'w;*
1'

----->

HI "\_

o_,/cylH 'H

H2" "Yt-t\r,
CH"

H l'' --------> \ .1--.n, nf-.vn


H

3"H

HHHH \/\/

/c-c \
l"

o/

H ------> c-c-\ H / o /'

"YJt"
Figure 4-13

,,f-'Y#.,
2"

Allvlic

which of the following carbocations is apt to undergo rearrangement?

A. B. C. D.

(H3C)3C+ (H3C)2CH+ H3CH2C+ (H3C)2CHCH2+

Solution
Choice A is already a tertiary carbocation, so it has no reason to rearrange, least of all to a primary carbocation. Choice A is eliminated. Choice B is a secondarr-

carbocation, but can only rearrange to form a primary carbocation, so it is eliminated. choice C is a primary carbocation that only has primary carbons, so it is eliminated. In choice D, a hydride shift can covert a primary carbocation into a tertiary carbocation, making choice D the best answer.

Copyright O by The Berkeley Review

272

The Berkeley Revieu'

Organic Chemistry

tlydrocarbons and Reactions

flydrocarbon Reactions

Electrophilic Addition Reactions Although isolated alkenes are not a topic tested on the MCAT, polyenes are a topic, so generic electrophilic addition reactions are viable. Electrophilic addition reactions involve a n-bond actrng as nucleophile by attacking an electrophile to form a substituted alkane product. In the first step of electrophilic addition reactions, the n-bond of the alkene donates its eiectron density to an electrophile. The first step of a generic electrophilic addition reaction is shown in Figure 4-14.

----------tStep

E* represents any electrophile (lone pair acceptor)

Figure 4-14

After the electrons from the n-bond are donated to the electrophile, a positive charge is situated on the most substituted carbon of the original alkene in the carbocation intermediate. This is the first step in almost all electrophilic addition mechanisms. In the second step of the mechanism, a nucleophile attacks the carbocation formed in the first step of the electrophilic addition reaction. The
second step of a generic electrophilic addition reaction is shown in Figure 4-15.

H-Nu:J
H-Nuc

\-/' \

-----------.r>

SteP

II

H-Nu

represents any nucleophile (lone pair donor)

Figure 4-15

It is important to recognize that when there is a carbocation intermediate, there is potential for rearrangement. If the carbocation is unstable, then prior to the attack by a nucleophile, the carbocation can rearrange by way of a hydride shift or alkyl shift to form a more stable carbocation. Rearrangement is not shown in this example, but it can occur between steps one and two of the mechanism. The nucleophile should be a weak base, otherwise it can deprotonate the intermediate to carry out the reverse reaction (elimrnation) and regenerate an alkene. To avoid this, eiectrophilic addition reactions are carried out under acidic conditions. The final step is the neutralization of the cationic product, which is carried out by a solvent basic enough to deprotonate the cationic species. The final step of a generic electrophilic addition reaction, a workup step, is shown in Figure 4-15.
sorv

-\

(workup) Step Solv represents any polar/protic solvent Figure 4-16 A protic solvent is capable of forming hydrogen bonds and transferring protons.

L-# \ \-/ lll

\-,/t

----------.>

Copyright

by The Berkeley Review

273

Exclusive MCAT Preparation

Organic Chemistry

Ilydrocarbons and Reactions

tlydrocarbon Reactions
to

mechanism with 1,2 versus 1,4 addition reactions of conjugated alkenes.

understood. Later in this section we will look at a variations on this same

dienes and to recognize the class of reaction. Know how to diaw the reaction if they describe the mechanism for the reaction in words. The MCAT doesn't focus much on memorization. The focus is on conceptual understanding, so be certain that the corresponding terms such as Markovnikov, syn and anti ire completeiy

It is a good idea to understand basic mechanism of electrophilic addition

Regioselectivity (Markovnikov versus Anti-Markovnikov Addition) The concept of regioselectivity occurs when the reactant electrophile and the alkene both lack mirror plane symmetry. This is to say that the double-bond carbons are not equaily substituted. Because the two carbons are not equally substituted, one is less sterically hindered than the other. As a mechanistii rule in electrophilic addition reactions, the first substituent attacks the less hindered carbon of the alkene, leaving the second substituent to add to the other carbon. Cenerally, the less hindered carbon of the intermediate is the aikene carbon that wasn't attacked by the first substituent. A Marlcoanikoa addition product results from the addition of the electrophile to the less substituted carbon of the alkene and the nucleophile to the more substituted carbon of the alkene. In the case where a strong acid reacts with an alkene the electrophile is a proton, you can simply say, "H goes where H is." An onti-Markoanikoa ad,dition prod,rct is the opposite of a Markovnikov addition. Stereoselectivity (Syn versus Anti Addition) The concept of stereoselectivity occurs when the alkene reactant or intermediate has asymmetric faces, and thus non-uniform steric hindrance. syn additiol refers to a reaction where the two new substituents add to the same side (face) of the alkene reactant or intermedrate. Anti addition refers to a reaction where the two new substituents add to opposite sides (face) of the alkene reactant or intermediate. As a mechanistic rule, a substituent attacks at (and adds to) the less hindered face of the alkene (or intermediate). If the two substituents add at the same time, they add in a syn fashion to the least crowded face of the alkene. If the two substituents add at different times, they add in an anti fashion to opposite faces of the alkene. once the first substituent adds to an alkene, it makes one side of the intermediate more crowded than the other side. This is referred to as stereoselectivity, because the face at which the substituent attacks dictates the stereclchemistry of any newly formed chiral centers.
Stereoisomer Formation You should always consider if stereochemistry is involved in a reaction; whether it forms a racemic mixture of enantiomers or a major/minor mixture of diastereomers. When no chiral center is present on an alkene reactant, there is no asymmetry to influence the reaction. There is an equally likeiy chance to attack the alkene from either face. The result is the formation of a racemic mixture of enantiomers, assuming that new chiral centers are formed. When a chiral center is present on an alkene reactant, its asymmetrv influences the stereoselectivity of the reaction. There is a greater chance to attack the alkene from the less hinclered face than the more hindered face. The result is the formation of a major/minor mixture of diastereomers. A good rule to follow is that if there is no optical activity in the reactants, then there can be no optical activity in the product mixture. This means that the presence of chirality influences further chirality.

Copyright

by The Berkeley Review

274

The Berkeley Review

Organic Chemistry

Hydrocarbons and Reactions

llydrocarbon Reactions

formed (thermodynamic control). At lower temperatures, the more stable intermediate dictates the product, so r,z-ad.d.Ltion is favored. This is known as kinetic control. There is always the option to add either 7,2 or 1,4 when the has conjugation. Figure 4-17 shows the product distribution for the fysJem l-rydration of 1,3-butadiene at two different temperatures.
H

1'2-Addition to a Conjugated Diene vs. 1,4-Addition to a Conjugated Diene Electrophilic addition reactions are not iimited to alkenes with isolated n-bonds, as these reactions also take_place on conjugated dienes. with conjugated ciienes, there are multiple potential products. t,z--aaaition refers to an addition reaction that adds substituents to the first and second atoms (usually carbon) in the conjugated n-network. 1,4-addition refers to an addition reaction that adds substituents to the first and fourth atoms in the n-network. As a general rule, r,4_ addition is favored at higher temperatures, because the more s-table product is

H2C: CH z-\

HC_ CH2

L H:SOq{aq)o"c

H,c-.')ot "r.\
62%

ttr.CH2

HC-

HC38%

CH2OH

1,2-AdditonProduct 1,4-Additonproduct
H

H.C.\

CH

H2SO,1(aq)

l-hL- c)ot '


\

H,c'r-\
CH2

cH

HC-

CHz

50"c

HC-

HC84%

CH2OH

16%

1,2-Additonproduct 1,4_Additonproduct
Figure 4-12

the carbocation on carbon two (a secondary carbocation). The carbocation is allylic meaning it can resonate through tie n-network. It is in essence a propylene cation that can have cationic density at either end of the rc-network. What this means is that the carbocation can resonate to the terminal carbon (carbon number four). This forms a primary carbocation, whicl-r is not as stable as a secondary carbocation. According to the Boltzmann distribution law, as temperature increases, the higher energy levels become more populated to absorb this increase in energy. This means that at higher temperatures there are more primary carbocations than at lower temperatuies. This is why we see the product distribution favoring substitution of the alcohol at carbon two at lower temperatures while we at higher temperatures we observe the substitution of the alcohol at the fourth carbon. Figure 4-18 shows the energy diagram and mechanism associated with this reaction, which includes t*o poisiute iuth*uyr.

In the example in Figure 4-77, there are two possible carbocation intermediates. The mechanism and both intermediates ur" ,ho*r, in Figure 4-1g. To start the reaction, the proton adds to the terminal carbon (least hiidered carbon) leavi'g

Copyright O by The Berkelev Review

275

Exclusive MCAT Preparation

Organic Chemistry

Ilydrocarbons and Reactions

Ilydrocarbon Reactions

Kinetic

Pathway

Thermodynamic Pathway

(favorable at lower temperatures) (favorable at higher temperatures)

2" carbocation is most stable

1" carbocation is least stable

I rirC=Cri :HrC-C^\ ' \ HC:CH2

HO

-J

o/,

H .e

H3C-CH '

)^)^.. HC:CH2

\HC-C@H /
\
H

1f."" H@ HsC-.i""'
HC:
H*
1r

1f.""
H?C"\o
CH

CH2

HC-

CH2OH2

1f".
H"C"\
CH2

HeC"\

C-

>oH
\ HC:

CH

HC-

CH2OH

Monosubstituted alkene
is least stable
1" carbocation

Disubstituted alkene
is most stable

2' carboca tion

Lower energy intermediate leads to the kinetic product. Higher energy intermediate leads to the thermodynamic product.

Reaction Coordinate

*--*-+
Figure 4-18

The energy diagram shows the two possible pathways for the reaction and their

relative energetics. The dashed pathway represents L,2-addition to the conjugated alkene, while the solid pathway represents 1,4-addition to the conjugated alkene. From this data, you should be able to predict the more
abundant product at a given reaction temperature.

Copyright O by The Berkeley Review

The Berkeley Review

Organic Chemistry

flydrocarbons and Reactions

Ilydrocarbon Reactions

rearrangement reactions, it is an intramolecular rearrangement that takes place.

come together, resulting in a single new compound. In sigmitropic

meaning that the formation and breaking of all bonds occur simultaneously. Pericyclic reactions include cycloaddition reictions,which most notably include the Diels-Alder reaction, sigmatropic rearrnngement, and electrocyclic reactiors. We will address only cycloadditions and sigrnatropic rearrangement. The significant difference between a cycloaddition reaction and sigi-Latropic rearrangement involves the number of molecules. In cycloaddition, two separate compounds

Pericyclic Reactions Pericyclic reactions involve the repositioning of both sigma-bonds and pi-bonds through a cyclic transition state. These reactions are believed to be concerted,

Paramount to understanding these reactions is having a good idea about orbital overlap in both sigma-bonds and pi-bonds. In everything we'll address in terms of the MCAT, we shall only consider the positioning oith" atoms and not the spin of the electrons within the moleculai orbitals. the first reaction we shall consider is the Diels-Alder reaction.

Diels-Alder Reaction The Diels-Alder reaction, an electrocyclic addition reaction, involves the addition of a conjugated diene (4 n-electrons) to an alkene (2 n-electrons) to from a six membered cyclohexene ring. The transition state for a Diels-Alder reaction is similar to the resonance of benzene, as shown in Figure 4-19.

49

_->

Die ls-Alder Reaction Figure 4-19

six n-electrons in a cyclic n-network make benzene aromatic, so we refer to the transition state of a Diels-Alder reaction as aromatic (containing 6 n-electrons in a ring). Diels-Alder reactions involve the addition of a 1,3-diene to a dieneophile. The diene must have cis orientation about the central sigma bond to undergo a Diels-Alder reaction. A sample Diels-Alder reaction is driwn in Figure 4-20.
diene

dieneophile

22"

,(+
4

t
@

.A (

cyclohexene derivative

o
CHs

cH2cH2

^*ll l+ 3\-/s
4

2//\6

CHg

cH2cH3
Second: draw the rest of the molecule connected to the numbered carbons

First: draw the cyclohexene ring

Figure 4-20

Copyright

by The Berkeley Review

Exclusive MCAT Preparation

Organic Chemistry

llydrocarbons and Reactions

Hydrocarbon Reactions

The six membered ring that is formed is cyclohexene. The carbons are numbered to help identify the product, which will make large, polycyclic products easier to evaluate. Figure 4-21 shows a more complex Diels-Alder reaction. Because both the diene and dienophile have substituents, there is a chance that stereoisomers can form. The stereoselectivity is driven by orbital overlap in the transition state. The two stereoisomers products, diastereomers, are drawn for the reaction.

o
1

ro

^*:o:::;1" no
Endo Product Exo Product

Figure

4-21.

The last thing for us to consider is regioselectivity. when the diene and dienophile have substituents, there exists the potential for different structural isomers. Regioselectivity can be predicted using resonance, where the most electron rich terminal carbon of the diene attacks the electron poor carbon of the dienophile. The reaction is optimized when the dienophile has electron withdrawing groups. Figure 4-22 shows the effect of electron donating and electron withdrawing groups on the diene through resonance.
EWG

EW G

/)
\

<:+

l_-.-.
Figure 4-22

l-/ <)

t) 4

EDG

EDG

EWG = electron withdrawing group

EDG = electron donating group

'a:

when it comes to predicting regiochemistry, it's as simple as plus attracts minus. Figure 4-23 shows a Diels-Alder reaction where regioselectivity is an issue.

H?co

z l.

o
H
6*

--+

.s-

*
Figure 4-23

H3CO

CHs

Major

Copyright

by The Berkeley Review

The Berkeley Review

Organic Chemistry
Example 4.12

Ilydrocarbons and Reactions

flydrocarbon Reactions

\A/hich of the following Diels-Aider reactions is fastest?

A.

2 l*
HrC\
B.

ll \A

4.", --+
CHa

CHe

nrao\
C.

2 l*

4.". tl- -^CHa

CHa

H3CO

(+

H.c^
D.

H:C

Ar-CHs

,4.',,

CH: cHs

2 I,
"r.o\
Solution

-^>
H3CO CHs

The rate of a Diels-Arder reaction is increaseci by the presence of an electron donating_ group on the diene and an erectron withdrawing group on the dienophile. The rate of a Diels-Alder reaction is decreased by the presence of bulky groups in the transition state. Choices C and D are eliminated because of the two methyl groups on the dienophile. No matter how the molecure aligns entering the transition state, one of the methyl groups wiil be in the way. In choice B, there is a methoxy group on the diene, *-hil" ,^ choice A is just a methyr Methoxy groups donate electron density through resonance, so choice ,grou.p. B has the more electron rich diene, resulting in a faster reaction than choice A.
Cope Rearrangement The Cope rearrangement is a sigmatropic rearrangement i.vorving two pi-bonds and one sigma-bond. Figure 4-24 shows a simpl"e Cope rearrangement carried out on a 1,S-diene. In more comprex examples, stereo.i-rerr-ristry rnay be an issue, because stereocenters can be both formed and rost as hybridizaiior-, changes.

HaC

'G -\g

A --+

4 ,-,,.4,
279

Figure 4-24 Copyright


@

by The Berkeley Review

Exclusive MCAT preparation

Organic Chemistry

Hydrocarbons and Reactions

Hydrocarbon Reactions

overlap. The reaction requires the input of energy to overcome the activation barrier. The structure of the product is dictated by the orbital overlap in the
transition state. Claisen Rearrangement The Claisen rearrangement is similar to Cope rearrangement, except that in a Claisen rearrangement, the reactant is a vinylic allyi ether. The rearrangement involves two pi-bonds and one sigma-bond and has a transition state that is similar to the one observed with the Cope rearrangement. The difference is the presence of an oxygen. Figure 4-25 shows a Claisen rearrangement.

The Cope rearrangement involves two pi-bonds and one sigma-bond aligned in such a way that the terminal p-orbitals of the two pi-bonds are close enough to

as

---l>
A

o^
\,

Figure -l-25

when the ether is benzylic instead of vinylic, the cyclic ketone can quickly
tautomerize to form a phenol. The preference of a phenol over the cyclic ketone is due to the aromaticity of the benzene ring.

Example 4.13 \z\4rich statement is

A. B. c. D.

valid in terms of the sigmatropic rearrangement reactions? Aldehydes are formed from a Claisen rearrangement of a vinylic allyl ether when the allylic ether carbon is unsubstituted. Aldehydes are formed from a Claisen rearrangement of a vinylic allyl ether when the vinylic ether carbon is unsubstituted. Aldehydes are formed from a Cope rearrangement of a vinylic allyl ether when the allylic ether carbon is unsubstituted. Aldehydes are formed from a Cope rearrangement of a vinylic aliyl ether when the vinyiic ether carbon is unsubstituted.

Solution
Choices C and D are eliminated immediately, because Cope rearrangement results in the conversion of one 1,5-diene into another 1,5-diene, not an aidehyde. The question is reduced to determining which carbon in the reactant forms the carbonyl group following Claisen rearrangement. The reaction is shown be1ow.
H

vinylicT\
I

aidehyde

altvtic

etner

---l>

It is the vinylic ether carbon that becomes the carbonyl carbon, not the allylic ether carbon. This means that the vinylic ether carbon must only have a hydrogen, and no carbons, in order to form an aldehyde and not a ketone. The best answer is choice B.

Copyright

by The Berkeley Review

The Berkeley Review

Organic Chemistry
Terp,he$

tlydrocarbons and Reactions

Terpenes

Classification Terpenes and terpenoids, btological molecules derived from terpenes, are natural hydrocarbons found in plants and animals that are made from S-carbon isoprene (2-methyl-1,3-butadiene) units. The five-carbon skeleton of isoprene can be found in terpenes. Terpenes are classified by their number of carbon atoms. Monoterpenes have ten carbon atoms, sesquiterpenes have fifteen carbons, diterpenes have twenty carbons, sesterterpenes have twenty-five carbons and so on, The in aitro synthesis of terpenes and terpenoids is called nstural product synthesis. Some naturally occurring monoterpenes are shown in Figure 4-26.

gH
Geraniol
(Oil of Germanium) Myrcene (Oil of Bay)

CH,O

Citronellol

Citronellal

N"
Limonene
(or Limin) o,-Terpinene (Oil of Coriander)

CH.O

tt Y\oH ^ i
Carvone (Oil of Spearmint)

t' u? lS ."-\1./
CH:

y-Terpinene (Oil of Coriander) CH"

Citral

(Oil of Lemongrass)

H"C. -CH"

"XJ

LR HeC
Camphor

cr-Pinene

Menthol
(Oil of Peppermint) Figure 4-26

(Oil of Turpentine)

Studies in biogenesis show that the large terpenes are synthesized using isopentenyi pyrophosphate rather than isoprene. Pyrophosphate adds across the

diene of isoprene to form either isopentenyl pyrophosphate or dimethylallyl


pyrophosphate, which are interconverted by isomerization. Figure 4-27 shows isoprene, isopentenyl pyrophosphate, and dimethylallyl pyrophosphate.

L,
Isoprene

o-

oo ilil
P-

Isopentyl pyrophosphate
Figure 4-27

oo-

ll

o-

P-

oH

o-

oo illt
P-

oDimethylallyl pyrophosphate

o-

tl

o-

P-

oH

These molecuies add to one another in a way where the n-bond of isopentyl pyrophosphate is the nucleophile and pyrophosphate of another molecule is the leaving group. A proton is lost from the nucleophilic moiety to regenerate a rubond. The reaction involves head-to-taii addition. When cyclizing, the bond that is formed to complete the ring is rarely connected head-to-tai1. Figure 4-28 shows the reaction of isopentenyl pyrophosphate and dimethylallyl pyrophosphate.

Copyright O by The Berkeley Review

281

Dxclusive MCAT Preparation

Organic Chemistry
oo ll
Isopentyl pyrophosphate
il

Hydrocarbons and Reactions

Terpenes

Geranyl pyrophosphate

- r HO- P- O- P- O- Y

oo ll

ll

HO- P- O- P- O

oo ilil
tt

oFigure 4-28

o-

Both plants and animals synthesize terpenes. Larger terpenes are built from multiple additions of isoprene units, including both isopentenyl pyrophosphate and dimethylallyl pyrophosphate. Geranyl pyrophosphate (a C-10 terpene derived from the head-to-tail connection of two isopentenyl pyrophosphate molecules) is the first monoterpene in many natural synthetic pathways. Another isoprene unit can be added to geranyl pyrophosphate to form farnesyl pyrophosphate (a C-15 terpene). These molecules can undergo further addition, dimerization, or modification into other terpenes and terpenoids. Figure 4-29 shows a generic pathway for the biosynthesis of larger terpenes.

4./\o-li-o-ll-o., tt
Isopentyl pyrophosphate (3-Methyl-3-butenyl pyrophosphate)

loo
I

,.^^o-ll_o_ll_on
or
, I
(3-Methyl-2-butenyl pyrophosphate) Isopentyl

l"o

o-

o-

Dimethylallr, Orr3pn"rf,i","

{PFophosthate

oMonoterpenes (C1s)

+_

Geranyl pyrophosphate (C1o-pyrophosphate)

tt o- P- oH O- OP-

oo illt

sesquiterpenes

(C15)

Farnesyl pyrophosphlt"

Diterpenes(C2s)

(C,5-pyrophosphate)\

tt o,

o(c36)

j
#
I

\ squalene tsoPentYl I rr-"r--""r-."".I pyrophosphate

C2g-pyrophosphate

Lanesterol (C36)

Tetraterpenes (Ca6)

{
Chloesterol (C27)

Figure 4-29 Copyright


@

bv The Berkeley Review

282

The Berkeley Review

Organic Chemistry

ffydrocarbons and Reactions

Terpenes

As Figure 4-29 shows, the triterpene squarene can undergo further reactivity to generate cholesterol, which does not have a number of carb'ons that is divisible by five. so while cholesterol may not be a terpene, its synthesis involves terpenes and terpenoids. The basic schematic for the biosynthesis of cholesterol startinj from isopentenyl pyrophosphate is shown in Figure 4_30.

-P-

j.

oo ilil O- P-

OH

Isopentenyl pyrophosphate

f"

Figure 4-30
a general , Asterpenes, rule, smaller terpenes are found primarily larger

carotene (a C-40 source of vitamin A), aie found in plants and animals. For instance, the monoterpene pinene is found only in plants while Vitamin 41, a diterpene, is found in both plants and animals. Frg.rr" 4-31 shows
larger terpenes. Elemene some selected Farnesol

in plants, while some such as lanesterol (a C-30 precursor to steroid hormones) and B-

(oiiofCoral)

G-Selinene
(Oil of Celery)

?or
Figure 4-31 by The Berkeley Review

Z+

(Oil of Lemon)

"
Vitamin

HeC

W
.A,1

CHg

CH:

CHs

OH

Copyright

283

Exclusive MCAT preparation

Organic Chemistry

flydrocarbons and Reactions

Terpenes

One of the skills that you must deveiop to do well on terpene related questions on the MCAT is to recognize terpenes and be able to identify the isoprene subunits in the carbon skeleton. Figure 4-32 shows the analysis of some terpenes for the isoprene units in the skeletal fragments.
Me

+Me
o,-Terpinene

,ll

^4{
^'r ,ll t

Me

Me

y-Bisabolene

t,,-=A'
M

"*A

+ix
Zingerberine

^4;
Patchouli Oil Figure 4-32

A common lab technique employed to isolate terpenes is steam distillation, where the pulp of some natural material is placed into water and boiled so that the natural oiis are distilled from the pulp. Steam distillation allows the essential oils to vaporize at a temperature lower than their boiling point, so they do not degrade. The distillate is a mixture of water and terpenes, which are easily separated using extraction techniques. Terpenes can also be extracted from pulp. When isolating terpenes, it is a mixture of geometrical isomers that is collected. The different geometrical isomers of a terpene are given the prefixes a"-, B-, y- and so on. The different geometrical isomers have similar physical properties, but because of differences in conjugation, they exhibit differences in the absorption of photons in the ultraviolet (UV) and visible range of the EM spectrum.
Terpenes are UV active, because of their n-bonds. An isolated alkene has a UV absorbance around 180 nm. A conjugated diene has a UV absorbance around 225 nrrl, which is significantly more intense than the absorbance of an isolated alkene. As the conjugation of a n-network increases, the wavelength of maximum absorbance, l,*u", and the intensity of absorbance, t, increase. Some terpenes contain oxygen, which is added in a way that does not alter the carbon skeleton. Carbonyls exhibit absorbances of greater wavelength than alkenes of the same conjugation, For instance, carvone (shown in Figure 4-26) is evident b,v a carbonyl absorption at 7744 cm-1 in its IR spectrum and a strong UV (e > 10,000) absorption at L,p31 = 242 nrrr. Terpenes are often isolated in educational laboratory experiments. Because of their biological significance and the fact thev are isolated in lab experiments, they are highly represented on the MCAT. If you have a fundamental understandrng of terpenes, then you should be fine.

Copyright

by The Berkeley Review

284

The Berkeley Review

Organic Chemistry
Alkanes

flydrocarbons and Reactions

Section Summary

Key Points for Hydrocarbons and Reactions (Section 4)

1'

Hydrocarbon compounds with only carbons, hydrogens, and sigma bonds a) Only contain C-C and C-H single bonds

i. ii. iii. b) i. ii. iii.

Can be aliphatic (straight chain) or cyclic Low water soiubility, row boiling point, and row melting point Relatively inert compounds that are used as solvents

Undergo free radical halogenation reactions with chlorine and bromine Involves initiation, propagation, and termination in that order Bromination is more selective than chlorination The relative stability for alkyl free radicals is: 3" > 2' > l" > methyl

Hydrocarbon Reactions (Reactions involving ru-bonds) 1. Elimination

a)

Loss of an H and a leaving group to form a ri-bond

i. ii. iii. b) i. ii. iii' c) i. ii. iii.

Requires high temperature Competes with nucleophilic substitution reactions Goes by way of one of two mechanisms: E1 or E2

E1 reactions are

similar to 5511 reactions Requires a strong acid (Bronsted-Lowry or Lewis) Forms carbocation intermediate so rearrangement is possible Forms most substituted and least sterically hindered alkene

E2 reactions are

similar to 5512 reactions Concerted reaction that requires a strong, bulky base Proton to be lost and the leaving group must be anti to one another

No intermediate formed, only a transition state

2. ElectrophilicAddition a) An electrophile can be added to a n-bond followed by nucleophilic attack i. The n,bond is a weak nucleophile ii. The reaction is driven by the strength of the electrophile b) Electrophilic addition reactions exhibit regioselectivity i' Steric hindrance and carbocation stability influence the site of attack ii. when the electrophile attacks the ress substituted carbon, the iii. when the electrophile attacks the more substituted carbon, the
reaction is said to be an anti-Markovnikov addition reaction is said to be a Markovnikov addition

c)

Electrophilic addition reactions exhibit stereoselectivity i. Steric hindrance is the most influential factor in stereoselectivity ii. when substituents add one at a time (the first is added before the second on attacks) the product exhibits anti addition stereochemistry iii. when substituents add simultaneously (both add at the same time) the product exhibits syn addition stereochemistry

Copyright

by The Berkeley Review

Exclusive MCAT Preparation

Organic Chemistry
3.

Ilydrocarbons and Reactions

Section Summary

1,2-Additionversus 1,4-Addition

a) Electrophilic addition reaction can proceed in multiple ways with


conjugated n-systems

i. Typically, \,2-addition is favored at lower temperatures


control)

(kinetic

ii. Typically, 1,4-addition is favored at higher


(thermodynamic controi)

temperatures

4.

Diels-AlderReaction

a)

Reaction of a diene and a dienophile (alkene)

i. ii. iii. iv. 5.

An electrocyclic reaction carried out with either light or heat Forms a cyclohexene product Stereoselectivity: Endo product is preferred over exo product Regioselectivity: Depends on the resonance nature of the groups on the reactants

Sigmatropic Rearrangement (Cope and Claisen Rearrangements)

a)

Both sigma-bonds and pi-bonds are broken and formed via a cyclic transition state i. Cope rearrangement converts a y,8-unsaturated alkene into another y,8-unsaturated alkene via a realignment of molecular orbitals ii. Claisen rearrangement converts a vinylic allyl ether into another y,5unsaturated carbonyl via a realignment of molecular orbitals iii. Sigmatropic rearrangement requires heat

Terpenes

1.

Natural products derived from the connecting of five-carbon units

a) b)

Derived via biosynthesis involving either isopentyl pyrophosphate or dimethylallyl pyrophosphate Can be cleaved into isoprene subunits i. Terpenes are named for their carbon count: 10 C = monoterpene, etc. ii. Isolated by steam distillation or extraction as natural oils iii. Presence of n-bonds results in UV absorbances. As conjugation increases, intensity and l"*u* both increase.

Copyright

by The Berkeley Review

286

The Berkeley Review

Alkanes : and,, ftr5rdroctrrbon' Keaction:s


I. Free Radical Halogenation Selectivity II. Free Radical Reactions III. Elimination and Stereochemistry IV. Elimination Study V. Phermones VI. Creen Synthesis VII. Conjugated n-Networks VIII. Diels-Alder Reaction Rate Study IX. Diels-Alder Reaction X. Claisen and Cope Rearrangements XI. Isoprene Units XII. Terpenes XIII. Fatty Acids and Oils XIV. Occidentalol Synthesis
Questions not Based on a Descriptive Passage

(r -7)
(B

14)

(r5 - 2t)
(22 - 28) (2e - 35) (36 -

4t)

(42 - 48) (4e - 54)

(s5 - 60)

(6r - 67)
(68 - 73) (74 - BO) (81 - 86) (87 - e2) (e5 - loo)

Alkanes and Hydrocarbon Reactions Scoring Scale

Kaw Score 84 - 100

MCAT Score

l5-15
10-12 7 -9

66-85
47 -65

34-46

4-6

t-33

t-5

Passage

(Questions 1 - 7)

The chlorination of methylcyclopentane would yield


how many different structural isomers?

Alkyl halides can be lormed tiom a halogen reacting with riith either chlorine gas and UV radiation to form chloroalkanes. or rvith bromine liquid and UV radiation to form bromoalkanes. The bromination of
an alkane. Alkanes can be treated

4.2 B. 3
D.
5

c. 4

an alkane is a slower and more selective reaction than the chlorination of an alkane. Because of this greater selectivity, bromination is preferred over chlorination in the synthesis of

highly substituted haloalkanes.

For the chlorination of an alkane, the reactivity


preference for carbon substitution through liee radicals follows the trend 3'carbon > 2" carbon > 1'carbon by a factor of 4 : 2.5 : I at a given temperature. This means that

4.

What is the most abundant product in the bromination


of 2-methylbutane?

for a compound like butane with four secondary hydrogens

is not based on random probability aione. Random


probability predicts the formation of two 2-chlorobutanes for every three 1-chlorobutanes. Because of the reactivity preference of secondary carbons over primary carbons by a ratio of 2.5 : l, the product distribution instead is five 2chlorobutanes to three 1-chlorobutanes. This implies that the percentage of secondary products is 62.5Vo (rather than 40Vo which is expected when there is no site preference). The calculation of the number of products is based on the reactivity coefficient times the number of unique hydrogens. In a molecule like pentane, there are three types of hydrogens in a 3 : 2'. I ratio. Six hydrogens are secondary and six are 5

and six primary hydrogens, the ratio of chlorination products

A. B. C. D.

1-bromo-2-methylbutane

2-bromo,2-methylbutane 2-bromo-3-methylbutane
1-bromo-3-methvlbutane

Following the free radical monochlorination of pentane what is the ratio of2-chloropentane to 1-chloropentane?

A. 2:3 B. 5:3 c.2.5:1 D. 3:


1

primary. The

abundance

of the primary hydrogens is

5.
1

hydrogens x 1 reactivity for 6. The abundance for carbon two and carbon four is 4 hydrogens x 2.5 reactivity for 10. The abundance for carbon three is 2 hydrogens x 2.5 reactivity for

This means that the chlorination product ratio is six lchloropentane to ten 2-chloropentane to five 3-chloropentane.

In the monochlorination of n-hexane, how can

1.07 2-chlorohexane
explarned?

to one

a ratio of 3-chlorohexane be

How many degrees of unsaturation are there in the


compound C5H9C1?

A. B. C. D.

Based on the relative reactivity of the carbons and the abundance of hydrogens, a ratio of 1.07:1 is
expected.

A. 0 B. C,2 D. 3
1

The second carbon of n-hexane is less sterically


hindered than the third carbon. The second carbon ofn-hexane can better stabilize a free radical due to resonance.

The third carbon of n-hexane can better stabilize a free radical due to the inductive effect.

How many structural isomers of C5H11Cl are there?

A. 5 B. 6

. If the alkane reactant


A

exhibits ring strain, the instability

of the free radical intermediate may cause the ring to


break. Which free radical alkane is LEAST stable?

c.1
8

D.

. B. C. D.

Propyl free radical (C:HZ.) Cyclopropyl free radical (CSHS.)

Butyl free radical (C+HS.) Cyclobutyl free radical (CqHl.)

Copyright @ by The Berkeley Review@

289

GO ON TO THE NEXT PAGE

Passage

ll

(Questions 8 - 14)

9.

A halide can be substituted onto an alkane by way of a free radical mechanism. Halogenation of an alkane is initiated by the homolytic cleavage of a diatomic halogen
molecule into free radical halogen atoms. During subsequent steps in the reaction, an alkane reacts with the halogen free radicals to form an alkyl halide. The reaction requires some source of activation energy to cleave the halogen-halogen bond. Depending on the halogen-halogen bond strength, the amount of activation energy necessary varies from thermal to ultraviolet radiation for the initiation step.

Which of the following energy diagrams corresponds to reaction of 12 with an alkane?

B.

I
bo

E n

Reaction Coordinate

Reaction Coordinate

The mechanism is a sequence broken down into steps that fit into one of three categories: initiation, propagation, and termination, in that order. The initiation step involves homolytic cleavage of a halogen-halogen bond to form two free radicals. The second phase of the reaction sequence is propagation where the free radical is transferred through a set of abstraction reactions. The last phase of the sequence is the termination step where two free radicals combine to fbrm a sigma bond. The reaction involves two transition states. in which the second is of higher energy than the first. Table 1 shows the bond energies of the halogens and the reaction enthalpies for the various halogenation reactions:

Reaction Coordinate

Reaction Coordinate

Compound
F2

B.D.E.

AH.t
-483 114
kJ kJ kJ

0.

The first propagation step in a free radical reaction is which of the following?

154

kJ

mole

mole mole

Clz
Br2
I2

n9

kI
kJ kJ

mole

193 149

mole mole

-33

A.XZ + R. -> RX + X. B. RH + X. -> HX + R. C.Xz 2X. -> D. X. + R. -> RX

mole

t1'.. kJ
mole
1

Table I
The enthalpy of a chemical reaction can be found by
using Equation

1.

The strongest halogen-halogen bond corresponds to


which of the following?

l.
Energy6onds broken - Energy6on6s fbrmed

AHr^n =

Equation

A. B. C. D.

The shortest halogen-halogen bond. The second shortest halogen-halogen bond. The longest halogen-halogen bond. The second longest halogen-halogen bond.

The average bond dissociation energy {br a sigma bond between an spJ-hybridized carbon and ahydrogen is 413 kJ per mole. A hydrogen free radical cannot be formed in this
reaction mechanism. 8

The MOST stable type of carbon fr-ee radical fonned in

12.

Which of the following steps is NOT found in a free


radical halogenation reaction
?

the monobromination
described as:

of (R)-3-methylhexane is best

A. B. C. D.

primary.
seconclary.

tertiary.
quatefnary.

A. RH + X. RX + H. B. XZ + R. -> RX + X. -> C. RH + X.-> HX + R. D. X. + R. RX ->

Copyright @ by The Berkeley Review@

290

GO ON TO THE NEXT PAGE

13.

with the alkane and minimai multiple halogenated


products ar.e isolated from the product mixture?

Which of the following conclusions can be inferred from the observation that usually only one halide reacts

Passage

lll

(Questions 1S- 21)

I. II. III.

Halogens do not help to stabilize free radical


intermediates.

Halogens, once on an alkane, increase the C-H


bond strength.

Halogens, once on an alkane, decrease the C_H


bond strength.

A. II only B. III only C. I and II only D. I and III only

predominantly the cis (Z) geometrical isomer. Reactions proceeding by the E1 mechanism result in the formation of mostly the trans geometrical isomer with some cis geometrical isomer formed. The E2 mechanism, on the other hand, allows for the formation of either the trans or cis geometrical isomers in high purity, if there are chiral centers present in the reactant. Reaction 1 is an E2 reaction carried out in the hopes of synthesizing Z-3-methylpentene, to be used in subsequent steps ofg total synthesis process:

important to be able to synthesize alkenes in a geometrically specific manner. This is to say that it is beneficial to be able to create predominantly the trans (E) geometrical isomer or

Due to the usefulness of alkenes in synthesis, they are often a starting material in many synthetic transformations. This can be attributed to the numerous addition reactions alkenes undergo. Because of their synthetic usefulness, it is

Br
1

4.

reaction associated with fluorination of an alkane?

Given that a C-F bond energy js 462 kJ per mole and a H-F bond energy is 588 kJ per mole, what is the heat of

CH"

'

HqC
-->
rbutOK
t-butOH

CHrCH'

H:C

CH2CHj

A. - 483 B. C.

kJ

><

CHr

mole

Reaction I
Reaction 1 proceeds by an E2 mechanism at elevated temperature, so the bromine leaving group and the hydrogen

-lt6 kJ rnole +116 kJ


mole mole

D. + +83 kJ

elimination, nor is the structure drawn in its most stable conformation. Counting the conformer shown in Reaction l, there are three staggered conformations total for the reactant. Only one of the three orientations has the anti orientation
necessary for the E2 reaction.

on carbon 3 must be aligned in the anti orientation. As drawn in Reaction 1, the bromine leaving group and the hydrogen on carbon 3 are not correctly-al-igned for f2

15.

What conclusion can be made about the elimination


reaction that generates the following data?

Trial

(H3C)3CBr 0.25 M 0.50 M 0.40 M

t-ButOK 0.25M

Rate

I il
III

4.61 x t0-3 M/s


9.29

0.25M
0.50M

x t0-3 Mls 1.47 x 10-2 Mts

A. The reaction proceeds by an E1-mechanism,


because the data indicate that the rate-determining step is unimolecular.

B. The reaction proceeds by an E1-mechanism,


because the data indicate that the rate-determining step is bimolecular.

The reaction proceeds by an E2-mechanism,


because the data indicate that the rate-determining step is unimolecular.

D.

The reaction proceeds by an E2-mechanism,


because the data indicate that the rate-determining step is bimolecular.

Copyright @ by The Berkeley Review@

291

GO ON TO THE NEXT PAGE

16.

Which of the staggered complexes is MOST stable for


the reactant?

20.

What is the role of concentrated sulfuric acid in an E1 elimination reaction?

A. B.
C

The conformer drawn in the example.

A. B. C.
D

Sulfuric acid protonates the leaving group making

The conformer fiom which the E2 elimination


takes place.

it

a better leaving group.

Sulfuric acid serves to dehydrate the solvent


preventing back reaction.

The conformer with carbons


another.

and 4 anti to one

D.

All three staggered conformers all equal in stability.

Sulfuric acid dissociates into sulfate which helps remove the proton allowing the leaving group to
leave.

Sulfuric acid stabilizes the carbocation intermediate by protonating the cationic carbon.

17. The

loss of optical rotation

in the reaction can be


21
and

explained by which of the following statements?

A. B. C.
D

Ez eliminates the carbon 2 chiral center.

EZeliminates the carbon 3 chiral center.

How many units of unsaturation are present in

the

Ez eliminates both the carbon 2 chiral center


the carbon 3 chiral center. The product is a pair of enantiomers.

product of chlorocyclohexane and strong base and heat?

A. 0 B. I
D.3

c.2

8.

Which of the following structures (from the perspective


of the eye) is the Newman projection for the reactant?

Br CH:

H$t),
H:C
A.
CHzCHT

",$.".
CH2CHj

Br

B.

CH:
H
CH2CH3

c.

Br

D.

Br

;"4; ;"V:
CH2CH3 CH2CH3 rea{

pru

reag

19.

The ENANTIOMER of the reactant has which of the


followin g stereochemical orientations
?

)'t

A. 2R, 3R B. 2R, 35
C. 25,3R D. 25, 35
Copyright @ by The Berkeley Review@
292

GO ON TO THE NEXT PAGE

copl:

Passage

lV

(Questions 22 - 28)

23.

Which of the reactions results in an optically active


product mixture?

Elimination results in the formation of a new n-bond following the loss of two groups from the molecule. The reaction can proceed by one of two possible mechanisms: E1 or 82.
In E1 reactions, the leaving group first leaves to form a carbocation intermediate. The carbocation has the potential to undergo rearrangement. The solvent serves as a base and deprotonates a hydrogen off of the carbon adjacent to the cationic carbon. The result is the formation of a new fi-bond. In E2 reactions, the leaving group leaves simultaneously as the proton is removed by a strong base. The proton and leaving group must be anti to one another in a staggered conformation to undergo an E2 reaction. An E2 reaction requires that the base be strong, to remove a weakly acidic hydrogen, and bulky to reduce the amount of side products formed from nucleophilic substitution. Figure I shows three
elimination reactions carried out concurrently.
Reaction I: CI

A. B. C. D.

Reaction I only Reactions I and

II only Reactions II and III only Reaction III only

24.

When the following reaction is carried out, why does


the optical activity disappear?

cH3
CHr

conc.

HrSOa

Major Product

60-

(crp=6';

A. B. C.

After the sulfate group substitutes for the hydroxyl


group, the chiral centers cancel one another. The product is meso.

@
Reaction Reaction

Yg

The major product is an achiral alkene, resulting


from rearrangement.
The product is an achiral alcohol.

t-butOK -----'+

t-butOH,50"C

D.

& &
CI

II:

25

Which of the following observations are consistent with


the mechanisms discussed in the passage?

t-butOK -..+ rbutOH,50'C

I. II.

Increasing the base concentration


increases the reaction rate.

in Reaction I
at

The amount of alkene product is maximized


lower temperatures.

III:

OH

_->

conc. HrSOa

60'c

III. Reaction II has a competing an Sp2 reaction. A. I only B. II only C. I andll only D. I and III only

Figure L. Three Elimination Reactions

The product shown in each of the reactions is the major

product. Among the minor side products for each of the three reactions is the alkene products from one of the other two
reactions.
2

26. Why do Reactions I


products?

and

II yield

different major

A. Et B. C.
D

reactions can undergo rearrangement hydrogen is on the correct side of the plane.

if

the

2.

Which of the elimination reactions in Figure


rearrangement?

involves

Et reactions are influenced by steric hindrance. EZ reactions require that the leaving group and
proton are anti to one another.

A. B. C. D.

Reaction I only
Reactions I and

II only Reactions II and III only Reaction III only

EZ reactions require that the leaving group


proton are syn to one another.

and

Copyright @ by The Berkeley Review@

293

GO ON TO THE NEXT PAGE

27

How does the hybridization change fbr the carbons common to both rings in Reaction III? A, sp2 to sp3

Passage

(Questions 29 - 35)

B. spz to sp C. sp3 to sp2 D. spi to sp

Phermones are chemicals secreted by animals (most commonly insects) that elicit a specific behavioral reaction in other members of their same species. They are effective in low concentration in sending signals between members of the same species for such things as reproduction, danger warnings, and aggregation (in the case of a food supply.) Many phermones are simple hydrocarbons. For instance, when in danger, ants secrete undecane (.Cy1H2$ or tridecane (Cf :HZS) to inform other ants of the trouble. Many of the

28. If diethyl
a'E,rN

amine, (H3CCH2)2NH, is used in Reacrion I instead of t-butOK, what is the major product?

traps and sprays we use to capture and kill insects take of sex attractants. The structures of four sex phermones are shown in Figure 1.
advantage

Tiger Moth sex attractant (2-Methyl heptadecane)

o4.",
Oriental Fruit Moth sex attractant
[(E)-8-DodecenI

-yl acetare]

HpCa

HH

\J /\

.CttHzt

House Fly sex attractant

(Muscalure [(9Z)-Tricosenel)

Silkworm Moth sex attractant (Bombykol)

Figure

Four random phermones

Phermones are specific to each species, because receptor proteins are highly selective in what they bind. In one of the rare cases where two geometrical isomers both elicit the same response, the Oriental Fruit Moth responds to both the E-

isomer, shown in Figure 1, and the Z-isomer. There are cases where two similar species to a phermone that is similar

in structure, but not exactly the same. For instance, the


Grape Berry Moth uses (Z)-9-dodecen-1-yl acetate as a sex attractant in roughly the same concentration that the Oriental Fruit Moth uses (Z)-8-dodecen-1-yl acatate.

29. The Silkworm Moth

sex phermone has following structural features EXCEPT:


one stereogenic center. one cis double bond. no tertiary carbons.

all of

the

A. B. C. D.
Copyright @ by The Berkeley Review@

conjugation.

GO ON TO THE NEXT PAGE

30.

Relative to the Oriental Fruit Moth sex phermone


shown in Figure 1, the follorving compound is:

5.

Which spectroscopic observation does NOT correlate


with the conesponding compound?

A.
cH.

The Oriental Fruit Moth sex phermone: an IR


absorbance at 1141 cm-l Bombykol: a UV-visible absorbance at22'l nm Muscalure: two signals around 5.00 ppm in its luNIrvlR

oA
A. B. C. D.
a confbrmational isomer. a geometrical isomer.

il

B. C.
D

an optical isomer.
a structlu'al isomer.

The Tiger Moth sex phermone: 14 signals in its

l3cNltR

spectrum

31.. Which of the following

statements accuraiely relates the


1?

four structures shown in Figure

I. II. m.

Muscalure has a shorter wavelength of rnaximum absorbance in UV-visible spectroscopy bombykol. The Tiger Moth sex phelmone has the more units
of unsaturation than undecane.

The Oriental Fruit Moth sex phermone can


classifled as a terpene.

be

A. I only B. iII only C. I and II only D. i and III only


3

2.

What physical property is NOT expected for muscalure?

A. Low miscibility in water B. A boiling point above room temperature C. High lipid solubility D. The ability to rotate plane-polarized light
33.
Which of the phermones in Figure
number of primary carbons?

has the greatest

A . The Tiger Moth sex phermone B. The House Fly sex phermone C. The Oriental Fruit Moth sex phermones D. The Silkworm Moth sex phermone

34.

The Green Peach Aphid def'ense phermone is shown


below.

What is NOT true of the structure?

A. It can be synthesized from isoprene units. B . It has *o.e sp2-hybridized carbons than spjhybridized carbons. C . It can potentially undergo 1,4-addition. D. lt is highly flexible.

Copyright O by The Berkeley Review@

295

GO ON TO THE NEXT PAGE

Passage

VI

(Questions 36 - 41)

-l 6

In Reaction I, the alkyne is besr described

as:

In the recent years, many chemists around the world have

shifted their focus to so called green chemistry. Green


chemistry, also called sustainable chemistry, aims to develop chemical reactions and processes that are environmentally safe. The goal is to reduce waste generation rather than

. B. C. D.
A

dienophile.

an electrophile. a nucleophile. an oxidant.

ernploying waste management--the "end of the pipe" solution. The most significant alteration to traditional
chemistry is the recycling of solvent, or in optimal cases, the
3

7.

elimination of solvent. This is achieved in many ways, including doing reactions under high pressure to make the
system act like a supercritical f'luid.

Which of the following intermediates is consistent with the two structural isomers formed in Reaction II?

A.o
il
C

B.o
il

Areas of current research in green chemistry include the use of renewable raw materials, direct oxidations using oxygen, improved separation during the course of a reaction, and all forms of catalysts. The aim is to maximize atont-

/\ HzC-

,zt\
CHCH3

economy, the tracking of how many atoms used in the reaction end up in the product, by not using solvents or protecting groups. Figure I lists three atom-economical
reactions used in green synthesis.

CHz

c'

HrC- C/
H2CO

Reaction I:

-tt

.cH,

D' HC-C H2C-O

H:C

.+ i.sd'u.":.:.
co2cHj
98%o vterd

COzCHT

tt

,/

CH.

38. Which of the reactions in Figure I involves the


fbrmation of new stereocenters?

Reaction

II:

H
H2C= CHCHj

A
Jt
O

CH:CHzCHr

+CO+Ho- - cat. Rh

. B. C. D.
A

III only II only Reactions I and III only Reactions II and III only
Reaction Reactions I and

CH(CHr)r

9.

Which of the following changes does NOT fit with th;


philosophy of green chemistry?

. B.
C

Using supercritical fluid as a solvent.

Pd, H2

Using protecting groups and not removing ther until the very last step of the reaction.

Supercritical CO2

. Using solid-state
equipment.

catalysts

built into the

1ar

cHr
HrC

l
Hrc
> 997o yield
Syntheses

D.

Running a constant stream of oxygen gas througi the reaction vessel fbr oxidation reactions.

Figure 1. Three Atom-economical Green

Reaction I is a Diels-Alder reaction. Reaction 2 is a hydrofbrmylation reaction, and Reaction 3 is a hydrogenation reaction. All o1' the reactions in Figure i start with alkenes, a common starting material in the production ol plastics and polymers. Green chemistry is ideal fbr polymerization reactions, which by design aims to minimize the material need to carry out the propagation reaction. Green synthesis
techniques can be applied to any reaction.

40.

Reaction III can be described by all of the follorvin_. terms EXCEPT:

A. reduction. B. hydrogenation. C. stereoselective. D. regioselective.


GO ON TO THE NEXT PAGE

Copyright @ by The Berkeley Review@

41

What is the major product of the reaction below?

Passage

Vll

(Questions 42 - 48)

e(+
A.

,\A

la" -:+
o

lt

A chemist set out to synthesize a series of conjugated dienes. Starting with an allylic alcohol, generating a
conjugated diene involves an acid catalyzed elimination reaction. Elimination by way of an E1 mechanism to form a conjugated diene is shown in Reaction 1.

oB'o
o
o
D.

tu
Compound

OH

conc. HrSO, A

Compound

II

C.

ee("q"
o uo o
o

Reaction

Reaction 1 is monitored using UV spectroscopy. Over the course of the reaction an intense UV absorban ce at 179 nm diminishes as a new peak at 222 nm appears. When the product of Reaction

I is treated with acidic

water, two products of detectable quantity are formed. Figure I shows the distribution of the two hydration products at 35'C, labeled Compound 3a and Compound 3b.

OH

@
I

OH Compound IV
60.\Vo

Compound
39.27n

III
Figure I

as the temperature

The percentage ofthe secondary alcohol formed increases ofthe hydration reaction increases. This is

attributed to a shift from kinetic control to thermodvnamic


control.

It is found that if the allylic alcohol in Reaction I is replaced by a new compound containing both an alcohol group and a carbon-carbon n-bond, with the exception of a
vinylic alcohol, a conjugated diene is formed upon treatment with concentrated strong acid at elevated temperatures.
4

2.

Which of the following starements accurately reflect


Reaction
I?

I. II. m.

Rearrangement is possible during the reaction.

A vinylic carbocation is formed


in the reaction.

as an intermediate

The first step of the reaction is the protonation of


the hydroxyl oxygen.

A. I only B. II only C. I and II only D . I and III only


Copyright @ by The Berkeley Review@

GO ON TO THE NEXT PAGE

43. Which

spectroscopy technique is MOST effective ln distinguishing Compound III from Compound IV?

8.

What is the major product of the reaction below?

a. IHNlaR B. Infraled C. Ultraviolet D. Visible


A.

O[.",
as:

H2O/H2SOa

75'C

B' oH -..>

44.

Cornpound

IV is best described

A. B. C. D.

a single, achiral molecule a single, chiral molecule.

d.,, d..,
"G;
is an allylic

pair of diastereomers. a pair of enantiomers.


a

45. Which of the following


alcohol?

^er* "ff* 'er-* 'er*


2,4-Hexadiene 2-methylcyclopentadiene 1,4-cycloheptadiene 1,3-cyclohexadiene

compounds

46.

1,2- and 1,4-addition is possible in all of the following


compounds EXCEPT:

A. B. C. D.
7.

Which of the following species is NOT an inrermediate formed during the hydration of Compound II?

A'

A^ \-ry

tgr,

t;,,\--

'eD

Copyright @ by The Berkeley Review@

GO ON TO THE NEXT PAGE

Passage

Vlll

(Questions 49 - 54)

way to synthesize polycyclic compounds. New carbon_


temperatures.

The Diels-Alder reaction is quicklv becoming the premier

The product of a Diels-Alder reaction is a cyclohexene derivative. The Diels-Alder reaction is classified as an electrocyclic reaction and it is believed to proceed by a
concerted mechanism that goes through a so called aromatic

carbon bonds are formed when a conju_sated diene reacts with a compound containing a n-bond (dienophile) at elevated

transition state, where the six n-electrons in the reactant


interact to form new bonds.

The reaction is fastest when the conjugated diene has an

electron-donating substituent and the dienophile has

an

electron-withdrawing substituent. Reaction 1 was carried out for a total of eight trials to determine the effects of electronics and steric hindrance on the reaction rate. Table I lists the data fbr the eight trials.

In the presence of a Lewis acid, the rate of the reaction increases substantially, which implies that the Diels-Alder reaction has an alternative mechanism, where the conjugated diene acts as a nucleophile by attacking the dienophile. This infbrmation is in agreement with the rate shift associated

with the addition of electron-withdrawing groups to

the

dienophile.

49

The product shown in Reaction


formed in Trial 6?

I is one of two

of the following pairs of molecules represents the product mixture


A.

enantiomers that is formed. Which

CH: CH: CH:

Reaction I

""1( CHr
X
H Relative
rate

&

cHr

Trial

o
CH:

t.0

"J( CH:
2

CH:

5.3

CHr

&

cHs
CHq

CH:

cHr

8.1

"ff::,
&
CH: CH:

CH:

CH:

0.00s0
.

'\'.
o

82

cHr

'Y'"',
o

4t1

'$f;:
&

cHr cHr

cHr

CHr

T,"'
o

613

o
0.34

CH:

CH:

'1('"'
o
1

Table

Copyright @ by The Berkeley Revierv@

299

GO ON TO THE NEXT PAGE

50. How would

the relative rate change

if ethyl groups were

s4. Which of the following compounds is the


dienophile?

WORST

used instead of methyl groups in Trial 4?

A. B. C. D.

The relative rate would be significantly lower than 0.005.


The relative rate would be about 0.005. The relative rate would be significantly higher than

A.NB.

0.005.

The relative rate would be 200, the reciprocal of


0.005.

Io'""''
COCHzCHT

51. Addition of which of the following


would increase the rate?

species to

Trial

c.o
7

D' /

cHrCHe

A. B. C. D.
(t

AICI3 CCla KH

("

()

LiAlHa

If Trial 2 proceeds by a nucleophilic mechanism,

rather

than a concerted mechanism, the intermediate carries charges. Which of the following structures best shows the structure of the intermediate in Trial 2?

c'

y*,(/*,
cHr oo D'
cHq oo
The methyl groups on the conjugated diene exhibit

53. How

can the significantly lower reaction rate in Trial 4 than Trial 2 best be explained?

A.

steric hindrance when the conjugated diene and


dienophile form the transition state.

B. The methyl group on the conjugated


diene.

diene

withdrawals electron density from the conjugated

C. The two methyl groups are acting like


intramolecular Lewis acids on the dienophile.

The carbonyl group is more electron-withdrawing on the dienophile in Trial 2 than it is in Trial4.

Copyright @ by The Berkeley Review@

GO ON TO THE NEXT PAGE

Passage

lX

(Questions 55 - 60)

6.

Which of the following conclusions can be drawn from


the data in Table

In a Diels-Alder reaction, the alignment of the diene and dieneophile determines the structural orientation of the substituents in the final product. If both the diene and the dieneophile are asymmetric, then there are two different orientations that the two reactants can assume when they align to form the transition state. The preferred alignment can be predicted using resonance theory. The two reactants align in a manner so as to have a partially positive site attacking a partially negative site. A generic Diels-Alder reaction of an asymmetric diene with an asymmetric dieneophile is shown in Figure 1.

A. B. C. D.

OCH3 is more electron donating than CH3 because OCH3 in the X position yields more producr A. OCH3 is more electron donating than CH3 because OCH3 in the X position yields more producr B, CH3 is more electron donating than OCH3 because CH3 in the X position yields more producr A. CH3 is more electron donating than OCH3 because CH3 in the X position yields more product B.

.+.
Figure

*ff..q.
X

7.

Predict the major product for the following reaction:

H3CH2CO )^.

o
Compound A
CompoundB

l+L^*
\
A.
cHrcr B.

Asymmetric Diels-Alder reaction

the reactants are asymmetric, the product distribution of Product A to Product B is never 50-to-50. When X is electron donating and Y is electron donating, Product A is the major product. When X is electron withdrawing and y is electron donating, Product B is the major product. Table I lists the product distributions for a series of reactions where

If

''""44.
\,4
cH2cl

CH2CI

the

and

Y
X

groups are varied.

In both Product A
A
B
6Vo l2%o

and

Product B, the Y group is always cis to the carbonyl group.

c. H3CH2CO

D.
H3CH2CO

Y
NHCH3

OCH3 OCH3

94Vo 88Vo 87Vo 63Vo

CH:
NHCH3

CH:

l3Vo
37Vo 82Vo 69Vo

eq: Oq:
o o ____>
150'C

cHr
COCH3 COCH3

CH:
NHCH3

lSVo

CH:

3lVo
1

Table

8.

What is the major product for the following reaction?

The product distribution in Table 1 supports


based
considered to be mildly electron donating.

the is

prediction about the electron donating and withdrawing effects

on resonance theory. A methyl substituent

,o

55. If

group (-CR=O), making the alkene reactant


symmetric, what would be predicted for the distribution
between Product A and Product B?

the Y-substituent is a second carbonyl functional

^O:")
c.

A. B. C. D.

>50Vo Product A;<S}VoProductB <50Vo Product A:>50Vo ProductB


507o Product A:50Vo Product B

Product A and Product B are the same compound

Copyright @ by The Berkeley Review@

301

GO ON TO THE NEXT PAGE

59. Two structural

isomers are formed fr.om Diels-Alder

Passage

(Questions 61 - 67)

reactions that involve:

A. B. C. D.

a symmetric diene with a symmetric dieneophile. an asymmetric diene with symmetric dieneophile.

symmetric diene with asymmetric dieneophile. an asymmetric diene with asymmetric dieneophile.
a

Pericyclic reactions are single-step reactions that involve the movement of electrons through cyclic transition states that involve pi and sigma orbitals. One class of pericyclic reactions is the sigmatropic rearrangement, which involves the migration of a sigma-bonded group across a pi-electron system. The two most common sigmatropic rearrangements are the Cope rearrangement and the Claisen reatangement. In the Cope rearrangement, one 1,5-hexadiene yields a new 1,5hexadiene. In the Claisen rearrangement, an allyl vinylic

60.

Counting stereoisomers, how many possible products can be formed from the followine reaction?

ether yields an unsaturated carbonyl compound. Both


reactions are shown in Figure 1 below. Cope rearrangement

H:Co
HrC

CH.

'----*

4.2 B.4
D.

1,5-diene

A new 1,5-diene

c.8

Claisen rearrangement

16

o/N
An allyl vinylic ether

v
l)\
1

t3,31 _

A y,&unsaturated carbonyl compound

Figure

Cope and Claisen rearrangements


a

64"

When the reactant in a Claisen rearrangement includes

benzene ring, the ketone formed from the allyl phenylic ether

undergoes tautomerization and converts into a phenol. The

ultimate product is the one that is most stable. Figure 2 shows a synthesis pathway that involves a Claisen
rearrangement, a Cope rearrangement, and tautomerization.

CH:
Step A

I __>

Hrc

Step

II
o

Ia

v
OH

Step <-

Hrc III

65. rn
A.
B. C.

CH:

cHr

Figure 2 Synthesis using Claisen and Cope rearrangements


Copyright @ by The Berkeley Review@ 302

D.
Copyright

GO ON TO THE NEXT PAGE

61.

How can it be supported that the Cope rearrangement is concerted rather than a multistep process involving
substitution?

6.

What spectroscopic evidence supports the formation of a product in a Claisen rearangement?

. B. C. D.
A

All of the stereocenters All of the stereocenters

are retained

A. B.

are inverted Several cross products are formed

Appearance of a signal at 9.5 ppm in the 1HNMR Appearance ofa broad absorbance around 3400 cm-t in infrared spectroscopy Disappearance of an absorbance around 1700 cm-l in infrared spectroscopy Disappearance of two signals between 5 and 6 ppm in rhe IHNMR

C.
D

No cross products are formed

62.

Step

III in the synthesis shown in Figure 2 occurs


6

because the product:

A. B. C. D.

loses steric hindrance after oxidation. gains aromaticity after reduction.


loses resonance after tautomerization.

7.

What is true of the Claisen and Cope rearrangements in Figure 2?

gains aromaticity after tautomerization.

Step I is Claisen rearrangement and step II is Cope rearrangement; the units of unsaturation decrease from 5 to 4 during the Claisen rearrangement. Step I is Cope rearrangement and step II is Clasien rearranggment; the units of unsaturation decrease from 5 to 4 during the Cope rearrangement. Step I is Claisen rearrangement and step II is Cope rearrangement; the units of unsaturation remain 5 during the Claisen rearrangement. Step I is Cope rearrangement and step II is Claisen rearrangement; the units of unsaturation remain 5 during the Cope rearrangement.

63

B.
Heat serves what role in the Claisen rearrangement?

A. To provide energy to overcome


barrier

the activation

B. To drive the exothermic reaction C. To generate pi-bonds D. To fbrm sigma-bonds

D.

64

Which of the following orbital arrangements represents the transition state of a Cope rearrangement? A.
B.

5.

The reactant in Figure 2 is best described as a:

A. B. C. D.

allyl benzylic erher. allyl phenylic ether. vinyl benzylic ether. vinyl phenylic ether.
303

Copyright @ by The Berkeley Review@

GO ON TO THE NEXT PAGE

Passage

Xl

(Questions 68 - 73)

The isoprene unit is one of nature's favorite building blocks. Isoprene (2-methyl-1,3-butadiene) reacts at carbons
one and two or one and four with other isoprene molecules to ibrm terpenes, a class of bio-organic molecules. Terpenes are found in such natural products as rubber and essential oils. Nearly all of the naturally occurring terpenes result from the head-to-tail connectivity of isoprene units. They connect by
i

il
1l

1i

undergoing either nucleophilic substitution or electrocyclic addition, such as a Diels Alder reaction. Figure 1 shows
some common terpenes.

H:C

-s
CH:
cr-Pinene

i"'

cHr

Caryophyllene

HSC CH:

CH:

cHr

OH

Vitamin.\

CHr
HO

*o^
Citronellol

Cholesterol

Figure 2. Biosynthesis of Cholesterol


6

Figure 1. Common Terpenes


Both plants and animals synthesize terpenes. Pinene, a monoterpene, is fbund in plants and Vitamin A1, a diterpene, is found in both plants and animals. Smaller terpenes are found primarily in plants, while some larger terpenes, such as lanesterol (a C-30 precursor to steroid hormones) and Bcarotene (a C-40 source of vitamin A), are found in plants and animals. Terpenes can be modified into other compounds,
known as terpenoids. Studies in biogenesis show that the large terpenes are synthesized starting from isopentenyl pyrophosphate. The pyrophosphate adds across the diene of isoprene to form either isopentenyl pyrophosphate or dimethylallyl pyrophosphate.

8.

Which of the following compounds is NOT a terpene?

A. Limonine (C16H16) B. Geraniol (C16H1gO) C. Patchouli alcohol (C15H26O) D. Stearol (C1gH3gO)

69.

What irregularity in a sample of a sesquiterpene (1-< carbon terpene) would indicate that the compound wa:
synthesized in lab as opposed to extracted from a plant.'

These molecules then add

to one

another

by way of

nucleophilic substitution reactions, where the pyrophosphate acts as a leaving group. Geranyl pyrophosphate (a name given to C-10 terpenes) is the first monoterpene in many natural synthetic pathways. More isoprene units are added to the monoterpene to forfir other terpenes and terpenoids. Figure 2 shows a basic schematic for the biosynthesis of cholesterol starting from isopentenyl pyrophosphate.
Copyright @ by The Berkeley Review@ 304

. The compound is not enantiomerically pure. B. The compound is not a racemic mixture. C. The compound had impurities with 16 carbons. D . The compound had impurities with 20 carbons.

GO ON TO THE NEXT PAGE

70. Which carbon

is most susceptible to nucleophilic attack

Passage

Xll

(Questions 74 - B0)

in isopentyl pyrophosphate?

o
il
I

o
il
I

o- P- o- PP
o()

oH

A. B. C. D.
7

Carbon Carbon

Terpenes are natural organic molecules found in plants and animals. They are formed from the basic subunit of isoprene, a five-carbon conjugated diene. Terpenes and terpenoids, biological molecules derived from terpenes, have a total carbon count that is divisible by five. Terpenes are classified according to the number of carbon atoms they contain. Monoterpenes have ten carbon atom, sesquiterpenes have filteen carbon atoms, diterpenes have twenty carbon atoms, sesterterpene has twenty-five carbons and so on. Because terpenes are natural products, they are common

Carbon 2
3

Carbon 4

in many household items, such as flavoring agents in various {bods and the active molecule in many drugs. Much of organic chemistry research involves the development of in

1.

Isoprene units are believed to be formed from three acetyl coenzyrne A molecules. What is a likely side
product from the reaction?

uilro synthesis of terpenes and terpenoids. Some naturally occurring monoterpenes are shown in Figure 1.

o
il
I

--.\ HrC- . B. C. D.
A
1''
Carbon dioxide gas

SCoA

Acetyl Coenzyme A Carvone

Limin

Ethanol Acetic acid Isopropanol

The biosynthesis
units?

of which of the following molecules likely involved a Diels Alder reaction with isoprene

Myrcene

A"
Camphor

. Caryophyllene B. Citronellol C. a-Pinene D. Vitamin A1


A

Figure 1. Four Common Monoterpenes


Some terpenes contain oxygen, which is added in a way that does not alter the carbon skeleton. Carvone is evident by a carbonyl absorption at 1750 cm-1 in the IR spectrum and a strong UV (e > 10,000) absorption at L.'nu* = 242 nm.
7

73.

Which of the labeled bonds in y-terpinene was formed in the biological synthesis fiom isoprene units?
Bond d

4.

When limin is converted into carvone, what type of


reaction has to transpire?

CH:

. B. C. D.
A
7

Oxidation of carbon
Reduction ofcarbon

Hydrolysis of

n-bond

Nucleophilic substitution

A. Bonda B. Bond b C. Bondc D. Bondd

5. Which compound in Figure 1 is LEAST likely to


r.rndergo ozonolysis when treated

with 03?

A. Camphor B. Carvone C. I-imin D. Myrcene


305

Copyright @ by The Berkeley Review

GO ON TO THE NEXT PAGE

76.

How many singlets does camphor show in its proron NMR spectrum?

Passage

Xlll

(Questions B1-

BG)

A. B. C. D.

Two
Three

Many processed fbod products often contain partially


hydrogenated vegetable oil as one oftheir ingredients. Partial hydrogenation serves to reduce some of the n-bonds found in

Six Nine

natural

oils.

Naturally occurring fatty acids, such

as

vegetable oil, often have long carbon chains. They can be hydrogenated to convert the alkyl chain, which may contain multiple double bonds, into to an aliphatic R group.

77

If myrcene reacts with another isoprene unit, what kind of terpene is fbrmed?

Hydrogenation raises the compound's rnelting point, and ofien converts a naturally occurring liquid into a solid. The
fatty acids can be found as either the carboxylic acid or as pan of a fatty acid triglyceride. Figure I shows the enzymaticalll' controlled conversion of a fatty acid triglyceride into glycerol and three fatty acids.

A. B. C. D.

Diterpene Monoterpene
Sesquiterpene Sesterterpene

-oA*, o
- oAo
Figure R2 + H2o
Li'ases

soAn,
o
o

78.

Camphor

is likely to show which of rhe following

physical and chemical proper ties'/

L II. m. A. B. C. D.

High water solubility

A boiling point above 298K


No specific rotation of plane polarized light

-oA*. 1

[I

* HoAnu

HoAn,

Enzymatic Hydrolysis of a Triglyceride

I only

II only and II only I and III only


I

79.

Does limin display a stlong (log e > 4) UV absorption?

A. B. C. D.

Yes, because the n-bonds are spaced fal apart. Yes, because of the six-membered ring,

The fatty acid is isoiated when a tatty acid triglyceride is hydrolyzed. Three carboxylic acids are formed from the fatty acid triglyceride. The R in Figure 1, represents any alkyl group. In naturally occurring fatty acids, the R has an odd number of carbons. Including the carbon of the carboxylic acid lunctional group, naturally occurring fatty acids have an even number of carbons. This is attributed to the fact thar fatty acid biosynthesis occurs two carbons at a time, via acetyi coenzyme A. Natural fats can be distinguished fiom synthetic lats by their carbon chain length. Table 1 lists some common fatty acids that are naturally found in animals:

Acid
Arachidic Arachidonic
Behenic

Formula
CH3(CH2)1sCO2H
CH
g

n
0

No, because there is no carbonyl group. No, because the n-bonds are not conjugated.

(C Hz)+( CH =C HC

H2) +(fH) zCO zH

4 0 0 0
2
3

CH3(CH2)2sCO2H

Lauric
8

CH:(CHz)roCO2H
CH3(CH2)22CO2H
CH j (CH 2 )a(C H=CHC Hz) z(CH) oCO zH

0.

Which compound is the direct product of a Diels Alder


condensation of two isoprene units?

Lignocaric Linoleic Linolenic

A. B.
C

Carnphor Carvone Myrcene

CH3CH2(CH=CHCHz): (CH2)6CO2H

D.

" Limin

Myristic
C)leic

CHj(CH2)12CO2H
CH 3 (C H2
)7

0
)7

CH =C H ( CHz

CO

2H

Palmitic Palmitoleic
Stearic

CH3(CH2)1aCO2H
CH 3 (C H 2 ).5 CH =C H (CHz
)1

0
CO

2H

CH3(CH2)16CO2H

Table 1 Common Fatty Acids


Copyright O by The Berkeley Review@

GO ON TO THE NEXT PAGE

Vegetable oils generally have more unsaturation than animal fats. For instance, corn oil is 63Vo linoleic acid and 26Vo oleic acid, with the rest being made of other saturated

3.

Addition of D2 with Pd catalysr reduces n-bonds by


adding deuterium to each n-bond carbon. Treatment

of

oleic acid with D2 and palladium yields a compound


with how many chiral centers?

fatty acids. Safflower

o1I

is'75Vc linoleic acid, l4To oleic

acid, and 4Vo linolenic acid with the rest being made of other saturated fatty acids. As the amount of unsaturation increases, the melting point of the fatty acid decreases, assuming that the number of carbons remains constant. For this reason, many animal fats are solids while many vegetable oils are liquids at room temperature. Fatty acids can play one of three roles in biological systems. They are found as the building blocks of cell walls as phospholipids and glycolipids. Fatty acids form derivatives that serve as hormones (intercellular messengers). Fatty acids are also used for fuel through fany acid metabolism.

A,7nro

B. One C. Two D. Four

4.

81.

What is the structure for the MOST abundant fatty acid found in corn oil?

Treatment of an alkene with potassium permanganate yields a vicinal diol at the alkene carbons. Where do the hydroxyl groups add when the most unsaturated fatty acid in safflower oil is treated with KMnO4 under basic
conditions?

A. Carbons 8, 9, 11, 12,14, and 15 B. Carbons 1,9, ll, 13, 15, and 17 C. Carbons 9,10,12, and 13 D. Carbons 9,10, 12,13, 15, and 16

B.o
HO
85

Bromine liquid is used as a quantitative test reagent to determine the amount of n-bonds per molecule of a compound. Which of the following acids consumes the
MOST Br2 per molecule?

HO D.

A. Arachidic B. Arachidonic C. Linoleic D. Linolenic

HO

86.
82. The n-bond of a fatty acid can be reduced via
hydrogenation when treated with hydrogen gas and a catalytic metal or by FADH2. Treatment of linoleic acid with FADH2 yields a product:

Complete hydrogenation of palmitoleic acid yields which of the following acids?

A. Myristic B. Palmitic C. Stearic D. Arachidic

A. with lower molecular

mass and a lower melting point than the reactant. B. with higher molecular mass but a lower melting point than the reactant.

C. with

a lower molecular mass but a higher melting point than the reactant. mass and a higher melting

D. with a higher molecular


point than the reactant.

Copyright @ by The Berkeley Review@

GO ON TO THE NEXT PAGE

Passage XIV (Questions 87 - 92)


Terpenes and terpenoids are natural compounds found in plants and animals that are built fiom 5-carbon reactants. Figure I shows the sesquitetpenoid (+)-occidentalol.

87

How many chiral canters are present in occidentalol?

A. I B. 2

c. 3

D.4
'rr4<
88.
Terpenes containing flfteen carbons are best described
AS:

loH
Figure

(t)-Occidentalol

synthesis leading to occidentalol begins with the conversion of Compound 1 into Compound 6, which can further react to form occidentalol. Fisure 2 shows the synthesis of Compound 6.

A multistep

A. B. C. D.

monoterpenes.
diterpenes.

triterpenes.
sesquiterpenes.

la"
150'C,
CO2Me

89. Which

compound has the longest maximum wavelength

of absorbance, Imax, in ultraviolet spectroscopy?

-Cq ---+ Step I

CO2Me

Compound

Compound 2 Step 2

A. B. C. D.

Compound I Compound 2 Compound


3

Compound 4

Y
SOj.Py, THF,0"C;

I l' HoaoH | rrou. phH. A


z. LeH, eroEt

90. What intermediate compound forms in Step I before


decarboxyl ation takes place'?

the

<-

. B. C. D.
A

A cyclohexadiene A cyclohexene An cr,B-unsaturated ketone Alactam

Compound 4

91. To synthesize occidentalol, Compound 6 is first


converted into a methyl ketone, which is then treated

Step

4ll
I

NHCI. HOAc

with MeLi in Et2O at *70'C. What is the role of


To act as an electrophile and accept electron density form the alpha carbon. To act as a nucleophile and donate electron density to the carbonyl carbon.

MeLi?

A.
Eto_o o

EtO\ -jSMe

fr

B.
C

Compound

HMPA, DME,62'C ---_->


Step 5 Compound 6

. To act as a base and deprotonate the alpha proton. D. To act as an acid and protonate the carbonyl
oxygen.

Figure

Synthesis of Compound 6

Step 1 involves a Diels-Alder cycloaddition followed by decarboxylation of the polycyclic system. Step 2 involves the conversion of the ketone group of Compound 2 into a ketal followed by the reduction of the ester into a primary alcohol" Further reduction of the vinylic alcohol group in Compound 3 forms a methyl group in Compound 4. The protecting group is removed in Step 4 to relorm the ketone. Compound 5 undergoes a variation on the Wittig reaction to form an alkene in Compound 6.

92. If Compound 4 were treated with


A

strong acid, at which carbon in the n-network is it most likely to gain H+?

. B.
C

Carbon a Carbon b Carbon d

D.

" Carbon c

Copyright O by The Berkeley Review@

GO ON TO THE NEXT PAGE

Questions

93 through 100 are NOT

based on

97, A

conjugated diene

is

necessary

in which of

the

descriptive passage.

following reactions? A . Claisen rearrangement B. Clemmensenreduction C . Cope rearangement D . Diels-Alder cycloaddition

93.

Leukotriene A4, LTAa, is derived from arachadonic acid. Its structure is shown below:

k
What is NOT true of LTA4?

cora
9

8. All of the following

observations are associated with an E2 reaction EXCEPT: the base must be bulky and strong. whether the major product has cis or trans geometry depends on the stereochemistry of the reactant.

A. B. C. D.

LTAa has six units of unsaturation.


LTAa has six n-electrons in a conjugated system. LTAa is capable of undergoin C 1,2-, 1,4-, 1,6-, or
1,8-addition when treated with an electrophile and nucleophile.

A. B.

C. that rearrangement is observed. D. heat is required to drive the reaction.


9

LTAa has more sp2-hybridized carbons than it spj-hyb.idir"d carbons.

has

9. All of the following


EXCEPT:

are physical properties of a terpene

94. What is the major organic product of the reaction


below?

n't\.0$1,
Br. __=+

A. High lipid solubiliry B. High boiling point C. Low volatility D. High specific rotation
I
0

t1cbHcH.,

hv

o."*,.:,'rlT,
I{TCCHCH3

0.

Which of the following molecules have dipoles NOT


equal to zero?

B.

H:C

HTCCHCH3

"'"r"
FIlCCHCH3

D'

nrc
I

..,noTr

I!CCBTCHT

L E-butene II. CzHq il. Z-butene A. I only B. II only C . iII only D. I and III only

95. Which of the following


reaction?

reactions

is a propagation

A. B. C. D.
9

H3C. + H3CCH2CH3 -+ 2 H3C. + H3CCH2.


H3C. + H3CCH2. -+ H3CCH2CH3

H3CH2C. + H3CCH3 -+ 2 H3CCH 2, + | t2H2 H3C' + H3CCH2CH3 -+ CH4 + (H3C)2CH.

6.

The terpene (+)-B-trans-Bergamontene shown below:

A. B. C. D.

is generated from three isoprene units. is a terpenoid and not a terpene. has three units of unsaturation. has sixteen possible stereoisomers.

1.B 1.8 13. c 19. c 2s. D 31. A 37. A 43. A 49. D 55. D 61. D 61. C 13. B 19. D 8s. B 91. B 91. D

2.D 8.C 14. A 20. A 26. C 32. D 38. A 44. D 50. A 56. A 62. D 68. D 14. A 80. C 86. B 92. A 98. C

3.C 4.8 5.8 9.D l0.B ll.B 15. D 16. C 1'.7. C 21. c 22. D 23. B 21. C 28. B 29. A 33. A 34. D 35. D 39. B 40. D 41. C 45. A 46. C 47. B 51. A 52. D 53. A 51. A 58. A 59. D 63. A 64. B 65. B 69. C 70. D ',71. A "/5. A 16. B 11. C 81. B 82. D 83. C 87. C 88. D 89. A 93. D 94. D 95. D 99. D 100. C

12.A
18. D

6.B

24. C 30. D 36. A 42. D 48. A


54. D

60. C 66. A 12. C 78. B 84. D 90. B 96. A

Copyright @ by The Berkeley Review@

ALL DONE, NO MORE!

Alkanes and Hydrocarbon Reactions passage Answers


Choice B is correct. Hahdes are like hydrogens when considering degrees of unsaturation, because they, like hydrogen, make only one bond with carbon. Plugging into the ronJ*itl[ formula for units of unsaturation yields an answer of 1. Pick B for best results.

Units of Unsturation = 2(#C) + 2- (#H) - (#cl)

_ 2(s)+2-(e)-(1)
2

_ 70

+2-9 -7 _12-10
2

22

_2_, r
___

Choice D is correct. According to the molecular formula, there are no units of unsaturation within the molecule, so there are three linear carbon backbones to be considered. No other carbon backbones are possible, because they would be either on9_of the following structures, but drawn differently, violaie the units of unsaturation, or not be structurally possible.
C

(-C-CC-C
I I

C-C
I

on each of the carbon backbones, the chlorine must be systematically placed to deduce the total number of structural isomers that are possible (stereoisomers do not count in this question). There are three non-equivalent carbons in the five carbon siraight chain, therefore there are three monochloro isomers of pentane. There are four non-equivalent carbons in the four carbon chain, therefore there are four monochloro isomers of 2methylbutane. There are two non-equivalent carbons in the three carbon chain, but one of the carbons (the central carbon) already has for-rr bonds, therefore there is only one monochloro isomer of 2,2-dimethylpropane. This means that there are eight structural isomers total, so pict< o for greatest success.

C_C-C_C-C

tti

C_C-C-C-C
C1

C_C-C_C-C
C1

1-chloropentane

2-chloropentane

3-chloropentane

C-C-C-C

clc
1

rl

C-C-C_C /\ CCI
2-chloro-2-methylbutane

C-C-C_C CCi
2-chloro-3-methylbutane

C-C-C_C CCI
1-chloro-3-methylbutane

lr

-chloro-2-methylbutane

C_
I

CI

C- C- C
I

7-chloro-2,2-dimethylpropane

total

C
J.

Choice C is correct. By symmetly, there are four unique carbons on methylcyclopentane, therefore chlorination can occur at a total of four different sites (four different carbons). This yields u tot^1 of four structural isomers. The correct answer is choice C.

&.{
Copyright @ by The Berkeley Review@

C1

HYDROCARI]ONS EXPLANATIONS

4.

Choice B is correct. Bromination is highly selective for tertiary carbons over secondary and primary carbons, so the exact quantity of each type of hydrogen need not be accounted for. The most abundant product from bromination results from the bromination of a tertiary carbon. Carbon number two is tertiary, so the most abundant product is 2-bromo-2-methylbutane. Pick B, and be a chemistry master.

5.

Choice B is correct. 1-chloropentane and 2-chloropentane are formed from chlorination of a primary and secondary carbon, respectively. There is a difference in reactivity between primary and secondary carbons in free radical chlorination reactions. Secondary carbons are 2.5 times as reactive as primary carbons. This means

that the ratio of hydrogens (abundance) and the relative reactivity must both be accounted for when estimating the final ratio. Drawn below is the application of quantity and reactivity. Hydrogens symbolized by Hu lead to 1-chloropentane and hydrogens symbolized by HU lead to 2-chloropentane. They are multiplied by their respective reactivity factor.

4-.^-c-.^-C\^,, L Hu-l
Ha

"\

/to

"\ l'
\H,

H,
Ha

@'ffi
n*'"irr{--

6Hux7=6:4H6x2.5=10

i----

2-chloropentane

The ratio is 10: 6, which reduces to 5 : 3. This makes choice B the best answer.
6.

Choice B is correct. Four secondary hydrogens lead to 2-chlorohexane and four secondary hydrogens lead to 3chlorohexane, sotheproductratio shouldbe 1:1. Becausethe ratio is7.07: l andnot 1: l,choiceAis eliminated. There is no resonance in the molecules (because there are no lone pairs and no n-bonds), so choice C is eliminated. The inductive effect wouid favor the formation of 3-chloropentane, so choice D is eliminated. Steric hindrance is the best explanation why the carbon-2 is selected over carbon-3, so choice B is best. Choice B is correct. The question addresses ring strain destabilizing not only the alkane, but the free radical intermediate. Based on that, choices A and C are eliminated, because they are not ring structures. A threemembered ring has more ring strain than a four membered ring, so the best answer is choice B.

7.

8.

Choice C is correct. The stability of the carbon free radical is attributed to the donation of electron density from the alkyl substituents through hyperconjugation. Because hydrogens cannot donate electron density through hyperconjugation, the stability of a free radical depends on the number of alkyl substituents attached. As a result, the stability of free radicals is tertiary > secondary > primary (3' > 2" > 1"). Quaternary free
radicals cannot exist, because the presence of four bonds and a free electron on carbon would exceed the octet rule for carbon. The best answer is therefore choice C.

9.

Choice D is correct. According to Table 1, the enthalpy change is positive for the reaction of iodine with an alkene, so it is an endothermic reaction. This means that the products are in a higher energy state than the reactants, which eliminates choices A and B. In the next to the last sentence of the first paragraph, it is stated that the second transition state is of higher energy than the first transition state, so the best answer is choice D. If you find yourself asking "do they really expect me to know this?", it's probably in the passage... find it! Choice B is correct. Initiation forms the halogen free radical (X.), so the first propagation step involves the halogen free radical as a reactant. This eliminates choices A and C. In propagation step 1, the free radical abstracts a hydrogen from an alkane, which is choice B. Eliminate choice D, because it is a termination step.
Choice B is correct. According to the data in Table 1, the strongest halogen-halogen bond is formed between two chlorine atoms. Chlorine is the second halogen from the top in the column VII of the periodic table. Atomic radius increases as you descend a column of the periodic table, so chlorine is the second smallest halogen. Therefore, the strongest halogen-halogen bond is formed between the second smallest halogen. The fluorinefluorine bond is an exception to the shorter bond equals stronger bond rule, because of inter-nuclear repulsion and the odd fact that the single bond is actually a pi-bond, rather than a sigma-bond. The best answer is choice B.

10.

1.L"

Copyright O by The Berkeley Review@

HYDROCARBONS EXPLANATIONS

12.

Choice A is correct. Hydrogen free radical is unstable, and thus it cannot be formed in the reaction mechanism. Choice A produces a hydrogen free radical, therefore the best answer is choice A. Cl'roice C is correct. If the halogen were to stabilize the free radical when attached, it would lower the transition state and make the reaction pathway for adding a second halogen more favorable. The halogen will not stabilize the free radical, because halogens are election withdrawiig by the inductive effect. Hilogens therefore do not heip stabilize the free radical intermediate so statement I is true. Because a second halide does not add to an alkyl halide, but a halide does add to an alkane, we assume that the alkane is more reactive' The difference is either in the bond broken or the bond formed. It is safe to assume that the wili choose a pathway of lowest energy so by breaking the C-H bond of the alkane preferentially over reaction the C-H of the alkyl halide, it can be concluded that the bond broken is of lower energy. rhi, makes statement II true. If statement II is true, then statement III cannot be true. The best answer is therefore choice C. As a note, an RH bond is stronger than an R-X bond, so if a halogen free radical (X.) were to react with an alkyl haiide, it would abstract the halide, not a hydrogen. This is the more logical explanation for the absence of multiple substitution products with free radical halogenation. Choice A is correct. The enthalpy of reaction for a reaction between an alkane and fluorine is listed in Table 1. By simply reading the chart, it can be seen that best answer is choice A. Sometimes answers are this easy to get, so don't be fooied into thinking every question on the MCAT will be difficult.

13.

14.

15

has increasea (bi7 sixty percent), and the rate has more than doubled. If the reaction depended just on the electrophile, the raie"would be less thang'2 x 10-3 M/s. Because the rate in Trial III is morl than tire rate in Trial I, ihe rate-determining step depends on both reagents, making it bimolecular. E2-reactions are bimolecular, so choice D is correct.

I with Trial III, the base concentration has doubled, the electrophiie concent.ation

Choice D is correct'-. Wl"l comparing Trial I with Trial II, both the electrophile concentration and reaction rate have doubled. This behavior is expected no matter what mechanism is employed. When comparing Trial

t6.

Choice C is correct. The most stable conformation has the largest substituents with anti orientation to one another' The bulkiest group on carbon two is the methyl group comprised of carbon 1 and the bulkiest group on carbon three is the ethyl group comprised of carbons + anJ s, so choiie C is correct. The Newman projections for the three staggered conformations are drawn below:

HsC

"-;,-4i'"'
Conformer from
the example CHs

cH2cH3

HsC

\->cir.cH" H\Y

Br.

.CH"

Br.

.*CH"CH3

HsC

"t,><

CHs

Conformer from which elimination takes place

Conformer with C1 and Cn anti

HsC-r

cH2cH3
CHs

H3CH2C

cHs

CH:
CHs

CH2CH3 'l'7'

choice C.

Choice C is correct" Carbons in the reactanthave sp3-hybridization while two carbons i1 the alkene product have sp2-hybridization, and thus those carbor-r, .ur-rr-roi have chirality. There are two stereocenters in the reactants (on carbon two and carbon three), and both stereocenters are lost in the elimination reaction. Because the alkene product has no chirai centers, it can have no optical activity. The best answer for the question is

18'

Choice D is correct. From the perspective of the eye as shown, the bromine atom is bonded to the rear (eclipsed) carbon, therefore choices A and C are eliminated. From the perspective of the eye, the methyl would stick out to the left and the hydrogen would stick out to the right. The-besi choice is thus answer choice D.
312

Copyright @ by The Berkeley Review@

HYDROCARBONS EXPLANATIONS

't9.

Choice C is correct. The enantiomer of the reactant is its mirror image, therefore both chiral centers must be opposite that of the reactant's chiral centers. The determination oiboth of the reactant's chiral centers is shown below. The chirai centers of the enantiomer are 23 and 3R. The best answer is choice C.
1

cHg cH2cH3
Carbon 2 = R
20.

Br

Compound is (2R,3S), so the enantiomer must be (2S,3R)

H$)
HsC Carbon 3 =
S

Choice A is correct. The role of a strong acid in an elimination reaction is to protonate the leaving group and increase its tendency to leave. Protonation occurs in the E1 reaction rather thin the E2 reaction. T;hI acii will not dehydrate the solvent, so choice B is eliminated. The sulfate ion is not a strong enough base to remove a

proton, so choice C is both wrong and impossible. Choice D should be eliminatia i*J"aiutely, because carbocation cannot be protonated. The best answer is choice A.
21..

Choice C is correct. Chlorocyclohexane when treated with a strong base und.ergoes an elimination reaction by way of an E2 mechani-sm to yield cyclohexene. There are two units of unsaturatiion in cyclohexene, one unit of unsaturation for the n-bond and one unit of nnsaturation for the ring. The correct answer is choice
C.

))

Choice D is correct. This question is another way of asking, "Which reaction proceeds via a carbocation intermediate in its mechanism?" The first two reactions proceed by an E2-mechanism, which is a one-step process having no intermediate. The E2-mechanism is predictable, becaur" of th" presence of the strong, bulky base. The base must be strong enough to remove a proton from carbon and it must te bulky enough to #inimize competition with an Sp2-reaction. Reaction III proceeds by an E1-mechanism, because of the preJence of strong acid, which protonates the hydroxyl group. This makes a better leaving group, and ultimaiely facilitates the formation of a carbocation. In fact, the location of the doubie bond in theiiial product can only be explained by a hydride shift. The hydride shift results in the conversion from a ,".orliury carbocation into a tertiary carbocation. Only Reaction iii exhibits rearrangement, so the best answer is choice D.
Choice B is correct. This question involves counting the chiral centers in each product in Figure 1 to see which molecules are chiral, and then analyzing to see if chirality is involved in the reaction. The m-ajor products from Reaction I and Reaction II both have chiral centers present. In Reaction I, two of the original three chiral centers are lost , but that still leaves one remaining chiral center, so the mixture is optically uJtirr". In Reaction II, one of the original three chiral centers is lost , which leaves two remaining chiral centers, so the mixture is optically active' In Reaction III, all three chiral centers are lost, so the fiial product mixture exhibits no optical activity. The best answer is choice B.

23.

,t

carbocation into a tertiary carbocation, which is the intermediate from which deprotonation to form the alkene product occurs. A11 three chiral centers are lost, which is also seen in Reaction III. This is best explained in choice C. On the MCAT, test-takers are expected to be able to see the analogy between the reaction in a question and a reaction in the passage.
25.

Choice C is correct. The reaction involves the use of concentrated strong acid at high temperature, so the reaction is apt to proceed via an E1-mechanism, An E1-mechanism entails the hydroxyl [roup being protonated and then leaving, producing a secondary carbocation. A hydride shift resuits in ihe conversion fropf secondary

Choice D is correct. Reaction I is an E2-elimination reaction. One of the characteristics of an E2-reaction is that the reaction is concerted, so the rate of the reaction depends on both reactants. If you increase the concentration of either reactant, in statement I the base, the reaction rate increases. This makes Statement I a valid statement. Elimination reactions require heat, so a decrease in temperature decreases the amount of elimination product that forms. You may recall that to maximize the subsiitution product and minimize the competing elimination product, temperatures are reduced. This makes Statement Il invalid, which eliminates choices B and C. Reaction II is an E2-reaction, so it has a competing Sp2-reaction occurring in the same flask. This makes Statement IiI valid, and makes choice D the best u.,r*"r.
JIJ

Copyright @ by The Berkeley Review@

HYDROCARBONS EXPLANATIONS

26.

it is not deprotonated. The best answer is choice C.


27

Choice C is correct' First and foremost, with,a strong, bulky base, the reaction proceeds by an E2-mechanism. This eliminates choices A and B' The E2-mechanism-requires that the hydrogeir being lost be oriented anti to the- leavrng group, in this case chlorine. In Reaction I, there are two anti irydrigens, and the one that is chosen, is the one that leads to the more substituted alkene product. In Reaction Ii, the"re is only one anti hydrogen, so only one-product may be. formed. The hydrog"n o., ii-," more substituted alpha carbon has gauche orientation, so

'

Choice C is correct. The two carbons common to both rings finish as doubie bond carbons in Reaction III, so they both finish with sp2-hybridization. Only choice C finish"es witn spi 1-rrAr1ii")ution, so it is the best answer. Choice B is correct. Diethyl amine is a weak base, while potassium tert-butoxide, t-butoK, is a strong bulky base' For elimination by an E2-mechanism, a strong, bulky base is necessary. This means that when using diethyl amine, there can be no elimination reaction, which eliminates choices C and D. The reaction with diethyl amine is nucleophilic substitution. The electrophiie is a secondary alkyl chioride, which could go by either an 5511 or S52 mechanism. Because the nucleophile is a good ,,rr.l"ophil", the best choice is aJ sNz reaction' An 5512 reaction results in the inversion of the reactive iite, which puts tne amine group behind the

28.

plane of the molecule and makes choice B the best answer.

29. Choice A is correct' The Silkworm Moth sex phermone has two double bonds, one that is cis substituted and the other that is trans substituted. This eliminates choice B. All of the carbons are part of a straight chain with no branching, so it has two terminal carbons that are primary and all the internal .uiborr, are secondary. There are C.is eliminar:d sinsle bond separates the two doubie bonds, so they are in l"^:"1:::I_:11,,""?,:: eliminates choice D. 91y:ne fact coni Ali of the carbons l1::^t"T1q111,_lllt so there are no stereogenic centers. This have at least two identical substituents br they nave sp'-hybridization, makes choice A the best answer.

t:tu:

30'

Choice D is correct. The Oriental Fruit Moth sex phermone differs from the structure in the question at the position of the double bond. The structure in the question ha9 cis geometry, while the phermone in Figure t has trans geometry. This can be misleadingand tempt you to pick choice g. gut, the conneitivity is not the same, so the structures are structural isomers. The n-boncl is between carbons B ancl 9 in the Oriental Fruit Moth sex phermone, but it is between carbons 9 and 10 in the compound that is shown. This makes choice D the best answer' The two structures are not interchangeable by a rotation about a bond, so they are not conformational isomers' This eliminates choice A. There is no itereogenic carbon, so the two structures cannot be optical isomers. This eliminates choice C. The two structures are structural isomers, making choice D the best answer.
Choice A is correct. UV-visible spectroscopy is used to detect n-bonds. Bombykol has conjugated n-bonds, while muscalure just one.n-bond. Conjugation tednces the transition energy, so the #avelength oi ria*i^.rm absorbance increases with conjugation. This means, that bombykol has a greatJi l,*u* than making Statement I a valid statement. Choice B is eliminated. The Tiger Moth r"* ho.-on"1s an aliphatic alkane, ^1rr."ulrrr", roit hu, no units of unsaturation. Undecane is an 11-carbon aliphalic hydrocarbon, so it too has no units of unsaturation. The two compounds have the same units of unsaturation, zero, so Statement II is invalid. This eliminates choice C. Terpenes and terpenoids contain a number of carbons that is divisible by five and a predictable connectivity that can be partitioned into isoprene subunits. The oriental Fruit Moth sex ih"r-o1" has twelve carbons in its chain and two more for the acetate group. Fourteen is not divisible by five nor is the structure one that can be broken into isoprene subunits. This makes statement III invalid and makes the choice A the best answer.

31'

32' Choice D is correct. Muscalure is a cis alkene made of 23 carbons and 46 hydrogens. It is a long chain hydrocarbon, so it has low water miscibitity. Choice A is a valid statement, and ttereby eliminated. It is excreted, so it must be a liquid under ambient conditions. This makes choice B a valid statement, which eliminates it. Because it is a long chain-hydrocarbon, lipids can dissolve into it. It has high lipid solubility, making choice C a valid statement, thereby eliminating it. There are no stereogenic centers on muscalure, so it
will not rotate plane-poiarized light. This makes
choiJe D an ir-rvalid statement] so choice D is the best answer.

33'

primary carbons. Choice A is the best answer.


Copyright O by The Berkeley Review@

Choice A is correct. All of the compounds in Figure t have straight chains (no rings), so each has at least two primary carbons' only 2-methylheptadecane his branching, ro lt hu, an extra primary carbon. No other str.ucture in Figure t has any branching, so 2-methylheptadeiane, the Tiger Moth sex phermone, has the rnost
314

HYDROCARBONS EXPLANATIONS

34.

Choice D is correct. The Green Peach Aphid defense phermone has 15 carbons connected in such a way that it can be partitioned into three isoprene subunits. Choice A is a valid statement, so it is eliminated. The Green peach Aphid defense phermone has four double boncls, so it has eight sp2-hybridized carbons. With fifteen carbons total, more than half of the carbons are sp2-hybridized. Choiie B is a valid statement, so it is eliminated. The Green Peach Aphid defense phermone has a conjugated diene, so it can undergo 1,4-addition of an electrophile. Choice C is a valid statement, which eliminates it. Because of all of the n-donds in the Green peach Aphid defense phermone, it is not very flexible. Choice D is an invalid statement, which makes it the best answer. Choice D is correct. The Oriental Fruit Moth sex phermone has an ester group. A carbonyl has an IR absorbance above 1700 crn-l , so an ester accounts for the IR absorbance at1741 .i',-r.'Ch"i;; A is a valid correlation of structure to spectroscopic observation, so it is eliminated. Bombykol has two n-bonds in a conjugated network. The presence of n-bonds in a structure results in an absorbance in the ultraviolet-visible tuig" of the EM spectrum, so an absorbance of 227 nm seems viable for a conjugated diene. Choice B is a valid iorrelation of structure to spectroscopic observation, so it is eliminated. Muscalure has two carbons involved in double bonds, each of which has a hydrogen attached. This means that the hydrogens on those carbons will be found downfield, resuiting in two signals with values around 5.00 ppm in-lUx"traR. Choice C is a valid correlation of structure to spectroscopic observation, so it is eliminated. The Tiger Moth sex phermone has eighteen carbons There are ?*.i:-{""e oj.srynmetry.signals if it two equivalent methyl gtorrp!, so there arJseventeen uniqui carbons. The TTCNMR would show 17 were of high enough iesolution. The reality is that ,.uny of the signals would overlap, so it would likely show less. It will not show fourteen signals in its 13CNMR spectrum, so choice D is an invalid correlation of structure to spectroscopic observation, makLg it the best answer.

35.

36.

Choice A is correct. Reaction I is a Diels-Alder reaction. Diels-Alder reactions involve the reaction of a conjugated diene and a dienophile. Normally we think of the dienophile as an alkene, but the only requirement is that it has a r-bond. The alkyne meets this requirement, so it is a dienophile. The best answer is choice A. There is no nucleophilic substitution gorng on, so choices B and C are eliminated. There is no change in oxidation state, so the compound is not an oxidant or reductant, eliminating choice D.
Choice A is correct. Each of the answer choices is lacking two hydrogens from the formula of the final product, CaHgO. This means that hydrogenation converts the intermediate to tne final product. The final product is an aldehyde and hydrogenation cannot convert a cyclic ether into a carbonyl, so choices C and D can be eliminated. Hydrogenation adds two hydrogen atoms to neighboring carbons. Because an aldehyde is generated, we know that one of the hydrogen atoms is added to the carbonyl carbon. The ring is cleaved, so iydrogenation must break the strained ring by adding a hydrogen to the carbonyl carbon and a h"ydrogen to the neighbJring atom. In choice A, the hydrogen atoms are correctly displaced on the intermediate-(CH2-CH-CH3) to lorm both aidehyde products (CH2-CH2-CH3 and CH3-CU-CH3) when a single hydrogen is added to tne akyl chain. This is not the case in choice B (CH2-CHZ-CHZ), so it is eliminated. Choice A is correct. On the product of Reaction I, ail of the functional groups are bonded to sp2-hybridized carbons, so there are no stereocenters. This eliminates choices B and C. Bised bn the remaini.,g ur,r*", choices, Reaction III must have formed a new stereocenter. The new stereocenter is located on the rin{ carbon with the methyl substituent. Hydrogenation can occur from above or below the ring, so the methyl is albove the plane in
racemic mixture. In Reaction II, the products have no chiral centers, so no new stereocenters were formed during the reaction. This eliminates choice D and makes choice A the best answer.

J/.

38.

fifty percent of the product mixture and below the plane in fifty percent of tnu mixture. Reaction III forms

39'

Choice B is correct. The basic tenet of green chemistry is to minimize waste and side products and maximize atom-economy. Atom-economy aims to get every atom added to the reaction containe. et-rait-rg up in the product. Using a supercritical fluid as a solvent makes for easy recovery and reuse of the solventl Iieaction III uses supercritical CO2, so choice A is valid and thus eliminated. Using protecting groups adds extra atoms to the solution that are not destined to be part of the product, so it violatei ine princfof, of atom-ecor-romy. protecting groups are difficult to recycle without spending a great deal of solvent, so they do not fit the green chemistr! philosophy. Choice B is an exception. If catalysts are part of the lab equipment, such as cataljrtic beads, then they are easily recovered and reused. This makes choice C in philosopnicat agreement with ti-re principles of green chemistry. This eliminates choice C. It is stated in the passage that direcl oxidation using oxygen fits in the philosophy of green chemistry, so choice D is eliminated. Choose B and be on top of your gui".
315

Copyright O by The Berkeley Review@

HYDROCARBONS EXPLANATIONS

40.

Choice D is correct. Reaction III is described in the passage as hydrogenation, the term applied to a reaction that adds hydrogen atoms. This eliminates choice g. rne gain of Uoias to hydrogen, u i"r, electronegative atom than carbon, is defined as reduction, so choice A is eliminated. Although it is not specified in the passage, when hydrogenating with a metal catalyst, the process adds the hydrJgen atoms in a syn fashion, which means that the reaction is stereoselective. This eiiminates choice C.- Both carbons gain a hydrogen atom, so there is no regioselectivity. This makes choice D the best answer. Choose D for the safe of correctness.
Choice C is correct. The reaction is a Diels-Alder reaction, similar to Reaction I, except an alkene is serving as the dienophile, rather than an alkyne. A Diels-Alder reaction results in the formation of a new six-membered ring, which is observed in all of the choices. When the reaction involves a conjugated cliene and an alkene, the product is a cyclohexene ring, so choices B and D are eliminated. To form choice"B, the dienophile would have needed to be an alkyne, like Reaction I. The n-bond in the product is located between the two internal carbons of the-original conjugated diene, so they should be found otr the left side of the central ring. This makes choice C the best arswer. Choose C for a brighter smile when scoring iike you did.

41..

eK-r+
42.

_^*

e[("

Choice D is correct. Reaction 1 is an elimination reaction by way of an E1 mechanism. In an E1 reaction, a lt is observed iepends on the compo,ind, but it is possible. Statement I is a valid statement. The intermediate is an allylic carbocation, where the cationic carbon is bonded to one of the carbons in the double bond, not a vinylic carbocation, where the cation carbon is one of the carbons in the double bond. Statement II is invalid, which eliminates choices B and C. The first step in acid catalyzed reactions is the protonation of some functional group on the reactant. In this particular case, the hydroxyl group is the most basic site, so it is protonated. Thii generates a good leaving grorrp, which then ieaves in the second step. Because the hydroxyl group is protonated to stait the reaction, Statement III is vaiid. This makes choice D the best answer.
carbocation is formed, so rearrangement is possible. Whether

{).

Choice A is correct. Compound III and Compound IV are structural isomers of one another. They each have a hydroxyl .group and an aikene functionality, so infrared spectroscopy yields the same key absorbances. This makes infrared spectroscopy ineffective at distinguishing the two iilyiic alcoho1s, so choice B is eliminated. Ultraviolet spectroscopy is great for determining the amount of conjugation in a system. However, both compounds have the same number of rc-bonds, one, so ultraviolet spectroscopy yields essentially the same spectrum for both compounds. Choice C is eliminated. Neither structure absorbs light in the visible range, given that neither structure has extensive conjugation. This can be inferred from the passage when thly mention that the peak at 779 nm disappears. That peak is associated with Compound I, which tappens to be one of the two enantiomers represented by Compound III. Choice D is eliminatei, because 1,79 nmis not in the visible range of the EM spectrum. The best method is 1HNMR, which can distinguish structurai isomers by their equivalent hydrogens. Choice A is the best answer. Choice D is correct' The tertiary carbon with the hydroxyl group has four unique substituents, so it is chiral. This eliminates choice A. There is only one chiral center, so diastereomers are not possible (there must be at least two chiral centers for diastereomers to be possible). This eliminates choice b. Because the hydroxyl group can be above the plane or below the plane, there is more than one structure possible for Compound IV. This eliminates choice B and makes choice D the best answer.

44.

OH Compound IV
Copyright O by The Berkeley Review@

m
OH

Enantiomers

HYDROCARBONS EXPLANATIONS

45.

Choice A is correct. The first paragraph of the passage implies that Compound I is an allylic alcohol. If you recall your general nomenclature, then you should know that when a functional group, in this case the hydroxyl group, is on the carbon bonded to the alkene carbon, it is said to be allylic. Choice A fits this description. Choice D should have been eliminated early, because it does not contains a n-bond. Choice B is eliminated, because it is a vinylic alcohol (hydroxyl group directly bonded to the alkene carbon). Choice C has the double bond too far from the alcohol group to be allylic, so it is eliminated as well. Choice C is correct. The option for either 1,2-addition or 1,4-addition occurs when the reactant has conjugated n-bonds. Choices A and D should be eliminated immediately, because when the two numbers describing the nbonds differ by 2, then the n-bonds are conjugated. Cyclopentadiene has only five carbons, so one n-bond must be between carbons one and two. The second ru-bond must be between carbons three and four, because in a five-carbon ring, no matter how you place two double bonds, for the ring to not be so strained it can't exist, they must be conjugated. Only in choice C are the n-bonds not conjugated, so choice C is the best answer. Choice A Choice B

46.

.A/
47.

f'*
H,o. \ -'
.:--\
lra

Choice C

Choice D

Choice B is correct. The hydration of Compound II starts with the addition of a proton to the conjugated nnetwork. The easiest carbon to protonate, because of steric hindrance and resonance stability, is the secondary, terminal carbon of the system. This generates the structure in choice C, so choice C is eliminated. That structure can undergo resonance to generate the structure shown in choice D. This eliminates choice D. If water were to attack the structure shown in choice C, the structure in choice A, a new intermediate, forms. This eliminates choice A. By default, the best answer is choice B. Choice B is not possible, because the structure would have to gain a proton at the more sterically hindered terminal carbon of the n-system.

, Ha
)

eI).\eD

Choice C

m
48.

Choice A is correct. At 35"C, the hydration of Compound II using sulfuric acid and water yields abottt 40"h secondary alcohol and 60"h tertiary alcohol. It is stated in the passage that "the percentage of the secondary alcohol formed increases as the temperature of the hydration reaction increases. This is attributed to a shift from kinetic control to thermodynamic control." This means that at 75'C,it is reasonable to suspect that the secondary, allylic alcohol is the major product. This eliminates choice C. The product is not a vinylic alcohol, so choices B and D are eliminated. In choice A, a secondary allylic alcohol is formed, so it is the best answer. Taking the information in the third paragraph of the passage and Figure 1, and erasing the cyclopentane ring could aiso solve this question.

49.

Choice D is correct. It is easiest to start by evaluating which pair represents enantiomers. Enantiomers, you recall, are non-superimposable mirror images. In this case, it is easier to compare the chiral centers rather than reorient the structures to see if they are mirror images. If all of the chiral centers differ, then the two structures are enantiomers. In choices A and C, only one of the three chiral centers differs between the pair, so

they are diastereomers, not enantiomers. The next factor to consider is the alignment of the carbonyl substituents. They are cis to begin with (on the alkene reactant), so they should finish cis. In choice B, the two carbonyl substituents are trans, so choice B is eliminated. In choice D, the groups are cis, so it is the best answer.
Copyright @ by The Berkeley Revierv@
31'/

HYDROCARBONS EXPLANATIONS

50.

Choice A is correct. In Triai 4, with methyi groups in positions A and B of the conjugated diene, the rate is roughly 0'005 that of 1,3-butadiene. The reason for thli reduced rate is the steric hindrance associated with the methyl qrgup pointing to the middle of the transition state. No matter how the molecule contorts, one of the two methyl groups is aiways pointing inwards, where the transition state forms. If ethyl groups were used in lieu of methyl groups, then the steric hindrance would be even more substantial and the reaction rate would be even slower. Only choice A presents a slower reaction rate, so choice A is the best answer.
Choice A is correct' According to the passage, the addition of a Lewis acid to the system increases the rate of the Diels-Alder reaction. Choice A, AlC13, is a Lewis acid, because the aluminum lacks a complete octet. Choice B, CCl4, is a common organic solvent where each atom has a satisfied octet. Because CCI4 acts like a solvent and not a Lewis acid, choice B is eliminated. Choice C, KH, is a strong base that reaaity donates electrons. Because KH acts like a base and not a Lewis acid, choice C is eliminaied. Choice D, LiAiH4, is a strong reducing agent where each atom has a satisfied octet. Because LiAIH4 acts like a reducing agent and not a Lewis acid, choice D is eiiminated. The best answer is choice A.

51.

52.

electrophiie is a ttack.

Choice D is correct. If the reaction proceeds by a nucleophilic mechanism, then we must determine which molecule is acting as the nucleophile and which is acting as the electrophile. The reaction is best when the dienopirile has an electron-withdrawing substituent, so lei us assume that the diene is the nucleophile and the dienophile is the electrophile. Based on this mechanism, the part of the intermediate that originally came from the diene should carry a positive charge (because it donated electrons) and the part of the intermediate that originally came from the dienophiie should carry a negative charge (because it accepted electrons). This is observed in each answer, choice eicept choice A, so choicJ A is eliminatecl. on the bright side, based on the answer choices, we know that our assumption about the diene being the nucleophiie anJdienophiie being the

valid. The drawing belolt' shows the forination

o?

th" first intermediate after nucleophilic

cHe

The second intermediate drawn matches choice D, so choice D is the best answer. Choices B and C could have been elimi'ated by the incorrect location of the positive charge.

53'

Choice A is correct. The dienophile is the same in Trial 2 and Trial 4, so the difference in reactivity must be attributed to the conjugated diene' This eiiminates choice D. Methyl groups, when bonded to a structure, do not act as Lewis acids, because their octets are complete. This eliminates choice C. The methyl groups are mildiy electron-donating, not eiectron-withdrawing, to .hoi." B is incorrect. The most significant iacior in the reaction rate is that the extra methyl group on the diene in Trial 4 causes steric hindrance in the transition state. No matter how the diene contorts, one of the two methyl groLlps interferes with the incoming dienophiie. The best
answer is choice A.

,f.

U r.3

__>

Minimal steric hindlance

&ilf*'--->U;
Signi licant steric hindrance

Choice D is correct. It is stated in the passage that a dienophile is enhanced when it has an electronwithdrawing group conjugated to the alkene. Choice A is enhanied by the carbonyl groups conjugated to the alkene, so choice A is eliminated. Choice B, albeit an alkyne and not an alkene, has el-ectronlwlthdrawing ester groups conjugated with the n-bond, so it is enhanced as a dienophile. Choice B is eliminated. Choice C is enhanced by the carhonyl groups conjugated to the aikene, so choice C is eliminated. In choice D, the amine group is an electron-donating group that lessens the reactivity of the dienophile. Choice D is the best answer.
Copyright @ by The Berkeley Review@

3r8

HYDROCARBONS EXPLANATIONS

55.

Choice D is correct. If the Y-group is a carbonyl group, then it is the exact same substituent as the other carbonyl group (on the adjacent carbon), making the compound symmetric and thus indistinguishabie. Both Product A and Product B are the same compound, if the reactant is symmetric, so choice D is the best answer. Choice A is correct. In the second paragraph of the passage it is stated that when X is an electron-donating group/ product A is the major product. Because the OCH3 reactant yields more Product A than the CH3 reactant in comparable reactions, it can be concluded that an OCH3 group is more electron donating than a CH3 group. The best answer is choice A. Choices B and D should have been eliminated, because tie major prlJuct is product A, not product B.
Choice A is correct. By analogy, OCH2CH3 (ethoxy) is an electron donating group like OCH3 (methoxy). The presence of the eiectron-donating group makes Product A the more favorable product. Product A fiom the generic reaction of the passage is choice A. Be careful not to choose B without paying attention to the location of the double bond. The double bond in choice B is on the side opposite from where it should be.

5b.

57.

58.

Choice A is correct. Two five-carbon species are combined, so the final product can have only ten carbons altogether. Choice D is eliminated for having twelve carbons total. One o] the double bonds is in the wrong location in both choice B and choice C. The best answer is therefore choice A. The stereochemistry with the new cyclopentyl ring trans to the bridging carbon is what is referred to as the "endo product." The arrowpushing schematic from the reactant to the product is drawn below:

ryO->OrO
59.

..rrrl\\

.,,tttlll

Choice D is correct. Structural isomers have different bonds (connectivity of atoms). Product A and Product B in the sample reaction in Figure 1 are structural isomers. Structural isomers result when both reactants are asymmetric. The best answer is choice D. Choice C is correct. For the reaction as drawn in the question, with two asymmetric reactants, there are two possible structural isomers (corresponding to product A and product B in the generic reaction) that can form. In both structural isomers, there are two new chiral centers formed. For a compound with two chiral centers, there are four (22) possible stereoisomers, meaning that there are four possible stereoisomers for each structural isomer. The result is that there are eight possible isomers total, so the best answer is choice C. In reality, not

50.

all eight isomers are observed to any measurable level in a Diels-Alder reaction. The major product results from the transition state of least steric hindrance. In a typical Diels-Alder reaction suclt as thir, th" major products are an enantiomeric pair of one of the two possible structural isomers. The less favorable structural isomer may also be formed, resulting in an enantiomeric pair, but it is generally in much lower concentration than the more favorable structural isomer.

6't.

Choice D is correct. A concerted reaction occurs in one step. Given that a sigmatropic rearrangement involves just one molecule, if it occurs in just one step, then only one product can be formed. This eliminates choice C, because there are not multiple products, let alone cross products. The stereochemistry can be lost at centers that go from sp3-hybridizationl, sp2-hybridization and it can be gained at centers that go from sp2-hybridization to spr-hybridization. Carbons that do not change hybridization cannot experience a change in stereochemistry. This means that there is no set rule about the complete retention or the complete inversion of all stereocenters. This eliminates choices A and B. The only possible answer is the one that supports no cross products being formed, because the molecule only reacts one way. Choice D is the best answer.

62.

Choice D is correct. Step III converts a cyclic ketone into a phenol, so the product has aromaticity that the reactant does not. The gain of aromaticity drives the reaction, so choices A and C are eliminated. The conversion from a ketone to phenol shifts the n-bond from the carbonyl to the benzene ring, so it is the result of tautomerization, not reduction. The best answer is choice D.
319

Copyright @ by The Berkeley Reviervo

HYDROCARBONS EXPLANATIONS

63.

released when bonds are formed, sigma or pi, so choices C and D are elimilated.
64.

its orbitals to achieve the transition state. The best answer is choice A. c-hoice B should have been eliminated, because exothermic reactions generate heat, so no heat must be added to drive them. Heat is

Choice A is correct. The role of heat in any pericyclic reaction is to provide energy for the reactant to realign

Choice B is correct. The Cope rearrangement involves a 1,S-diene, so there are six carbons within the molecular orbital of the transition state. Choice A is elimrnated because it has only four carbons. Choice C is eliminated because the orbitals show no fi-overiap between adjacent carbons. Choice D is eliminated because there is no overlap across the complete cycle. The best overlap is choice B, where the sigma-bond is present o1 the left and the terminal orbitals are aligned correctly to form i pi-bond. Choice B is correct. The oxygen is directly bonded to the benzene ring in the reactant, so it is phenylic a'd not benzylic. This eliminates choices A and C. The oxygen is also bonded to the carbon alpha to ihe aikene. This makes the carbon allylic, so choice B is the best answer. Choice A is correct. The Claisen rearrangement converts an ether into a carbonyl, so the spectroscopic evidence must depict either the loss of an ether or gain of a carbonyl group. Aldehyde protons shor,n, a signai around 9.5 ppm in the 1HNMR, so the formation of in aldehyde wouliin fact correspond with the appearince of a signal around 9.5 ppm. Choice A is the best answer. Infrared absorbances uro.r,-rd 1700 cm-l inaicate the presence of a carbonyl grouP and broad infrared absorbances around 3400 cm-1 indicate the presence of an alcohol group. No hydroxyl group appears in either the reactant or product, so choice B is eliminated. The reaction would be supported by the appearance of an absorbance around 1700 cm-1 in infrared spectroscopy, not a disappearance, so choice C is eliminated' Signals between 5 and 6 ppm in the 1HNMR co.respond to vinylic hydroge-ns bonded to aikene carbons, which are present before and afier the reaction, so choice D is eliminated.

65.

66.

67.

Choice C is correct- The first reaction in the synthesis in Figure 2, Step I, invoives the oxygen. The Claisen rearrangement involves oxygen as the ether is converted into a ketone. This eliminates ihoices B and D. According to the remaining choices, Step II is a Cope rearrangement. To determine the best answer, we must decide if the units of unsaturation decrease by one cluring the Claisen rearrangement or r,r.hether they remail constant at five. In all compounds in Figure 2, there are four n-bonds and one .lig, ,o there are aiways iive units of unsaturation. This makes choice C the best answer. You could also conclud"J thut the units of unsaturation do not change by looking at the Claisen rearrangement in Figure 1, where there are two n-bonds in both the redctant and product.

68.

a molecule must have a nurnber of carbons that is divisible by a terpene. The correct choice is D.

Choice D is correct. Terpenes are composed of isoprene subunits which are made of five carbons. To be a terpene, five. Stearol has eighteen carbons, so it cannot be

69.

Choice C is correct- If the sesquiterpene were derived from a natural source (such as extraction or distillation from a plant), then any impurities would be naturally occurring impurities. If there were two enantiomers present, that would be explained by attack at a planar site from two sides. This can occur in nature although enzymes strongly favor synthesis of one enantiomer over another. Choices A and B are eliminated, because chiral impurities can occur in nature. The dead give-away would be an impurity with sixteen carbons. Terpenes have muitiples of five for their carbon values. Because sixteen carbons is not possible, choice C is the best choice. A twenty-carbon impurity is a terpene, thurs it is naturally occurring.

v0.

Choice D is correct. The carbon that is most susceptible to nucleophilic attack is the carbon with a leaving group attached" Carbon four, with the pyropl-rosphate leaving group; is the most electrophilic. Alkene carboni do,act as electrophiles oir occasion, but in this iornpound, carbon four is more electrophilic than an alkene carbon" The best answer is choice D.
Choice A is correct. Combining three acetyl coenzyme A molecules result in six carbons total. Isoprene units have only five carbons, so one carbon must be in a side product. Carbon dioxide contains only one carbon, so choice A is the best choice. Ethanol and acetic acid each contain two carbons ancl isopropanol contains three carbons. Choices B, C, and D are all eliminated.

71"

Copl,right @ by The Berkeley Review@

HYDROCARBONS EXPLANATIONS

nt

Choice C is correct. A Diels-Alder reaction forms cyclohexene, so caryophyllene and citronellol cannot have been formed from a Diels-Alder reaction. This eliminates choices A and B. Both c,-pinene and Vitamin 41 have a cyclohexene moiety, so we must look closer. Diels-Alder reactions involve a di"ne and dienophile, so we can look at the compounds in a retrosynthetic fashion. In Vitamin ,A.1, the retro Diels-Alder reaction does not generate terpene fragments, so choice D is eliminated. Choice C is the best answer by default. Choice B is correct. Bond a can be eliminated immediately, because the fragments formed from the break are three carbons and seven carbons. Bond c can be also eliminated immediately, because the fragments formed from the break are nine carbons and one carbon. These are not multiples of five, therefore the two fragments cannot be involved in the synthesis. This eliminates choices A and C. Bond b and Bond d when broken can leave a ten carbon molecule, so neither can be eliminated. The trouble with bond d is that the fragment to the right of the break cannot form a 2-methylbutene, because it loses the tertiary carbon. Choice D is eliminated. Isoprene

,/

J.

units must be isopentenyl, not straight chain pentenyl, thus the break is not allowed. The two possible retro synthesis pathways are shown below, and only Bond b is involved. Choice B is the best answer.
Bond d CFI?

Bond d

CIT

Bond b must have been formed to connect the isoprene units.

None of the labeled bonds were


formed to connect the isoprene units.

74.

Choice A is correct. Carvone differs from limin by a carbonyl group. To go from iimin to carvone, a carbon must lose two bonds to hydrogen and gain a double bond to oxygen. This is oxid.ation, so choice A is the best answer. Choice A is correct. Ozonolysis is the oxidative cleavage of a double bond between two carbons. The resulting products are carbonyl compounds that vary from aldehydes to ketones to carboxylic acids, depending on the work up step. To undergo ozonolysis, the reactant must contain an alkene functional group. All of the compounds have an alkene functionality except for camphor. This makes choice A, camphor, the best answer. Choice B is correct. Singlets in the proton NMR are caused by unique hydrogen atoms in an environment where the adjacent atoms have no bonds to hydrogen, and thus there are no neighboring hydrogens with which coupiing can take place. In camphor, ali of the methyl groups are bonded to quaternary carbons, so they all fit this description. Because the cyciic structure is incapable of rotation, like an alkene, the two methyl groups bonded to the bridge carbon are not equivalent, causing them to express different NMR signals. The result is that each of the methyl groups are represented by a singlet in the proton NMR. All of the remaining hydrogens on camphor are on carbons adjacent to neighboring carbons with hydrogens, so there are no other singlets than the ones from the methyl groups. This generates three proton NMR singlets, so the best answer is choice B. No Hs on neighbor, cannot be rotated to be equivalentwith other bridge methyl group. No Hs on neighbor, cannot be rotated to be equivalent with other bridge methyl group. (3H singlet)
H

75.

76.

g H3cs
77.

)g--

No Hs on neighbor, isolated methyl grorrpl (3H singlet)

Choice C is correct. Myrcene contains ten carbon atoms, so the addition of another isoprene unit would result in a product with fifteen carbons total. According to the first paragraph of the passage, terpenes having fifteen carbons are referred to as sesquiterpene, making choice C the best answer.
321

Copyright @ by The Berkelel, Revie*'3

HYDROCARBONS EXPLANATIONS

Choice B is correct. Can-rphor has a carbonyl group (water soluble) and a iarge alkyl ring system (not water soluble). It is hard to decide based on the structure. It happens that the compound is watei soluble, which you may know first hand from using camphor-containing cleaning agents for skin. The question is whether or not it is highly water soluble. Because there is some ambiguity, let's say for now thai it 1ikely not highly water soluble, and consider statement I to be invalid. Camphor is a liquid at room temperature, as you ti-tignt n^rr" seen if you synthesized it in a lab experiment. Being a liquid at standard temperature, its boiling point"is above 298 K. Statement II is valid. Camphor has two chiral carbons, so it rotates plane-polarized lighi. This makes statement iII invalid. Choice B is the best answer, but not with one hundred percent certainty.
79.

information on UV spectroscopy. The minimum you should know is that n-bonds ire UV active, and with conjugation, the intensity of the absorbance increases and the energy of the absorbance decreases.
80.

Choice D is correct. It is stated in the passage that carvone has a strong UV absorbance (e > 10,000). Carvone has conjugation, which causes its intense UV absorbance. On the other hand, limin has no conjugation, so its UV absorbance is not as intense as that of carvone. This means that the UV absorbance for limin his an e less than 10,000 (therefore, log e < 1og 104 = 4). The best answer is choice D. This question required some background

Choice C is correct. A Diels-Alder reaction is a cyclization reaction that involves the addition of a diene to a dienophile (alkene) to form a cyclohexene product. Both limin and carvone are cyclohexene compounds, eliminating choices A and D, but carvone has a carbonyl group and isoprene contains only Cs and Hs. The best answer is limin, choice C.

81.

Choice B is correct. It is stated in the passage that corn oil is 63'/, iinoleic acid. Looking at table 1 shows that linoleic acid is made up of eighteen carbons and has two n-bonds. Choices C and D are eliminated, because they only have sixteen carbons. Choice A is eliminated, because it l-ras three n-bonds. The best answer is choice B. You can try to match the exact location of each n-bond from the formula in Table 1 to the drawing in the answer choices, but doing so is not time efficient.

82.

four), and the loss of unsaturation results in more molecular flexibility, which results in a higher melting poini. Unsaturated fats, with less flexibility and therefore less ability to engage in intermoleculaiinteractioni, have lower melting points than saturated fats of comparable mass. This is Co**orl organic chemistry knowledge that you should have addressed when comparing vegetable and animal fats. The coirect answer is choice D.
83.

with FADH2, the result is hydrogenation of the diene and the formation of the aliphatic carboxylic acid (of eighteen carbons) stearic acid. The gain of four hydrogen atoms increases the moleCular mass of tne acid (by

Choice D is correct. Linoleic acid contains two n-bonds, both with cis geometry. When linoleic acid is treated

Choice C is correct. Oleic acid is an eighteen-carbon acid with one n-bond between the eighth and ninth carbons. The a-bond in oleic acid has cis orientation. Treating oleic acid with deuterium (D2) and a catalytic metal like pailadium adds two deuterium atoms across the n-bond of the alkene molecule. The two deuterium atoms add syn to one another at carbons eight and nine. The result is the formation of two new chiral centers. There are no chiral centers to begin with, so the product has tr.tro chiral centers. The best answer is choice C.
Choice D is correct. Potassium permanganate reacts with alkenes to form diols by adding two hydroxyl groups is a syn addition fashion to the carbons of the n-bond. Linolenic acid has three n-bonds, located between carbons 9 and 10, carbons 12 and L3, and carbons 15 and 1,6 when the carboxylic acid carbon is considered to be carbon one (IUPAC convention). Hydroxyl groups form at all sp2-hybridized carbon sites. This results in a product with hydroxyl groups at carbons 9,70, 12,13, 15, and 16, as listed in choice D. Choice B is correct. The most bromine per moiecule is consumed by the fatty acid with the greatest number of nbonds present. For every n-bond present, one molecule of bromine tiquid will be consumed. Arachidonic acid has four n-bonds. Arachidic acid has no n-bonds present in its structure, linoleic acid has two n-bonds present in its structure, and linolenic acid has three n-bonds present in its structure. Arachidonic acid is the mosiunsaturated of the choices. The correct answer is thus choice B.

84.

85^

86.

Choice B is correct. Paimitoleic acid has sixteen carbons and one n-bond. When palmitoleic acid is fuliy hydrogenated, it forms the aliphatic acid of sixteen carbons (listed in Table t as pilmltic acid). The best
answer choice is B.
322

Copyright O by The Berkeley Review@

HYDROCAR.BONS EXPLANATIONS

87.

Choice C is correct' Figure 1 shows occidentaiol with three specified chiral centers, so choices A and B.ffi or they are methyrene (cHz) sroups' rhe methvr group and tertiirv arcohol "itn".iul" ::,il;lXl*"1'fli,*:lr":::,-".11",::lE_::ly:lt1: carbon o" chiral centers on the structure. The best answer is choice
C.

,;a_i;;fi;ii;" ""inl.i#J:T:.ffi:ffil":J""f:tJ:ii::

88.

Choice D is correct' The first paragraph states that occidentalol is a sesquiterpenoid. occidentalol has fifteen carbons total' so it is a tuatotlibl" conclus.ion that sesquiterpenes hurr" iift""r-r carbons. Choice D is the best answer' Monoterpenes have ten carbons, diterpenes trave twenty carbons, and triterpenes have thirty carbons.

89.

choice A is correct' The maximum wavelength of absorbance, r*u*, increases as the conjugation of the rcnetwork increases' All four choices are conjugaied dienes, but Compound 1 also has a carbonyl in the conjugated networks' As such, the compound with the ioigest L'u* is Compound 1, choice A.
Cl'roice B is correct' step 1 involves a Diels-Alder reaction followed by decarboxylation. The intermediate compound is the Diels-Alder product' There is. no nitrogen present in either reactant, so the compound cannot be a lactam (cyclic amide)' Thii eliminates choice ? Tie p.oar.t of a Diels-Alder reaction involving a diene and an alkene (dienophile) is cyclohexene, so choice s is the best answer. The diene in Compound 2 is regenerated from cyclohexene after decarboxylation, so choice A is eliminated. The reaction is shown below:

90.

150'C ____*> Cyclohexene Lactone (not lactam) Carbonyl is NOT conjugated to alkene

o -

C02Me
Compound
1

CO2Me CO2Me

Compound 1.5 ( intermediate compound in Step

1)

The compound is a cyclohexene with a lactone that is not conjugated to the alkene.
9"t_.

Choice B is correct' occidentalol has one. more me-thyl group than compound 6, so the role of methyl lithium, MeLi' must be to add a methyl group to compouna o. int eliminates choices C and D. Methyt lithium has an anionic carbon' so it acts as a nucleophile rither than an electrophile. This eliminates choice A and makes choice B the best answer' The methyi anion attacks the carbonyl carbon in the same fashion as an alkyl magnesium bromide anion attacks a caibonyl in a Grignard reaction. Choice A is correct' A conjugated diene can be protonated at either terminal carbon of the ru-network, because the carbocation that results is resonance stabilized. This eiiminates choices B and C. Carbon a is a secondary carbon while carbon d jgrtiary carbon. It is easier to protonate the less hindered site, so carbon a is the site 1s that is most likely to gain lH+. The best answer is choice A.

92.

93'

Choice D is correct' Leukotriene 44, LTA4, has four alkene n-bonds, one carbonyi n-bond, and an epoxide ring. choice,A is a valid sratement, which eliminates it. LrAa has three Il':-:"::i:i::::i::^:l,T'"*'l.lion. alkene n-bonds in conjugation, resulting in six n-electrons in a conjugated system. cnrr.?i:r, Jtffnri"1"""."",, which eliminates it' Because of the extensive conjugation, there are several sites at which a" nucleophile and electrophile may add' For instance, if the epoxide o*yg"n were protonated, a nucleophile could attack the ring or the left carbon of any n-bond to add u"ros the slstem, Tilis means that r,2-addition, r,4-addition, 1,6addition, and 1,8-addition are all possible. Choice C is a valid statement, which eliminates it. In all likelihood, this answer choice uurrr"d the coveted "huh?", meaning you can,t eliminate it, because you're just not sure" on a multiple choice exam, this is not a problem. You just need to look at choice D and use your testing logic' LTAa has four alkene n-bonds and one carbonyt n-bond, so tn"." ur" are twentv carbons total in LTA4, so eleven of them.are 'ri'r"-rp'r+yrrialzed carbons. There sp3-hybridlzed. There _.1",pl;;,[;;""urUo.* than sp2-iybridized carbons, ,o .hoi." D is an invalid statement, which makes"r" the best answer. it
.11i

Copyright @ by The Berkeley Revierv@

HYDROCARBONS EXPLANATIONS

94.

Choice D is correct. in the presence of light, adds to the most substituted carbon of an alkane by way ,Bromine, of a free radical mechanism. The most substituted carbon is tertiary, so the choice with bromine added to the tertiary carbon (most substituted) is the best answer. This makes choice D the correct choice. T'he product shown, as well as its enantiomer, are both formed.

95.

there is one free radical on the reactant side and three free radicals on the product side, so it is not a propagation step. Choice A is eliminated. In choice B, there are two free radicals on the reactant side and no free radicals on the product side, so it is a termination step and not a propagation step. Choice B is eliminated. In choice C, there is one free radical on the reactant side and two free radicals on the product side, so it is not a propagation step. Choice C is eliminated. In choice D, there is one free radical on the reactant side and one free radical on the product side, so it is a propagation step. Choice D is the best answer. You may not recognize the reaction from the overall mechanism, but it converts a less stable free radical into a more stable free radical, which ultimately impacts the product distribution.
Choice A is correct. Bergamontene contains fifteen carbons, so it is likely made from three S-carbon isoprene units. We can't be sure without analyzing the structtire to find the isoprene fragments, but that is not time efficient. Choice A is the best answer so far, and shall remain our choice until a better one comes. Bergamontene is a hydrocarbon with no heteroatoms, so it is a terpene and not a terpenoid. This eliminates choice B. Bergamontene has two n-bonds and a cyclohexane ring, so at first look choice C is tempting. But the molecule is bicyclic, meaning it has a second ring, the four-membered ring connected to the cycioheiane ring. Bergamontene has four units of unsaturation, not three. This eliminates choice C. To verify this, bergamontene

Choice D is correct., Free radical propagation reactions keep the free radical reaction going, so to be a propagation reaction, there must be the same number of free radicals on each side of the equation. In choice A,

has 24 hydrogens and therefore a formula of C15H24. The units of unsaturation are 1z1iS; + Z -24\/2 = 4, so choice C is eliminated. There are three chiral centers on bergamontene, so 8 (23) is the maximum number of stereoisomers, not 16. This elimir-rates choice D and secures choice A as the best answer.
97

Choice D is correct. Both the Claisen rearrangement and the Cope rearrangement require dienes, but they need not be conjugated. The two n-bonds must be separated by three sigma bonds, so choices A and C are eliminated. Clemmensen reduction converts a carbonyl into an alkane, so no diene of any kind is required. Choice B is eliminated. A Diels-Alder reaction involves the cyclization of a conjugated diene and a dienophile, so it must l-rave a conjugated diene. This makes choice D the best answer.
Choice C is correct. For an E2 reaction, the base must be strong enough to remove a proton from carbon and bulky enough to not undergo substitution. This makes choice A a valid statement and thereby eliminates it. For an E2 reaction, the leaving group and proton being lost from carbon must be positior-red anti to one another, so the geometry of the product is dependent upon the alignment of the reactant. Cis versus trans results from the orientation and stereochemistry, so choice B is a valid statement and thereby eliminated. For an E1 reaction, a leavilg group first leaves, resulting in a carbocation. With carbocations, rearrangement can be observed, so it is with E1 reactions that we see rearrangement, not E2 reactions. Because E2 reactions are concerted, there is no rearrangement, so choice C is an invalid statement and thereby the correct answer. Heat is required to drive both El and E2 reactions, so choice D is a valid statement. It is eliminated, leaving choice C as our choice.

99.

from consideration. It is important that you use all of your test taking stcllls. The specific rotation of a compor-rnd is dictatecl by its chiral centers, which a terpene may or may not have. Given that there is no general rule about the chirality of terpenes, we cannot conclude that they have high specific rotations" Choice
D is the best answer.
100.

Choice D is correct. Terpenes are hydrocarbons of 70,15,20, etc... carbons, so they are somewhat massive lipids. Because they are hydrocarbons, they are lipid soluble, so choice A is a valid statement. Choice A is consequently eliminated. Terpenes have molecular masses of about 140 g/mole, about 210 g/mole, about 280 g/moie, etc..., so they have somewhat high boiling points. High is a relative term, so we ian't be certain in eliminating choice B. However, choices B and C are the same concept, so they mutually exclude one another

Choice C is correct. To have a dipoie nol eqr-ral to zero is to have a dipole. To have a dipole is to be polar. Cis compouuds are always polar so Compound III is polar. Ethvlene is perfectly symmetric, io choice Compound II is nonpolar. The question comes down to: "Is Compound I polar?" Compound I is not polar, because thl methyl groups on t1-ie alkene cancel one another and sum to a resultant vector of 0. Choose C for best results.
)1+

Copyright @ by The Berkeley Review@

HYDROCARI}ONS EXPLANATIONS

Das könnte Ihnen auch gefallen